You are on page 1of 282

CASOS CLNICOS SAEI

DE ENFERMEDADES
INFECCIOSAS
2015
PARA RESIDENTES
EDITOR
RAFAEL LUQUE MRQUEZ
CASOS CLNICOS SAEI EDITOR

DE ENFERMEDADES Rafael Luque Mrquez


INFECCIOSAS Unidad de Gestin Clnica de E. Infecciosas,

PARA RESIDENTES Microbiologa y M. Preventiva.


Hospital Universitario Virgen del Roco. Sevilla.

EDICIN 2015 EDITORES ASOCIADOS

Francisco Javier Martnez Marcos


Unidad de Gestin Clnica de E. Infecciosas.
Complejo Hospitalario Universitario de Huelva.

Antonio Plata Ciezar


Unidad de Gestin Clnica de E. Infecciosas
y Microbiologa.
Hospital Regional Universitario. Mlaga.

David Vinuesa Garca


Unidad de Gestin Clnica de E. Infecciosas
y Microbiologa.
Hospital Clnico San Cecilio-Complejo Hospitalario
Universitario de Granada.

Elisa Fernndez Fuertes


Unidad de Medicina Tropical. Servicio
de Medicina Interna.
Hospital de Poniente. El Ejido. Almera.
CASOS CLINICOS SAEI
DE ENFERMEDADES INFECCIOSAS
PARA RESIDENTES
Edicin 2015

Sociedad Andaluza de Enfermedades Infecciosas (SAEI)

ISBN: 978-84-608-4882-0

Diseo y maquetacin: lvaro Pedrero

Los contenidos de esta obra reflejan la opinin de sus autores y no representan la


opinin de la Sociedad Andaluza de Enfermedades Infecciosas.

Los derechos de copia corresponden a la Sociedad Andaluza de Enfermedades


Infecciosas. Cualquier forma de reproduccin, distribucin, comunicacin pblica
o transformacin de esta obra slo puede ser realizada con su autorizacin que
debe ser solicitada a su Secretara Tcnica (secretariatecnica@saei.org)

*Los editores han comprobado que los tratamientos recomendados estn de


acuerdo con la prctica clnica aceptada en el momento de la publicacin. Sin
embargo, se recomienda a los lectores consultar los ltimos datos aportados por
los fabricantes y las agencias reguladoras sobre las dosis, vas de administracin,
duracin de tratamiento y contraindicaciones.
ndice

CAPTULO 1
INFECCIONES RESPIRATORIAS Y DE ORL 11
CAPTULO 2
INFECCIONES ABDOMINALES
(INCLUIDAS GASTROENTERITIS) 21
CAPTULO 3
INFECCIONES URINARIAS
Y GINECOLGICAS 39
CAPTULO 4
INFECCIONES DEL SISTEMA
NERVIOSO CENTRAL 49
CAPTULO 5
INFECCIONES DE PIEL,
PARTES BLANDAS Y PI DIABTICO 71
CAPTULO 6
INFECCIONES OSTEOARTICULARES 85
CAPTULO 7
ENDOCARDITIS, INFECCIONES ASOCIADAS
A DISPOSITIVOS ENDOVASCULARES
Y OTRAS INFECCIONES CARDIOVASCULARES 95
CAPTULO 8
INFECCIN VIH
Y ENFERMEDADES ASOCIADAS 121
CAPTULO 9
INFECCIONES DE TRASMISIN SEXUAL 147
CAPTULO 10
INFECCIONES TROPICALES
Y DEL VIAJERO 157
CAPTULO 11
INFECCIONES EN PACIENTES
INMUNODEPRIMIDOS Y TRASPLANTADOS 189
CAPTULO 12
INFECCIONES POR HONGOS 211
CAPTULO 13
INFECCIONES POR MICOBACTERIAS 227
CAPTULO 14
INFECCIONES
POR PATGENOS ESPECIALES 261
Prlogo

Transcurridos ya los primeros quince aos del Siglo XXI, la especiali-


dad de Enfermedades Infecciosas sigue sin estar reconocida oficialmente
en nuestro pas, que se desmarca as inexplicablemente de los estados de
nuestro entorno socioeconmico. Afortunadamente, inmunes al desen-
canto que dicha situacin provoca, cientos de clnicos espaoles dedica-
mos nuestros esfuerzos cotidianos a la asistencia, investigacin y docencia
en esta amplia rea de conocimiento. Los resultados de dichas labores se
plasman en la literatura cientfica nacional e internacional y su excelencia
queda demostrada por el hecho de que nuestro pas ocupa el cuarto lugar
en el mundo y el tercero en Europa en cuanto a produccin cientfica en
enfermedades infecciosas. Ensayos clnicos sistematizados, estudios de co-
hortes multicntricas, documentos de consenso y guas clnicas llevados a
cabo o elaborados en Espaa proporcionan evidencia y ayudan a mdicos
de todo el mundo a tomar decisiones correctas para sus pacientes con
infecciones.
Pero fuera del marco de la investigacin organizada, los clnicos nos en-
contramos con frecuencia en nuestro trabajo diario cuadros patolgicos
concretos cuya originalidad, dificultad diagnstico-teraputica o sorpren-
dente desenlace los convierten en valiosas y amenas herramientas docen-
tes, pero que no tienen cabida, o muy escasa, en la literatura mdica con-
vencional.
Por ello, la Sociedad Andaluza de Enfermedades Infecciosas (SAEI) efec-
ta una convocatoria anual, dirigida en especial a sus socios ms jvenes,
para que stos aporten casos clnicos interesantes, que luego son rigurosa-
mente evaluados por un Comit de Seleccin. De esta forma, los cuatro
de mayor puntuacin son expuestos presencialmente en la Reunin de
Primavera de la Sociedad y luego todos los que han superado el proceso
de seleccin son publicados para dar forma a este libro que ahora tienes
ante tus ojos. Quiero felicitar a su Editor, a los Editores asociados y a los
autores, sin cuyo entusiasta esfuerzo y contribuciones esta obra no hubiera
visto la luz. Deseo agradecerles su excelente trabajo y animarlos para que
este libro se siga publicando a lo largo de tantos aos que llegue a conver-
tirse en un clsico en nuestras bibliotecas.
Como presidente de la SAEI es para m un honor y un gran placer presen-
tar esta edicin 2015 de Casos clnicos SAEI de enfermedades infeccio-
sas para residentes, pues constituye un signo patognomnico del pujante
presente de la infectologa andaluza y, dada la juventud de la mayora de
sus autores, tambin un indiscutible prdromo de su brillante futuro.

Salvador Prez Corts


Presidente de la Sociedad Andaluza de Enfermedades Infecciosas
Presentacin

El caso clnico ha sido una forma clsica de publicacin en las revistas


biomdicas en la que los autores, a partir de una experiencia personal que
consideran paradigmtica, realizan una reflexin sobre la prctica clnica,
con especial nfasis en los apartados de diagnstico diferencial, estrategia
diagnstica y tratamiento. Adems, llevan a cabo una revisin actualizada
del problema clnico, destacando las ideas claves que puedan ser aplicables
a la prctica diaria.
La Sociedad Andaluza de Enfermedades Infecciosas (SAEI) ha conside-
rado que la publicacin de casos clnicos constituye una herramienta
formativa de gran utilidad para los residentes que se aproximan al co-
nocimiento de las enfermedades infecciosas. Tanto para los autores que,
utilizando como referencia un caso clnico personal han revisado la bi-
bliografa actual sobre el tema, siempre supervisados por un especialista
con formacin y experiencia en enfermedades infecciosas, como para los
lectores que podrn encontrar informacin actualizada sobre algunos de
los problemas ms interesantes a los que con frecuencia se enfrentan en su
prctica diaria.
Pero adems estos casos clnicos suponen para algunos su primera ex-
periencia con el complejo mundo de las publicaciones biomdicas. Ello les
obliga a ordenar las ideas, expresarlas de manera concisa, y adaptarse a
las normas internacionales que se exigen para la publicacin en las revistas
de nuestro entorno.
La SAEI realiz una convocatoria dirigida a residentes de nuestra comu-
nidad con una gran acogida. La seleccin de casos se realiz tras una
evaluacin por los miembros del comit editorial donde se puntuaron la
originalidad, el inters clnico, la calidad expositiva y la calidad de la dis-
cusin. Finalmente se aceptaron sesenta y cinco casos, contenidos en este
libro y clasificados por captulos, que recogen algunas de las reas ms
importantes de las enfermedades infecciosas, lo que esperamos facilite su
identificacin y lectura. Contienen abundante documentacin grfica que
seguro facilitar su aprendizaje.
Existe el compromiso de la SAEI de mantener una convocatoria anual de
la que se seleccionaran los casos para futuras publicaciones. Los editores
confan en que en las prximas ediciones se puedan mejorar las deficien-
cias que inevitablemente puedan encontarse en esta publicacin.

Los editores
CAPTULO 1
INFECCIONES RESPIRATORIAS Y DE ORL
Paciente de 44 aos con fiebre, disnea e infiltrado
y afectacin radiogrfica multilobar bilateral
Martnez Posada, P; Lojo Cruz, C;
De la Campa Gestido, A; Merchante Gutirrez, N.
Hospital Universitario de Valme. Sevilla.

Caso clnico Da 1 Da 3 Da 6 Da 10
Leucoctios K/uL 6,5 1,9 22,5 9,9
Linfocitos K/uL 0,74 0,78 0,47 1,92
Varn de 44 aos obeso, sin otros factores de riesgo cardio-
Neutrfilos K/uL 4,85 0,9 21,24 6,97
vasculares conocidos, ex adicto a drogas va inhalada desde
hace 10 aos y ex bebedor desde hace un ao, sin hbitos Hemoglobina g/dL 17,1 14,1 8,8 9,8

txicos en la actualidad y con antecedentes de pancreatitis Plaquetas K/uL 124 22 37 109

enlica en 2006. Reside, desde hace un ao, en un centro de INR 1,43 1,07 1,28

deshabituacin. Acude a Urgencias por clnica de tres das Urea mg/dL 46 61 83 160

de evolucin de intensa tos con expectoracin purulenta, Creatinina mg/dL 2,46 4,95 2,11 3,83

fiebre de 41C con tiritona y deposiciones lquidas. Horas Sodio mEq/L 132 144 135 133

antes de acudir al hospital comienza con disnea progresiva, Potasio mEq/L 4,1 4,3 4,6 6,5

hemoptisis y hematuria. Procalcitonina >100 75

En la exploracin fsica a su llegada se constata mal estado Tabla 1. Evolucin determinaciones analticas.
general, est consciente y orientado, palidez mucocutnea e
intenso trabajo respiratorio. Saturacin del 98% con CPAP bilateral, de aspecto nodular a nivel paracardial izquierdo,
de alto flujo y tensin arterial de 220/130 mmHg. A la aus- con pinzamiento posterobasal bilateral y atrapamiento a-
cultacin se encuentra taquicrdico y el murmullo vesicu- reo retroesternal en la proyeccin lateral (Ver Figura 1).
lar est disminuido con crepitantes generalizados en ambos Tras las medidas iniciales el paciente se deteriora, aparece
campos pulmonares. No hay otros hallazgos de inters en la hipotensin, mayor hipoxemia y se traslada a UCI donde
exploracin. precisa intubacin orotraqueal (IOT). Tras la IOT se ex-
Se inicia antibioterapia emprica con cefotaxima (2 gr intra- traen varias muestras de aspirado bronquial (BAS).
venosos iv-cada 6 horas) y levofloxacino (500 mg iv cada 12
horas) con el objetivo de cubrir los microorganismos ms fre- Diagnstico diferencial
cuentes causantes de neumona de adquisicin comunitaria
(NAC) y sueroterapia. Se extraen hemocultivos seriados y se Nos encontramos ante un paciente que ingresa en situa-
solicita la determinacin en orina de los antgenos de Neumo- cin de sepsis por una NAC. Los principales microorganis-
coco y Legionella. Se realiza una primera determinacin ana- mos causantes de NAC son:
ltica donde destaca un fallo renal no conocido previamen-
te, trombopenia, linfopenia y alteracin de los parmetros Streptococcus pneumoniae: microorganismo principalmen-
bsicos de la coagulacin (ver tabla 1). En la radiografa de te productor de neumona aunque puede ocasionar
trax (RX) inicial se aprecia un infiltrado pulmonar alveolar infecciones a otros niveles (peritonitis primaria en pa-
cientes cirrticos enfermos renales, meningitis, etc).
Haemophilus influenzae: su incidencia ha disminuido tras
la introduccin de la vacunacin frente al tipo b, pero
debemos sospecharla en pacientes ancianos o con en-
fermedad pulmonar obstructiva crnica (EPOC).
Escherichia coli y Klebsiella pneumoniae: pueden encontrar-
se, como otras colonizando la faringe de personas an-
cianas, cirrticos, diabetes mellitus, enfermedad renal
crnica, desnutricin o neoplasia activa.
Coxiella burnetii: hay que sospecharla en pacientes ex-
puestos a animales (ganado, roedores, aves o con pica-
Figura 1. Rx: infiltrado pulmonar alveolar bilateral nodular. duras de garrapatas) en reas endmicas.

12
CAPTULO 1
INFECCIONES RESPIRATORIAS Y DE ORL

En el caso de las NAC de presentacin grave, como la que en pacientes con neutropenia grave y prolongada, en-
presenta nuestro paciente, deberamos pensar, adems en: fermedad granulomatosa crnica, EPOC, alcoholis-
mo, cirrosis heptica y/o diabetes mellitus.
Staphylococcus aureus: sobre todo si existe algn tipo de
inmunodepresin (alcohlicos, diabticos, usuarios de Evolucin
droga va parenteral, que traumatismo craneoencef-
lico, infeccin por el virus de la gripe o IOT) si bien se En las primeras 12 horas tras su ingreso en la UCI se man-
han descrito casos en pacientes previamente sanos. Las tiene el deterioro progresivo, con precisa aminas vasoacti-
cepas productoras de leucocidina de Panton-Valenti- vas, se establece el fracaso renal en y el fallo respiratorio.
ne, sobre todo cepas de Staphyloccocus aureus meticilin Ante la persistencia de la situacin de shock se aade al
resistente (MRSA) pueden ocasionar una neumona tratamiento inicial Linezolid (600 mg cada 12 horas iv) y
necrosante grave. Por tanto debe considerarse en un se ampla el estudio microbiolgico, aadindose a las pe-
adulto joven previamente sano o nio con una infec- ticiones la serologa de VIH, virus de hepatitis (VH) y de
cin pulmonar rpidamente progresiva que y/o que se neumonas atpicas as como la determinacin de PCR
acompae de necrosis pulmonar, shock y neutropenia. de virus de la gripe en BAS extrado tras la IOT.
Virus de la gripe (Influenzavirus A,B o C): su diagnsti- En este perodo comenzamos a obtener los resultados mi-
co suele ser clnico y los sntomas ms frecuentes son crobiolgicos de las muestras extradas inicialmente: en la
escalofros, fiebre, mialgias, cefalea, tos no productiva primera tanda de hemocultivos crece Staphylococcus aureus
y/u odinofagia. Puede originar, adems, neumonas meticilin resistente (MRSA), al igual que en el BAS, donde
vricas o por sobreinfecciones bacterianas. En el caso se asla el MRSA (con el mismo perfil de sensibilidad) y
de infeccin por el virus H5N1 se puede producir una crece Aspergillus fumigatus. La baciloscopia fue negativa, as
infeccin grave con neumona en ocasiones hemorr- como la antigenuria para neumococo y legionella y la de-
gica y de curso grave. terminacin de gripe. La serologa fue negativa para VIH,
Legionella pneumophila: por inhalacin de aerosoles con- VHB y neumonas atpicas: Mycoplasma pneumoniae, Chlamy-
taminados. Tienen mayor riesgo personas con en- dophila pneumoniae, Coxiella burnetti y Legionella pneumophila. La
fermedades crnicas, tabaquismo, ancianos o inmu- determinacin de anticuerpos IgG VHC fue positiva por
nodeprimidos, fundamentalmente trasplantados. La lo que se realiza la PCR para deteccin de ARN viral que
antigenuria para Legionella pneumophila es un mtodo de es negativo. Ante estos resultados se ampla el estudio y
deteccin rpido, con una alta sensibilidad y especifi- se pide B-D Glucano y Galactomanano en suero y BAS y
cidad, debe ser complementada con tcnicas de detec- seguidamente se modifica la antibioterapia actual por Li-
cin de ADN por PCR e inmunofluorescencia directa. nezolid y Voriconazol.
Chlamydophila (Chlamydia) pneumoniae: es difcil estimar
su incidencia por las dificultades que conlleva su diag-
nstico, que se puede realizar mediante serologa (mi-
croinmunofluorescencia), inmunohistoqumica, PCR
o cultivo.
Mycoplasma pneumoniae: patgeno principalmente de
nios y adolescentes pero cada vez ms frecuente en
adultos, especialmente ancianos. Los mtodos de diag-
nstico incluyen el cultivo (de crecimiento lento), PCR
y la serologa con IgM (aumento al final de la primera
semana) e IgG.
Infeccin por anaerobios: en aspiracin, presencia de
cavidad pulmonar en un paciente con predisposicin a
broncoaspiracin o existencia de secrecin purulenta
(esputo o empiema).
Infeccin por hongos, como Aspergillus spp: ocasionan
principalmente alveolitis alrgica y asma extrnsecas,
sinusitis crnica en atpicos, o aspergilosis broncopul-
monar alrgica. La neumona aparece generalmente Figura 2. Rx 72 horas. Progresin con infiltrado multilobar y cavitaciones.

13
CAPTULO 1
INFECCIONES RESPIRATORIAS Y DE ORL

Los hemocultivos extrados a las 24 horas del inicio de tra- elevada sospecha (clnica, epidemiolgica o en las pruebas
tamiento continan positivos para MRSA. A las 48 ho- complementarias).
ras se negativizan. La determinacin de Galactomanano En relacin al tratamiento dirigido, Vancomicina contina
y B-D Glucano en suero as como de Galactomanano en siendo una opcin teraputica aceptable a dosis objetivo
BAS fueron positivas. individualizada (segn los parmetros de farmacocintica
Se consigue la estabilizacin clnica, analtica y radiolgica y farmacodinamia) pero al menos a una concentracin de
a las 72 horas de ingreso (ver tabla 1) (ver Figura 2). Se entre 15-20 mcg/mL en suero en adultos sanos, aunque
mantiene antibioterapia instaurada hasta el da 7 cuando esto no asegura concentraciones pulmonares adecuadas.
aparece nuevo pico febril acompandose de deterioro cl- Los resultados de varios estudios retrospectivos muestran
nico franco. Se recogen nuevas muestras microbiolgicas: que Linezolid es mejor opcin en los casos de neumona
hemocultivos seriados, urocultivo, BAS. Se revisa el catter por MRSA asociada a ventilacin mecnica, adems del
central que no presenta datos de infeccin y se realiza eco- efecto inmunomodulador asociado, siendo la asociacin de
cardiograma transesofgico donde no hay lesiones sospe- Linezolid y Clindamicina recomendada por expertos en
chosas de endocarditis. pacientes jvenes, con taquicardia, taquipnea, hemoptisis
El da 11 el paciente fallece. Se reciben los resultados de los y presencia de infeccin cutnea. Estudios in vitro mues-
ltimos hemocultivos, urocultivo y BAS: negativos. tran que Ceftarolina puede ser una opcin teraputica en
estos casos, incluidos aquellos con sensibilidad intermedia
Diagnstico final a Vancomicina.
Por otra parte nuestro paciente presenta un cultivo de BAS
Shock sptico con fallo multiorgnico por NAC bacteri- donde se asla Aspergillus fumigatus, aislamiento que se con-
mica por MRSA. Colonizacin por Aspergillus fumigatus. firma en los resultados del BAS extrado a las 24 horas.
Tambin es positiva la determinacin de Galactomanano
Discusin en suero y BAS. Dado que las especies de Aspergillus son
ubicuas en la naturaleza, la inhalacin de conidias es fre-
Las infecciones por MRSA se han relacionado general- cuente. El diagnstico de certeza ser el aislamiento de hi-
mente con el mbito hospitalario pero en los ltimos aos fas en la biopsia de tejidos afectos.
se ha constatado mayor incidencia (en torno al 5% en Es- Entre los factores de riesgo para presentar una aspergilosis
paa) en personas sin contacto reciente con el sistema sa- invasiva se encuentran la presencia de neutropenia seve-
nitario. ra y/o prolongada, la toma de altas dosis de corticoides
En relacin a los factores de riesgo los datos de las distintas o de otros tratamientos inmunosupresores. El riesgo ms
series son discordantes, algunas describen gran parte de los elevado se ha observado en pacientes con trasplante de c-
casos en pacientes jvenes sin comorbilidad, mientras en lulas hematopoyticas o de rgano slido o con neoplasias
otras se observa una mayor frecuencia en casos con otras hematolgicas que condicionen neutropenia severa. Tam-
patologas. Su presentacin clnica tambin es variable y bin se ha observado una alta incidencia en pacientes con
la aparicin de tos y fiebre suele ser constante, pero no as un menor grado de inmunodepresin pero que se encuen-
el dolor pleurtico. Los hallazgos en las pruebas comple- tran en una UCI, sobre todo si padecen de EPOC y se han
mentarias caractersticos son la afectacin multilobar en la tratado con corticoides.
RX, que adems est presente en el 85-94% de los casos En cuanto a las manifestaciones clnicas, la aspergilosis in-
y la aparicin de cavitaciones durante la evolucin. Se ha vasiva afecta principalmente al pulmn y puede aparecer
descrito la presencia de metstasis spticas, aunque la en- fiebre, dolor torcico, tos y/o hemoptisis. Otras formas de
docarditis secundaria es anecdtica. Los hemocultivos casi presentacin pueden ser la traqueobronquitis, aspergilosis
siempre son positivos. necrotizante crnica, rinosinusitis, afectacin del sistema
Suele cursar con datos de gravedad asociados principal- nervioso central, endoftalmitis, endocarditis y la aspergi-
mente a la presencia de insuficiencia respiratoria y aso- losis cutnea o gastrointestinal. Los hallazgos radiolgi-
cia una mortalidad elevada (entre 25-50%). Uno de los cos ms frecuentes son la aparicin de ndulos mltiples,
problemas observados en la literatura es la frecuencia de cavitados o no, consolidaciones mltiples o parcheadas o
tratamiento emprico inadecuado administrado ya que en imagen en rbol en brote en la TAC, as como derrame
las recomendaciones actuales no est incluido el uso de un pleural o infiltrado en vidrio deslustrado.
antimicrobiano activo frente a MRSA en las NAC comu- El valor predictivo positivo del cultivo de esputo (equiva-
nitarias (por su baja incidencia) salvo cuando existe una lente al BAS) o del lavado bronquioalveolar (BAL) depen-

14
CAPTULO 1
INFECCIONES RESPIRATORIAS Y DE ORL

a) Patologa de base:
Otros mtodos diagnsticos disponibles son la PCR, la tin-
1. Neutropenia (<500 cls/mm3) reciente durante ms de 10 das. cin directa con calcoflor y mtodos en desarrollo como
2. Transplante alognico mdula sea.
3. Tratamiento con corticoides a dosis >0,3mg/kg/da de equivalente de
el dispositivo de flujo lateral (detecta antgenos extracelula-
prednisona durante al menos tres semanas. res excretados por Aspergillus) o la deteccin de metabolitos
4. Tratamiento inmunosupresor de clulas T en los ltimos 90 das.
5. Inmunodeficiencia severa innata. en aire espirado (por cromatografa/espectrometra).
Por tanto, el diagnstico de aspergilosis invasiva es com-
b) Criterios clnicos y radiolgicos:
plicado tanto por el riesgo de colonizacin y contamina-
1. Infeccin del tracto respiratorio inferior con presencia de uno de los cin como por el bajo valor predictivo de los cultivos de
siguientes signos: Lesiones densas, bien delimitadas (con o sin signo
del halo), signo de atrapamiento areo, cavitacin. muestras respiratorias, fundamentalmente esputo. La Or-
2. Traqueobronquitis (lcera, ndulo, pseudomembrana, placa o erosin ganizacin Europea para la Investigacin y Tratamiento
en estudio broncoscpico).
3. Infeccin senos paranasales, en asociacin a uno de los siguientes del Cncer (EORTC) y el Grupo de Estudio de Micosis
signos: Dolor agudo localizado, lcera nasal con escara negra, rotura Americano (MSG) elaboraron unas recomendaciones
de barrera sea incluyendo rbita.
4. Infeccin del sistema nervioso central, con presencia de uno de los diagnsticas para infeccin fngica invasiva de modo que
siguientes signos: lesin focal en las pruebas de imagen, engrosamiento
menngeo.
se establecen tres criterios diagnsticos -infeccin proba-
da, infeccin probable o infeccin posible (ver tabla 2) tras
c) Criterios microbiolgicos (micolgicos):
analizar los antecedentes, la presentacin clnica (incluyen-
1. Anlisis directos (citologa, microscopa directa o cultivo). do la imagen RX) y la positividad microbiolgica o histo-
2. Hongo filamentoso en esputo, lavado broncoalveolar, cepillado bron-
quial o aspirado de senos paranasales, indicado por: lgica. De este modo la infeccin es probada cuando existe
Presencia de hifas confirmacin por histologa o por cultivo de tejido estril,
Cultivo de hongo filamentoso.
3. Anlisis indirectos (deteccin de antgeno). es probable cuando se cumple a)+b)+c) y posible si se
4. Aspergilosis: galactomanano en suero, plasma, lavado broncoalveolar o cumple a)+b) (al menos un factor de cada grupo).
lquido cefalorraqudeo.
5. Infeccin fngica invasiva (no criptoccica o mucoral): -D-glucano en Voriconazol y anfotericina liposomal constituyen la base
suero.
del tratamiento en todas las formas de aspergilosis y po-
Tabla 2. Recomendaciones diagnsticas EORTC y MSG. Pauw B et al. saconazol, itraconazol, caspofungina y otras equinocandi-
nas son alternativas eficaces.
de del husped y de la clnica y es mayor en los casos de Nuestro paciente desarrolla un shock sptico por una NAC
pacientes con factores de riesgo para aspergilosis invasiva bacterimica por MRSA. Si bien es una etiologa poco
y cuando la muestra se ha obtenido del tracto respiratorio frecuente, el curso de la enfermedad cuando se establece
inferior. La deteccin del antgeno Galactomanano (poli- puede ser grave y por tanto es de vital importancia la sos-
sacrido integrante de la pared celular de varias especies pecha clnica pues el tratamiento actualmente propuesto
de hongos) en suero puede ser positiva incluso antes del en las guas no es activo frente a este microorganismo. Para
inicio de la clnica y si bien la sensibilidad observada vara establecer dicha sospecha clnica debemos guiarnos por los
entre el 30-100%, la especificidad es alta en varias revi- antecedentes epidemiolgicos, la clnica y los hallazgos ra-
siones, por encima del 75%. Hay que tener en cuenta en diolgicos descritos. En cuanto al aislamiento de Aspergillus
relacin a esta determinacin que la sensibilidad disminu- fumigatus en nuestro caso podra tratarse de una coloniza-
ye con la administracin de antifngicos y han observado cin pues no existen antecedentes epidemiolgicos (criterio
falsos positivos en tratamientos con ciertos betalactmicos necesario en las recomendaciones diagnsticas), la clnica
como la piperacilina-tazobactam o la amoxicilina-clavul- se explica por la infeccin por MRSA y por el escaso valor
nico, en infecciones por Histoplasma, Penicillium o Fusarium predictivo de las muestras de cultivo respiratorias tomadas
y en el tratamiento con Plasma-Lyte. Numerosos estudios y de la determinacin de Galactomanano en ausencia de
objetivan un valor predictivo positivo bajo (<50%) y un clnica y sin disponer del punto de corte de la positividad.
valor predictivo negativo elevado (>90%) y por tanto tiene Se instaur tratamiento frente a este microorganismo dada
un mayor rendimiento productivo en los casos en los que la severidad clnica y el riesgo vital del paciente.
la sospecha clnica es alta. Su determinacin en el BAL
puede aportar una mayor sensibilidad, si bien no est claro
cul puede ser el punto de corte ptimo. Por otro lado, la
determinacin de B-D-Glucano es una prueba diagnsti-
ca ms sensible pero menos especfica que la anterior y
presenta igualmente falsos positivos (infecciones por otros
hongos, infecciones por Pseudomonas aeruginosa, etc.).

15
CAPTULO 1
INFECCIONES RESPIRATORIAS Y DE ORL

Bibliografa

1. Obed M, Garca-Vidal C, Pessacq P, Mykietiuk A, Via-


sus D, Cazzola L, et al. J.Clinical features and outcome
of community-acquired methicillin-resistantStaphylo-
coccus aureuspneumonia. Enferm Infecc Microbiol Clin
2014; 32: 23-7.
2. Cercenado E, Luis de Gopegui E. Staphylococcus au-
reusresistente a la meticilina de origen comunitario.
Enferm Infecc Microbiol Clin 2008; 26 (Supl 13):
1924.
3. Mandell LA, Wunderink RG, Anzueto A. Infectious
Diseases Society of America/American Thoracic
Society consensus guidelines on the management of
community-acquired pneumonia in adults. Clin Infect
Dis 2007; 44 (Suppl 2): S27S72.
4. Vardakas KZ, Matthaiou DK, Falagas ME. Incidence,
characteristics and outcomes of patients with severe
community acquired-MRSA pneumonia. Eur Respir
J 2009; 34: 114858.
5. Nathawani D, Morgan M, Masterton RG. Guidelines
for UK practice for the diagnosis and management of
methicillin-resistant Staphylococcus aureus(MRSA) infec-
tions presenting in the community. J Antimicrob Che-
mother 2008; 61: 97694.
6. Peyrani P,Ramirez J. What is the best therapeutic
approach tomethicillin-resistant Staphylococcus au-
reuspneumonia? Curr Opin Infect Dis2015; 28: 164-70.
7. Fortn J, Meije Y, Fresco G, Moreno S. Aspergillosis.
Clinical forms and treatment. Enferm Infecc Micro-
biol Clin 2012; 30: 201-8.
8. Ascioglu S, Rex JH, de Pauw B, Bennett JE, Bille J,
Crokaert F, et al.Defining opportunistic invasive fun-
gal infections in immunocompromised patients with
cancer and hematopoietic stem cell transplants: an in-
ternational consensus. Clin Infect Dis. 2002; 34: 7-14.
9. Vandewoude KH, Blot SI, Depuydt P, Benoit D,
Temmerman W, Colardyn F, et al.Clinical relevance
of Aspergillusisolation from respiratory tract samples
in critically ill patients. Crit Care. 2006; 10:R31.doi:
10.1186/cc4823.

16
Varn de 57 aos con tos,
fiebre y dolor torcico
Cornejo Saucedo, MA; Brun Romero, FM;
Guerrero Snchez, F.
Hospital Universitario Puerta del Mar. Cdiz.

Caso clnico

Varn de 57 aos, ex fumador desde hace 28 aos, con


gammapata monoclonal IgG kappa de bajo riesgo, y vida
totalmente activa e independiente (acude al gimnasio y
hace ejercicio a diario).
Dos semanas antes de su ingreso comienza con tos seca y
fiebre que atribuye a un cuadro catarral de vas altas. Das
ms tarde sufre dolor torcico de caractersticas atpicas,
con irradiacin a regin interescapular y escpula izquier-
da, motivo por el que acude a Urgencias y es diagnostica-
do de posible contractura muscular. Inicia tratamiento con
analgsicos, antiinflamatorios y relajante muscular, ade-
ms de un antitusgeno. Ante la persistencia de la tos y el
dolor a pesar del tratamiento pautado, acude nuevamente
al Servicio de Urgencias. En ningn momento haba pre-
sentado disnea.
En la exploracin presenta un buen estado general, TA Imagen 2. Radiografa lateral de trax. Derrame pleural bilateral e infiltrado en LII.
110/74 mmHg, FC 95 lpm, T 36 C, SpO2 basal 93%,
sin taquipnea ni trabajo respiratorio. La auscultacin car- En la valoracin inicial, las pruebas complementarias rea-
diaca fue normal, y en la auscultacin pulmonar se oan lizadas fueron:
crepitantes bibasales. El resto de la exploracin no mostr
hallazgos patolgicos. Analtica bsica (hemograma, bioqumica, coagula-
cin), donde destaca 11.260 leucocitos/l (91,3% neu-
trfilos), PCR 547,9 mg/l, procalcitonina 1,69 ng/ml,
fibringeno 1.200 mg/dl, CK 124 U/L, troponina T
ultrasensible 15 ng/l, GOT 103 U/L y GPT 90 U/L.
Radiografa de trax, en la que se objetiva la presencia
de derrame pleural bilateral de predominio derecho,
con infiltrado neumnico en lbulo inferior izquierdo
(imgenes 1 y 2).
ECG: taquicardia sinusal a 124 lpm, sin otros hallaz-
gos patolgicos.
Sistemtico de orina sin hallazgos patolgicos.

Diagnstico diferencial

Desde un punto de vista sindrmico, y una vez realizadas


las pruebas complementarias bsicas en el momento de su
llegada a Urgencias, el diagnstico ms probable es una
sepsis respiratoria por neumona de adquisicin comunita-
ria en lbulo inferior izquierdo (tos, fiebre, dolor torcico
Imagen 1. Radiografa posteroanterior de trax. Derrame pleural bilateral. atpico que podra ser pleurtico, elevacin de reactantes

17
CAPTULO 1
INFECCIONES RESPIRATORIAS Y DE ORL

de fase aguda, foco de condensacin neumnica en la ra-


diografa de trax, taquicardia), si bien esto no explicara
la existencia de derrame pleural derecho y crepitantes bi-
basales en la auscultacin.
La aparicin de dolor torcico e interescapular, a pesar de
tratarse de un dolor de caractersticas atpicas, hace nece-
sario descartar la presencia de patologa cardiaca tipo sn-
drome coronario agudo o aneurisma disecante de aorta in-
tratorcica. La ausencia de cambios elctricos y elevacin
de enzimas cardiacas hace improbable el primer diagnsti-
co; para el segundo requeriramos de nuevas exploraciones
complementarias (ecocardiograma, TC de trax), aunque Imagen 3. Ecografa abdominal. Derrame pleural derecho.
parece menos probable por su menor frecuencia y ausen-
cia de factores de riesgo cardiovascular como hipertensin
arterial o tabaquismo.
Con respecto al derrame pleural bilateral con crepitantes
bibasales, adems de la posibilidad de que el derrame pleu-
ral izquierdo pudiera ser paraneumnico o tratarse incluso
de un empiema, hace pensar en la posibilidad de insufi-
ciencia cardiaca izquierda. sta, descartadas las patologas
cardiovasculares previamente citadas, podra encuadrarse
en el contexto de una endocarditis infecciosa sobre vlvula
nativa u otras valvulopatas, afectacin pericrdica (peri-
carditis, derrame pericrdico), o enfermedades con afecta-
cin cardiopulmonar mucho menos prevalentes (mixoma
auricular, amiloidosis cardiaca, conectivopatas). En la ex-
ploracin no se objetivaron soplos ni roce pericrdico, y el Imagen 4. Ecocardiograma transtorcico. Derrame pericrdico severo.
ECG no mostr signos de pericarditis aguda. No obstante,
para descartar totalmente estas afecciones sera necesaria Evolucin
la realizacin de un ecocardiograma y otras determinacio-
nes analticas. El paciente qued hospitalizado en la Unidad de Medicina In-
Otras posibilidades diagnsticas que pudieran explicar la terna. A su ingreso se realiz serologa de VIH, VHB, VHC,
presencia de crepitantes bilaterales sera la existencia de una sfilis, CMV y VEB, todas ellas negativas. La determinacin
fibrosis pulmonar o una linfangitis carcinomatosa, aunque en de ANA y ANCA fue negativa, descartndose patologa au-
ausencia de otros sntomas y signos parece razonable descar- toinmune. En la ecografa abdominal se visualizaron signos de
tarlas al menos en el momento inicial. esteatosis heptica, que podra justificar la discreta elevacin
El dolor escapular pudiera tambin estar en relacin con le- de transaminasas, adems del derrame pleural (imagen 3). Se
siones osteomusculares (contractura secundaria al ejercicio, realiz ecocardiograma transtorcico, en el que se inform
lesin ltica tipo plasmocitoma o metstasis de primario no de derrame pericrdico severo con datos ecocardiogrficos de
conocido). Para la primera haba recibido tratamiento sin xi- compromiso hemodinmico (imagen 4).
to, mientras que las segundas pueden descartarse al no visua- Ante la presencia de derrame pleural y pericrdico se proce-
lizarse lesiones seas en la radiografa de trax. di a la realizacin de toracocentesis diagnstica y pericar-
La elevacin de transaminasas podra estar en relacin con diocentesis con colocacin de drenaje pericrdico y salida
un amplio abanico de posibilidades: esteatosis heptica, in- de material purulento, cuyas respectivas citobioqumicas
feccin crnica por virus de hepatitis B o C, infeccin aguda fueron compatibles con exudado polimorfonuclear sin ca-
por otros virus hepatotropos (fiebre Q, citomegalovirus, virus ractersticas de empiema (lquido pleural derecho: pH 7,43,
de Epstein Barr, etc.), enolismo no reconocido, bajo gasto por 1.102 leucocitos con 77% polimorfonucleares, glucosa 100
insuficiencia cardiaca, lesiones ocupantes de espacio a nivel mg/dl, protenas 5,7 g/dl, LDH 631 U/L; lquido pericr-
heptico, etc, siendo necesarias nuevas determinaciones ana- dico: pH 7,39, 31.729 leucocitos con 93% polimorfonuclea-
lticas y la realizacin de ecografa abdominal para aclararlo. res, glucosa <2 mg/dl, protenas 6,8 g/dl, LDH 2317 U/L).

18
CAPTULO 1
INFECCIONES RESPIRATORIAS Y DE ORL

La determinacin de ADA fue normal y los marcadores tu- productora de pericarditis constrictiva y derrame pericrdi-
morales y cultivos (incluido el de micobacterias) fueron ne- co, siendo la ecocardiografa una herramienta esencial para
gativos. En el anlisis citolgico nicamente se encontraron su diagnstico y seguimiento. Adems, esta tcnica nos per-
datos de importante inflamacin aguda inespecfica. mite detectar la existencia de derrame pleural en pequea
En dos muestras de hemocultivos se aisl Haemophilus in- cuanta, con una sensibilidad muy superior a la radiografa
fluenzae no capsulado. de trax. Por otro lado, nos permite identificar sus caracte-
El paciente recibi tratamiento antibitico (ceftriaxona + rsticas (tabicaciones y contenido pus, sangre, trasudado-),
levofloxacino) intravenoso durante 14 das, completando diferenciar el pioneumotrax del absceso de pulmn y loca-
posteriormente un total de 4 semanas de tratamiento oral lizar el punto adecuado para la realizacin de toracocentesis
con levofloxacino. Adems, se llev a cabo tratamiento de- o colocacin de tubo de trax5.
plectivo con furosemida y antiinflamatorio con ibuprofeno. En cuanto a la etiologa, Streptococcus pneumoniae es el microor-
La evolucin fue favorable desde el punto de vista clnico ganismo aislado con mayor frecuencia en estas situaciones,
(desaparicin de la tos, dolor torcico e interescapular, y seguido de Staphylococcus aureus y grmenes gram-negativos
disminucin de los estertores crepitantes), analtico (nor- como Proteus, Escherichia coli, Pseudomonas y Klebsiella spp. Hae-
malizacin de los reactantes de fase aguda), radiolgico mophilus influenzae ha sido implicado de forma inusual en esta
(reduccin del derrame pleural) y ecocardiogrfico (al alta, enfermedad, sobre todo en casos de neumona complicada
derrame pericrdico ligero sin datos de pericarditis). en pacientes inmunocomprometidos6,7, aunque tambin se
han descrito casos en pacientes inmunocompetentes8,9.
Diagnstico final Su tratamiento consiste en antibioterapia asociada a peri-
cardiocentesis evacuadora. Para reducir la inflamacin del
Finalmente, el paciente fue diagnosticado de neumona de pericardio pueden usarse antiinflamatorios no esteroideos
adquisicin en la comunidad en lbulo inferior izquierdo, o colchicina. En ocasiones, y especialmente en los casos
bacteriemia por Haemophilus influenzae y pleuropericarditis producidos por Haemophilus influenzae, puede ser necesaria
purulenta con derrame pericrdico severo e insuficiencia la pericardiectoma e incluso, a veces, la instilacin de fr-
cardiaca secundaria. macos trombolticos como la estreptoquinasa10.
En nuestro caso podemos considerar que se trataba de un
Discusin paciente inmunocompetente (presentaba una gammapata
monoclonal benigna y de bajo riesgo). En cuanto al origen
La pericarditis aguda es la patologa ms frecuente del de la pleuropericarditis, no se pudo establecer la etiologa
pericardio. En los pases desarrollados, ms del 80% de con certeza debido a que los cultivos de lquido pleural y
los casos son idiopticos, con una causa presumiblemente pericrdico fueron tomados tras varios das de ingreso y
viral, teniendo habitualmente un curso benigno y autoli- tratamiento antibitico, por lo que stos fueron negativos.
mitado o con buena respuesta al tratamiento antiinflama- No obstante, por el hecho de que el lquido pleural y pe-
torio; no obstante, en su estudio deben descartarse la etio- ricrdico extrados tuvieran un aspecto purulento y pre-
loga neoplsica, tuberculosa, la pericarditis secundaria a sentaran caractersticas de exudado, y en ausencia de otra
enfermedad sistmica y la pericarditis purulenta1. causa que lo justifique, damos por hecho que el origen se
sta ltima viene definida por la presencia de pus macros- encuentra localizado en el pulmn, y que el agente causal
cpica o microscpica en el espacio pericrdico, siendo la debi ser Haemophilus influenzae, bien por contigidad o de
infecciosa una de sus posibles causas2. Tpicamente se pre- forma secundaria a bacteriemia. El antecedente de un cua-
senta con fiebre, taquicardia, tos y dolor torcico, adems dro catarral en los das previos podra hacer pensar tambin
de disnea y un posible roce pericrdico a la auscultacin en la posibilidad de una pericarditis aguda de origen vrico,
cardiaca, si bien algunos de estos signos y sntomas pue- pero stas suelen ser benignas y autolimitadas y las caracte-
den estar ausentes. En la era preantibitica, la extensin rsticas del lquido pleural y pericrdico suelen ser diferentes;
directa desde el pulmn durante el curso de una neumona adems, la buena respuesta al tratamiento antibitico frente
era la va de adquisicin ms frecuente. En la actualidad al microorganismo sealado apoya nuestra teora.
este mecanismo fisiopatolgico es infrecuente. Existen una Por ltimo, queremos resaltar dos aspectos fundamenta-
serie de factores predisponentes para su desarrollo como les en la prctica clnica diaria: por un lado, la gran uti-
la inmunosupresin, el abuso de alcohol, la insuficiencia lidad de la ecografa clnica en los servicios de Urgencias
renal crnica o el traumatismo torcico3,4. y Medicina Interna para el diagnstico precoz y el segui-
La pericarditis purulenta est reconocida como potencial miento de patologas graves, ya que se trata de una tcnica

19
CAPTULO 1
INFECCIONES RESPIRATORIAS Y DE ORL

no invasiva, poco costosa, de una gran rentabilidad y que


puede ser realizada a pie de cama del enfermo, con todas
las ventajas que esto supone en el manejo y el pronstico
de nuestros pacientes; por otro, la necesidad imperiosa de
practicar una toracocentesis diagnstica a aquellos pacien-
tes que presentan un derrame pleural de nueva aparicin
antes de iniciar cualquier tratamiento que pueda interferir
en su diagnstico etiolgico, salvo que exista alguna con-
traindicacin para su realizacin y que la sospecha clnica
no permita la demora en el inicio del tratamiento.

Bibliografa

1. Imazio M. Contemporary management of pericardial


diseases. CurrOpinCardiol. 2012;27:308-17.
2. lvarez Navascus R, Snchez Cembelln M, Carrio-
Montes I, Quiones Ortiz L, Guerediaga Madariaga
J. Pericarditis purulenta por Streptococcus agalactaie. An.
Med. Interna. 2005;22:198.
3. Sagrist-Sauleda J, Barrabs JA, Permanyer-Miralda
G, Soler-Soler J. Purulent Pericarditis: Review of a
20-Year Experience in a General Hospital. J Am Coll
Cardiol. 1993;22:1661-5.
4. Albal Martnez N, Moneo Gonzlez A, Waez Tata-
ri B, Argelles Baquero A, Ferrero Zorita J, Martn
Bentez JC. Pericarditis neumoccica: presentacin
de un caso y revisin de la literatura. Med Intensiva.
2005;29:308-12.
5. Henrquez-Camacho C, Garca-Casasola G, Gi-
lln-Astete C, Losa J. Ultrasound for the diagnosis of
infectious diseases: Approach to the patient at point
of care and at secondary level. J Infect. 2015. Mar
19. pii: S0163-4453(15)00077-8. http://dx.doi.or-
g/10.1016/j.jinf.2015.03.003. [Epub ahead of print]
6. Varghese V, George JC. Purulent Pericarditis Caused
by Haemophilus influenzae Type B. J Invasive Cardiol.
2011;23:110-12
7. Yeh YH, Chu PH, Yeh CH, Jan Wu YJ, Lee MH, Jung
SM et al. Haemophilus influenzae pericarditis with tam-
ponade as the initial presentation of systemic lupus
erythematosus. Int J Clin Pract. 2004; 58:1045-47.
8. Garg P, Gupta R, Szalados JE. Bacterial Pericarditis
and Tamponade Due to Non encapsulated Haemoph-
ilus influenzae Complicating a Case of Adult Commu-
nity-Acquired Pneumonia. Med Gen Med. 2006;8:48.
9. Iggo R, Higgins R. Bilateral empyema and purulent
pericarditis due to Haemophilus influenzae capsular
type B. Thorax. 1988;43:582-3.
10. Keersmaekers T, Elshot SRE, Sergeant PT. Primary
bacterial pericarditis. Acta Cardiol. 2002;57:385-7.

20
CAPTULO 2
INFECCIONES ABDOMINALES
(INCLUIDAS GASTROENTERITIS)
Paciente de 17 aos que acude por ictericia,
elevacin de transaminasas y plaquetopenia
Navarro Marn, LJ; Campos Calero, A;
Lpez Snchez, MV; Herrero Rodrguez, C.
Complejo Hospitalario Universitario de Jan.

Caso clnico incluyeron serologas para infecciones por grmenes ms


infrecuentes: Brucella, Leptospira, VIH y fiebre Q (capaces
Varn de 17 aos sin antecedentes personales ni familia- de producir un cuadro patolgico similar) que igualmente
res de inters, que consult al Servicio de Urgencias por fueron negativas.
astenia y molestias abdominales difusas de dos semanas de Otro de los puntos a tener en cuenta en el diagnstico del
evolucin, tras la vuelta de un viaje a Melilla por motivos caso presentado y que debamos descartar son las hepatitis
deportivos. Negaba ingesta de medicamentos, productos inducidas por frmacos drogas ilcitas y las de etiologa
de herbolario o drogas de abuso. No tena tatuajes, pier- isqumica.
cings ni conductas de riesgo sexual. Nuestro paciente negaba consumo de txicos y medica-
En la exploracin fsica estaba consciente y orientado, bien mentos (se realizaron test en orina: negativos) adems las
hidratado y perfundido. T 36,5 C Llamaba la atencin elevaciones de las transaminasas suelen estar por encima
una ictericia cutneo-mucosa y mnimas lesiones petequia- de 100 veces el valor de referencia en esos casos.
les en regin torcica y espalda. Auscultacin cardiopul- Otros planteamientos que consideramos como causa, an
monar sin hallazgos patolgicos. Abdomen con dolor di- menos frecuente, de hepatitis en nuestro paciente fueron la
fuso a la palpacin del hipocondrio derecho. El resto de la hepatitis autoinmunes. Se practicaron estudios inmuni-
exploracin era normal. tarios con el fin de descubrir alguna anomala en el sistema
Las exploraciones complementarias iniciales fueron: inmune del enfermo, no encontrando ninguna alteracin.
Hemograma: leucocitos 4.110/mm3 (63%N; 18.9%L; La enfermedad de Wilson y otras causas no hepticas: en-
11.8%M); Hb 15.3 g/dL; Hcto 44.6% y plaquetas fermedad celaca, miopatas, hipotiroidismo, insuficiencia
21.000/mm3. Bioqumica: GOT 4020 U/L, GTP 5698 suprarrenal fueron tambin descartadas.
U/L, GGT 181 U/L, bilirrubina total 10,20 mg/dL y
PCR 9,4 mg/dL. Perfil renal, iones y glucemia normales. Evolucin
Estudio de coagulacin: actividad de protrombina 68% y
tiempo de protrombina 13.9 seg. El paciente permaneci durante las primeras semanas del
En radiologa de trax y ecografa abdominal no se eviden- ingreso con buen estado general mantenindose la eleva-
ciaron hallazgos de inters. cin de transaminasas y no remontando la plaquetopenia a
pesar de las medidas adoptadas por nuestros hematlogos
Diagnstico diferencial (necesit mltiples transfusiones de plaquetas, se inici tra-
tamiento con inmunoglobulina humana intravenosa, cor-
Una elevacin de transaminasas 10 veces por encima del ticoides y ciclosporina, sin respuesta alguna).
lmite superior de la normalidad junto a cifras de fosfatasa Ante la negatividad de las pruebas solicitadas y ante el em-
alcalina que no superen 3 veces el lmite superior, sugieren peoramiento progresivo de la plaquetopenia y de la coagu-
la existencia de una hepatitis aguda. lacin se plante derivar al paciente a una Unidad de He-
Hasta el 90% de los casos de hepatitis aguda son de etio- patologa de referencia para programar biopsia heptica,
loga viral, entre los que cabe destacar: virus hepatotropos la cual fue descartada por el alto riesgo de sangrado.
(A, B, C, D, E ), citomegalovirus, virus de Epstein-Barr o Mientras tanto se complet el estudio mediante TAC tora-
virus Herpes simplex. co-abdominal y nueva ecografa abdominal objetivndose
En el caso clnico que nos ocupa, el origen infeccioso fue en ambas exploraciones como nico hallazgo de inters
nuestra primera sospecha diagnstica, motivo por el que se una esplenomegalia de 13 cm.
solicitaron las serologas anteriormente indicadas, las cua- Recibimos la serologa positiva a VHE, pero la evolucin
les resultaron negativas (a expensas de recibir el resultado clnica no era la esperable y el paciente evolucion hacia
VHE, al no estar en nuestra cartera de servicios y precisar un empeoramiento del estado general y en la analtica se
de laboratorio externo de referencia). Posteriormente, se objetiv una llamativa pancitopenia (leucocitos 550/mm3

22
CAPTULO 2
INFECCIONES ABDOMINALES
(INCLUIDAS GASTROENTERITIS)

(50%N; 41.9%L; 5.8%M ); Hb 9.6 g/dL; Hcto 26.5%; puede conducir a la insuficiencia de rganos. El trmino
plaquetas 15.000/mm3) junto con una elevacin de ferri- hemofagocitosis (se refiere literalmente a "comer" he-
tina y triglicridos, y el mantenimiento de la elevacin de mates) se caracteriza por la presencia de glbulos rojos,
transaminasas y bilirrubina (bilirrubina total 17 mg/dl; plaquetas o glbulos blancos o fragmentos de estas clulas,
GOT 2003 U/l; GPT 3925 U/l). Ante dichos hallazgos dentro del citoplasma de los macrfagos. Se puede obser-
analticos y la mala respuesta clnica, sospechamos la aso- var en las biopsias de los tejidos inmunes (ganglios linfti-
ciacin de un sndrome hemofagoctico que se confirm cos, bazo, hgado) o de aspirados de mdula sea. Aunque
con un aspirado de mdula sea. puede ser un marcador de la activacin excesiva de los ma-
A pesar de las medidas adoptadas y el tratamiento precoz crfagos y apoya el diagnstico, la hemofagocitosis por s
iniciado por Hematologa (entre los que se incluy trata- sola no es patognomnica.
miento con etopsido y planteamiento de TMO) el pacien- La HLH se presenta como una enfermedad febril junto
te falleci a consecuencia de una aspergilosis broncopul- con la afectacin de mltiples rganos. Por lo tanto, los sig-
monar invasiva. nos y sntomas iniciales pueden imitar cualquier infeccin
y debutar como una fiebre de origen desconocido, una he-
Diagnstico final patitis o una encefalitis.
Los signos clnicos principales incluyen los siguientes:
Sndrome hemofagoctico secundario a hepatitis por virus E.
Hepatomegalia 67- 95 %
Discusin Esplenomegalia 69%
Linfadenopata 33 %
La linfohistiocitosis hemofagoctica (HLH) o sndrome he- Sntomas neurolgicos 33 %
mofagoctico, es un sndrome agresivo y potencialmente Rash 31 %
mortal de la activacin inmune excesiva. Es ms frecuente Otras manifestaciones pueden incluir fallo respiratorio
en los pacientes hasta los 18 meses de edad, pero la enfer- (SDRA), fallo renal, inestabilidad hemodinmica, etc.
medad tambin se observa en adultos de todas las edades.
Puede ocurrir como un trastorno familiar o espordico, y Es tpico encontrar citopenias (especialmente anemia y
puede ser desencadenado por una variedad de eventos que trombocitopenia), niveles de ferritina elevados (superior a
perturban la homeostasis inmune. La infeccin es un dis- 500), elevacin de transaminasas (mayor de tres veces el
parador comn, tanto en aquellos con una predisposicin lmite superior) LDH y bilirrubina.
gentica como en los que no la tienen. Existen varios tipos:
Para establecer su diagnstico se requieren cinco de los si-
Primario: tambin llamado linfohistiocitosis hemofagoctica guientes signos:
familiar, causada por la mutacin de un gen.
Secundario (adquirido o espordico): se ha utilizado Fiebre 38.5 C
para describir a aquellos adultos que no presentan una Esplenomegalia
mutacin familiar conocida y en los cuales se identifica Citopenia sangre perifrica, con al menos dos de los si-
un desencadenante claro del episodio de HLH siendo guientes: hemoglobina <9 g / dl; plaquetas <100.000
las infecciones virales, especialmente el virus de Eps- / microlitro; recuento absoluto de neutrfilos <1.000
tein-Barr, el ms frecuente, seguido de las enfermeda- / microL
des autoinmunes y procesos neoplsicos. Hipertrigliceridemia (Triglicridos en ayunas >265
Sndrome de activacin macrofgica sndrome he- mg/dL) y/o hipofibrinogenemia (fibringeno <150
mofagoctico reactivo: es una forma de HLH que mg/dL)
se presenta principalmente en pacientes con artritis Hemofagocitosis en la mdula sea, el bazo, los gan-
idioptica juvenil u otras enfermedades reumatolgi- glios linfticos o el hgado
cas. Actividad baja o ausente de las clulas NK
Ferritina> 500 ng/mL
En general, se cree que la inflamacin excesiva es causada CD25 elevada (soluble de IL-2 receptor alfa) de dos
por una falta de regulacin de macrfagos activados y lin- desviaciones estndar por encima de las normas espe-
focitos. Los macrfagos se activan y segregan cantidades cficos para su laboratorio ajustadas por edad.
excesivas de citoquinas, causando dao tisular grave que

23
CAPTULO 2
INFECCIONES ABDOMINALES
(INCLUIDAS GASTROENTERITIS)

En nuestro caso, el paciente cumpla 5 de los 8 criterios: immunodeficiency virus infection. Int J Hematol.
hemofagocitos en el aspirado de mdula sea, ferritina ele- 2003; 7:450-2.
vada, hipertrigliceridemia, citopenia (de las tres series) y 6. Otrock ZK, Eby CS. Clinical characteristics, prognos-
esplenomegalia. tic factors, and outcomes of adult patients with hemo-
La hepatitis E, generalmente es asintomtica, aunque pue- phagocytic lymphohistiocytosis. Am J Hematol. 2015;
de presentar una sintomatologa solapable a la del resto de 90:220-4.
hepatitis por virus hepatotropos: astenia, malestar general, 7. Price J. An update on hepatitis B, D, and E viruses.
dolor abdominal, hepato-esplenomegalia, ictericia y ele- Top Antivir Med. 2014; 21:157-63.
vacin de transaminasas, tal y como ocurra con nuestro 8. Sridhar S, Lau SK, Woo PC. Hepatitis E. A disease of
paciente. El largo tiempo de incubacin de la hepatitis por reemerging importance. J Formos Med Assoc. 2015;
virus E (unos 40 das de media) hace poco probable que se 114:681-90
contagiara en su viaje a Melilla, pero tampoco lo descar-
ta. Es su mala evolucin y su escasa respuesta lo que hace
buscar explicaciones alternativas etiolgicas a su cuadro
sindrmico y a su fatal desenlace.
Lo excepcional de este caso es que el desencadenante de
la HLH fuese el virus de la hepatitis E, generalmente in-
dolente y con baja mortalidad (02-03%). No hemos en-
contrando en la literatura ningn caso publicado de dicha
asociacin, siendo siempre los desencadenantes vricos
ms frecuentes: los virus Epstein-Barr y el citomegalovirus,
lo que hace pensar en la existencia de factores predispo-
nentes, que no pudieron comprobarse.
Este caso es un claro ejemplo de la rapidez de progresin
del sndrome de HLH secundario a una infeccin vrica
(excepcional que sea por VHE), que nos obliga a una alta
sospecha clnica, sobre todo al presentar una evolucin ca-
tastrfica con unas tasas de mortalidad superiores al 90% a
pesar de las medidas adoptadas de forma precoz.

Bibliografa

1. Filipovich A, McClain K, Grom A. Histiocytic Disor-


ders: Recent Insights into Pathophysiology and Prac-
tical Guidelines. Biol Blood Marrow Transplant 16:
S82-S89 (2010).
2. Jordan MB, Allen CE , Weitzman S, Filipovich AH,
McClain KL. How I treat hemophagocytic lympho-
histiocytosis. Blood 2011; 118: 4041-52.
3. Trottestam H, Horne A, Aric M, Egeler RM, Filipo-
vich AH, Gadner H, Imashuku S, Ladisch S, Webb D,
Janka G, Henter JI. Histiocyte Society: Chemoimmu-
notherapy for hemophagocytic lymphohistiocytosis:
long-term results of the HLH-94 treatment protocol.
Blood. 2011;118:4577.
4. Ramos-Casals M, Brito-Zern P, Lpez-Guillermo A,
Khamashta MA, Bosch X: Adult haemophagocytic sy-
ndrome. Lancet. 2014; 383:1503-16.
5. Chen TL, Wong WW, Chiou TJ. Hemophagocytic
syndrome: an unusual manifestation of acute human

24
Colecistitis de etiologa atpica
y otros hobbies
Macas Dorado, S; Valiente Mndez, A.
Hospitales Universitarios Virgen Macarena y Virgen del Roco. Sevilla.

Caso clnico Cuando acude a CCEE persiste la fiebre, termometrndo-


se en la consulta 39.2C, y se encuentra muy afectado con
Presentamos el caso de un varn de 35 aos sin alergias signos de irritacin peritoneal a la exploracin, por lo que
medicamentosas conocidas. Como hbitos txicos, es acepta ingreso en planta.
fumador de medio paquete diario y bebedor de 120 gra- Ante la presencia de un sndrome febril subagudo de pro-
mos de alcohol diarios, as como ex-usuario de drogas por bable foco abdominal, aunque sin certeza del mismo, se
va parenteral. Entre sus antecedentes mdico-quirrgicos inicia antibioterapia con ceftriaxona 1 g/24 h de acuerdo a
destaca infeccin por virus de la hepatitis B y C y una es- la gua de antibioterapia de nuestro centro para cobertura
plenectoma postraumtica hace 12 aos. Tiene 2 perros, de patgenos entricos y se extrae un nuevo hemograma,
2 gatos y 2 pjaros como animales domsticos. bioqumica, hemocultivos, urocultivo, serologa frente a
Inicialmente acude por presentar desde hace 5 das fiebre fiebre Q , rickettsiosis, parvovirus y VIH y se solicita TC
de hasta 39 C, vmitos y dolor abdominal generalizado. urgente para diagnstico y descartar complicacin, donde
Consulta en Urgencias e inicia tratamiento con amoxicili- se objetiva pared vesicular edematosa sin litiasis en su in-
na/cido clavulnico 875/125 mg, 1 comprimido cada 8 terior (Figura 1).
horas y tratamiento sintomtico del dolor y la fiebre. A las
48 horas consulta nuevamente por aumento del dolor ab- Diagnstico diferencial
dominal, que adems ahora se focaliza en flanco derecho y
epigastrio. A la exploracin: paciente febril (38 C) y afecta- El engrosamiento parietal vesicular en la mayora de las
do por el dolor pero sin signos de peritonismo y con signo ocasiones est relacionado con una colecistitis aguda de
de Murphy negativo. Hemograma normal, bioqumica con etiologa litisica; sin embargo existen otras patologas bi-
hiponatremia (126 mEq/l), transaminasas elevadas (GOT liares o extravesiculares que tambin provocan edema ve-
107 U/L [0-37], y GPT 141 U/L [0-40]) y PCR 96 mg/l sicular (Grfico 1).
(0-5). Orina con presencia de nitritos positivos y leucocitu- Entre las causas vesiculares, encontramos la colecistitis
ria. Se realiza ecografa abdominal urgente que sugiere una aguda alitisica, que se da habitualmente en enfermos con
pielonefritis derecha no complicada. El paciente solicita alta factores de riesgo predisponentes. En la colecistitis crnica
voluntaria, por lo que se indica tratamiento ambulatorio con tambin puede existir edema, aunque con menor frecuen-
amoxicilina/cido clavulnico 1000/62.5 mg 2 comprimi- cia que en la colecistitis aguda (25% frente a 50-75%). El
dos cada 12 horas y se cita en Consultas Externas (CCEE)
de Enfermedades Infecciosas para revisin a las 48 horas.

Figura 1. Engrosamiento pared vesicular en TC abdominal. Grfico 1. Algoritmo en el Diagnstico diferencial del edema de pared vesicular.

25
CAPTULO 2
INFECCIONES ABDOMINALES
(INCLUIDAS GASTROENTERITIS)

15-30% de los carcinomas de vescula provocan un engro- burnetii de fase I y II negativos. Al reinterrogar de manera
samiento focal o difuso de la pared. La adenomiomato- dirigida al paciente, refiere abonar con excrementos de ca-
sis es una colecistosis hiperplsica que se caracteriza por ballo un huerto del que dispone.
engrosamiento dela pared vesicular, ya sea segmentaria Inicia tratamiento con doxiciclina 100 mg cada 12 horas
o difusa,con exageracin de los pliegues epiteliales intralu- durante 15 das y se solicita segunda serologa en la que
minales. La colangitis o colangitis esclerosante asociada se observa seroconversin (IgG frente antgeno de fase II
al VIH es un sndrome en el que existe una obstruccin positivo a ttulo 1/256).
biliar como resultado de una estenosis de causa infecciosa
en la va biliar, que generalmente se da en pacientes con Diagnstico final
un recuento celular de CD4 <100/mm3. Las causas extra-
vesiculares son variadas y comprenden desde fallo orgni- Colecistitis aguda alitisica por Coxiella burnetii.
co a procesos locales. En la insuficiencia heptica, renal o
cardiaca existe un aumento de la presin venosa en cava y Discusin
en porta as como una disminucin de la presin osmti-
ca, dando lugar a edematizacin local. Dicho aumento de La colecistitis alitisica supone el 10% de todas las causas
presin venosa tambin se da en situaciones de hipopro- de colecistitis. Se produce por una respuesta inflamatoria
teinemia o cirrosis heptica. Secundariamente a procesos en la pared vesicular secundaria a estasis e isquemia vesi-
inflamatorios perivesiculares (pancreatitis, pielonefritis) la cular, que conduce a una necrosis que, en caso de progre-
pared vesicular tambin se puede ver afectada. sar, termina produciendo una perforacin vesicular.
La imagen ecogrfica del engrosamiento de la pared no Los factores de riesgo para desarrollar una colecistitis ali-
permite distincin entre colecistitis aguda y los engrosa- tisica son varios: reanimacin cardiopulmonar, fallo car-
mientos de causa no biliar; ser el contexto clnico el que diaco, ventilacin mecnica, SIDA, nutricin parenteral
nos ayude a establecer un diagnstico. (1) total, trasplante de mdula sea, leucemia aguda, infec-
ciones, enfermedad renal crnica, inmunosupresin o es-
Evolucin clnica tenosis ampular, entre otros. Es por esto que la colecistitis
alitisica suele darse en pacientes crticos, en los cuales
Con los hallazgos radiolgicos de colecistitis alitisica se confluyen uno o ms factores de riesgo.
consulta con Ciruga General, que descart realizar in- Es en presencia de estos factores de riesgo que se producen
tervencionismo urgente. El dolor persiste, aunque la fie- infecciones secundarias por patgenos entricos (Escherichia
bre progresivamente va descendiendo llegando a picos de coli, Enterococcus faecalis, Klebsiella spp, Pseudomonas spp, Pro-
37.5C. En los siguientes das, sin embargo, va cediendo teus spp y Bacteroides sp.), pero tambin existen patgenos
el dolor por lo que el paciente solicita el alta voluntaria que de forma primaria provocan una colecistitis, entre los
tras 6 das de ingreso hospitalario. Hasta el momento, los que se encuentra Coxiella burnetii, responsable del cuadro en
resultados disponibles eran un urocultivo negativo, hemo- nuestro caso. Otros microorganismos son Ascaris lumbricoi-
gramas con leucocitosis mantenida y pruebas bioqumicas des, Brucella sp., Campylobacter jejuni, Candida spp, Flavivirus,
con PCR elevada (valores entre 50-120 mg/l, siendo nor- Hepatitis A y B, Isospora belli, Leptospira spp, Mycobacterium
mal hasta 5 mg/l) y alteracin en las transaminasas (GOT tuberculosis, Plasmodium spp, Salmonella spp (S. enterica, S. typhi),
68 U/L [0-37], GPT 81 U/L [0-40], GGT en 197 U/L Cryptosporidium spp, Citomegalovirus, E. granulosus, Vibrio
[11-49], fosfatasa alcalina 480 U/L [90-258]) con bilirru- cholerae y virus de Epstein-Barr (2).
bina total normal, por lo que al alta queda pendiente de los Por ser una patologa que habitualmente se diagnostica en
resultados, tanto de los hemocultivos y las serologas, que pacientes crticos hospitalizados, la incidencia de la colecis-
se haban solicitado al ingreso. Dado que el paciente no titis alitisica no est bien definida en los pacientes extra-
haba completado tratamiento para el cuadro sospechado hospitalarios. En una serie de 1990 el 77% de los pacientes
se indica empricamente ciprofloxacino 750 mg cada 12 de una serie de 47 pacientes registrados a lo largo de 7
horas durante 7 das y cita en Consultas Externas de En- aos desarrollaron la enfermedad en domicilio y el 72%
fermedades Infecciosas en una semana. presentaba enfermedad vascular significativa de base (3).
En dicha revisin, se dispone ya de los resultados de la Los sntomas y signos son similares a aquellos observados
serologa solicitada inicialmente, que pone de manifiesto en la colecistitis litisica: dolor en hipocondrio derecho
una infeccin aguda por Coxiella burnetii, con un ttulo de irradiado hacia costado derecho, fiebre, signo de Murphy
IgM de 1.88 U/ml (tcnica ELISA) e IgG para antgeno C. positivo, leucocitosis y alteraciones en las pruebas de fun-

26
CAPTULO 2
INFECCIONES ABDOMINALES
(INCLUIDAS GASTROENTERITIS)

cin heptica (hiperbilirrubinemia, elevacin de fosfatasa registrndose 17 en total, destacando de ellos que 7 se co-
alcalina srica y de las aminotransferasas). rresponden a una serie de Raoult publicada en 2003, 3
El diagnstico se realiza por pruebas de imagen (ecografa han sido publicados sobre soldados americanos destinados
o TC) y los hallazgos sugestivos de colecistitis alitisica son, en Iraq y Afganistn y otros 3 se han dado en territorio
en primer lugar, la ausencia de litiasis o barro biliar y el espaol, sin que en este caso se observe una distribucin
engrosamiento de la pared vesicular mayor de 3-3.5 mm geogrfica especfica (Cdiz, Huesca y Len), si bien la
(el corte en 3 mm ofrece una sensibilidad del 100% y una muestra es escasa.
especificidad del 90%, mientras que en 3.5 mm la sensibili- Se ha postulado que los anticuerpos anticardiolipina po-
dad es del 80% y la especificidad del 99%) (4). Otros datos dran tener un papel en la fisiopatologa del cuadro, ya que
indicativos son la presencia de lquido perivesicular, signo se detectan en la mayora de los pacientes con fiebre Q ,
de Murphy ecogrfico positivo, distensin vesicular (>5 aunque generalmente slo de manera transitoria y sin sig-
cm) o el signo del halo (tejido graso normal que contrasta nificacin patolgica. Sin embargo, podran provocar una
con los tejidos circundantes inflamados). vasculitis local causante de una microangiopata trombti-
El tratamiento de la colecistitis alitisica consiste en el ca en la vescula que desencadenase la isquemia vesicular
tratamiento antibitico especfico, en caso de obtener de que es el evento primario en la colecistitis (8).
aislamientos microbiolgicos, o tratamiento quirrgico Por lo tanto, recalcamos que la colecistitis alitisica es una
mediante colecistectoma o colecistostoma o bien drenaje forma de presentacin muy rara de las infecciones por C.
endoscpico; estas dos ltimas tcnicas, menos invasivas, a burnetii. El diagnstico se establece mediante pruebas de
menudo sern las elegidas cuando se trate de un paciente imagen y tests serolgicos y habitualmente tiene una evolu-
crtico con alto riesgo quirrgico. cin favorable al instaurar tratamiento antibitico espec-
En cuanto a la infeccin por Coxiella burnetii, puede ser asin- fico, siendo de vital importancia una adecuada anamnesis
tomtica hasta en la mitad de los casos, y cuando da snto- dirigida que nos haga orientar el agente etiolgico, ya que
mas puede hacerlo de manera aguda (40%) o crnica (1- aunque se trate de una zoonosis generalmente relacionada
5%). Las infecciones agudas cursan en forma de neumona con vacas, ovejas y cabras, tambin infecta a caballos, cer-
o hepatitis aguda en su mayora, aunque tambin pueden dos, conejos, perros, gatos y camellos, habiendo adquirido
cursar como miopericarditis, meningitis, pancreatitis, cole- en los ltimos aos una mayor importancia en la deteccin
cistitis, gromerulonefiritis y anemia hemoltica, entre otras. de casos equinos por su relacin con abortos.
Las infecciones crnicas fundamentalmente se manifiestan
como endocarditis en pacientes con afectacin valvular
previa, trasplantados o inmunodeprimidos (5).
El diagnstico en la prctica clnica habitual es serolgico.
Las infecciones agudas presentan antgenos de fase II (IgM
1/32; IgG 1/128) y las infecciones crnicas antgenos
de fase I (IgG 1/800). El tratamiento de las infecciones
agudas se realiza con doxiciclina a dosis de 100 mg cada 12
h durante 15-21 das mientras que las infecciones crnicas
deben recibir antibioterapia con dos agentes y de manera
prolongada (doxiciclina ms ciprofloxacino o doxiciclina
ms hidroxicloroquina durante 2-3 aos, hasta que el ttulo
IgG frente a antgeno de fase II sea menor de 1/200) (6).
Como hemos comentado, habitualmente las infecciones
agudas por C. burnetii cursan como sndromes febriles, neu-
monas y/o hepatitis, existiendo en Espaa una tendencia
a la distribucin geogrfica de estos cuadros, de manera
que en el norte de Espaa se manifiesta con ms frecuen-
cia en forma de neumona, mientras que en el sur lo hace
como hepatitis aguda (7).
Con respecto a la colecistitis alitisica como forma de pre-
sentacin de una infeccin por C. burnetii, se trata de un
hecho poco frecuente. En la literatura existen pocos casos,

27
CAPTULO 2
INFECCIONES ABDOMINALES
(INCLUIDAS GASTROENTERITIS)

Bibliografa

1. Middleton WD, Kurtz AB, Hertzberg BS. Ecografa.


1 ed. Marban. 2006.
2. Uptodate.com [Internet]. Colecistitis alitisica. [Ac-
tualizado Febrero 2015; citado Abril 2015]. Disponi-
ble en www.uptodate.com.
3. PE Savoca, WE Longo, KA Zucker, MM McMillen,
M Modlin. The increasing prevalence of Acalculous
cholecystitis in Outpatients. Results of a 7-year study.
Ann Surg 1990; 211: 4337.
4. PS Barie,SR Eachempati. Acute acalculous cholecys-
titis. Curr Gastroenterol Rep2003;5:302-9.
5. MT Fraile Farias, C Muoz Collado. Infeccin por
Coxiella burnetii (fiebre Q). Enferm Infecc Microbiol
Clin. 2010; 28(Supl 1):29-32.
6. JA Herrero, E Garca-Vzquez, A Hernndez, J. G-
mez. Infecciones por rickettsias y fiebre Q. Medicine.
2010; 10:3881-8.
7. A de Alarcn, JL Villanueva, P Viciana. Q fever:
epidemiology, clinical features and prognosis. A
study from 1983 to 1999 in the South of Spain. J In-
fect2003;47:1106.
8. D Ergas, A Abdul-Hai, Z Moshe Sthoeger. Acalculous
Cholecystitis: An Unusual Presentation of Acute Q
Fever Masquerading as Infectious Endocarditis. Am J
Med Sci2008; 336:356-7.

28
Absceso heptico por Klebsiella pneumoniae
hipervirulenta K1. Una enfermedad emergente
Alarcn Manoja, E; Miranda Sancho, E;
Moreno Maqueda, I; Canueto Quintero, J.
H. Punta Europa. Algeciras.

Caso clnico te, en el que destacaban numerosas lesiones hipodensas


confluentes, mal delimitadas que globalmente medan
Varn de 63 aos que acudi a Urgencias por un cuadro 85x80x75 mm que afectaban a gran parte del lbulo he-
de 6 das de evolucin de fiebre elevada de hasta 40C, ptico izquierdo (LHI) y a una mnima porcin adyacente
que se acompaaba de intensa disuria y polaquiuria, por lo en lbulo heptico derecho (LHD). De igual forma, se ob-
que en Atencin Primaria le haban pautado 5 das antes, servaron muy probables milimtricas litiasis en coldoco
Ciprofloxacino 500mg cada 12h sin mejora. distal con va biliar de calibre normal. La conclusin radio-
Entre sus antecedentes destacaban: dudosa alergia a la lgica fue, que en el contexto clnico del paciente, la mejor
Propifenazona, Diabetes Mellitus tipo 2 no insulindepen- posibilidad diagnstica era una colangitis aguda piognica
diente, hiperlipemia mixta y haba sido apendicectomiza- que estuviera iniciando un absceso heptico (no organiza-
do en la infancia y colecistectomizado va laparoscpica un do an) en LHI.
mes antes del ingreso. Ante dichos hallazgos, se ingres al paciente, se solicitaron
A la exploracin estaba febril (39C), consciente y orien- hemocultivos, se inici antibioterapia emprica con Pipera-
tado, eupneico en reposo, bien nutrido, hidratado, per- cilina/tazobactam 4/0,5 cada 6h y se tramitaron una eco-
fundido y hemodinmicamente estable. La auscultacin grafa con contraste y una colangio-RMN para completar
cardiopulmonar era rtmica, sin soplos, con murmullo ve- estudio de imagen.
sicular conservado, sin ruidos sobreaadidos. El abdomen A las 24 horas del ingreso se realiz la ecografa con con-
era blando, depresible, no se palpaban masas, ni megalias. traste donde se apreci una zona heterognea de unos
Ligeras molestias a la palpacin de epigastrio, sin defen- 90x80 mm de dimetro en LHI muy sugestivas de abscesos
sa ni peritonismo. Ruidos hidroareos conservados y pu- hepticos mltiples, adems de quistes septados en LHD.
opercusin renal no dolorosa bilateral. En la exploracin La Colangio-RMN se realiz a las 48 horas del ingreso y
neurolgica no se detectaron hallazgos de inters. No se fue informada como probable barro biliar en heptico co-
encontraron lesiones cutneas, ni otros datos destacables. mn, y probable absceso intraheptico en LHI, sin poder
Exploraciones complementarias: en el hemograma desta- descartar colangiocarcinoma.
caba una leucocitosis 18.600 /mm, con 15.070 granulo- Ante las discrepancias entre las diferentes tcnicas de ima-
citos y 6% de cayados, linfocitos: 1.300/mm, plaquetas: gen, se opt por la solicitud de una colangiopancreatogra-
70.000 /mm, con el resto de parmetros en rango nor- fa retrgrada endoscpica (CPRE).
mal. En la coagulacin: TP 13.3 seg, TPTA normal, fibri- Al 4 da del ingreso al persistir picos febriles diarios de
ngeno 1.161 mg/dL y en la bioqumica: glucemia 223 39-40 grados, se cambi la antibioterapia a Imipenem/
mg/dl, creatinina 1.2 mg/dl, Na 130 mEq/L, Ca 8.2 mg/ cilastacina 500mg/6h iv an de forma emprica al no ob-
ml, BT 3,7mg/dL, BD 2,6mg/dL, GOT 125 U/L, GPT tenerse crecimiento en los hemocultivos seriados.
160 U/L, amilasa, GGT, FA y LDH normales. PCR 17,9 La CPRE descart obstruccin de la va biliar (realizaron
mg/L. En el anlisis de orina se detectaron nitritos positi- esfinterotoma y limpieza de la va sin extraccin de mate-
vos y el sedimento mostraba piuria y bacteriuria intensa. rial), permitiendo llegar al diagnstico de abscesos hep-
La radiografa de trax y abdomen, as como el electrocar- ticos piognicos mltiples.
diograma, no presentaban hallazgos de inters.
Aunque el diagnstico de presuncin en Urgencias ante Diagnstico diferencial
los datos descritos previamente era de sepsis de probable
origen urinario, ante los hallazgos de laboratorio de bili- Sobre la patogenia de los abscesos hay que decir que sue-
rrubina elevada con discreta citolisis y el antecedente de len desarrollarse tras peritonitis (a travs de circulacin
colecistectoma va laparoscpica un mes antes, se solicit portal), tras infeccin biliar (va directa), mediante disemi-
previamente al ingreso una ecografa abdominal, que el nacin va hematgena por infeccin sistmica o a travs
radilogo complet con un TAC abdominal con contras- de herida quirrgica o penetrante. Conocer la patogenia

29
CAPTULO 2
INFECCIONES ABDOMINALES
(INCLUIDAS GASTROENTERITIS)

es fundamental y orienta hacia al patgeno responsable en tivos seriados negativos), se realiz una hepatectoma iz-
la mayora de los casos. quierda debido a la existencia de mltiples abscesos, per-
Con respecto a la microbiologa, suelen ser polimicrobia- sistiendo abscesos en el lbulo heptico derecho.
nos (anaerobias y facultativas entricas), aunque se han A las 24 horas de la ciruga, el paciente estaba afebril por
descrito una alta variabilidad de patgenos como respon- primera vez desde su llegada al hospital y los reactantes de
sables de esta patologa, lo que justifica la bsqueda de un fase aguda comenzaron a descender progresivamente.
diagnstico microbiolgico en casi todos los casos. En la muestra de tejido intraoperatoria se aisl una Klebsie-
Los patgenos potenciales que se incluyeron en el diagns- lla pneumoniae hipervirulenta serotipo K1 (magA+, rmpA+),
tico diferencial de los abscesos hepticos en nuestro caso con un antibiograma sensible a ambos tratamientos previa-
fueron: mente pautados (tanto a Piperacilina/tazobactam como a
Imipenem/cilastacina), responsable de una patologa co-
1. El grupo de Streptococcus milleri o S. anginosus (incluyendo nocida como Sndrome del absceso heptico invasivo por
S. constellatus y S. intermedius) es una importante causa Klebsiella pneumoniae (por lo que se mantuvo durante 4 se-
de absceso heptico. Cuando estn implicados, debe manas el tratamiento con Imipenem/cilastacina).
motivar la bsqueda de infecciones metastsicas simul-
tneas en otros lugares. Diagnstico final
2. S.aureus, S. pyogenes y otros cocos Gram positivos son
patgenos a considerar en circunstancias especficas, Sndrome del absceso heptico invasivo por Klebsiella pneu-
como por ejemplo, en pacientes que se sometieron a moniae hipervirulenta K1 magA+, rmpA+.
la embolizacin transarterial por carcinoma hepato-
celular. Discusin
3. Candida spp tambin han sido implicadas en el absceso
heptico pigeno y representan hasta el 22 por ciento Los abscesos hepticos son el tipo ms comn de abscesos
de los abscesos hepticos en series publicadas. La can- viscerales. La incidencia anual de absceso heptico se ha
didiasis hepatoesplnica puede ocurrir en pacientes estimado en 2,3 casos por 100.000 habitantes y es mayor
que han recibido quimioterapia y presentan recupe- entre los hombres que entre las mujeres (3,3 frente a 1,3
racin en recuento de neutrfilos tras un episodio de por 100.000);tasas sustancialmente ms altas han sido re-
neutropenia. portados en Taiwan (17,6 casos por cada 100.000).
4. Klebsiella pneumoniae es un importante patgeno emer- El Sndrome del absceso heptico invasivo (SAHI) cau-
gente.Sobre todo implicado en Asia y Sudfrica. sado por Klebsiella pneumoniae hipervirulenta K1 magA+,
5. Abscesos hepticos tuberculosos son poco comunes, rmpA+ es diferente a otros abscesos hepticos, debido a su
pero deben ser considerados cuando los hemocultivos agresividad y a la posibilidad de producir metstasis spti-
son negativos. cas a distancia (ms comnmente endoftalmitis, meningitis
6. Las infecciones hepticas por Listeria monocytogenes son y absceso cerebral).
excepcionales. Los pacientes afectados por esta con- Es una patologa que tiene mayor prevalencia en Asia,
dicin suelen ser inmunodeprimidos y la mayora de principalmente en Taiwan, Singapur y Corea, aunque
los casos se presentan como cuadros de meningitis o tambin se ha descrito en pacientes asiticos que viven en
bacteriemia primaria. otros pases y en Sudfrica. En EEUU, en Europa y en Ca-
7. Burkholderia pseudomallei (el agente de Melioidosis) debe nad es una patologa mucho menos prevalente.
ser considerado en pacientes procedentes de zonas en- En Espaa solo hemos encontrado en la literatura revisada
dmicas (sudeste de Asia y el norte de Australia). 4 casos del SAHI causado por Klebsiella pneumoniae hipervi-
8. Y la amebiasis se debe considerar como una causa rulenta K1, dos en el Hospital de Gran Canaria durante el
de absceso heptico primario, especialmente en los periodo 1998-2003, uno en el 12 de Octubre en 2007 y el
pacientes procedentes de reas endmicas o que han ltimo en Valencia en 2013.
viajado a un rea endmica en los ltimos seis meses. En los casos publicados una minora de pacientes desa-
rroll meningitis y/o endoftalmitis (2% o menos), siendo
Evolucin tambin este porcentaje mayor en Asia frente al resto; as
como tambin se apreci una mayor incidencia de mets-
l7 da del ingreso al continuar con picos febriles de 39-40, tasis spticas en los abscesos por Klebsiella que en los absce-
a pesar del cambio de antibioterapia emprica (hemocul- sos por otros patgenos.

30
CAPTULO 2
INFECCIONES ABDOMINALES
(INCLUIDAS GASTROENTERITIS)

La diabetes es el factor de riesgo ms frecuentemente aso- la extensin de la infeccin y la respuesta clnica del pa-
ciado a esta patologa (70-78%), aunque su mecanismo se ciente al manejo inicial y suele mantenerse un perodo de
desconoce. Otro factor a tener en cuenta, es el uso de anti- entre cuatro a seis semanas. Los pacientes que han tenido
biticos los 30 das previos (ambos factores coexistieron en una buena respuesta al drenaje inicial deben ser tratados
el caso descrito). con dos a cuatro semanas de terapia parenteral, mientras
Su hipervirulencia viene conferida por su serotipo capsu- que los pacientes con drenaje incompleto deben recibir
lar K1, su hipermucoviscosidad (magA+, rmpA+), por los cuatro a seis semanas de terapia parenteral. El resto de
lipopolisacrido de la membrana externa, los siderforos tratamiento puede entonces ser completado con antibiote-
y las fimbrias o adhesinas. Los elementos anteriormente rapia oral adaptada a los resultados del cultivo (en nuestro
citados tienen todos ellos una gran importancia en la agre- paciente se continu al alta con levofloxacino segn anti-
sividad de esta bacteria. Su patogenicidad es el resultado biograma).
de la accin conjunta de varios de estos factores, que per- Las tcnicas de drenaje incluyen los drenajes percutneos
miten a la bacteria entrar y multiplicarse en el interior del guiados por TAC o ecografa (con o sin colocacin de ca-
hospedador, resistir su sistema inmune (o simplemente no tter), el drenaje quirrgico y el drenaje mediante colan-
estimularlo) y producirle un dao. giopancreatografa retrgrada endoscpica (CPRE), en
Los sntomas y signos son similares a los que presentan pacientes con procedimientos biliares anteriores cuya in-
los pacientes con abscesos hepticos causados por otras feccin se comunica con el rbol biliar.
bacterias: fiebre (93%), hipersensibilidad en hipocondrio Para abscesos aislados con un dimetro 5 cm, se reco-
derecho (71%), nuseas, vmitos, diarreas, dolor abdomi- mienda drenaje percutneo o con aspiracin con aguja
nal (38%), leucocitosis, elevacin GOT y GPT (59-68%), fina. Si el absceso individual es > 5 cm de dimetro, se
elevacin de FA (78%) y elevacin de bilirrubina (26%). prefiere el drenaje percutneo o quirrgico.
El diagnstico es similar al del absceso heptico piognico, El drenaje quirrgico, que se realiz en el caso que descri-
basado en la historia clnica, la exploracin y la imagen bimos por presentar mltiples abscesos, est tambin indi-
radiolgica, seguida por la aspiracin y cultivo del material cado en los casos de absceso mayor de 5 cm, si stos estn
del absceso. loculados, en aquellos abscesos con contenidos viscosos
La tomografa computarizada (TC) y la ecografa son las que obstruyen el catter de drenaje, si existe enfermedad
modalidades de eleccin. La TC suele mostrar una colec- subyacente que requiere tratamiento quirrgico primario
cin fluida con edema circundante, y en ocasiones tambin o si existe respuesta inadecuada al drenaje percutneo tras
puede presentar tabiques y subcolecciones loculadas.La un plazo de siete das.
resonancia magntica es menos til en esta patologa. Para finalizar destacar del caso que describimos la escasa
El material obtenido mediante aspiracin guiada por CT frecuencia de aparicin, su gran agresividad y la posibili-
o ecografa debe ser enviado al laboratorio para cultivar dad de producir metstasis a distancia (las cuales deben
(tanto aerbico y anaerbico), as como los hemocultivos, tratarse mediante desbridamiento o tratamiento local).
ya que son esenciales (positivos en hasta el 50 por ciento de
los casos). Por el contrario, los cultivos obtenidos de drena-
jes existentes, no suelen ser suficientes para guiar la terapia
antimicrobiana ya que a menudo estn contaminados con
la flora de la piel y otros organismos.
El tratamiento debe incluir el drenaje y tratamiento antibitico.
No existe ningn ensayo clnico controlado y aleatorizado
que haya evaluado los regmenes de antibiticos empri-
cos para el tratamiento del absceso heptico pigeno.Las
recomendaciones de los diferentes tratamientos se basan
en la probable fuente de infeccin y deben guiarse por los
patrones de resistencia bacteriana locales, si se conocen.
Independientemente del rgimen emprico inicial, el rgi-
men teraputico debe ser revisado una vez que se obtenga
el cultivo y en los casos en los cuales la evolucin del pa-
ciente no se satisfactoria, como lo fue en nuestro caso.
La duracin del tratamiento se determina tpicamente por

31
CAPTULO 2
INFECCIONES ABDOMINALES
(INCLUIDAS GASTROENTERITIS)

Bibliografa

1. Siu LK1, Yeh KM, Lin JC, Fung CP, Chang FY. Kleb-
siella pneumoniae liver abscess: a new invasive syndro-
me. Lancet Infect Dis. 2012; 12:881-7.
2. Chang Z, Kheng J, Ma Y, Liu Z. Analysis of clinical
and CT characteristics of patients with Klebsiella
pneumoniae liver abscesses: an insight into risk factors
of metastatic infection. Int J Infect Dis 2015; 33:50-4.
3. Popescu GA, Tanase D, Petrescu AM, Florea D. Li-
ver abscess associated with severe myopathy caused by
Klebsiella pneumoniae serotype K1 in Romania. J In-
fect Dev Ctries. 2014; 8:1491-3.
4. Holms K, Fostervold A, Stahlhut SG, Struve C, Hol-
ter JC. Emerging K1 serotype Klebsiella pneumoniae
primary liver abscess: three cases presenting to a single
university hospital in Norway. Clin Case Rep. 2014;
2:1227.
5. Lin JC, Koh TH, Lee N, Fung CP, Chang FY, Tsai YK,
et al. Genotypes and virulence in serotype K2 Klebsie-
lla pneumoniae from liver abscess and non-infectious
carriers in Hong Kong, Singapore and Taiwan. Gut
Pathog. 2014; 6:21.
6. Gundestrup S, Struve C, Stahlhut SG, Hansen DS.
First Case of Liver Abscess in Scandinavia Due to the
International Hypervirulent Klebsiella Pneumoniae
Clone ST23. Open Microbiol J. 2014; 8:22-4.
7. Shon AS, Bajwa RP, Russo TA. Hypervirulent (hyper-
mucoviscous) Klebsiella pneumoniae. A new and dan-
gerous breed. Virulence. 2013; 4:10718.
8. Nadasy KA, Domiati-Saad R, Tribble MA. Invasive
Klebsiella pneumoniae Syndrome in North America.
Clin Infect Dis. 2007; 45:2528.
9. Fang CT1, Lai SY, Yi WC, Hsueh PR, Liu KL, Chang
SC. Klebsiella pneumoniae genotype K1: an emerging
pathogen that causes septic ocular or central nervous
system complications from pyogenic liver abscess. Clin
Infect Dis. 2007; 45: 284-93.

32
Mujer de 56 aos con colangitis recurrentes
y malformacin congnita
de la va biliar intraheptica
Navarro-Amuedo, MD; Ruz Ruz, F;
Chacn Mora, N; Luque Mrquez, R.
Hospital Universitario Virgen del Roco. Sevilla.

Caso clnico

Se presenta el caso de una mujer de 56 aos, psicloga de


profesin. Tiene una alergia documentada a pirazolonas
y es fumadora de 20 cigarrillos /da (consumo acumulado
32 paquetes /ao). Dislipemia en tratamiento con estati-
nas, niega diabetes mellitus ni hipertensin. Estudiada en
2004 en Servicio de Digestivo por historia de nuseas y
pirosis con endoscopia digestiva alta sin hallazgos patolgi-
cos. Intervenida de ligadura de trompas uterinas y prtesis
mamarias. Tratamiento habitual: omeprazol 20 mg /da,
rosuvastatina 10 mg/da y paroxetina 20 mg/da.
Consulta por primera vez en 2006 por un cuadro clni-
co y ecografa compatible con colecistitis aguda litisica,
realizndose colecistectoma laparoscpica en las prime-
ras 48 horas de ingreso. El postoperatorio transcurrir sin
incidencias clnicas salvo por la persistencia de discretas
molestias en el lecho quirrgico (LQ) que motivarn una
ecografa abdominal a las 72 horas, donde se informar de
la presencia de una coleccin hemtica a dicho nivel. Es Figura 1. Colangioresonancia.
dada de alta tras 14 das de ingreso con excelente estado
general. con esfinterotoma sin obtencin de material tras repetidos
A los 30 das refiere fiebre (hasta 39 C), nuseas, vmi- arrastres con baln de Fogarty. El TC de abdomen previo
tos y dolor en hipocondrio derecho, diagnosticndose por a alta sugiere sobreinfeccin de quiste en segmento VI he-
ecografa de coleccin abscesificada del LQ que requerir ptico como probable foco de la colangitis.
drenaje externo durante 12 das y antibioterapia intrave- En 2012, nuevo ingreso hospitalario por idntico cuadro
nosa prolongada (Imipenem 14 das) con favorable aunque clnico y aislamiento en hemocultivos de E. coli BLEE. En
lenta resolucin posterior. En el material cultivado creci bioqumica sangunea destacan: elevacin de GGT (95
E. coli productor de BLEE. UI/l) y FA (136 UI/l) siendo el resto del estudio normal.
Ingresar 4 aos ms tarde en Servicio de Digestivo por PCR: 10.3 mg/l, VSG: 49.0 mm/h. Hemograma sin ha-
fiebre con escalofros, nuseas, vmitos y de nuevo dolor en llazgos. Urocultivo negativo. Se realiza ecografa abdomi-
hipocondrio derecho y epigastrio. A la exploracin, abdo- nal que evidencia nuevamente dilatacin de coldoco e
men blando y depresible, doloroso a la palpacin profun- imagen ecognica con dudosa sombra acstica posterior,
da, sin signos de irritacin peritoneal y ruidos hidroareos decidindose nueva realizacin de CPRE sin hallazgos.
conservados. En analtica inicial destaca discreta elevacin Tras cumplimiento de antibioterapia intravenosa con Er-
de GGT (80 UI/l), FA (293 UI/l) y leucocitosis con neu- tapenem durante 14 das y resolucin del cuadro clnico, la
trofilia (12.900 leucocitos, 88% PMN). En hemocultivos paciente es dada de alta
extrados crece nuevamente E. coli productor de BLEE. Se
solicita colangioresonancia que evidencia dilatacin mo- Evolucin
derada del coldoco con posible foco de colangitis en rama
subsegmentaria de rbol izquierdo y donde se informa la Reingresar en los prximos doce meses hasta en 7 ocasio-
presencia de microhamartomas biliares mltiples hepti- nes por colangitis recurrente con distintos aislamientos mi-
cos como hallazgo incidental (Figura 1). Se realiza CPRE crobiolgicos y patrones de sensibilidad (K. pneumoniae, E.

33
CAPTULO 2
INFECCIONES ABDOMINALES
(INCLUIDAS GASTROENTERITIS)

coli multisensible, E. faecium). En Octubre de 2012 se decide tamao, redondeadas, hipo-anecoicas, con aspecto hete-
inicio de profilaxis secundaria con cido ursodeoxiclico rogneo y zonas hiperecognicas, mostrando a veces un
(300 mg cada 12 horas), quinolonas (Ciprofloxacino 500 marcado refuerzo acstico posterior. En la TC se muestran
mg/24 horas) y macrlidos (Azitromicina 250 mg/24 ho- como lesiones de baja atenuacin, que no cambian de den-
ras), pese al cual, ha presentado hasta 14 nuevos episodios sidad tras administrar contraste. La resonancia magntica
de colangitis bacterimica. Acude peridicamente a con- nuclear (RMN) y colangio-RMN muestran los hallazgos
sultas y se realiza controles radiolgicos anuales para des- ms relevantes para establecer un diagnstico no invasivo:
cartar desarrollo de colangiocarcinoma. Ha sido remitida con gadolinio se muestran lesiones hipointensas en T1 e
a consultas de trasplante heptico siendo por el momento hipertensas en T2.
desestimada para dicho tratamiento. El diagnstico diferencial de la hamartomatosis mltiple
En resumen, se trata de una paciente de 56 aos de edad biliar, debe realizarse principalmente con 2 entidades: la
sometida a una colecistectoma laparoscpica por colecis- metstasis heptica y la enfermedad de Caroli. Las lesio-
topata aguda litisica cuyo postoperatorio se complicar nes metastsicas generalmente presentarn contornos peor
con absceso de lecho quirrgico. En dicho ingreso se detec- definidos, una distribucin ms heterognea y mayor va-
ta adems una anomala congnita de la va biliar (hamar- riabilidad de tamaos lesionales. El antecedente de neo-
tomatosis mltiple biliar complejos de Von Meyenburg). plasia extraheptica nos obligar a pensar en primer lugar
Entre los aos 2006-2015 presenta bacteriemias recurren- en esta posibilidad. La enfermedad de Caroli es tambin
tes por enterobacterias de probable origen biliar (colangitis una malformacin congnita de la va biliar intraheptica,
de repeticin) siendo las CPRE repetidamente normales. cuya extensin puede ser difusa segmentaria. La colan-
gio-RMN pondr en evidencia la presencia de dilataciones
Diagnstico final saculares de los ductos intrahepticos que tpicamente co-
municarn con la va biliar. Este ltimo aspecto y el com-
Colangitis recurrentes bacterimicas en paciente con ha- portamiento clnico propio de esta entidad, nos permitir
martomatosis mltiple de la via biliar (Complejos de Von diferenciarla de la hamartomatosis mltiple biliar. Quistes
Meyenburg) hepticos simples peribiliares, microabscesos pigenos
y otras entidades menos frecuentes como la enfermedad
Discusin poliqustica hepatorrenal, la hidatidosis tumores benig-
nos del hgado (cistoadenoma, hemangiomas etc.) sern
Descrita por primera vez en 1918 por Hans Von Meyen- fcilmente descartables tras la valoracin conjunta de una
burg, es una anomala congnita benigna originada por un completa y detallada historia clnica, pruebas de imagen y
fallo del desarrollo embrionario de la placa ductal. Consis- contexto clnico del paciente.
te en la dilatacin de los conductos biliares intrahepticos Dada su naturaleza benigna y comportamiento clnico si-
embrionarios que formarn mltiples estructuras qusticas lente, no existe actualmente tratamiento especfico para
dilatadas rodeadas por estroma fibroso y epitelio cbico esta enfermedad. Sin embargo, nuestro caso ejemplifica la
normal. dificultad de manejo de un paciente con colangitis recu-
Suele ser asintomtica, constituyendo un hallazgo ca- rrente no subsidiario de tratamiento endoscpico o quirr-
sual en los estudios de diagnstico por imagen, con una gico y la importancia de realizar un correcto diagnstico
incidencia del 0.6-5.6%1. Cuando hay mltiples lesiones diferencial en pacientes con este tipo hallazgo en la va
puede haber un aumento selectivo de la gammaglutamil- biliar.
transferasa (GGT), que en nuestra paciente era manifiesto La colangitis aguda bacteriana recurrente (CABR) es una
en los controles bioqumicos y que puede acompaarse de enfermedad hepatobiliar, por lo general, consecuencia de
otras alteraciones analticas (elevacin de GOT, FA y bili- una combinacin de factores: el deterioro del flujo de la
rrubina, leucocitosis) en los casos excepcionales en los que bilis y la colonizacin bacteriana de las vas biliares. En la
la entidad se complica. La colangitis recurrente por infec- fase aguda, el tratamiento se basa en el drenaje biliar y la
cin del contenido bilioso del quiste ser la complicacin administracin de antibioterapia intravenosa. Cuando la
ms frecuente. Se han descrito casos aislados de colangio- enfermedad es recurrente y existen alteraciones anatmi-
carcinoma perifrico asociados a hamartomas, sugiriendo cas o morfolgicas subyacentes, el tratamiento de eleccin
que la transformacin neoplsica es posible aunque muy es quirrgico. Existen pacientes en los que ste no es posi-
infrecuente5. ble, surgiendo as la necesidad de buscar nuevas alternati-
Ecogrficamente suelen ser lesiones mltiples, de pequeo vas teraputicas para su manejo.

34
CAPTULO 2
INFECCIONES ABDOMINALES
(INCLUIDAS GASTROENTERITIS)

Es posible que el curso de las CABR puede estar relacio- a los centros de trasplante heptico. Debe descartarse de
nado con la formacin de biofilm en el rbol biliar, a seme- forma exhaustiva la presencia de colangiocarcinoma antes
janza de lo que ocurre en las infecciones bronquiales recu- del mismo.
rrentes de pacientes con bronquiectasias o fibrosis qustica. Por tanto, dada la escasa casustica de esta entidad y los
Las bacterias contenidas en el biofilm muestran una ma- pocos casos reportados en la literatura cientfica, conclui-
yor resistencia antibitica ya que expresan mecanismos mos que el tratamiento de la CABR de los pacientes con
de multirresistencia y escapan a la actividad del sistema Complejos de Von Meyenburg ha de ser individualizado.
inmune, a pesar de encontrarse en una fase estacionaria La antibioterapia cclica profilctica no est sustentada por
de crecimiento. Varios ensayos han valorado el efecto de la ninguna evidencia, sin embargo, podra constituir una alter-
antibioterapia de mantenimiento en la prevencin secun- nativa en aquellos casos en los que un tratamiento quirr-
daria de estos episodios. En estas circunstancias, conside- gico o intervencionista no es posible en espera de un TH.
rar la excrecin biliar de los antimicrobianos a la hora de
su eleccin, es ms importante que en la colangitis aguda, Bibliografa
ya que no hay una obstruccin significativa asociada y no
existe disminucin en su eliminacin. Aunque no dispone- 1. Redtson M, Wanles I. The hepatic von Meyenburg
mos de estudios randomizados para evaluar su efectividad, complex: prevalence and association with hepatic and
se ha propugnado la utilizacin de pautas intermitentes de renal cysts among 2843 autopsies. Mod Pathol 1996;
antibiticos como medida preventiva eficaz en estos pa- 9:2337.
cientes. Los agentes seleccionados deben ser activos frente 2. Brancatelli G, Federle M, Vilgrain V, Vullierme MP,
a los patgenos biliares habituales, poco txicos empleados Marin D, Lagalla R. Fibropolycystic Liver Disea-
a largo plazo y estar disponibles para administracin por se: CT and MR Imaging Findings. Radiographics
va oral. Entre ellos, el trimetoprim-sulfametoxazol (160 2005;25(3):659-70.
/ 800 mg VO /12 h), las fluoquinolonas (ciprofloxacino 3. Jain D, Sarode VR, Abdul-Karim FW, Homer R, Ro-
500 mg VO/12 h) y la asociacin amoxicilina-clavulnico bert ME. Evidence for the neoplastic transformation
(500 / 125 mg VO/8 h) son los que mejor cumplen estos of Von Meyenburg complexes. Am J Surg Pathol.2000
requisitos. Se han descrito distintos regmenes de adminis- A; 24(8):1131-9.
tracin y duracin de los ciclos sin claro beneficio de uno 4. Wisniewski B, Tordjman G, Tran Van Nhieu J, Bettan
sobre otro. Se recomienda realizar ciclos de 3-4 meses para L, Courillon-Mallet A. Cholangiocarcinoma develo-
posteriormente evaluar si pueden ser retirados sin que rea- ping in von Meyenburg complexes in haemochroma-
parezcan las crisis; reinstaurando la terapia en caso contra- tosis. Gastroenterol Clin Biol.2002 Oct; 26(10):922-4.
rio. El conocido efecto inmunomodulador de otras familias 5. Tang CN, Tai CK, Siu WT, Ha JP, Tsui KK, Li MK.
de antimicrobianos como los macrlidos podra tener im- Laparoscopic treatment of recurrent pyogenic cholan-
portancia en estos casos aunque no existen estudios bien gitis. J Hepatobiliary Pancreat Surg. 2005;12(3):243-8
diseados para justificar su utlizacin. 6. Brooun A, Liu S, Lewis K. A dose-response study of
En nuestro caso las opciones quirgicas son muy limitadas. antibiotic resistance in Pseudomonas aeruginosa biofil-
Dada la naturaleza difusa de enfermedad, tcnicas como ms. Antimicrob Agents Chemother. 2000;44(3):640-6
la CPRE CTPH, quistectoma incluso segmentecto- 7. Van den Hazel SJ, Speelman P, Tytgat GN, Dankert J,
ma heptica no resultaran eficaces para el tratamiento van Leeuwen DJ. Role of antibiotics in the treatment
de esta complicacin. Tan slo, la hepatectoma total y el and prevention of acute and recurrent cholangitis.
trasplante de hgado (TH) ofrecern la oportunidad de un Clin Infect Dis. 1994; 19(2):279-86.
tratamiento definitivo, si bien la experiencia es muy escasa, 8. lvarez Martn MJ, Garca Medina J. Recurrent pyo-
siendo su indicacin extrapolada de otras enfermedades genic cholangitis in a western patient. Gastroenterol
congnitas de la va biliar como la cirrosis esclerosante pri- Hepatol. 2000;23(4): 170-3
maria (CEP) enfermedad de Caroli. El deterioro de la 9. M. Prieto, G. Clemente, F. Casafont, N. Cuende, V.
funcin heptica, la elevada frecuencia la gravedad de Cuervas-Mons, J. Figueras, L. Grande, J.I. Herrero, P.
los episodios de colangitis, sern criterios a tener en cuenta Jara, A. Mas, M. de la Mata y M. Navasa. Documen-
para la eleccin de esta alternativa. Aunque el tiempo pti- to de consenso de indicaciones de trasplante heptico.
mo de realizacin del trasplante no est del todo estableci- Gastroenterol Hepatol 2003;26(6):355-75
do, la evaluacin pre-TH no debe retrasarse, recomendn-
dose que estos pacientes sean remitidos tempranamente

35
Trombosis portal y fiebre
en paciente inmunocompetente
Hidalgo Jimnez, A; Mariscal Vzquez, G;
Javier Jimnez-Ruz, F; Martnez Marcos, FJ.
Complejo Hospitalario Universitario de Huelva.

Caso clnico

Historia clnica. Anamnesis

Exponemos el caso de un varn de 34 aos, natural de


Huelva, grusta de profesin, sin antecedentes familiares
de inters, sin factores de riesgo cardiovasculares conoci-
dos ni hbitos txicos, cuyo nico antecedente mdico des-
tacable consiste en una intervencin de fractura de tibia y
peron a la edad de 24 aos.
Acude a urgencias por un cuadro consistente en fiebre de
hasta 39 C, de 2 semanas de evolucin, bien tolerada, en
tratamiento con antitrmicos, habiendo consultado en va-
rias ocasiones en atencin primaria. Refiere que dos se-
manas antes de la aparicin de la fiebre present faringitis
aguda, resuelta con AINEs. Niega tos, expectoracin, do-
lor torcico, diarrea, disuria o lesiones cutneas. Tampoco Imagen 1.
ha presentado prdida de peso, astenia, anorexia, prurito
ni sudoracin nocturna. No ha hecho viajes al extranjero. procalcitonina 0,4 ng/mL. GPT 120 U/L, GOT 83 U/L.
Tiene caballos, perros y gallinas en una finca familiar a la Sistemtico de orina normal. Se extrajeron 2 tandas de he-
que acude en contadas ocasiones. mocultivos que fueron negativos a los 5 das. Urocultivo
negativo. Radiografa de trax con silueta cardiomediast-
Exploracin Fsica nica dentro de los lmites de la normalidad y parnquimas
pulmonares sin hallazgos patolgicos.
Buen estado general, consciente, orientado en persona, En planta, se ampli la bioqumica, destacando GGT 110
tiempo y espacio, bien hidratado y perfundido, eupneico en U/L, Fostasa alcalina 180 U/L, Ig A, G y M normales,
reposo. TA 120/65 mmHg, FC 75 lpm, sat de O2 al 99% complemento normal, ANA y ANCAs negativos, coleste-
con fiO2 de 0,21. Afebril. Peso: 82 Kg. No presenta ade- rol 107 mg/dL, HDL 9 mg/dL, LDL 71 mg/dL, Triglic-
nopatas ni lesiones en la piel ni mucosas. Orofaringe nor- ridos 136 mg/dL. Se solicit serologa infecciosa para virus
mal. Auscultacin cardaca con tonos rtmicos, sin soplos de Hepatitis A, B, C, Ac heterfilos, Ac VEB, Ac CMV,
audibles. Auscultacin respiratoria con buen murmullo Ac VIH, Ac Toxoplasma, Ac Rickettsia spp, Ac Coxiella, Ac
vesicular, sin ruidos sobreaadidos. El abdomen es blando, Mycoplasma pneumoniae y Rosa de Bengala. El Mantoux fue
depresible, no doloroso, no se palpan masas ni megalias, negativo.
sin objetivar signos de irritacin peritoneal. Los miembros A la espera de los resultados de la serologa infecciosa, se
inferiores no presentan edemas ni signos de trombosis ve- realiz una ecografa de abdomen donde llama la atencin
nosa profunda, con pulsos perifricos simtricos. la presencia de material hiperecognico en la luz vascu-
lar de un segmento de la rama portal derecha, la cual no
Pruebas Complementarias muestra aumento significativo de su dimetro; la vena por-
ta principal presenta un dimetro ligeramente aumentado
En urgencias se realiz un Hemograma que mostraba las 3 (1,2 cm), demostrndose su permeabilidad con doppler co-
series normales. VSG 19 mm/hora. Coagulacin normal. lor y pulsado. Adems, ligera esplenomegalia homognea.
Bioqumica con glucosa, creatinina, urea, sodio, potasio, Los hallazgos mencionados sugieren una pileflebitis o pro-
amilasa, lipasa y bilirrubina normales. PCR 1,5 mg/dL, bable trombosis sptica portal derecha. Se solicita TC de

36
CAPTULO 2
INFECCIONES ABDOMINALES
(INCLUIDAS GASTROENTERITIS)

abdomen urgente para mejor valoracin, mostrando trom- con enoxaparina, desapareciendo la fiebre al da siguien-
bosis de la rama anterior de la porta derecha, sin identifi- te (sexto da de ingreso). Se extrajo una nueva analtica al
car causa que lo justifique, que condiciona una alteracin mes del alta, siendo los Ac Ig M e Ig G positivos. Se envi
de la perfusin del parnquima heptico; discreta hepato- una muestra a un laboratorio externo para la medicin de
megalia y discreta paniculitis mesentrica. (Imagen 1) ADN-CMV y viremia (ya que en nuestro centro no se reali-
Se le realizaron a su vez una endoscopia digestiva alta y zan de rutina), siendo positivo, con ms de 100000 copias/
una colonoscopia que fueron normales. mL. Se realiz estudio de hipercoaglabilidad (Factor V de
Leiden, mutacin G20210A del gen de la prototrombina,
Diagnstico diferencial anticoagulante lpico, Ac anticardiolipina IgG, Ac anti
beta-2 glucoprotena 1, homocistena, dficit de la prote-
El diagnstico diferencial inicial al ingreso se hizo entre na C y dficit de la protena S, resistencia a la protena C
aquellas causas que producen fiebre prolongada. Las cau- activada y dficit de antitrombina III) que result negativo.
sas principales son las infecciones, neoplasias, enfermeda- Se repiti una ecografa de control al mes, no mostrando
des del tejido conectivo y frmacos. Esta ltima se descarta alteraciones, suspendindose la anticoagulacin. A los dos
desde el principio, al no tomar nuestro paciente ninguna meses, se mostr una seroconversin frente al CMV con
clase de medicacin. Entre las infecciones, las ms frecuen- desaparicin de la Ig M.
tes son la tuberculosis y los abscesos generalmente localiza-
dos en abdomen y pelvis, osteomielitis o endocarditis suba- Diagnstico final
gudas, sin olvidar las infecciones causadas por virus de la
Hepatitis, virus de Epstein Barr o Citomegalovirus (CMV). Por tanto, el diagnstico final de nuestro paciente es de
Entre las neoplasias, el linfoma (especialmente no-Hodg- trombosis portal por infeccin aguda por citomegalovirus
kin), leucemia, carcinoma de clulas renales, carcinoma en paciente inmunocompetente.
hepatocelular o metstasis hepticas. De las enfermedades
del tejido conectivo, la enfermedad de Still del adulto, la Discusin
arteritis de clulas gigantes, poliarteritis nodosa, arteritis
de Takayasu, enfermedad de Wegener y crioglobulinemia La infeccin por CMV en pacientes inmunocompetentes
son las ms frecuentes. ocurre con mayor frecuencia en la adolescencia y juventud,
Ante el hallazgo de las pruebas de imagen, nos plantea- y es generalmente indolente. Tiene una clnica muy varia-
mos un diagnstico diferencial entre aquellas causas que da, desde un cuadro pseudogripal a formas ms graves.
provocan trombosis portal. Las causas de trombosis portal Como complicaciones, hay descritos cuadros trombticos
extraheptica incluyen las causas idiopticas, la asociacin en territorios esplnico, portal y supraheptico, que gene-
a estados de hipercoagulabilidad, principalmente deficien- ralmente se producen en el curso de infecciones graves en
cias de protenas C y S, postraumtica, manipulacin por- inmunodeprimidos, existiendo slo 18 casos descritos en
tal, pancreatitis, obstruccin o invasin tumoral, as como la bibliografa en pacientes inmunocompetentes, uno de
a infecciones bacterianas o vricas. Las principales causas ellos en Espaa. El diagnstico de infeccin por CMV se
de trombosis portal intraheptica son la cirrosis heptica realiza fundamentalmente mediante tcnicas serolgicas,
y la invasin tumoral por tumores hepticos primarios o aunque tambin determinando la viremia y el ADN vrico.
secundarios. Adems, en aquellos en los que hay alguna complicacin
trombtica est indicada la realizacin de un estudio de
Evolucin trombofilia. Los primeros casos informados en la literatura
mdica de trombosis asociada a infeccin por CMV fueron
Al diagnosticarse de pileflebitis se inici tratamiento con en pacientes con algn tipo de inmunodeficiencia, como
piperacilina-tazobactam a dosis de 4/0,5 g cada 8 horas de en el VIH o en pacientes transplantados. La mayor parte
forma emprica y enoxaparina a dosis anticoagulantes (80 de los casos en inmunocompetentes anteriormente men-
mg cada 12 horas), persistiendo la fiebre, que incluso era cionados corresponden a personas en la tercera o cuarta
ms rebelde al tratamiento oral, precisando antitrmicos dcada de la vida, siendo la forma de presentacin ms
intravenosos. Al cabo de 5 das obtuvimos los resultados frecuente dolor abdominal difuso presentando todos ellos
de la serologa infecciosa, siendo todos los solicitados ne- un aumento de las cifras de GOT y GPT. La mayora de
gativos excepto los Ac Ig M para CMV que fueron posi- los diagnsticos se realizaron mediante serologa, aunque
tivos. Se decidi suspender la antibioterapia y continuar en algunos se determinaron la viremia y el ADN frente a

37
CAPTULO 2
INFECCIONES ABDOMINALES
(INCLUIDAS GASTROENTERITIS)

CMV. De los 18 casos de trombosis portal asociada a infec- 2. Squizzato A, Ageno W, Cattaneo A, Brumana N. A
cin aguda por CMV, 7 no tenan ningn factor de riesgo Case Report and Literature Review of Portal Vein
de trombosis y en otros 6 el uso de anticonceptivos orales Thrombosis Associated with Cytomegalovirus Infec-
fue el nico factor de riesgo. La evolucin en 14 de los 18 tion in Immunocompetent Patients. Clin Infect Dis.
casos fue la resolucin completa de la trombosis entre 10 2007; 44:13-6.
das y 5 meses tras la administracin del tratamiento anti- 3. Spahr L, Cern A, Morard I, Rubbia-Brandt L, Schren-
coagulante con heparina de bajo peso molecular y/o anta- zel J. Acute partial Budd-Chiari syndrome and portal
gonistas de la vitamina k (slo uno no recibi tratamiento vein thrombosis in cytomegalovirus primary infection:
anticoagulante). De los 4 pacientes que no presentaron re- a case report. BMC Gastroenterol. 2006; 6:10.
solucin completa, uno presentaba dficit de protena C y 4. Valla DC, Condat B. Portal vein thrombosis in adults:
S, otro era heterocigoto para la mutacin G20210 del gen pathophysiology, pathogenesis and management. J
de la protrombina, otro era heterocigoto para el factor V Hepatol 2000; 32:865-71.
de Leiden y una tomaba anticonceptivos orales. En la pa- 5. Schreiner M, Barck T, Foroutan B, Baumgarten U. A
togenia, la mayor parte de los autores defienden como me- rare cause of portal vein thrombosis in a previously
canismo principal la induccin de un estado procoagulante healthy young man with acute hepatitis. J Clin Virol.
secundario a factores locales precipitantes (sobreexpresin 2011; 51:152-4.
de las molculas de adherencia y una estimulacin de la 6. Girszyn N, Leport J, Baux N, Kahn JE, Bltry O.
adherencia de plaquetas y leucocitos), existiendo un riesgo Thrombose portale au cours dune hpatite aigu de
incrementado de desarrollo de episodios tromboembli- primo-infection cytomgalovirus de limmunocom-
cos por alteracin del endotelio vascular. Otro mecanismo ptent. Rev Med Interne 2006; 27:4268.
propuesto es la activacin del factor X como causante de 7. Babyatsky MW, Keroack MD, Blake MA, Rosenberg
la formacin de trombina. Estudios in vitro han demostra- ES, Mino-Kenudson M. Case records of the Massa-
do que el CMV tiene efecto procoagulante, por la accin chusetts General Hospital. Case 35-2007. A 30-year-
directa sobre las clulas endoteliales y la inflamacin sis- old man with inflammatory bowel disease and recent
tmica, que activan la cascada de la coagulacin debido onset of fever and bloody diarrhea. N Engl J Med
a un aumento de la generacin de trombina tisular y a la 2007;357:206876.
disminucin de los mecanismos anticoagulantes y fibrino- 8. Ladd AM, Goyal R, Rosainz L, Baiocco P, DiFabri-
lticos. En los casos graves de pacientes inmunodeprimidos zio L. Pulmonary embolism and portal vein throm-
estara indicado el tratamiento con ganciclovir, aunque no bosis in an immunocompetent adolescent with acute
debe usarse ordinariamente en los pacientes con inmuni- cytomegalovirus hepatitis. J Thromb Thrombolys
dad preservada. Se recomienda iniciar tratamiento anti- 2009;28:4969.
coagulante en cuanto se diagnostique de trombosis portal, 9. Massoure MP, Ezanno AC, Millot I, Fixot K, Rey P,
aunque no hay recomendaciones acerca de la duracin del Sockeel P. Thrombose aigu de la veine porte asso-
mismo. ciee une primo-infection cytomgalovirus chez un
adulte immunocomptent et complique dun syndro-
Qu aporta el caso? me du compartiment abdominal. Gastroenterol Clin
Biol 2010;34:e35.
Se trata del segundo caso documentado en Espaa de
trombosis portal asociada a primoinfeccin por CMV en
paciente inmunocompetente y el decimonoveno en el mun-
do hasta la fecha. Ante la presencia de trombosis portal sin
causa que lo justifique, hay que tener en cuenta la infec-
cin por CMV, adems de realizar estudio de trombofilia.

Bibliografa

1. Aomar Milln IF, Prez Fernndez L, Parejo Snchez


MI, Hernndez Quero J. Hepatitis aguda por citome-
galovirus como causa de trombosis portal y mesentri-
ca. Med Clin (Barc) 2010; 135:3389.

38
CAPTULO 3
INFECCIONES URINARIAS
Y GINECOLGICAS
Sndrome febril persistente a pesar de antibioterapia
dirigida en paciente con infeccin del tracto urinario
Vzquez Mrquez, M; Carabantes Rueda, JJ.
Plata Ciezar, AJ.
Hospital Regional Universitario de Mlaga.
Hospital Comarcal de Antequera

Caso clnico izquierdo, esteatosis heptica sin LOES, riones sin


alteraciones ecogrficas y vejiga con paredes engrosadas
Varn de 59 aos sin alergias medicamentosas conocidas y por hipertrofia prosttica.
sin hbitos txicos con antecedentes personales de dislipe- Dos das ms tarde el paciente persista con fiebre de hasta
mia, EPOC con grado funcional II/IV (GOLD) y cardio- 39C, sntomas miccionales y dolor suprapbico a pesar
pata isqumica crnica tipo angor estable. Realiza trata- de tratamiento.
miento domiciliario habitual con cido acetil-saliclico 100
mg/24 horas, atorvastatina 40 mg/24 horas, diltiazem 120 Diagnstico diferencial
mg/12 horas y losartan 50 mg/24 horas.
Es ingresado de forma programada en el Servicio de Car- Ante la persistencia de fiebre tras 48-72 horas de antibio-
diologa para realizacin de un cateterismo cardaco. Tras terapia dirigida, nos planteamos los siguientes diagnsticos
el mismo, y durante su estancia en UCI, el procedimiento diferenciales:
se complica con infarto agudo de miocardio en territorio
inferior, fibrilacin auricular lenta e hipotensin severa, 1. Infeccin por organismo resistente.
requiriendo nuevo cateterismo y soporte hemodinmico 2. Complicaciones locales:
(no requiri ventilacin mecnica, aunque s sondaje vesi- Nefritis focal aguda: Infiltrado inflamatorio renal
cal y catter venoso central). A las 72 horas es trasladado (flemn renal).
al Servicio de Cardiologa dada su buena evolucin. Absceso renal cortical (hematgeno) o crtico-me-
Al da siguiente el paciente comienza con sntomas miccio- dular (uropatgenos).
nales, dolor suprapbico y fiebre de hasta 39C con inten- Pielonefritis enfisematosa.
sa tiritona. En la exploracin no existan datos de inters, Necrosis papilar.
salvo dolor a la palpacin en hipogastrio (la sonda vesical Absceso prosttico/prostatitis.
haba sido retirada a su llegada al servicio). Valorado por Obstruccin de la va urinaria.
su cardilogo de planta y, a la vista de un sedimento de 3. Otras: Fiebre medicamentosa, flebitis, otros procesos
orina patolgico (incontables leucocitos en sedimento), co- infecciosos aadidos
mienza tratamiento antibitico con amoxicilina-clavulni-
co 875/125 mg cada 8 horas por va oral. A las 48 horas se Evolucin
recibe un urocultivo positivo a Morganella morganii resistente
a amoxicilina-clavulnico y sensible a cefalosporinas de 3 Valorado de nuevo, se decide extraer nuevos hemocultivos,
generacin. En este momento el paciente persista con la se realiza TC abdomino-plvico y se modifica de nuevo el
misma sintomatologa, por lo que se modific tratamiento tratamiento antibitico a meropenem 1 g/8 h IV y genta-
a cefotaxima 1 gr/8 horas IV. micina 240 mg/24 horas IV.
Se realiz analtica sangunea en la que presentaba Hb En el nuevo hemocultivo se asla Morganella morganii fenoti-
12.2 g/dL, plaquetas 206000 mm3, leucocitos 12200 mm3 po AmpC (resistente a ampicilina, amoxicilina/cido cla-
(86% neutrfilos), glucosa 120 mg/dl, urea 41 mg/dl, vulanico y cefotaxima). Y en el TAC de abdomen (Figuras
creatinina 1,0 mg/dl, Na 138 mEq/l, K 4.3 mEq/l, Ca 7.9 1 y 2) se objetivan varios abscesos prostticos.
mg/dl, fsforo 2 mg/dl, bilirrubina total 0.4 mg/dl, GOT Ante este hallazgo y, dado que el germen es portador de
30 U/L, GPT 55 U/L, F. alcalina 127 U/L, GGT 126 AmpC y ha expresado una resistencia a cefalosporinas
U/L, proteinas totales 5.1 g/dl, albumina 2.2 g/dl y PCR de 3 generacin por exposicin a stas, se prosigui con
72.7 mg/L. En un nuevo sedimento persista una modera- meropenem aunque se aument la dosis a 2 gr/8 horas
da leucocituria. La radiografa de trax puso de manifies- y en perfusin extendida, por la presencia de los abscesos
to un pequeo derrame pleural izquierdo. Una ecografa prostticos (no se pautaron quinolonas por presentar re-
abdominal fue informada como pequeo derrame pleural sistencia tambin a este grupo de antibiticos). Valorado

40
CAPTULO 3
INFECCIONES URINARIAS
Y GINECOLGICAS

por Urologia se descart ciruga en este momento optando


por tratamiento conservador. Tras completar 2 semanas de
tratamiento con meropenem el paciente fue dado de alta
y complet dos semanas ms de tratamiento en domicilio
con rrtapenem 1 gr/24 horas IM y trimetropim/sulfame-
toxazol 800/160 mg via oral evolucionando sin complica-
ciones en revisiones posteriores.

Diagnstico final

Infeccin del tracto urinario bajo complicada con abscesos


prostticos por Morganella morganii fenotipo AmpC.

Discusin

1. Infeccin por Morganella morganii Figura 1. TC de abdomen con contraste IV. Abscesos prostticos (flechas).

Morganella morganii es un bacilo Gram negativo que perte-


nece a la familia Proteae, junto a Proteus y Providencia (den-
tro de la familia de enterobacteriaceae). Se encuentra en
el medio ambiente (tierra, agua y alcantarillado) y en el
tracto digestivo humano y de reptiles, pero, a pesar de su
amplia distribucin, es una causa poco comn de infeccin
comunitaria, siendo la mayor parte de los aislamientos no-
socomiales. Generalmente responde bien a antibioterapia
adecuada, pero su resistencia intrnseca a muchos antibi-
ticos beta-lactmicos puede retrasar el inicio de un trata-
miento eficaz.
Respecto a su etiopatogenia, es la infeccin del tracto uri-
nario la causa ms frecuente de infeccin por Morganella spp
y, al igual que Proteus spp, es capaz de generar urolitiasis y
es frecuente su presencia en portadores de catteres. Tras
sta, la infeccin de la herida quirrgica y la afectacin Figura 2. TC de abdomen con contraste IV. Abscesos prostticos (flecha).
de la va biliar son los focos ms frecuentes. Con menor
frecuencia tambin se han descrito infecciones en SNC, que muchas cepas (especialmente las hospitalarias) poseen
osteomuscular, neumona, endoftalmitis, pericarditis -lactamasas AmpC cromosmicas inducibles. Tambin
Como factores de riesgo para la infeccin por Morganella hay descritos mutantes espontneos con BLEA pero son
se encuentra la exposicin previa a ampicilina y otros be- infrecuentes.
ta-lactmicos, diabetes mellitus, edad avanzada, procedi- Este caso nos ensea que ante una Morganella spp, Entero-
miento quirrgico previo, absceso o infeccin secundaria a bacter spp, Providencia spp, Citrobacter spp o Serratia spp, grme-
mordedura de serpiente. nes que pueden ser portadores de genes inducibles AmpC,
En cuanto a su respuesta a los antimicrobianos, Morganella debemos evitar el uso de cefalosporinas de 3 generacin
spp es resistente de forma intrnseca a penicilina, ampicili- pues podran generar aparicin de resistencias a las mis-
na, ampicilina/sulbactam, oxacilina, cefalosporinas de 1 y mas como sucedi en este paciente.
2 generacin, eritromicina y colistina. Generalmente son
sensibles a piperacilina/tazobactam, 3 y 4 generacin 2. Absceso prosttico
de cefalosporinas, carbapenemas, aztreonam, fluorquino-
lonas y aminoglucsidos. Actualmente el amplio uso de ce- En cuanto al absceso prosttico, constituye una patologa
falosporinas de 3 generacin se ha asociado con la apari- poco frecuente, con una incidencia en torno al 0,5 % del
cin de resistencias de Morganella spp a estos antibiticos, ya total de las enfermedades prostticas. Su incidencia ha

41
CAPTULO 3
INFECCIONES URINARIAS
Y GINECOLGICAS

disminuido con la introduccin de los antimicrobianos, 3. George J. AmpC- Lactamases. Clin Microbiol Rev
especialmente con la aparicin de antibiticos con buena 2009; 22: 161-82.
penetracin prosttica como las fluorquinolonas y cotri- 4. Ing-Kit L, Jien- Wei L. Clinical characteristics and risk
moxazol. factors for mortality in Morganella morganii bactere-
Es una patologa de difcil diagnstico clnico, debido a la mia. J Microbiol Immunol Infect 2006; 39: 328-34.
similitud de sntomas con otras patologas del tracto urina- 5. Rodrguez-Crexems M, Alcal L, Muoz P, Cerce-
rio bajo. Por ello su diagnstico definitivo requiere, gene- nado E, Vicente T, Bouza E. Bloodstream Infections.
ralmente, estudios de imagen (ecografa plvica o transrec- Evolution and Trends in the Microbiology Workload,
tal y/o TC). Incidence and Etiology, 1985-2006. Medicine 2008;
Fisiopatolgicamente, la formacin de abscesos prostti- 87: 234-49.
cos se debe bien a reflujo de orina contaminada hacia la
prstata (siendo los organismos ms frecuentes asociados a
este mecanismo las enterobacterias) o bien a diseminacin
hematgena desde un foco primario (siendo los organis-
mos ms frecuentes en este caso S. aureus, Candida spp, M.
tuberculosis y Brucella melitensis).
Como factores de riesgo para la formacin de abscesos
prostticos se han descrito la manipulacin de la va urina-
ria (biopsia, sondaje), la uropata obstructiva baja, pros-
tatitis aguda y crnica, diabetes mellitus y cirrosis heptica,
terapias inmunosupresoras e infeccin por VIH.
Como se ha comentado anteriormente la clnica es simi-
lar a otras muchas patologas urolgicas y principalmen-
te consiste en: disuria, urgencia miccional, poliaquiuria,
disminucin del calibre del chorro miccional, retencin
aguda de orina, hematuria, secrecin uretral, dolor hipo-
gstrico y fiebre.
Respecto al tratamiento se considera la posibilidad de tra-
tamiento conservador con antibioterapia cuando las colec-
ciones son inferiores a 1-1,5 cc de volumen. En caso con-
trario se ha de optar, como primera opcin, por drenaje
transrectal o transperineal eco-dirigido (ya que se trata de
un procedimiento relativamente sencillo con anestesia lo-
cal y con una baja morbilidad), dejando como segundas
opciones la incisin y reseccin transuretrales o la incisin
perineal.

Bibliografa

1. Bosquet Sanz M, Gimeno Argente V, Palmero Mar-


tn JL, Bonillo Garca MA, Salom Fuster JV, Jimnez
Cruz JF. Absceso prosttico: revisin de la literatura
y presentacin de un caso. Actas Urol Esp 2005; 29:
100-4.
2. Sang-Ho C, Jung Eun L, Su Jin P, Seong-Ho C,
Sang-Oh L, Jin-Yong J, et al. Emergence of Antibio-
tic Resistance during Therapy for Infections Caused
by Enterobacteriaceae Producing AmpC -Lactamase:
Implications for Antibiotic Use. Antimicrob Agents
Chemother 2008; 52: 9951000.

42
Mujer de 35 aos con disnea y fiebre puerperal
Rodrguez-Torres, P; Chacn Mora, N;
Fernndez Perea, Y; lvarez Marn, R.
Hospital Universitario Virgen del Roco. Sevilla.

Caso clnico

Mujer de 34 aos gestante a trmino, con diabetes gestacio-


nal bien controlada con dieta, que ingresa por trabajo de
parto. Se realiza parto instrumentado mediante esptulas
que precisa realizacin de episiotoma, con evolucin ini-
cial favorable, dada de alta a los 3 das. Cinco das despus
comienza con fiebre de hasta 39C y dolor en el lugar de la
episiotoma, motivo por el que acude al Servicio de Urgen-
cias. A su llegada se observa que la herida de la episiotoma
tiene los bordes entreabiertos y eritematosos, con exudado
purulento escaso. Se comprueba adems la existencia de
loquios malolientes y desgarro en cara lateral izquierda
de vagina, sin sangrado activo. Se toman hemocultivos y
muestra de exudado endometrial antes de ingresar a la pa-
ciente en planta de Obstetricia, donde inicia tratamiento
segn protocolo con ceftriaxona 2 g al da, doxiciclina 100
mg cada 12 horas y metronidazol 500 mg cada 8 horas
por sospecha de endometritis. La fiebre desaparece, pero
contina con postracin y, en los das siguientes, comienza
de manera progresiva con sensacin disneica y dolor en
regin precordial y hemitrax derecho de caractersticas
pleurticas, que empeoran significativamente en decbito, Imagen 1. Rx de trax PA. Mltiples ndulos diseminados por ambos campos
pulmonares, con obliteracin de seno costofrnico derecho.
y reaparicin de fiebre intermitente sin tiritona. No pre-
sentaba tos, expectoracin, dolor abdominal o plvico ni
otra sintomatologa. tes de fase aguda (RFA), con una protena C reactiva de
En aquel momento, a la exploracin se encontraba afebril, 280 mg/l, leucocitosis que lleg a alcanzar 36.600 clulas/
bien perfundida, con una TA 100/60 mmHg, frecuencia mm3 con un 90% de neutrfilos, hemoglobina 107 g/l con
cardiaca de 100 lpm. Presentaba ligera taquipnea mien- VCM normal, coagulacin normal y fibringeno de 7,2
tras estaba sentada, con la que mantena una saturacin g/l. En la ecografa transvaginal se vea un tero bien in-
capilar de O2 del 94%, y ortopnea muy significativa con volucionado con imgenes en su interior compatibles con
descenso de la misma al 88% a 0. No tena ingurgitacin cogulos y un endometrio fino, sin colecciones extraute-
yugular. La auscultacin cardiaca mostraba tonos rtmicos, rinas.
sin soplos ni roce pericrdico, y la pulmonar una hipoven- En aquel momento recibimos el resultado del cultivo de
tilacin generalizada. El abdomen presentaba nicamente exudado endometrial que se tom en su primera visita a
molestias a la palpacin profunda, con tero postgrvido Urgencias segn protocolo con crecimiento de Staphylococ-
bien involucionado, sin signos de peritonismo. Los geni- cus aureus resistente a meticilina (SARM).
tales externos mostraban una cicatriz de episiotoma sin
datos inflamatorios indolora a la palpacin y loquios blan- Diagnstico diferencial
quecinos no malolientes. En los miembros inferiores tena
edemas leves, sin signos de trombosis venosa profunda. Nuestra paciente presentaba una fiebre puerperal seguida
Pulsos conservados a todos los niveles. de insuficiencia respiratoria de instauracin subaguda, con
Analticamente destacaba un gran ascenso de los reactan- aislamiento de SARM en exudado endometrial.

43
CAPTULO 3
INFECCIONES URINARIAS
Y GINECOLGICAS

La presentacin inicial de fiebre en el puerperio precoz,


con un aislamiento microbiolgico en endometrio, permi-
te diagnosticar una endometritis post-parto por SARM. La
reaparicin de la fiebre y los datos de sepsis, as como la
elevacin de RFA, hacen sospechar la presencia de una
complicacin supurada. Entre las complicaciones loca-
les ms frecuentes se encuentran el absceso plvico y la
tromboflebitis de la vena ovrica.
Con respecto a la insuficiencia respiratoria hipoxmica
planteamos un amplio diagnstico diferencial: neumona Imagen 2. Corte apical de TC de trax con ventana de parnquima en el que se
aprecian lesiones nodulares mltiples, una de ellas cavitada.
nosocomial, tromboembolismo pulmonar con origen en
trombosis venosa profunda (ovrica, plvica o de miem-
bros inferiores), distrs respiratorio en el contexto de la
sepsis o edema alveolar de origen cardiognico, como de-
but de insuficiencia cardiaca congestiva, ante la presencia
de ortopnea. No obstante, dado el aislamiento microbio-
lgico, se plantea una infeccin por SARM con disemina-
cin hematgena con, al menos, endometritis y neumo-
na, especialmente si existiera tromboflebitis sptica como
complicacin local.

Evolucin

Tras recibir el aislamiento microbiolgico, se inici trata-


miento con vancomicina en perfusin continua. Ante las Imagen 3. TC de abdomen: trombosis en vena ilaca comn e interna izquierdas.
sospechas clnicas, se solicitaron nuevos hemocultivos y
una radiografa de trax (Imagen 1) que mostraba mlti-
ples ndulos diseminados por ambos campos pulmonares,
con obliteracin de seno costofrnico derecho. Se ampli
el estudio con TC de trax y abdomen. En trax (Imagen
2) se aprecian mltiples ndulos pulmonares, distribuidos
de forma bilateral, principalmente perifrica y subpleural,
con una consolidacin triangular de base pleural en el l-
bulo inferior derecho, algunos ndulos cavitados y un pe-
queo derrame pleural bilateral. En abdomen (Imagen 3)
se observa la existencia de trombosis en vena ilaca comn
e interna izquierdas. Estos hallazgos confirman el diagns-
tico de sospecha de tromboflebitis sptica de venas ilacas
con existencia de embolismos spticos pulmonares a dis-
tancia. En los hemocultivos se aisl en un solo frasco S. au-
reus resistente a meticilina y sensible al resto de antibiticos
testados. En determinacin posterior, se demostr que era
una cepa productora de leucocidina de Panton-Valentine
(PVL).
Realizamos adems una ecocardiografa transtorcica y Imagen 4. Ecocardiografa transesofgica con reconstruccin 3D. Visin de AD.
1: Endocarditis derecha con vegetacin en red de Chiari. CVC: catter venoso
transesofgica (Imagen 4) que mostr la existencia de una central entrando por la vena cava superior.
endocarditis derecha con vegetacin en red de Chiari, re-
manente de la vlvula seno coronario en la unin de la Con los diagnsticos reseados se traslad a la planta de
vena cava inferior con la aurcula derecha, con volmenes Enfermedades Infecciosas, donde recibi tratamiento con
y funciones valvulares y miocrdica normales. vancomicina en perfusin continua durante 28 das y clin-

44
CAPTULO 3
INFECCIONES URINARIAS
Y GINECOLGICAS

damicina iv durante 2 semanas. Se instaur adems anti- antibiticos, es que SARM-Co suele ser susceptible a nu-
coagulacin con dosis teraputicas de HBPM durante 6 merosos agentes antimicrobianos como clindamicina, qui-
semanas. La hospitalizacin dur 2 semanas en total, el nolonas y cotrimoxazol, y con frecuencia es resistente solo
resto del tratamiento se administr con el programa de an- a antibiticos betalactmicos. Hay que tener en cuenta que
tibioterapia domiciliaria DOMUS. el uso de betalactmicos puede incluso aumentar la secre-
La evolucin fue favorable quedando afebril a las 48 h de cin de PVL. La clindamicina, por su parte, inhibe su ex-
antibioterapia dirigida, con desaparicin de la insuficien- presin. Por tanto, un tratamiento antibitico correcto de
cia respiratoria y hemocultivos de control negativos a las una cepa PVL+ debe asociar un frmaco bactericida para
72 h. No ha presentado nuevos sntomas tras la finaliza- S. aureus en funcin de su resistencia y un antibitico que
cin del tratamiento. bloquee la produccin de PVL. En nuestro caso, la pacien-
te recibi doxiciclina como tratamiento emprico, quedan-
Diagnstico final do inicialmente apirtica. Aunque no es un tratamiento
apropiado para una infeccin invasiva por S. aureus, cabe
Endometritis postparto e infeccin de episiotoma por S. suponer que contribuy a evitar la progresin a una sepsis
aureus resistente a meticilina comunitario con tromboflebi- grave. Posteriormente, la combinacin de vancomicina a
tis sptica de vena ilaca comn e interna izquierdas, en- dosis elevadas y clindamicina logr la curacin clnica y
docarditis derecha sobre una red de Chiari y embolismos microbiolgica.
pulmonares spticos. No se conoce la prevalencia exacta de poblacin espaola
portadora de SARM-Co y an menos se sabe acerca de la
Discusin colonizacin vaginal por el mismo. No obstante, aunque
hasta ahora la incidencia de infecciones documentadas por
El inters de nuestro caso clnico radica en la confluencia este microorganismo en nuestro medio ha sido muy baja y
de varios factores que son bastante infrecuentes, como el ha afectado principalmente a inmigrantes procedentes de
desarrollo de una infeccin endovascular por SARM feno- Sudamrica, diversos autores alertan de que su presencia
tipo comunitario (SARM-Co) con foco inicial en endome- como etiologa de infecciones de piel y partes blandas po-
tritis postparto vs. infeccin de episiorrafia o la existencia dra estar en ascenso, y la escasa informacin procedente
de una endocarditis derecha sobre una red de Chiari. de estudios de portadores sita la prevalencia de coloniza-
Los seres humanos somos un reservorio natural para S. au- cin por SARM-Co en torno al 37,4% en Europa. Por este
reus, siendo flora colonizante habitual de nasofaringe, peri- motivo, es posible que en los prximos aos asistamos al
neo, vagina o piel. La colonizacin asintomtica es mucho aumento de infecciones provocadas por SARM-Co, apro-
ms comn que la infeccin. Su paso a la sangre puede ximndonos a las cifras del continente americano.
producirse desde las zonas colonizadas cuando la integri- La endometritis post-parto es la causa ms frecuente de
dad de la piel se ve comprometida o la mucosa inflamada. fiebre puerperal. Se produce en el 1 a 6% de los partos va-
A diferencia de SARM nosocomial, que se relaciona fun- ginales, aumentando el riesgo relativo en 20-30% en ces-
damentalmente con la presin antibitica, el uso de dispo- reas. Los principales factores de riesgo son la va del parto,
sitivos y los procedimientos invasivos, las infecciones por duracin del mismo, horas de bolsa rota, alumbramiento
SARM-Co se suelen producir en personas sin contacto con manual, prematuridad, as como factores maternos como
el medio sanitario, y sus caractersticas fenotpicas y geno- inmunosupresin, anemia, diabetes mellitus o positividad
tpicas son muy distintas de las de las cepas hospitalarias. para Streptococcus agalactiae (grupo B). La etiologa habitual
Una de las principales diferencias radica en los factores de es flora vaginal (enterobacterias y anaerobios), seguida
virulencia. S. aureus posee una amplia variedad de ellos, en- por microorganismos de transmisin sexual (gonococo o
tre los que cabe destacar la secrecin de PVL que produce Chlamydia trachomatis). Por ese motivo, las guas recomien-
la mayora de las cepas de SARM-Co, mientras que en las dan un tratamiento emprico con actividad frente a estos
cepas hospitalarias o de S. aureus sensible a meticilina es microorganismos, que en el caso de nuestras guas loca-
excepcional. Esta leucocidina es una citotoxina que causa les es ceftriaxona, metronidazol y doxiciclina. Suele tener
lisis leucocitaria y necrosis de tejidos, asocindose especial- un curso clnico benigno con antibioterapia. Las compli-
mente a infecciones de piel y partes blandas, incluidas las caciones ms habituales son locales (infeccin de herida
mastitis, y a neumona hemorrgica grave en nios y adul- quirrgica, absceso plvico, tromboflebitis plvica) y las
tos jvenes. Otra diferencia relevante respecto a las cepas complicaciones a distancia son muy infrecuentes. Aunque
hospitalarias, que normalmente son resistentes a mltiples existe escasa la bibliografa al respecto, la presencia de mi-

45
CAPTULO 3
INFECCIONES URINARIAS
Y GINECOLGICAS

croorganismos multirresistentes es poco comn al tratarse imagen descarta el diagnstico. En la tromboflebitis spti-
mayoritariamente de poblacin sana con mnima relacin ca plvica profunda, la defervescencia no se produce habi-
con el medio hospitalario. S. aureus es una etiologa rara de tualmente hasta las 48 h de iniciar tratamiento anticoagu-
endometritis y solo encontramos en la literatura algunos lante con heparina, que es recomendado por la mayora
casos anecdticos de infecciones diseminadas por SARM- de los expertos, y que por lo general puede interrumpirse
Co con origen ginecolgico (endometritis postparto o aso- poco despus de la resolucin de la fiebre.
ciadas a dispositivo intrauterino, infeccin de episiotoma, En esta paciente, la manifestacin ms grave fue la neu-
aborto sptico). mona embolgena bilateral, que condicion insuficiencia
Aunque probablemente motivado por un sesgo de publica- respiratoria. La mayora de las neumonas embolgenas
cin, todos los casos descritos de endometritis por SARM- por S. aureus provienen de endocarditis derechas, frecuen-
Co presentan tambin infecciones diseminadas y graves, temente en pacientes adictos a drogas por va parenteral,
con neumona necrotizante embolgena e infeccin endo- sin embargo, las producidas por SARM-Co suelen darse
vascular (endocarditis derecha y/o tromboflebitis plvica), en pacientes jvenes sin comorbilidades, con una infec-
y evolucin infausta en muchos casos. Encontramos en la li- cin de piel y partes blandas como puerta de entrada, y
teratura dos casos clnicos similares al nuestro descritos por es frecuente que cursen sin endocarditis. El curso de la en-
Rotas y Maurell respectivamente. En los dos se describen fermedad suele ser grave afectando a varios lbulos en la
embolismos spticos pulmonares con origen en trombofle- gran mayora de las ocasiones, tendiendo a la necrosis y
bitis plvica profunda por SARM-Co con PVL+. La dife- cavitacin en ms de la mitad de los casos. Pueden presen-
rencia entre ambos es que en el primero la bacteriemia fue tar empiema o embolismos spticos en otras localizaciones
consecuencia de una episiotoma infectada y en el segundo (artritis, endoftalmitis). El comportamiento agresivo que
de una endometritis en paciente portadora de DIU. muestra SARM-Co a nivel pulmonar genera una mortali-
En nuestro caso, adems de la infeccin leve de la episio- dad global en torno al 25%, producindose la mayor parte
rrafia, la paciente presentaba una endometritis clnica. La de los fallecimientos de forma precoz (en las primeras 48
etiologa de esta se puso de manifiesto en los primeros das h) y estando relacionado en la mayora de los casos con la
gracias al cultivo precoz de aspirado endometrial realizado eleccin de una antibioterapia emprica ineficaz o incom-
por protocolo en la primera visita a Urgencias del servicio pleta, es decir, sin aadir un inhibidor de la PVL.
de Ginecologa. La importancia de dicho aspirado fue cru- La endocarditis derecha est especialmente descrita en
cial en el inicio del tratamiento antibitico dirigido y en usuarios de drogas por va parenteral, aunque puede
la estabilizacin clnica de un cuadro potencialmente tan producirse tambin por una bacteriemia con origen en
grave, ya que los hemocultivos iniciales fueron negativos y tromboflebitis sptica o cualquier foco supurado. La vl-
los siguientes tardaron 48 horas ms en crecer en un solo vula tricspide suele ser la principal afectada en este tipo
frasco, a pesar de la existencia de infeccin endovascular. de endocarditis, ya sea sola o en conjuncin con vlvula
Nuestra paciente present tres complicaciones derivadas pulmonar. La red de Chiari, que es un remanente de la
del foco inicial: una tromboflebitis plvica, una endocardi- vlvula del seno coronario que queda en la unin de la
tis derecha y una neumona embolgena necrotizante bila- vena cava inferior con la aurcula derecha, aparece en el
teral. La tromboflebitis sptica plvica, cuando se expresa 2-3% de la poblacin. Esta red se presenta como una es-
como una tromboflebitis sptica profunda, suele comenzar tructura filamentosa, fenestrada, de movimiento irregular
con fiebre a los pocos das del parto o de la intervencin tipo ltigo que generalmente no tiene importancia clnica,
ginecolgica, con estado general conservado, y la explo- pero que en situaciones de bacteriemia puede ser asiento
racin abdominal suele ser anodina. Debemos sospechar de microorganismos, como ocurri en nuestro caso.
su presencia cuando una fiebre puerperal persiste a pesar En resumen, presentamos el caso de una infeccin disemi-
antibioterapia dirigida o de amplio espectro, tras haber nada y grave por un microorganismo infrecuente y difcil
descartado la existencia de un foco supurado, y aunque no de tratar con una puerta de entrada muy poco habitual en
exista evidencia radiolgica de trombosis, ya que las prue- el que la toma precoz de muestras para cultivo y el traba-
bas de imagen convencionales (TC o RM) tienen una baja jo coordinado de obstetras, infectlogos, microbilogos y
sensibilidad para su deteccin. Se trata, por tanto, en mu- especialistas en imagen contribuy a la resolucin del ad
chas ocasiones de un diagnstico de exclusin. Esto no su- integrum del cuadro.
cede en la trombosis asptica o la tromboflebitis de la vena
ovrica, en cuyo caso la exploracin abdominal suele ser
muy dolorosa y la ausencia del trombo en las pruebas de

46
CAPTULO 3
INFECCIONES URINARIAS
Y GINECOLGICAS

Bibliografa

1. Rotas M, McCalla S, Liu C, Minkoff H. Methicillin-re-


sistant Staphylococcus aureus necrotizing pneumonia ari-
sing from an infected episiotomy site. Obstet Gynecol.
2007; 109: 5336.
2. Maurel V,Mons S, Quenee V, Levy M,Conil JM. En-
dometritis secondary to Staphylococcus aureus producing
Panton-Valentine leukocidin. Ann Fr Anesth Rea-
nim.2012; 31: 927-8.
3. Asnis D,Haralambou G,Tawiah P. Methicillin-resis-
tant Staphylococcus aureusnecrotizingpneumoniaarising
from infectedepisiotomysite. Obstet Gynecol.2007;
110: 188.
4. Mostafavifar AM,Guilfoose J,Sarwari AR. Septic pel-
vic thrombophlebitis due toStaphylococcus aureus. W V
Med J.2009; 105: 20-2.
5. Aslam AF,Aslam AK,Thakur AC,Vasavada
BC,Khan IA. Staphylococcus aureus infective endocardi-
tis and septic pulmonary embolism after septic abor-
tion. Int J Cardiol.2005; 105: 233-5.
6. Josey WE,Staggers SR. Heparin therapy in septic pel-
vic thrombophlebitis: a study of 46 cases. Am J Obstet
Gynecol.1974; 120: 228-33.
7. Garcia J, Aboujaoude R,Apuzzio J,Alvarez JR. Septic
pelvic thrombophlebitis: diagnosis and management.
Infect Dis Obstet Gynecol; 2006: 15614.
8. Mousavi N,Bhagirath K,Ariyarajah V,Fang T,Ah-
madie R,Lytwyn M et al. Chiari network endocar-
ditis: not just an innocent bystander. Echocardiogra-
phy.2008; 25: 642-5.
9. Obed M,Garca-Vidal C,Pessacq P,Mykietiuk
A,Viasus D,Cazzola L, et al. Clinical features and
outcome of community-acquired methicillin-resistant
Staphylococcus aureus pneumonia. Enferm Infecc Micro-
biol Clin;2014; 32: 23-7.
10. Li S,Li J,Qiao Y,Ning X,Zeng T,Shen X. Preva-
lence and invasiveness of community-acquired me-
thicillin-resistant Staphylococcus aureus: a meta-analysis.
Indian J Pathol Microbiol.2014; 57: 418-22.

47
CAPTULO 4
INFECCIONES DEL SISTEMA
NERVIOSO CENTRAL
Meningitis por Staphylococcus aureus:
el dilema del huevo o la gallina
Lanz, JF; Martnez Prez-Crespo, PM; Lpez-Corts, LE.
Hospital Universitario Virgen Macarena. Sevilla.

Caso clnico caba la presencia de leucocitosis intensa (32.440cls/mm3


con 93% de segmentados), mientras que el nico dato pa-
Mujer de 69 aos de edad con antecedentes de hiperten- tolgico en la bioqumica era un valor de protena C reac-
sin arterial y artritis reumatoide en tratamiento basal des- tiva (PCR) de 479 mg/L.
de hace un ao con metotrexato subcutneo a dosis de 15
mg semanal. En el contexto de una reagudizacin reciente Evolucin
de dicha patologa se haba aadido a su tratamiento ha-
bitual 90 mg de prednisona oral diario dos semanas atrs, En base a la historia clnica y los antecedentes epidemio-
con una dosis inicial de 80 mg de metilprednisolona por lgicos de la paciente, en un primer momento pensamos
va intramuscular. que el resultado de la determinacin del antgeno de neu-
Encontrndose previamente bien, la paciente presenta de- mococo en LCR pudiera corresponder a un falso negativo,
terioro cognitivo de 48 horas de evolucin caracterizado por lo que se inici tratamiento intravenoso emprico con
por desorientacin y bradipsiquia, as como sensacin fe- cefotaxima (300 mg/kg/da), y vancomicina (1 g cada 8
bril no termometrada, motivo por la que es trasladada por horas) siguiendo las guas especficas actualizadas. Sin em-
sus familiares a Urgencias de un Hospital de Alta Resolu- bargo, la evolucin clnica fue desfavorable en las horas
cin. En las horas posteriores progresa el deterioro de nivel posteriores, desarrollando afasia global y cifras de hiper-
de conciencia, motivo por el que se decide trasladarla a su tensin arterial compatibles con emergencia hipertensiva,
Hospital de referencia de tercer nivel. La primera explo- motivo por el cual se decidi su ingres en la Unidad de
racin clnica realizada a la paciente revel los siguientes Cuidados Intensivos (UCI) del centro. En dicha unidad se
datos: temperatura axilar 38.1C, tensin arterial 130/80 aadi al tratamiento ampicilina (2 gramos cada 4 horas)
mmHg, buena perfusin distal con llenado capilar menor a debido a la gravedad clnica y a que la paciente era mayor
dos segundos, y frecuencia cardiaca 80 latidos por minuto. de 50 aos, dato que ha mostrado ser un factor de riesgo
La exploracin cardiorrespiratoria, abdominal y cutnea para el desarrollo de meningitis por Listeria monocytogenes.
no presentaba alteraciones destacables. En relacin con la El Servicio de Microbiologa Clnica aisl Staphylococcus
exploracin neurolgica, la paciente presentaba lentitud aureus en el LCR, cuyo perfil de susceptibilidad poste-
ideomotora marcada y desorientacin temporo-espacial, riormente mostr una concentracin mnima inhibitoria
con una puntuacin de 15 en escala de Glasgow. En dicho (CMI) a cloxacilina a 0,25 g/mL, en base a lo cual se
momento era capaz de obedecer rdenes sencillas aunque procedi a realizar un tratamiento dirigido en monotera-
no aquellas de mayor complejidad. El signo de Brudzinski pia con cloxacilina IV 2 gramos cada 4 horas. No se objeti-
estaba presente siendo el resto de la exploracin normal. v bacteriemia en ningn momento del ingreso.
En base a estos datos clnicos se procedi a extraccin de Debido a que se desconoca la puerta de entrada del pat-
hemocultivos seriados, as como a realizar una tomografa geno al sistema nervioso central se realiz una resonancia
axial computerizada (TAC) de crneo urgente, en la cual magntica de trax y abdomen, en la que se objetiv una
no se apreciaron datos patolgicos. Tras la realizacin de extensa coleccin multiloculada paravertebral derecha que
dicha tcnica de imagen, se realiz una puncin lumbar se introduca al canal raqudeo entre los cuerpos vertebra-
para extraccin de lquido cefalorraqudeo (LCR), que les L3, L4 y arcos posteriores, extendindose a retroperi-
mostr hiperproteinorraquia (584 mg/dL), incontables toneo y psoas derecho. Se procedi a realizar un drenaje
leucocitos a expensas de polimorfonucleares, y glucorra- percutneo de la coleccin por medio de tcnica de Seldin-
quia de (36 mg/dL), correspondiente a un cociente LCR/ ger por parte del Servicio de Radiologa Intervencionista.
suero de 0.23. En la tincin de Gram se observaron cocos En una muestra del material purulento drenado se aisl de
grampositivos, con una determinacin de antgeno de neu- nuevo S. aureus con el mismo perfil de resistencias y CMIs.
mococo mediante inmunoanlisis de LCR (BinaxNOW) Posteriormente se produjo una mejora paulatina del pa-
negativa. En el hemograma realizado en Urgencias desta- ciente, incluidos los sntomas neurolgicos descritos hasta

50
CAPTULO 4
INFECCIONES DEL SISTEMA
NERVIOSO CENTRAL

Figura 2. Drenaje percutneo de absceso paraespinal derecho por tcnica de


Seldigner guiado por tomografa.

Figura 1. TC de abdomen en corte coronal. Se muestra absceso paraespinal en nuestro medio es Streptococcus pneumoniae, con una pre-
derecho (flecha roja).
valencia que llega al 50% en las distintas series descritas1.
En base a ese dato y al resultado de la tincin Gram, el
quedar asintomtica. En un control de TAC posterior se resultado de la determinacin de antgeno de neumococo
objetiv la desaparicin de la coleccin paravertebral, dato en LCR se interpret como un posible falso negativo ya
que junto con la normalizacin de los reactantes de fase que como es sabido la sensibilidad de esta prueba (Strepto-
aguda se emple para decidir la duracin final del trata- coccus pneumoniae Antigen Card BinaxNOW) puede ser en
miento IV. ciertos casos insuficiente (69% - 100%)2.
La incidencia global de meningitis aguda por S. aureus se
Diagnstico final estima que se encuentra entre un 3%1 y un 9%3. Esta en-
tidad consta de dos mecanismos patognicos bien diferen-
El diagnstico final fue meningitis por S. aureus sensible a ciados. En el primer grupo distinguimos las meningitis
meticilina secundaria a un absceso paraespinal en una pa- postquirrgicas o postraumticas, que abarca el 30%
ciente inmunosuprimida por tratamiento con corticoides y de los casos, en el que las bacterias penetran durante pro-
metotrexate. A pesar de las distintas pruebas complemen- cedimientos neuroquirrgicos, fstulas de lquido cefalorra-
tarias realizadas, no pudo demostrarse una puerta de en- qudeo, traumatismos, o en menor medida, desde un foco
trada. En nuestra opinin, el origen de la coleccin pudo contiguo de infeccin (como otitis, sinusitis o mastoiditis)3.
ser la puncin intramuscular ipsilateral que la paciente El segundo grupo son las meningitis hematgenas o es-
recibi dos semanas antes de iniciarse la clnica descrita pontneas, en las que el S. aureus alcanza las meninges por
como dosis inicial del tratamiento corticoideo prescrito va bacterimica desde un foco ajeno al sistema nervioso
para su brote de artritis reumatoide, ya que no hallamos central, afectan principalmente a pacientes ancianos con
otra posible puerta de entrada en su historial clnico ni en importantes comorbilidades de base, como enfermedades
las conversaciones con la paciente. En cualquier caso, tam- cardiovasculares, enfermedad renal crnica y diabetes,
poco podemos descartar con absoluta certeza que dicho aunque tambin incluye aquellos casos que se originan de
absceso apareciese posteriormente a la clnica menngea. manera primaria directamente en el sistema nervioso cen-
Creemos igualmente que el papel de la iatrogenia en este tral, mucho menos frecuentes3.
caso es muy importante, ya que la inmunosupresin celu- En nuestro caso, a pesar de la extraccin de hemocultivos
lar y humoral secundaria a metrotrexate y corticoides tuvo previos al inicio de la antibioterapia emprica, no fue po-
un papel importante en la fisiopatologa inicial del cuadro sible demostrar una potencial diseminacin hematgena
y en su evolucin clnica subaguda. que explicara la afectacin menngea. Este dato y los ha-
llazgos de las pruebas de imagen nos llevan a pensar que
Discusin el mecanismo fisiopatognico fue la diseminacin por con-
tigidad desde el absceso paraespinal. A pesar de ello, no
La clnica, exploracin neurolgica y pruebas complemen- puede descartarse la diseminacin hematgena.
tarias descritas son definitorias de meningitis bacteriana Respecto a la indicacin de realizar una TAC craneal pre-
aguda. El agente etiolgico ms comn de esta patologa via a la puncin lumbar ante la sospecha de meningitis

51
CAPTULO 4
INFECCIONES DEL SISTEMA
NERVIOSO CENTRAL

aguda comunitaria, se recomienda la realizacin de una inflamatoria que vancomicina por no inducir lisis celular
prueba de imagen previa en las siguientes circunstancias: bacteriana9. Tambin se considera como opcin linezolid.
pacientes con inmunosupresin (VIH, postransplante, en Mltiples estudios han demostrado que la concentracin el
tratamiento farmacolgico inmunosupresor), enfermeda- LCR es elevada (superior a 80%), aunque su mecanismo es
des del sistema nervioso central (lesin ocupante de espacio bacteriosttico puede utilizarse en esta indicacin10.
con efecto de masa, ictus o infeccin conocida), convulsin
reciente (se considera dentro de la ltima semana), papile- El caso clnico de la paciente expuesta en este trabajo se
dema, alteracin del nivel de conciencia, dficit neurolgi- consider de particular inters por la presentacin atpica
co focal y mayores de 60 aos4,5. No obstante, algunas de de un cuadro menngeo infeccioso de etiologa bacteriana,
estas recomendaciones no estn exentas de debate, entre tanto por el germen causal como por la coleccin de partes
ella la que nos ocupa en este caso: la alteracin del nivel blandas, la cual se presume fue el foco inicial.
de conciencia. Algunos autores sostienen que en determi-
nados supuestos de gravedad como el mencionado, la rea-
lizacin de una prueba de imagen previa retrasa innecesa-
riamente la puncin lumbar y consecuentemente el inicio
de antibioterapia precoz, pudiendo afectar negativamente
a la supervivencia y aparicin de secuelas posteriores, sin
aportar en cambio beneficios frente al riesgo de daos ce-
rebrales por herniacin4.
El tratamiento de eleccin en la meningitis aguda por
S. aureus depende directamente del perfil de su perfil de
susceptibilidad. En aquellos sensibles a meticilina se debe
utilizar cloxacilina a dosis elevadas (2g/4h) como primera
opcin. En aquellas cepas de S. aureus resistentes a meti-
cilina (SARM) se debe utilizar vancomicina 40-60 mg/
kg/da, aunque existen opciones alternativas como cotri-
moxazol, linezolid o teicoplanina2, 6. La concentracin de
cloxacilina en LCR es muy baja incluso en situacin de
meningitis (ABCLCR/ABCplasma menor al 5%). La pene-
tracin de vancomicina en LCR es algo mayor, aunque
no supera el 20% de la concentracin plasmtica6. Con
el objetivo de conseguir concentraciones efectivas en LCR
se aconseja alcanzar unas concentraciones valle plasm-
ticas elevadas (15 20 g/mL), mediante la utilizacin
de dosis comprendidas entre 35 y 45 mg/kg/da. El pa-
rmetro farmacocintico predictor de eficacia para van-
comicina es el ABC24h/CMI del microorganismo aislado,
cociente que no debe ser menor de 4008. Algunos estudios
contemplan la opcin de asociar rifampicina o cotrimoxa-
zol debido a su potencial sinergia farmacolgica, aunque
la evidencia al respecto es limitada7. En la actualidad hay
varios estudios en marcha focalizados en profundizar so-
bre opciones teraputicas distintas a vancomicina en el
tratamiento de infecciones del sistema nervioso central por
SARM. Una de estas opciones es la utilizacin de cotri-
moxazol asociado a rifampicina en aquellos pacientes en
los que han tenido un fracaso teraputico con vancomicina
o cuando no es posible su utilizacin. Otra es el empleo de
daptomicina debido a su alto poder bactericida frente a
grmenes grampositivos y a que genera menor respuesta

52
CAPTULO 4
INFECCIONES DEL SISTEMA
NERVIOSO CENTRAL

Bibliografa

1. Roos KL, Tyler KL. Meningitis, Encephalitis, Brain


Abscess, and Empyema. In: Longo DL, Kasper DL,
Jameson JL, Fauci AS, Hauser SL, Loscalzo JL, edi-
tors. Harrison's Principles of Internal Medicine. 18th ed.
New York: McGraw-Hill; 2012.
2. Tunkel AR, Hartman BJ, Kaplan SL, Kaufman BA,
Roos KL, Scheld WM, et al. Practice Guidelines for
the Management of Bacterial Meningitis. CID. 2004;
39: 1267-84.
3. Aguilar J, Perri M. Staphylococcus aureus meningitis: case
series and literature review. Medicine. 2010; 89: 117-
25.
4. Glimker M, Johansson B, Grindborg , Bottai
M,Lindquist L,Sjlin J. Adult bacterial meningitis:
Earlier treatment and improved outcome following
guideline revision promoting prompt lumbar punctu-
re. Clin Infect Dis. 2015; 60: 1162-9.
5. Colmenero JD, Gonzlez M, Jimenez E. Sociedad
Andaluza de Enfermedades Infecciosas, Avances en
enfermedades infecciosas: Meningitis bacteriana en
pacientes adultos, Documento de Consenso. 2005; 7.
6. Mensa J, Gatell JM, Jimnez de Anta MT, Prats G,
Domnguez-Gil A. Gua de teraputica antimicrobia-
na. 24. Ed. Barcelona: Masson, 2014.
7. Pintado V, Pazo R, Jimnez-Mejas ME, Rodr-
guez-Guardado A, Gil A, Garca-Lechuz JM, Methici-
llin-Resistant Staphylococcus aureus Meningitis in Adults:
A Multicenter Study of 86 Cases. Medicine. 2012; 91:
10-17.
8. Olaechea PM, Garnacho J, Grau S, Rodrguez O,
Palomar M, Zaragoza R, Muoz P, Cerd E, lvarez
F, Recomendaciones GEIPC-SEIMC y GTEI-SEMI-
CYUC para el tratamiento antibitico de infecciones
por cocos grampositivos en el paciente crtico. Enferm
Infecc Microbiol Clin. 2007; 25: 446-66.
9. Lee DH, Palermo B, Chowdhury M. Successful Treat-
ment of Methicillin-Resistant Staphylococcus aureus Me-
ningitis with Daptomycin. Clin Infect Dis. 2008; 47:
588-90.
10. Myrianthefs P, Markantonis SL, Vlachos K, Anag-
nostaki M,Boutzouka E,Panidis D, et al. Serum and
Cerebrospinal Fluid Concentrations of Linezolid in
Neurosurgical Patients.Antimicrob Agents Chemo-
ther. 2006; 50: 3971-6.

53
Varn de 53 aos con clnica de rombencefalitis
y elevacin de adenosn deaminasa (ada) en LCR
Cruces Moreno, MT; Guilln Zafra, L;
Vinuesa Garca, D; Muoz Medina, L.
Hospital Clnico San Cecilio. Granada.

Caso clnico Caractersti- Normal Meningitis aguda Meningitis Meningitis


cas del LCR purulenta subaguda linfocitaria
o crnica o asptica
Varn de 53 aos, diabtico en tratamiento con sitaglipti- Presin de 8-18 Elevada Elevada Normal o
apertura cmH2O elevada
na/metformina, fumador activo, bebedor y consumidor de
Aspecto Claro Purulento Claro o turbio Claro
cocana, que acude a urgencias por cuadro febril de hasta
Recuento 0-5 1000-10000 50-500 5-1000
39C, de cuatro das de evolucin asociado a inestabilidad celular/mm3
en la marcha, diplopia e intensa cefalea holocraneal. Sin Celularidad Mono- Polimorfonuclear Mononuclear Mononu-
otros datos de inters en la anamnesis por rganos y sis- predominante nuclear clear

temas. Glucorraquia 60-80% <60% <25% Habitual-


repecto a mente
A la exploracin fsica destaca: T 38C; TA 160/102 glucemia % normal
mmHg; FC 99 lpm. SpO2 94% (Fi02 21%), eupneico, Proteinor- 40-50 >100 50-300 <100
regular estado general. Neurolgicamente est consciente raquia mg/dl

y orientado, presenta una parlisis facial del Vll par de- Etiologas S. Pneumoniae Tubercolosis Virus,
infecciosas H. Influenzae C. Neoformans parsitos
recho, dismetra en miembro superior derecho y marcha N. Neningitidis Listeria
Enterobacterias Brucella
con aumento de la base de sustentacin con lateropulsin Listeria
derecha. Fuerza y sensibilidad conservada en las 4 extre-
midades. No signos de irritacin menngea, hepatomegalia Figura 1. Caractersticas del LCR en las diferentes infecciones del SNC.
no dolorosa de 2 cm a la palpacin abdominal, siendo el
resto de la exploracin fsica y neurolgica normal, excep- Diagnstico diferencial
to ginecomastia, eritema palmar bilateral y contractura de
Dupuytren entre el cuarto y el quinto dedo. Ante un cuadro febril con afectacin neurolgica nos plan-
Los resultados de las primeras pruebas realizadas en ur- teamos el diagnstico diferencial de las meningitis agudas.
gencias fueron las siguientes: En la analtica destacaba La primera aproximacin diagnstica se realiza atendien-
leucocitosis (13.000mm3) con neutrofilia (N 81%), resto de do a las caractersticas del LCR (figura 1), donde se obser-
series y coagulacin normales. Bioqumica: PCR 44 mg/l, va un predominio de clulas mononucleares, caracterstica
glucosa: 124mg/dl, funcin renal, LDH, iones y funcin que la diferencia de las meningitis de etiologa bacteriana
heptica normales. Radiografa (Rx) de trax y tomogra- que habitualmente presentan un predominio de polimor-
fa computerizada (TC) craneal sin contraste normales. Se fonucleares. La importancia de esta clasificacin inicial ra-
realiza puncin lumbar y se obtiene un lquido cefalorra- dica en poder descartar entidades que conllevan un mane-
quideo (LCR) a presin normal de aspecto algo turbio, con jo especfico tanto del paciente como de su entorno como
aumento marcado de protenas (239.2mg/dl), consumo de la meningitis meningoccica. Se recomienda ser prudente
glucosa (39mg/ml) y el siguiente recuento diferencial: he- en la interpretacin del LCR y ante la ms mnima duda
mates 18/mm3 y leucocitos 840/mm3 (7%PMN y 93%L). comenzar con tratamiento antibitico emprico ya que es
Se extraen hemocultivos, urocultivos y se solicita en LCR, una entidad que compromete la vida del paciente y el LCR
ADA, tincin de Gram y para bacilo alcohol resistentes puede alterar sus caractersticas si ha recibido antibiticos
(BAAR), cultivos y PCR para virus herpes simple (VHS), previos a la puncin o sta se ha realizado de forma muy
virus varicela zoster (VVZ), tuberculosis (TBC) y enterovi- precoz.
rus. Se informa la tincin de Gram inicial como negativa. Cuando predominan clulas mononucleares, en la etiolo-
Tras inicio de antibioterapia emprica se ingresa en Infec- ga se distinguen causas no infecciosas como diferentes en-
ciosas con el diagnstico de meningitis aguda. fermedades autoinmunes, vasculitis, neoplasias o toxicidad
medicamentosa; o con mayor frecuencia, causas infeccio-
sas por lo que siempre es necesario descartarlas. La agentes
infecciosos causales que se aslan con mayor frecuencia son

54
CAPTULO 4
INFECCIONES DEL SISTEMA
NERVIOSO CENTRAL

los virus; en nuestro medio los ms frecuentes son enterovi- en lbulos temporales bilateral y asimtrica, sin em-
rus, VHS tipo 2, VVZ y Toscana. Otros microorganismos bargo, en raras ocasiones cursa afectacin restringida
menos frecuentes son Listeria monocytogenes, Mycobacte- al tronco del encfalo.
rium tuberculosis (TBC), Treponema pallidum, Brucella y Cryp- Toxoplasmosis: es la infeccin cerebral oportunista
tococcus (1). Nuestro paciente, sin embargo, asocia clnica ms frecuente en pacientes con VIH. En la TC o re-
de afectacin del tronco del encfalo, cuadro conocido sonancia magntica (RM) se suelen observar lesiones
como romboencefalitis y un LCR con protenas elevadas nodulares que captan contraste en anillo, con edema
y consumo de glucosa, lo que descarta la mayora de los en la fase aguda.
virus que se aslan en nuestro medio, siendo quizs en este Enfermedad de Lyme: es importante tenerla en cuenta
escenario Listeria monocytgenes, la etiologa ms frecuente. a pesar de tener una mayor distribucin a nivel de la
Otras causas menos frecuentes incluyen VHS, TBC, toxo- Cornisa Cantbrica y el Nordeste de Espaa, ya que
plasmosis, criptococcosis, enfermedad de Lyme, brucelosis se han reportado casos en la zona de Andaluca. Las
y virus JC. La historia clnica y los diferentes patrones de manifestaciones iniciales suelen ser autolimitadas y en
exposicin del paciente pueden ayudarnos a discriminar ocasiones suelen pasar desapercibidas e incluso pue-
de inicio algunas de ellas. den cursar sin la aparicin de la lesin tan caracters-
tica como es el eritema migratorio que aparece solo
Listeria: como bien hemos dicho, es la causa ms fre- en una cuarta parte de los pacientes. Por lo tanto pode-
cuente y afecta principalmente a pacientes de edad mos recibir un paciente en un estado de infeccin di-
media previamente sanos. El curso clnico suele ini- seminada temprana donde pueden aparecer sntomas
ciarse con prdromos de sntomas inespecficos (cefa- neurolgicos como el descrito en nuestro caso.
lea, malestar general, Naseas, vmitos y fiebre de das Criptococosis y otras infecciones fngicas: son poco
de evolucin) seguido de clnica de disfuncin progre- frecuentes y suelen afectar a pacientes inmunodepri-
siva del tallo cerebral: afectacin de pares craneales, midos. En el caso de Cryptococcus los hallazgos clnicos,
paresias, dficit sensitivo, insuficiencia respiratoria, de- de LCR y radiolgicos plantean el diagnstico diferen-
terioro del nivel de conciencia y, ocasionalmente con- cial con la meningitis tuberculosa. El diagnstico de-
vulsiones. El diagnstico de certeza se establece con la finitivo viene de la demostracin del hongo mediante
positivizacin de los cultivos de LCR o de sangre. tincin con tinta china, antgeno o cultivo.
TBC: suele afectar a adultos, generalmente como
reactivacin secundaria en una situacin de inmu- Evolucin
nodepresin; menos frecuentemente sigue a una pri-
moinfeccin. La fase de afectacin florida del tronco Ante la situacin clnica del paciente, y siguiendo las re-
encfalo viene precedida por un periodo prodrmico comendaciones de la gua local de tratamiento antibi-
de enfermedad febril subaguda de 2-3 semanas aso- tico emprico, se ingres en tratamiento con linezolid,
ciada a malestar general, astenia, cefalea y cambios en ceftriaxona y Aciclovir. Durante su ingreso se reciben las
el comportamiento. Alrededor de un tercio de los pa- pruebas solicitadas en urgencias; PCR para VHS tipo 1
cientes, asocian TBC miliar en cuyo caso la inspeccin y 2, enterovirus, VVZ y TBC, todas negativas por lo que
del fondo de ojo a menudo pueden mostrarnos tubr- se suspende Aciclovir. Se realizan serologas para VHC,
culos coroides, pista muy valiosa para el diagnstico VHB, sfilis y VIH, tambin negativas. Posteriormente, a
etiolgico. Podremos ver anomalas en la Rx de trax las 48h del ingreso, Microbiologa informa del crecimiento
hasta en un 50% de los casos y generalmente tendre- en el LCR de Listeria monocytogenes, por lo que se ajusta tra-
mos un mantoux positivo pero su negatividad no ex- tamiento segn antibiograma y se inicia ampicilina IV a
cluye dicha etiologa. La tincin y cultivo para BAAR dosis elevadas, asociada a gentamicina, retirndose el resto
es el mejor mtodo diagnstico siendo stos positivos de antibiticos. Das despus, se recibe un ADA de 53.7
inicialmente en un 37% de los casos, aumentando a un U/L aunque ello no modific nuestra actitud terapeti-
87% si se examinan nuevas muestras en los das sucesi- ca. Se completa el estudio con una RM donde se observa
vos a pesar de haber iniciado terapia tuberculosttica. un absceso en hemiprotuberancia y pednculos cerebrales
La determinacin del ADA nos ayuda a establecer el derechos con efecto masa sobre la mitad derecha del lV
posible diagnstico aunque el punto de corte no est ventrculo compatible con absceso cerebral (Figura 2a).
bien establecido. A pesar del tratamiento con ampicilina y gentamicina, el
VHS: suele dar un cuadro de encefalitis necrotizante dficit neurolgico del paciente continu progresando y

55
CAPTULO 4
INFECCIONES DEL SISTEMA
NERVIOSO CENTRAL

la literatura de pacientes afectos de absceso cerebral por


Listeria, observaron que la mayora de los pacientes eran
varones, con una edad media de 50 aos y que asociaban
alguna enfermedad debilitante, como en nuestro caso, o
algn grado de inmunodepresin. Al diagnstico, todos
ellos presentaron fiebre o febrcula y solo 5 de ellos, cur-
saban con sintomatologa neurolgica; pese a esto, el diag-
nstico inicial de absceso cerebral (en base a la clnica y a
los hallazgos en la TC inicial) se realiz nicamente en un
paciente (2).
Analizando el LCR, en una romboencefalitis por Listeria
podemos encontrar caractersticas de una meningitis lin-
focitaria (Figura 1), la tincin de Gram presenta muy baja
Figura 2. a) RM donde se aprecia absceso en hemiprotuberancia y pednculos sensibilidad, y es positiva slo en una tercera parte de los
cerebrales derechos con efecto masa sobre la mitad derecha del lV ventrculo.
b) RM de control tras dos meses de tratamiento.
casos; y cuando lo es, puede llevarnos a confusin por el
polimorfismo que Listeria es capaz de adoptar similar a
empeor el nivel de conciencia, por lo que se traslad a otros microorganismos. Los cultivos, tanto de LCR como
la Unidad de Cuidados Intensivos (UCI) donde precis de de sangre son imprescindibles para establecer el diagns-
intubacin y manejo con vasoactivos por inestabilidad he- tico de certeza. En LCR son positivos en un 38-41% de
modinmica asociada. Tras varias semanas de estancia en los casos (3), sin embargo, ante la presencia de un absceso
UCI se estabiliza y, a pesar de una extubacin dificultosa, cerebral, como en el caso que nos ocupa, los hemoculti-
consigui una recuperacin parcial del dficit neurolgico vos tienen una mayor rentabilidad diagnstica alcanzando
manteniendo una parlisis facial derecha, oftalmopleja del positividad en un 90% de los casos (2), dato importante
Vl par craneal y hemiparesia derecha residual. Se realiz cuando existe contraindicacin para la realizacin de una
RM de control aprecindose una disminucin significativa puncin lumbar, lo que en ocasiones permite llegar al diag-
de la lesin a nivel de tronco (Figura 2b) nstico sin el cultivo de LCR. La PCR en tiempo real para
el gen hly que codifica listeriolisina O, se ha desarrollado
Diagnstico final y probado para la deteccin de Listeria en el LCR, siendo
ms especfica y sensible que los cultivos pero an no se
Romboencefalitis por Listeria con absceso cerebral asociado. encuentra disponible comercialmente (4).
Es imprescindible, hasta confirmar el diagnstico, iniciar
Discusin antibioterapia emprica. Segn las guas clnicas, ante un
paciente con meningitis aguda, teniendo en cuenta la edad
Listeria monocytgenes, es una importante causa de zoonosis y y determinadas caractersticas (mayor de 50 aos y alco-
contaminacin de muchos alimentos, siendo sta ltima, la hlico en nuestro caso), el tto de eleccin sera la combina-
puerta de entrada ms frecuente para causar infeccin en cin de Vancomicina, ceftriaxona, ampicilina y Aciclovir
el ser humano. Presenta especial tropismo por el SNC sien- cubriendo todos los agentes causales tanto de meningitis
do la meningitis la forma ms frecuente de presentacin; bacterianas como vricas. linezolid, al igual que Vancomi-
en nuestro caso se presenta como una romboencefalitis. cina, es efectivo frente a la mayora de los microorganismos
sta entidad, descrita por primera vez en 1957 por Eck, se Gram positivos, incluyendo cepas resistentes, aunque ste
trata de una forma infrecuente de Listeriosis y al contrario presenta una mayor penetracin a nivel del SNC consi-
que en las meningitis agudas que se asocian con frecuencia guiendo, a las dosis habituales, una concentracin mnima
a determinados grupos de riesgo como alcohlicos, inmu- inhibitoria mayor que Vancomicina. Vancomicina necesita
nodeprimidos o pacientes de edad avanzada, sta puede dosis ms elevadas para llegar a dichos niveles aadiendo,
aparecer en pacientes previamente sanos. por tanto, mayor toxicidad especialmente renal. Adems,
En el caso que nos ocupa, el paciente adems asocia un en algunos ensayos in vitro, linezolid demuestra una buena
absceso cerebral que no se visualizaba en la TC inicial. actividad frente a Listeria monocytogenes que, a pesar de ser
El absceso cerebral aparece en el 10% de las infecciones algo menor que ampicilina, justificara su uso junto a cef-
por Listeria del SNC y en una revisin realizada por Mi- triaxona y Aciclovir como terapia emprica (5). Ante el au-
reia Moragas et al, donde analizaron 76 casos descritos en mento de resistencias a penicilina de neumococo, la mayor

56
CAPTULO 4
INFECCIONES DEL SISTEMA
NERVIOSO CENTRAL

frecuencia de S. aureus de adquisicin comunitaria como Hasta la fecha, hay descritos varios casos de meningitis
etiologa de meningitis de adquisicin por va hematge- por Listeria, asociadas a elevacin de los niveles del ADA.
na y la mayor tasa de fracaso con Vancomicina cuando la En algunos de ellos, se obtienen niveles de ADA elevados
CMI a sta es mayor de 1,5, adems de lo comentado ante- previamente a la positivizacin de los cultivos. En otros al
riormente, son los motivos por lo que linezolid nos parece contrario, la positivizacin de los cultivos fue previa, por
una buena opcin como terapia emprica de inicio (6). lo que iniciaron terapia dirigida a Listeria. En todos ellos,
En nuestro caso se podra haber planteado iniciar tra- el paciente mejora clnicamente asociando una normaliza-
tamiento tuberculosttico ante los resultados del LCR y cin de dicho parmetro (9).
con los niveles de ADA que se recibieron, si no hubiera Con respecto al tratamiento, el antibitico de eleccin ante
un diagnstico alternativo, por la gravedad de esta enti- una infeccin por Listeria es la ampicilina, no obstante,
dad y la importancia del retraso en el tratamiento. El tra- sta presenta cierto retraso en la actividad bactericida, por
tamiento de la meningitis tuberculosa, debe iniciarse ante lo que, en general, en pacientes con infecciones en el SNC
un paciente con afectacin del SNC asociado a pleocitosis se recomienda aadir gentamicina para un efecto sinr-
linfocitaria, protenas elevadas y glucosa baja en LCR, si gico bactericida, excepto ante la presencia de un absceso
hay indicios de afectacin de tuberculosis en otros luga- cerebral donde el beneficio es cuanto menos dudoso dada
res y no se logra establecer un diagnstico alternativo. La la escasa o nula actividad de ste antibitico en presencia
determinacin de ADA en LCR se utiliza para apoyar el de contenido purulento. No obstante dado que la combi-
diagnstico de meningitis tuberculosa, aunque el punto de nacin puede ser til en la primera fase de la enfermedad
corte no est bien establecido. Dependiendo de las series para controlar la sepsis inicial o la posible meningitis con-
consultadas, vara desde 6,5 hasta 11,5 U/L, y numerosos comitante, se recomienda el tratamiento combinado pero
autores le confieren una elevada sensibilidad y especifici- sin exceder la semana en la mayora de los casos. El tra-
dad, por encima del 90%. En un metaanlisis publicado en tamiento con dexametasona en la meningitis bacteriana,
2010 intentan determinar el papel del ADA, y realizan una aporta escasos beneficios, salvo en alguna etiologa con-
revisin sistemtica donde incluyeron un total de 10 estu- creta como meningitis por neumococo, y en determinados
dios publicados hasta la fecha, observando una sensibilidad escenarios como podra ser la presencia de efecto masa
de un 79% y una especificidad de 91% demostrando con como ocurre en nuestro paciente (10) La antibioterapia
ello una elevada utilidad en el diagnstico de meningitis en pacientes inmunocompetentes, debe durar al menos de
tuberculosa. La mayor parte de los estudios que se incluye- dos a cuatro semanas, siempre aplicando el juicio clnico
ron se basaban en poblaciones con una alta prevalencia de valorando la necesidad de extender la misma en funcin
meningitis tuberculosa. Un estudio ms reciente, estudio de los resultados de la RM o el cultivo de LCR.
Haydarpasa-1, donde incluyen una de las mayores series
de casos jams registrada de meningitis tuberculosa y don-
de participan pases con una menor prevalencia, observan
una menor sensibilidad del ADA (29.9%) para un mismo
punto de corte. En el mismo estudio, adems, establecen
una sensibilidad de un 81.8% para los cultivos automatiza-
dos en medio lquido y de un 72.7% para cultivo en medio
de Lowenstein- Jensen aumentando si se combinan ambos.
(7) El retraso en la positivizacin de los cultivos, hace que
el ADA se utilice para llegar al diagnstico de meningitis
tuberculosa y algunos autores defienden que para mejo-
rar estos problemas de sensibilidad en las zonas de baja
prevalencia lo mejor es incluirlo en un algoritmo predic-
tivo, junto con otros parmetros del LCR como glucosa
o los leucocitos, alcanzando una sensibilidad del 91%. La
interpretacin del valor del ADA debe hacerse acorde a la
prevalencia de la zona o a la probabilidad preprueba de la
regin (8). La PCR para TBC es muy til cuando es positi-
va por su elevada especificidad pero un resultado negativo
no excluye el diagnstico (9).

57
CAPTULO 4
INFECCIONES DEL SISTEMA
NERVIOSO CENTRAL

Bibliografa

1. Navarro Mar JM, Prez Ruz M, Vicente Anza. Diag-


nstico de laboratorio de las meningitis linfocitarias.
Enferm Infecc Microbiol Clin. 2010; 28 (Supl 1):56-
61.
2. Mireia Moragas, Sergio Martnez-Ylamos, scar
Murillo, Pedro Fernndez-Viladrich. Absceso cerebral
del adulto por Listeria Monocytgenes: presentacin
de 6 casos clnicos y revisin de la literatura mdica.
Enferm Infecc Microbiol Clin. 2010; 28:8794.
3. Armstrong RW, Fung PC. Brainstem encephalitis
(rhombencephalitis) due to Listeria monocytogenes:
case report and review. Clin Infect Dis. 1993; 16:689
702.
4. Le Monnier A, Abachin E, Beretti JL, Berche P, Kayal
S. Diagnosis of Listeria monocytogenes meningoen-
cephalitis by real-time PCR for the hly gene. J Clin
Microbiol. 2011;49:3917.
5. Roland Nau, Fritz Sorgel, Helmut Eiffert. Penetration
of Drugs through the Blood- Cerebrospinal Fluid/
Blood-Brain Barrier for Treatment of Central Ner-
vous System Infections. Clin, Microbiol. Rev. 2010;
23: 858-883.
6. Aguilar J, Urday-Cornejo V, Donabedian S, Perri M,
Tibbetts Robert, Zervas M. Staphylococcus Aureus,
Case series and literature review. Medicine. 2010; 89:
117-125.
7. Erdem H, Ozturk-Engin D, Elaldi N, Gulsun S, Sen-
goz G, Crisan A Et al. The microbiological diagnosis
of tuberculous meningitis: results of Haydarpasa-1
study. Clin Microbiolol Infect. 2014 Oct; 20:O600-8.
8. Parra-Ruiz J, Ramos V, Dueas C et al. Rational appli-
cation of adenosin deaminase activity in cerebrospinal
fluid for the diagnosis of tuberculous meningitis. Infec-
tion. 2015;43:531-5
9. Cabezas P, Ruiz A, Morales JL, Porcel JM. Meningitis
caused by Listeria monocytogenes with high levels of
adenosine deaminase. Enferm Infecc Microbiol Clin.
2011 Mar; 29:240.
10. Diederik van de Beek, M.D., Ph.D., Jan de Gans, M.D.,
Ph.D., Allan R. Tunkel, M.D., Ph.D., and Eelco F.M.
Wijdicks, M.D., Ph.D. Community-Acquired Bacterial
Meningitis in Adults. N Engl J Med 2006;354:44-53.

58
Lesin ocupante de espacio cerebral
en paciente inmunodeprimida
Hidalgo Castelln, Antonio J.; Lpez Gonzlez, C;
Nadejda Gutirrez Cortizo, E; Martnez Marcos, FJ.
Complejo Hospitalario Universitario de Huelva.

Caso clnico Origen/Situacin Microorganismos


Pulmonar (absceso Estreptococos del grupo viridans, S. aureus,
pulmonar, empiema, enterobacterias (Proteus), P. aeruginosa, Haemo-
Mujer de 68 aos con cardiopata hipertensiva y sarcoi- bronquiectasias o philus, Nocardia, Cryptococcus, microorganismos
fibrosis qustica) anaerobios (Bacteroides, Prevotella, Fusobacteri-
dosis pulmonar en tratamiento con corticoides (20 mg de um, Propionibacterium, Actinomyces).
prednisona al da). La paciente refera astenia de 3-4 meses Endocarditis S. aureus, estreptococos del grupo viridans
de duracin, y desde la semana anterior inestabilidad en la Bacteriemia con o Staphylococcus aureus, enterobacterias, estrepto-
marcha con desviacin a la derecha; presentaba hemipa- sin foco aparente coco del grupo viridans, Listeria, Actinomyces.

resia derecha (fuerza miembro superior derecho 4/5 y en Inmunodepresin Nocardia, Toxoplasma gondii, Listeria, hongos
(incluyendo (Aspergillus, Cryptococcus, Candida, Mucorales,
inferior derecho 3/5) y afasia sensitiva con ecolalia; el resto pacientes con sida) hongos dematiceos), M. Tuberculosis, enterobac-
de la exploracin fue normal. En la analtica la serie roja terias, Rhodococcus equi

y plaquetas fueron normales, leucocitos dentro de la nor- Pacientes procedentes Schistosoma japonicum. Entamoeba histolytica.
de reas tropicales Paragonimus. Balamuthia mandrillaris. Taenia
malidad sin desviacin a la izquierda, VSG 5 mm/h, coa- solium (cisticercosis). Echinococcus
gulacin normal, bioqumica con glucosa, iones, funcin Tabla 1. Microorganismos causales de absceso cerebral
renal y heptica normales, PCR 0.1mg/dL; serologa VIH
negativa y toxoplasma IgG positiva, antgeno criptocci- siella pneumoniae, Staphylococcus aureus, Pseudomonas aeruginosa
co srico negativo. En la tomografa computerizada (TC) o Stenotrophomonas maltophilia son menos frecuente en los
craneal se hall lesin ocupante de espacio (LOE) parietal pacientes inmunodeprimidos. La afectacin por S. aureus
izquierda de 2 cm de eje mximo rodeada de edema digi- es generalmente la complicacin de una endocarditis. Los
tiforme, que con contraste presentaba realce anular. Dado abscesos cerebrales debido a Mycobacterium tuberculosis tam-
que la imagen cerebral era compatible con neoplasia pri- bin se han descrito, pero son raros, pudiendo estar asocia-
maria o metstasis, se solicit TC de trax y abdomen no dos o no a infeccin diseminada.
observndose lesiones compatibles con tumor primario. Se La toxoplasmosis cerebral suele presentarse con cambios
realiz resonancia magntica (RM) de crneo, con hallaz- en el estado mental del paciente, convulsiones, cefalea y
go de dos LOEs, una frontal izquierda y otra parietoocci- dficits neurolgicos focales. Las lesiones, generalmente
pital derecha que por sus caractersticas sugirieron como mltiples, se localizan la en zona periventricular, y realzan
primera posibilidad abscesos cerebrales. en anillo con contraste. En pacientes con probable toxo-
Ante estos hallazgo se traslad a Neurociruga para toma plasmosis, la terapia emprica est justificada en ausencia
de muestras y drenaje de lesin frontal izquierda mediante de diagnostico microbiolgico cuando la serologa anti-
ciruga estereotxica. toxoplasma (anticuerpos IgG) es positiva.
Los pacientes con infeccin del sistema nervioso central
Diagnostico diferencial (SNC) por Listeria presentan normalmente bacteriemia.
Aunque la meningitis es la forma ms comn de presenta-
As como la etiopatogenia de un absceso cerebral nico sue- cin, tambin se ha descrito en abscesos cerebrales, ence-
le ser extensin por contigidad desde foco paramenngeo, falitis o romboencefalitis.
la inmunodepresin es un factor de riesgo para sufrir abs- Los abscesos cerebrales por Aspergillus se presentan normal-
cesos cerebrales mltiples por diseminacin hematgena. mente con afectacin pulmonar concurrente. Clnicamen-
Despus de descartar posibles focos de infeccin a distancia te los pacientes desarrollan alteracin del estado mental
(endocarditis, absceso pulmonar, fibrosis qustica, empiema) con o sin convulsiones; la presencia de dficit neurolgico y
y comprobar la procedencia no tropical del paciente (en este signos de irritacin menngea son menos comunes. Las le-
caso valorar esquistosomiasis, cisticercosis o amebiasis entre siones suelen ser multifocales con predileccin por la unin
otros), debemos centrarnos en patgenos que se aslan ms entre materia gris y blanca de los hemisferios cerebrales
frecuentemente en inmunodeprimidos (tabla 1). (ms frontoparietales), pero tambin puede implicar al ce-
Los abscesos intracraneales debido a bacterias como Kleb- rebelo o mesencfalo. Tiene una alta mortalidad.

59
CAPTULO 4
INFECCIONES DEL SISTEMA
NERVIOSO CENTRAL

Cryptococcus puede ser causa de meningitis en paciente in-


munodeprimidos. La presentacin como absceso cerebral
criptoccico es casi exclusivo de los paciente con SIDA, y
normalmente afecta a ganglios basales.
La afectacin del SNC por Candida ocurren en el contexto
de una candidemia, y pueden aparecer en coinfecciones
junto a Aspergillus. En las pruebas de imagen suelen ser le-
siones mltiples de pequeo tamao con realce en anillo
tras contraste, y situadas en la sustancia blanca o zonas
profundas de la sustancia gris.
Nocardia tambin tiene propensin a la infeccin del SNC
en forma de absceso, siendo el sitio ms frecuente de infec-
cin secundaria (la infeccin pulmonar suele ser la puer-
ta de entrada). La afectacin cutneas o del tejido blando
puede estar presente. Los abscesos debido a Nocardia pue- Foto 1. Colonias de Nocardia en agar sangre.
den ser loculados, individuales o mltiples (hasta el 40%) y
muestran realce en anillo tras contraste.
Los hongos dematiceos (gneros Dactylaria, Ochroconis, Sco-
ledobasidium), son patgenos muy poco frecuentes, pudien-
do causar abscesos cerebrales nicos o mltiples con realce
en anillo tras contraste. Los pacientes afectados suelen pre-
sentar tambin infeccin cutnea o pulmonar. Puede ser
difcil decidir si un hongo dematiceo est implicado o no
en el proceso patolgico, ya que pueden presentarse como
contaminantes de las muestras clnicas.

Evolucin

Tras la biopsia cerebral iniciamos tratamiento con ceftria-


xona, metronidazol, pirimetamina y sulfadiazina (tabla 2)
a la espera de los resultados microbiolgicos y anatomo-
patolgicos. Tras 14 das, la paciente no present mejora
clnica, con empeoramiento radiolgico (crecimiento de la
lesin frontal, sin cambios la occipital). Durante este tiem-
po la paciente comenz con nauseas y vmitos que provo- Foto 2. A y B- RM craneal previa a tratamiento antibitico. C- TC de crneo tras
15 das de tratamiento eficaz. D- TC de crneo tras 25 das de tratamiento eficaz.
caron alteraciones hidroelectrolticas; al atribuir estos sn-
tomas al tratamiento antimicrobiano, se decidi sustituir la
sulfadiazina por clindamicina y suspender el metronidazol, microorganismo se identific posteriormente como Nocar-
con escasa mejora de la intolerancia gastrointestinal. dia asteroides sensu stricto.
El resultado de anatoma patolgica descart tumor cere- Pese a tratamiento antibitico dirigido para Nocardia (cef-
bral y los cultivos fueron negativos. Tras 28 das de tra- triaxona y cotrimoxazol), radiolgicamente la paciente si-
tamiento antibitico se realiz TC craneal sin observarse gue empeorando. Desde microbiologa nos informaron de
mejora; valoramos junto a neurociruga la actitud a seguir, resistencia a ceftriaxona, por lo que la suspendimos y aa-
programndose nueva ciruga estereotxica. Se suspen- dimos imipenem y linezolid al cotrimoxazol. Esto provoc
dieron antibiticos durante una semana para aumentar la aumento de la intolerancia gastrointestinal en la paciente,
rentabilidad de las muestras. Minutos despus de la inter- con afectacin clnica y analtica, vindonos obligados a
vencin nos informaron desde microbiologa de la visua- suspender el tratamiento durante 24 horas, y reiniciarlo
lizacin en el Gram de abundantes formas de Nocardia posteriormente sin cotrimoxazol, no presentando de nue-
(confirmndose das despus con cultivo, foto 1) por lo que vo estos sntomas. En control analtico se observa pancito-
iniciamos tratamiento con ceftriaxona y cotrimoxazol. El penia grave que precis transfusin de 3 concentrados de

60
CAPTULO 4
INFECCIONES DEL SISTEMA
NERVIOSO CENTRAL

Da 0 Tratamiento emprico para Ceftriaxona - Metronidazol - Pirimet-


hemates y oblig a suspender linezolid, aadiendo levo- absceso cerebral. amina - Sulfadiazina
floxacino (tabla 2). Da 21 Toxicidad sulfadiazina. Ceftriaxona - Pirimetamina - Clin-
La paciente sigui evolucionando favorablemente, con me- damicina
jora de los sntomas neurolgicos y disminucin del tama- Da 32 Biopsia. Sin tratamiento
o de las lesiones cerebrales en las pruebas de imagen (foto Da 40 Tratamiento Nocardia. Ceftriaxona - Cotrimoxazol
2). Tras 40 das de tratamiento eficaz segn antibiograma, Da 49 Nocardia resistente a ceftri- Imipenem - Cotrimoxazol - Linezolid
axona.
se decide cambio a monoterapia con levofloxacino va oral
Da 62 Toxicidad cotrimoxazol. Sin tratamiento
y, una vez comprobada tolerancia, se le dio el alta hospi-
Da 63 Reintroduccin tratamiento. Imipenem - Linezolid
talaria.
Da 68 Pancitopenia. Imipenem - Levofloxacino
Tras un mes de seguimiento ambulatorio la paciente se en-
Da 85 Tratamiento al alta. Levofloxacino
cuentra en su domicilio con buena evolucin clnica y ra-
diolgica; tolera tratamiento antibitico oral (levofloxacino Tabla 2. Evolucin en el tratamiento antibitico de la paciente.
500 mg cada 12 horas), y se realizan controles peridicos
en las consultas externas de enfermedades infecciosas. de infeccin (como en nuestra paciente), pudiendo diag-
nosticarse errneamente como una neoplasia primaria o
Diagnostico final metstasis antes de la biopsia.
El diagnstico definitivo de absceso cerebral por Nocardia
Diagnostico principal: Abscesos cerebrales por Nocardia aste- requiere el aislamiento del microorganismo en una mues-
roides sensu stricto resistente a ceftriaxona, en paciente in- tra clnica obtenida por puncin-aspiracin estereotxica.
munodeprimida por tratamiento crnico con corticoides. Incluso cuando se obtienen muestras adecuadas, su aisla-
Otros diagnsticos: Intolerancia a sulfadiazina y cotri- miento y cultivo puede ser difcil, por lo que ante su sospe-
moxazol y Pancitopenia grave por linezolid. cha debemos informar al laboratorio. Las pruebas de sen-
sibilidad se deben realizar en todos los aislamientos dado
Discusin que los patrones de resistencia varan con frecuencia de
unas cepas a otras.
Nocardia asteroides es una bacteria filamentosa, ramificada, No hay ensayos prospectivos aleatorizados que hayan de-
grampositiva y aerobia, de crecimiento intracelular fa- terminado el rgimen antimicrobiano ms eficaz; la serie
cultativo. Se encuentran en la tierra, materia vegetal en ms amplia de casos de abscesos cerebrales por Nocardia
descomposicin, medios acuticos y el aire (partculas de constaba de 11 pacientes, con una mortalidad del 27,3%.
polvo). La inhalacin se considera el modo ms comn de Las recomendaciones de tratamiento se basan en estudios
entrada, aunque tambin puede contagiarse por alimentos retrospectivos, modelos animales y actividad in vitro, con
o heridas contaminadas. sensibilidades variable segn las cepas. El tratamiento em-
El riesgo de infeccin se incrementa en inmunodeprimidos, prico de eleccin sigue siendo el cotrimoxazol. Si el pa-
particularmente en aquellos con defectos en la inmunidad ciente est inmunodeprimido, como es nuestro caso, es
mediada por clulas (trasplantados, tratamiento con glu- aconsejable aadir imipenem, amikacina y/o una cefalos-
cocorticoides, VIH, neoplasia maligna, diabetes mellitus; porina (3 o 4 generacin), estas ltimas de preferencia en
tambin enfermedad pulmonar crnica y alcoholismo). caso de afectacin del SNC. La mayora de especies descri-
Los sitios de infeccin ms frecuentes son: diseminada (2 tas de Nocardia son susceptibles al linezolid; en los estudios
o ms rganos) 32%, pulmonar 39%, SNC 9%, cutnea revisados no se hallaron resistencia in vitro, y la respuesta
o linfocutnea 8%, extrapulmonar 12% (hueso, ojo); tiene clnica a la infeccin fue buena (casos aislados). En nuestra
una alta capacidad para difundir a otros rganos desde su paciente se aisl N. asteroides sensu stricto, tpicamente sensi-
foco de infeccin inicial. ble a cotrimoxazol, cefalosporinas de 3 generacin y ami-
Nocardia tiene especial tropismo por el SNC (afectado en kacina, con resistencia variable a imipenem.
el 44% de las infecciones diseminadas), formando tpica- El tratamiento inicial debe realizarse por va intravenosa
mente abscesos. Los estudios de imagen cerebral se deben durante tres semanas y/o hasta mejora clnica, para pos-
realizar en todos los pacientes con infeccin por Nocardia, teriormente realizar el cambio a va oral. El tratamiento en
incluso los que no tienen sntomas de afectacin del SNC, pacientes con afectacin del SNC debe prolongarse duran-
ya que su afectacin variar la duracin del tratamiento. te al menos un ao, con frecuentes revisiones y controles
En pacientes inmunodeprimidos la clnica de presenta- radiolgicos (TC o RM craneal).
cin habitual es compatible con LOE cerebral sin signos

61
CAPTULO 4
INFECCIONES DEL SISTEMA
NERVIOSO CENTRAL

Bibliografa

1. Brouwer MC, Tunkel AR, McKhann GM, van de


Beek D. Brain abscess. N Engl J Med. 2014; 371: 447-
56.
2. Brouwer MC, Coutinho JM, Beek D. Clinical cha-
racteristics and outcome of brain abscess. Neurology.
2014; 82: 806-13.
3. Arlotti M, Grossi P, Pea F, Tomei G, Vullo V, Rosa FG,
et al. Consensus document on controversial issues for
the treatment of infections of the central nervous sys-
tem: bacterial brain abscesses. Int J Infect Dis.2010;
14: 79-92.
4. Singh N, Husain S. Infections of the central ner-
vous system in transplant recipients. Transpl Infect
Dis.2000; 2: 101-11.
5. Mamelak AN, Obana WG, Flaherty JF, Rosenblum
ML. Nocardial Brain Abscess: Treatment Strategies
and Factors Influencing Outcome. Neurosurgery.
1994; 35: 62231.
6. McNeil MM, Brown JM. The Medically Important
Aerobic Actinomycetes: Epidemiology and Microbio-
logy. Clin Microbiol Rev. 1994; 7: 357-417.
7. Uhde KB,Pathak S,McCullum I,Jannat-Khah
DP,Shadomy SV,Dykewicz CA, et al. Antimicro-
bial-resistant Nocardia isolates, United States, 1995-
2004. Clin Infect Dis.2010; 51: 1445-8.
8. Pea F,Cojutti P,Pagotto A,Cristini F,Furlanut M,Via-
le P. Successful Long-Term Treatment of Cerebral
Nocardiosis with Unexpectedly Low Doses of Line-
zolid in an Immunocompromised Patient Receiving
Complex Polytherapy.Antimicrob Agents Chemother.
2012; 56: 3438-40.
9. Loeffler JM,Bodmer T,Zimmerli W,Leib SL. Nocar-
dial Brain Abscess: Observation of Treatment Strate-
gies and Outcome in Switzerland from 1992 to 1999.
Infection.2001; 29: 337-41.
10. Lin YJ, Yang KY, Ho JT, Lee TC, Wang HC, Su FW.
Nocardial brain abscess. J Clin Neurosci. 2010; 17:
250-3.

62
Varn de 62 con hemiparesia izquierda de
una semana de evolucin
Cantero Nieto, L; Arenas Miras, MM;
Martnez Egea, I; Pasquau Liao, J.
Hospital Universitario Virgen de las Nieves. Granada.

Caso clnico

Nuestro paciente era un varn de 62 aos con anteceden-


tes personales de asma bronquial y ex-fumador hasta hace
8 aos con una carga acumulada de 36 paquetes-ao. Vi-
va en un cortijo donde tena perros, gatos y pjaros.
Estaba en tratamiento con inhaladores de budesonida/
formoterol y montelukast sin presentar alergias farmaco-
lgicas.
Acude a urgencias por cuadro de prdida de fuerza en he-
micuerpo izquierdo de una semana de evolucin junto con
torpeza de mano izquierda y alteracin de la deambula-
cin, agudizada esta clnica el da de su visita a urgencias;
asociando adems episodio autolimitado hace mes y me-
dio de alteracin en la emisin del lenguaje.
Profundizando en la anamnesis, refera haber presentado
haca dos meses cuadro de dolor pleurtico en costado iz-
quierdo, as como tos con expectoracin blanquecina, sin
fiebre asociada ni hemoptisis y no habiendo realizado en
domicilio ningn tratamiento antibitico; clnica que en
el momento del ingreso no presentaba. Relataba adems
prdida de peso de unos 8Kg en el ltimo mes sin otra cl-
nica desde el punto digestivo ni urinario. Tampoco refera
extracciones dentales.
A la exploracin, la tensin arterial era de 133/75 mmHg, Imagen 1. Radiografa de trax al ingreso.
frecuencia cardiaca de 80 latidos/min, temperatura de
36.3C, saturacin de oxgeno basal del 98% y frecuencia llamaba la atencin unos discretos edemas perimaleolares
respiratoria de 16 respiraciones/min. Presentaba un buen +/+++, sin presentar lesiones cutneas.
estado general, estando consciente, orientado y alerta. En
la exploracin neurolgica, el lenguaje era coherente sin Entre las pruebas complementarias realizadas en urgen-
rasgos disrtricos ni disfsicos. Con PINLA, MOEC y cias se encontraban:
campimetra por confrontacin normal. No presentaba
rigidez de nuca ni signos menngeos destacando una dis- Una analtica donde destacaba un hemograma con
minucin de fuerza en el hemicuerpo izquierdo de pre- hemoglobina de 12 gr/dl con 13.200 leucocitos/uL
dominio proximal, siendo de 3/5 en miembro superior (80.2% de neutrfilos y 14.7% de linfocitos) y 391.000
izquierdo y 4/5 en miembro inferior izquierdo. Tena una plaquetas. En la bioqumica, la funcin renal e iones
extincin sensitiva en miembros izquierdos, con una mar- estaban dentro de la normalidad y la coagulacin era
cha no atxica. A la exploracin de la orofaringe destaca- tambin normal, destacando nicamente un fibrin-
ba la ausencia de varias piezas dentales y mltiples caries. geno de 644.7 mg/dl.
En la Auscultacin cardiorrespiratoria solo destacaban al- Se realiz una radiografa de trax (imagen 1) don-
gunos sibilantes en ambos campos pulmonares, con unos de se visualizaba un aumento de densidad en lbulo
tonos cardiacos rtmicos y sin soplos audibles. La explora- superior izquierdo con retraccin hilar y prdida de
cin abdominal era normal y en miembros inferiores solo volumen de dicho pulmn.

63
CAPTULO 4
INFECCIONES DEL SISTEMA
NERVIOSO CENTRAL

En el ECG el ritmo era sinusal con una frecuencia de


70 lpm, sin alteraciones de la repolarizacin.
Finalmente se realiz tambin de urgencias un TAC
craneal (imagen 2) con y sin contraste intravenoso don-
de se aprecia una lesin a nivel frontoparietal derecha
con edema asociado y captacin de contraste en anillo.

Diagnstico diferencial

En el diagnstico diferencial de lesiones cerebrales que


captan contraste en anillo, habra que tener en cuenta las
siguientes causas:

Neoplasias

Primarias o del SNC (supratentoriales): Glioblastomas,


astrocitomas, linfoma del SNC.
Secundarias: Metstasis, a tener en cuenta sobre todo
en este paciente el origen pulmonar, por ser ex-fuma-
dor y la posibilidad de masa tumoral subyacente cau-
sante de la atelectasia de la radiografa de trax. Otras Imagen 2. TAC con/sin contraste iv al ingreso.
neoplasias a tener en cuenta seran el melanoma, cn-
cer de origen gastrointestinal o carcinoma renal.

Infecciones

Absceso cerebral, por contigidad o por diseminacin


hematgena (dental, endocarditis, pulmonar).
Toxoplasmosis (ms frecuente en pacientes inmunode-
primidos)

Evolucin

Despus de los hallazgos en Urgencias, con la clnica neu-


rolgica establecida y la lesin objetivada en el TAC se de-
cidi ingreso a cargo de Neurociruga para estudio.
Al da siguiente del ingreso se procedi a realizacin de
RMN (imagen 3) con objetivo de dilucidar el origen de la
LOE cerebral, donde se describa una lesin redondea-
da de 2 x 2,1 x 2,5 cm localizada en las reas posteriores
del centro semioval derecho, rodeada de extenso edema Imagen 3. RMN en T2 al da siguiente del ingreso.
perilesional y que presenta una pared irregular de baja se-
al en las secuencias T2, con un centro que presenta baja tracin de CIV presenta realce intenso de su pared delimi-
seal en T1, intermedia la secuencia FlairT2 y alta en la tndose con claridad ciertas irregularidades de la misma.
secuencia spin eco potenciada en T2. La lesin presenta No se observan otras zonas de realce patolgico intracra-
restriccin de difusin y un patrn de metabolitos en el neal. Los hallazgos descritos, salvo las irregularidades de la
estudio espectroscopia en el que destaca la presencia de un pared, sugieren un probable absceso cerebral.
gran pico de cido lctico y ligera elevacin de colina. El Tras dichos hallazgos se realiz un drenaje de la lesin por
estudio de perfusin no muestra hallazgos que sugieran la puncin estereotxica, cuyo material fue enviado a micro-
presencia de angiognesis neoformativa. Tras la adminis- biologa para su cultivo y se inici tratamiento antibitico

64
CAPTULO 4
INFECCIONES DEL SISTEMA
NERVIOSO CENTRAL

emprico con Vancomicina a dosis de 1,5gr cada 12 horas


iv. , Metronidazol 500 mg cada 12 horas iv y cefotaxima
2gr/8 horas iv.
El segundo da su estancia el planta el paciente present
pico febril de 38.2C por lo se extrajeron hemocultivos ve-
nosos y se ampli el estudio en espera de los resultados del
drenaje, solicitando una analtica general con marcadores
tumorales, un ecocardiograma y un TAC toraco-abdomi-
no-plvico.
En la analtica general solo destacaba una VSG de 9 mm,
con 17.700 leucocitos/uL, 90,2% de PMN y PCR de 0,6
mg/dl. Los marcadores tumorales (CEA 1,2, AFP 1,7, Ca
125, Ca 72,4, Ca 19,9, NSE, PSA y S100) fueron norma-
les. En el ecocardiograma no haba signos de endocarditis
y el TAC toraco-abdomino-plvico (imagen 4) fue infor-
mado como lesin que afecta al lbulo superior izquierdo
con un patrn alveolar y con la presencia de reas de ne-
crosis y comunicacin con el rbol bronquial en su interior.
Existe afectacin pleural y componente atelectsico. Se ob-
servan pequeas adenopatas en mediastino superior, ven-
tana aorto-pulmonar y para-traqueales derechas. A nivel
abdominal no se aprecian alteraciones significativas salvo
la existencia de herniacin inguinal izquierda. Conclusin: Imagen 4. Imagen de TAC torcico.
la lesin torcica debe tener una etiologa inflamatoria /
infecciosa no pudiendo descartar radicalmente la posibili- Discusin
dad de patologa tumoral
Tras el inicio del tratamiento antibitico fue remitiendo Los abscesos cerebrales comprenden un conjunto de infec-
la clnica neurolgica con una RMN craneal de control ciones graves que son ms frecuentes en pacientes inmu-
donde se fue reduciendo el tamao del absceso y eviden- nodeprimidos, o en pacientes a los que se realiza procedi-
cindose una pequea coleccin supratentorial izquierda, mientos invasivos craneales como ciruga, traumatismos,
que estaba presente en el estudio previo y que tambin ha- sinusitis, otitis con mastoiditis por contigidad, siendo estos
ba mostrado una reduccin de su tamao. Se recibieron focos los ms frecuentes en el 20-60% de los casos.
los resultados microbiolgicos, creciendo en el cultivo de La etiologa por diseminacin hematgena, como en el
absceso cerebral un Streptococcus intermedius sensible a peni- caso de nuestro paciente, tiene como orgenes ms fre-
cilina (CMI menor 0,064), cefotaxima (CMI menor 0,064), cuentes las infecciones respiratorias crnicas como un abs-
vancomicina (CMI 1), y sensible a clindamicina, linezolid y ceso pulmonar o un empiema, sobre todo en pacientes con
daptomicina. El hemocultivo venoso fue negativo. bronquiectasias o fibrosis qustica; infecciones cutneas,
Adems se realiz control radiolgico de la lesin pulmo- plvicas, intraabdominales, secundarios a dilatacin esof-
nar mediante TAC de trax que mostr una resolucin gica o esclerosis endoscpica de varices, cardiopatas con-
prcticamente completa de la ocupacin alveolar descrita, gnitas o shunt intrapulmonares y endocarditis infecciosa.
con presencia de atelectasia fibrtica o cicatricial del l- Los microorganismos ms frecuentemente encontrados
bulo superior izquierdo con bronquiectasias en su interior. son Staphylococcus spp y Streptococcus spp.
En cuanto a las manifestaciones clnicas, suelen ser ines-
Diagnstico final pecficas y de curso subagudo, por lo que es frecuente el
retraso diagnstico. El sntoma de presentacin principal
Abscesos cerebrales por Streptococcus intermedius secundarios es la cefalea en el 69% de los casos (aunque este sntoma
a neumona cavitada. suele ser un motivo de consulta frecuente); presentando
fiebre nicamente del 43 al 53% de los casos. Los dficit
neurolgicos focales se presentan en el 50% das o semanas
despus del inicio de la cefalea.

65
CAPTULO 4
INFECCIONES DEL SISTEMA
NERVIOSO CENTRAL

En cuanto al diagnostico, a parte de las pruebas de imagen Neurosurg Focus. 2008; 24 (6): E4.
como TAC craneal y sobre todo RMN craneal por su ma- 3. Brouwer MC, Tunkel AR, McKhann GM et al. Brain
yor especificidad, es necesario contactar con neurociruga Abcess. N Engl J Med 2014;371:447-56. doi: 10.1056/
para realizacin de una toma de muestra antes de iniciar el NEJMra1301635.
tratamiento antibitico emprico si es posible. Actualmente 4. Helweg-Larson J, Astradsson A, Richhall H. et al. Pyo-
la ciruga estereotxica est dando buenos resultados con genic brain abscess, a 15 year survey. BMC infect Dis.
menores secuelas que la extirpacin del absceso. Habra 2012; 12: 332.
sin embargo, situaciones en las que la necesidad de aspi-
racin podra obviarse, como en el caso de abscesos que
ocurren en el contexto de una bacteriemia, en cuyo caso
el tratamiento ira dirigido al germen aislado en el hemo-
cultivo. En otras ocasiones, el drenaje puede ser retrasado
o no requerido, como en el caso de cerebritis sin presencia
de necrosis cerebral o bien abscesos localizados en regiones
vitales o inaccesibles. La aspiracin siempre ser preferida
a la extirpacin quirrgica del absceso por las menores se-
cuelas neurolgicas de la primera. En la toma de muestras,
es importante enviar muestras para estudio de aerobios y
anaerobios, hongos y Mycobacterium tuberculosis. Adems, en
pacientes inmunocomprometidos, habr que valorar la po-
sibilidad de Toxoplasma gondii, Listeria y Nocardia.
Respecto al tratamiento antibitico emprico, lo deseable
es que se gue por la etiologa ms probable y por el re-
sultado del Gram si est disponible. Un aceptable trata-
miento emprico, hasta resultados microbiolgicos, sera el
inicio de metronidazol (dosis de carga de 1gr iv seguida de
7,5 mg/Kg cada 8 horas iv) junto con cefotaxima (2 gr iv
cada 4-6 horas). Para cobertura emprica S. aureus meti-
cilin-resistente (SAMR) se podra iniciar tratamiento con
vancomicina (15-20 mg/Kg iv cada 8-12 horas), habiendo
autores que han utilizado con buenos resultados frmacos
como linezolid, daptomicina o cotrimoxazol. La duracin
del tratamiento antibitico es prolongada, generalmente
hasta 8 semanas, pudiendo reducirse la duracin del mis-
mo a 4-6 semanas si se ha realizado drenaje del mismo y
con controles mediante TAC cada 10-15 das.
Es importante en casos como ste, plantear todas las po-
sibilidades diagnsticas durante el estudio, incluyendo las
neoplasias pulmonares al tratarse de un paciente ex-fuma-
dor con una lesin cavitada pulmonar, as como estar en
contacto con el servicio de neurociruga para el manejo
ptimo del paciente.

Bibliografa

1. Brouwer MC, Coutinho JM, Van de Beek D. Clinical


characteristics and outcome of brain abscess: systema-
tic review and meta-analysis. Neurology. 2014; 82:806-
13.
2. Hakan T. Management of bacterial brain abscesses.

66
Sndrome confusional agudo
y crisis comicial en varn de 47 aos
Snchez Bern, I; Fernndez-Roldn, C;
Moya Roldn, S; Hernndez Quero, J.
Hospitales Universitarios Virgen de las Nieves y San Cecilio. Granada.

Caso clnico orotraqueal, conexin a ventilacin mecnica e ingreso en


UCI, donde precisa adems de empleo de drogas vasoac-
Hombre de 47 aos sin antecedentes personales de inters tivas por inestabilidad hemodinmica (tensin arterial sis-
que llega al Servicio de Urgencias en transporte sanitario, tlica 80 mmHg).
sin acompaantes. Presenta un sndrome confusional agu- Durante su estancia en UCI el paciente recobr la esta-
do acompaado de fiebre de 38C, no pudiendo establecer bilidad hemodinmica con el empleo de noradrenalina,
una cronologa de los sntomas por la situacin neurolgi- tolerando la progresiva retirada de la misma hasta su sus-
ca del paciente. En la exploracin neurolgica destacaba la pensin. De igual modo se produjo una mejora progresiva
mirada con nistagmo hacia la derecha e imposibilidad para del nivel de consciencia, que permiti retirar la ventilacin
mirar a la izquierda, as como pupilas midriticas medias mecnica y extubar, con progresiva mejora del nivel cog-
hiporreactivas, sin meningismo, y sin signos de hiperten- nitivo hasta alcanzar Glasgow 15. Se mantuvo la pauta an-
sin intracraneal, con reflejo cutneo-plantar indiferente, tibitica iniciada en Urgencias y se inici tratamiento con
y sin otras alteraciones de inters. En el resto de la explora- fenitoina.
cin fsica, cardiorrespiratoria, abdominal y de miembros,
no exista ninguna alteracin significativa en el momento Diagnstico diferencial
de la valoracin inicial. Ante la sospecha de infeccin del
sistema nervioso central se inici tratamiento antibitico Ante un paciente con un sndrome confusional agudo, fo-
emprico con ceftriaxona 2g cada 24 horas y aciclovir 800 calidad motora y fiebre el primer diagnstico a considerar,
mg cada 8 horas. por su gravedad e implicacin pronstica, es la existencia
La analtica urgente solicitada a la llegada el paciente a Ur- de una infeccin del sistema nervioso central, fundamen-
gencias destacaba la leucocitosis de 21.650 clulas/L con talmente del tipo encefalitis. En nuestro medio, en pacien-
80% neutrofilia, con normalidad de las otras dos series del tes inmunocompetentes, destacan las encefalitis virales,
hemograma. En la bioqumica presentaba una glucemia al siendo la causa ms frecuente la secundaria a enterovirus,
azar de 169 mg/dL, con funcin renal e iones normales, y la de mayor morbimortalidad la infeccin por virus her-
as como PCR 2,5 mg/dL. Coagulacin sin alteraciones. pes simple tipo 1, no pudiendo descartar otros agentes vi-
Gasometra venosa con pH 7.31, pCO2 36 mmHg, pO2 rales como el virus Toscana, virus de la varicela-zster, etc.
67 mmHg, HCO3 18.1 mEq/L, y lactato 5.2 mmol/L. Existen otras causas destacables de meningoencefalitis con
Determinacin de txicos en sangre, orina y jugo gstrico lquido claro, como pueden ser algunas infecciones bacte-
negativos. Se realiz un TAC craneal donde no se detecta- rianas (Mycobacterium tuberculosis, Treponema pallidum, etc) o
ron alteraciones significativas. fngicas, o bien causas no infecciosas como son las para-
Con estos resultados se procedi a la realizacin de una neoplsicas, secundarias a frmacos, trombosis venosas de
puncin lumbar, con salida de un lquido cefalorraqudeo senos subdurales, o las asociadas a enfermedades sistmi-
claro, con presin normal, con los siguientes datos en la cas inmunomediadas (lupus eritematoso sistmico, enfer-
citoqumica: protenas 133 mg/dL, glucosa 77 mg/dL, medad de Behet, sarcoidosis, etc) o vasculitis.
leucocitos 77 con 94% mononucleares, 3 hemates.
Estando an en Urgencias, el paciente presenta empeora- Evolucin
miento brusco del estado general por lo que se traslada a
Sala de Crticos, donde sufre una crisis parcial tnico-cl- Dada la evolucin satisfactoria del paciente en UCI, sin
nica en miembro superior izquierdo con generalizacin recidiva de la actividad convulsiva, se traslad a planta de
secundaria, que cedi tras la administracin de 10 mg de hospitalizacin de la Unidad de Enfermedades Infeccio-
diazepam intravenoso. Tras el episodio comicial, en situa- sas. En la exploracin neurolgica detallada realizada en
cin de coma con estado postcrtico, presenta 6 puntos ese momento destacaba la existencia de una anisocoria,
en la escala de Glasgow por lo que se decide intubacin con pupila derecha midritica, escasamente reactiva a la

67
CAPTULO 4
INFECCIONES DEL SISTEMA
NERVIOSO CENTRAL

luz y moderadamente a la acomodacin, con resto de pa-


res craneales normales; persista cierta paresia 2/5 de la
musculatura extensora de mano derecha, estando afecto
el supinador de dicho lado con preservacin del triceps
y siendo normales los msculos dependientes de cubital
y mediano derechos; reflejos osteotendinosos vivos; refle-
jo cutneo-plantar flexor bilateral; sensibilidad vibratoria
algo disminuida en tobillos, dismetra dedo-nariz bilateral
moderada, as como marcha moderadamente atxica, con
apertura de base de sustentacin, siendo la marcha en tn-
dem imposible sin ayuda; Romberg negativo. El resto de
la exploracin fsica no mostraba ninguna alteracin del
inters.
Durante su estancia en planta se solicit ecocardiografa
transtorcica, sin alteraciones estructurales significativas
ni signos de endocarditis infecciosa. Hemocultivos veno-
sos extrados a su llegada a Urgencias negativos. Se reali-
z igualmente una resonancia magntica nuclear cerebral
con captacin leptomenngea de contraste, sin alteraciones
parenquimatosas; en la angio-resonancia del polgono de
Willis se objetiv la existencia de un aneurisma sacular de
3mm a nivel de la arteria cerebral media izquierda (Figuras
1 y 2). El estudio electroencefalogrfico mostraba una ac-
tividad focal theta rtmica intermitente temporal derecha.
El estudio microbiolgico solicitado en Urgencias del lqui-
do cefalorraqudeo fue negativo, tanto el cultivo habitual
como la reaccin en cadena de la polimerasa (PCR) de
virus herpes simple (VHS) 1 y 2, enterovirus y virus vari-
cela-zoster.
Adems, se solicit serologa de les con el siguiente re-
sultado: RPR 1/128, sfilis Ig G + Ig M positivo, y FTA-
ABS positivo. Ante estos hallazgos se realiz nueva pun-
cin lumbar con persistencia de la hiperproteinorraquia y
pleocitosis con predominio de mononucleares, negatividad
de nueva PCR de VHS, y deteccin de RPR positiva en
lquido cefalorraqudeo.
Con el diagnstico de neuroles, dado que el paciente era
alrgico a penicilina y no haba opcin de realizar tera-
pia desensibilizadora, se mantuvo el tratamiento con cef- Figuras 1 y 2. Angio RNM que muestra aneurisma sacular de arteria cerebral
media izquierda de 3 mm.
triaxona durante 14 das, con mejora clnica marcada del
paciente, con normalizacin pupilar, si paresias residuales
ni nuevos episodios convulsivos, motivo por el que se pro- Discusin
cedi al alta hospitalaria con seguimiento estrecho en con-
sulta para monitorizacin de respuesta al tratamiento. La afectacin del sistema nervioso central por la sfilis se
denomina neuroles. La invasin neurolgica por las espi-
Diagnstico final roquetas se produce por va hemtica de forma precoz en
hasta el 80% de los pacientes infectados de les, la mayora
Neuroles: meningitis sifiltica versus sfilis meningovascular. de los cuales aclaran la bacteria del lquido cefalorraqu-
deo sin tratamiento antibitico. De este modo, el 20% de
los pacientes infectados por les desarrollan una neuros-

68
CAPTULO 4
INFECCIONES DEL SISTEMA
NERVIOSO CENTRAL

filis asintomtica, un pequeo porcentaje de los cuales de- del paciente. Se recomienda una monitorizacin mediante
sarrollan manifestaciones clnicas (en la era preantibitica examen neurolgico y puncin lumbar entre 3 y 6 meses
este porcentaje ascenda hasta el 6%, siendo en la actuali- tras el tratamiento para valorar evolucin citoqumica y
dad mucho ms infrecuentes). serolgica en el lquido cefalorraqudeo hasta la negativi-
Las manifestaciones neurolgicas de la neurosfilis pueden zacin de los mismos, previndose la primera en los 6 me-
aparecer en cualquier fase de la enfermedad, motivo por el ses iniciales y la segunda a lo largo de los 2 aos siguientes,
que en 2005 los Centers for Disease Control (CDC) eliminaron recomendndose pautas de retratamiento en caso de que
los conceptos de les secundaria y terciaria en la afecta- no se consigan estos resultados.
cin neurolgica y propugnaron la distincin entre formas Como conclusin debemos destacar la importancia de
precoces y tardas. Entre las manifestaciones de la neuros- iniciar tratamiento antibitico emprico precoz ante cual-
filis precoz se distingue: neuroles asintomtica, que suele quier caso de infeccin del sistema nervioso central, siendo
detectarse en fases iniciales de la infeccin ante alteracio- en este paciente fundamental considerar la etiologa her-
nes citoqumicas y test no treponmicos positivo en LCR; ptica por el mal pronstico asociado a la misma, sin poder
meningitis sifiltica, donde las alteraciones citoqumicas y descartar otras causas menos frecuentes, como la sfilis. De
microbiolgicas se asocian a un sndrome menngeo; sfilis igual modo un correcto diagnstico etiolgico nos permi-
ocular, con afectacin de cualquier estructura ocular aso- tirn establecer una pauta teraputica ptima y un segui-
ciada o no a meningitis; otosfilis, afectacin de cualquier mento mucho ms exhaustivo del paciente.
estructura tica asociada o no a meningitis; y sfilis menin-
govascular, donde se produce una arteritis infecciosa aso- Bibliografa
ciada al cuadro menngeo y que ocasiona trombosis local y
accidente cerebrovascular secundario. Por otra parte, entre 1. Huang C, Morse D, Slater B, Anand M, Tobin E, Smi-
las formas tardas se encuentra: parlisis general, consistente th P et al. Multiple-year experience in the diagnosis of
en una demencia rpidamente progresiva; y tabes dorsal, viral central nervous system infections with a panel of
donde se asocia ataxia sensitiva por afectacin de cordones polymerase chain reaction assays for detection of 11
posteriores medulares y dolor neuroptico generalizado. viruses. Clin Infect Dis 2004;39:630-5.
Es importante destacar, tal y como ocurri en nuestro pa- 2. Vinuesa Garca D, Muoz Medina L, Pea Monje A,
ciente, la tpica afectacin oftalmolgica que puede apa- y Hernndez Quero J. Protocolo diagnstico y trata-
recer en la neurosfilis: la pupila de Argyll Robertson, que miento emprico de las encefalitis. Medicine 2006; 9:
consiste en la abolicin del reflejo fotomotor directo y con- 3545-3548.
sensuado manteniendo el reflejo de acomodacin; si bien 3. Tinoco Racero I, Prez Cano R, Martn Aspas A.
podemos encontrarla en otras enfermedades, su presencia Encefalitis. Concepto: Clasificacin. Etiopatogenia.
en un paciente como el nuestro es bastante sugerente de Clnicas. Pronstico. Terapticas. Medicine 2003; 8:
enfermedad sifiltica. 5242-5246.
Para el diagnstico de neuroles, se recomienda la reali- 4. Radolf JD, Tramont EC, Salazar JC. Syphilis (Tre-
zacin de puncin lumbar a todos los pacientes diagnos- ponema pallidum). En: Mandell, Douglas, Bennetts,
ticados de sfilis que presentan sntomas neurolgicos. Es Editores. Principles and Practice of Infectious Disea-
tpico encontrar un lquido cefalorraqudeo con aumento ses. 8 ed. Elsevier; 2015. P.2684-2709.
del nmero de clulas y protenas, asociados a un test no 5. Ghanem KG. Review: Neurosyphilis: A historical
treponmico (VDRL o RPR) positivo. Los hallazgos radio- perspective and review. CNS Neurosci Ther 2010;
lgicos suelen ser bastante inespecficos. 16:e157-68.
La pauta antibitica de eleccin en el tratamiento de la 6. Workowski KA, Berman S; Centers for Disease Con-
neurosfilis, tanto precoz como tarda, es la penicilina G trol and Prevention (CDC). Sexually transmitted di-
18-24MU diarias repartidas en 6 dosis, o bien en infusin seases treatment guidelines, 2010. MMWR Recomm
continua durante 10 a 14 das. En pacientes alrgicos a Rep 2010;59(RR-12):1-110.
penicilina, estara indicado realizar pauta de desensibiliza- 7. Marra CM. Neurosyphilis. [Monografa en Internet].
cin por personal experto en Alergologa y, en caso de no Walthman (MA): UpToDate 2015 Marzo [acceso 2 de
ser posible, como era nuestro caso, se realiza tratamiento mayo de 2015]. Disponible en: http://www.uptodate.
con ceftriaxona 2g cada 24 horas tambin durante 10 a com/
14 das, habiendo menos experiencia con esta pauta por
lo que se suele recomendar una vigilancia ms estrecha

69
CAPTULO 5
INFECCIONES DE PIEL,
PARTES BLANDAS Y PI DIABTICO
Dolor, lesiones purpricas y necrosis
en miembros inferiores tras ciruga
por hernia discal lumbar
Moreno Rosauro, JM; Sillero Herrera, A;
Garca Romero, P; Corzo Delgado, JE.
Hospital Universitario de Valme. Sevilla

Caso clnico

Paciente varn de 51 aos con antecedentes de Vasculitis


de Takayasu y criterios analticos de sndrome antifosfol-
pido en tratamiento con prednisona 5 mg/da y Azatiopri-
na 150 mg/da que ingresa en nuestro hospital por una
lumbociatalgia izquierda refractaria a tratamiento mdico
secundaria a una hernia discal L5-S1 decidindose inter-
vencin por Traumatologa que realiza una hemilaminec-
toma izquierda L5 con foraminotoma S1 y discectoma
L5-S1.
A las 72 horas de la ciruga el paciente comienza con dolor
severo y tumefaccin de miembro inferior izquierdo distal
a la rodilla, objetivndose lesiones cutneas pretibiales de Imagen 1. Primera cura quirrgica de la lcera.
coloracin violcea y algunas lesiones aisladas de aspecto
equimtico. El cuadro se hizo extensivo a la pierna con- mentalmente dos posibles orgenes, uno infeccioso y otro
tralateral donde desarroll tambin lesiones de aspecto autoinmune. El paciente presenta un contexto de autoin-
purprico sin antecedente traumtico que evolucionaron munidad con una vasculitis de grandes vasos (Takayasu).
a placas necrticas y flictenas. Al mismo tiempo el pacien- En vista del deterioro del estado general, las alteraciones
te comenz con un exudado sero-sanguinolento a nivel de analticas (anemia, trombopenia, coagulopata, fallo renal)
la herida quirrgica as como un deterioro progresivo del y las lesiones purpricas habra que descartar la presencia
estado general con hipotensin (90/60 mmHg), oliguria y de un sndrome hemoltico-urmico (SHU) o una prpura
deterioro analtico con insuficiencia renal aguda (creati- trombtica Trombocitopnica (PTT) as como una afecta-
nina 1.7 mg/dl), plaquetopenia (70.000 plaquetas/mm3), cin vascultica de pequeo vaso (vasculitis de hipersensi-
anemia (hemoglobina 10 mg/dl), coagulopata y elevacin bilidad o poliangeitis microscpica).
de reactantes de fase aguda (protena C reactiva 244 mg/L Tambin habra que descartar la presencia de trombosis
y procalcitonina 11.81 ng/ml). Se solicit una valoracin venosa profunda (TVP) dado que el paciente presenta cri-
por Ciruga Vascular, se realizaron hemocultivos y cultivos terios analticos de un sndrome antifosfolpido.
de la herida quirrgica y se inici cobertura con antibiote- De forma alternativa habra que pensar en un origen in-
rapia de amplio espectro con cefepime y linezolid a las 24 feccioso del cuadro pudindose tratar en ese caso de una
horas de inicio de los sntomas, decidindose aumentar la sepsis grave con fracaso multiorgnico secundaria a una
cobertura a imipenem y linezolid tras 24 horas del inicio infeccin de la herida quirrgica, que presenta un exudado
del tratamiento ante la trpida evolucin del cuadro. En en el que se detecta un BGN, pudiendo estar la lesiones en
sendos aspirados de la lesin cutnea y de la herida quirr- miembros inferiores causadas por una afectacin de partes
gica se detect un bacilo Gram negativo (BGN). blandas a distancia.

Diagnstico diferencial Evolucin

Nos encontramos ante un paciente con un deterioro del La evolucin clnica del paciente fue trpida precisando
estado general con una intervencin lumbar reciente y una una limpieza quirrgica urgente de la herida lumbar a las
aumento de tamao miembros inferiores junto con lesio- 72 horas del inicio de los sntomas as como posterior in-
nes equimticas, purpricas y evolutivamente con zonas greso en UCI con necesidad de soporte hemodinmico.
necrticas. Ante estos hallazgos nos planteamos funda- Fue valorado por Ciruga Vascular que descart la pre-

72
CAPTULO 5
INFECCIONES DE PIEL,
PARTES BLANDAS Y PI DIABTICO

sencia de TVP, se descart analticamente la presencia de


hemlisis (haptoglobina normal, test coombs negativo y
frotis sin esquistocitos) y creci en todos los cultivos (he-
mocultivos, cultivo de la herida quirrgica y de las flictenas
del miembro inferior izquierdo) una Pseudomonas aeruginosa
multisensible. Con tratamiento antibitico dirigido y el res-
to de medidas el paciente present una clara mejora con
resolucin del cuadro sistmico y desaparicin del exuda-
do de la herida quirrgica y de las lesiones del miembro
inferior derecho. Sin embargo tras su vuelta a la planta de
Enfermedades Infecciosas las lesiones de la pierna izquier-
da aumentaron de tamao con extensas reas de necrosis y
cultivos repetidos positivos para Pseudomonas aeruginosa con
el mismo perfil de resistencia. Tras ms de 15 das de trata-
miento antibitico dirigido sin cambios sustanciales en las
lesiones de la pierna se decidi realizar un desbridamiento
quirrgico intensivo de las lesiones dejando al descubier-
to gran prdida de sustancia (imagen 1), requiriendo dos
nuevos desbridamientos en quirfano. En vista de la pr-
dida de sustancia y de la insuficiente respuesta a las curas
en planta se decidi el inicio de una terapia con sistema
de vaco cerrado (terapia VAC) de todas las lesiones (ima-
gen 2). En dos semanas con la terapia VAC y continuando
con la antibioterapia dirigida se negativizaron los cultivos
de la herida y apareci abundante tejido de granulacin Imagen 2. Terapia VAC sobre la lcera.
(imagen 3). Finalmente se valor por el Servicio de Ciruga
Plstica del Hospital de referencia y se traslad, tras tres
meses de hospitalizacin, al paciente para realizacin de
un autoinjerto cutneo con buena evolucin posterior.

Diagnstico final

1. Sepsis grave con fallo multiorgnico secundario a in-


feccin precoz de herida quirrgica por Pseudomonas
aeruginosa en paciente inmunodeprimido.
2. Fascitis necrotizante en miembro inferior izquierdo
por implantacin metastsica de Pseudomonas aeruginosa.

Discusin

La fascitis necrotizante es una infeccin rara pero fatal,


caracterizada por la rpida progresin de la necrosis de
la fascia, piel, tejido blando y msculo, en pared abdomi-
nal, extremidades y zona perineal ms frecuentemente (1).
Tambin se ha visto este tipo de infeccin en la zona facial,
siendo esta presentacin mucho ms rara. (9)
Lo ms comn es una infeccin polimicrobiana, hallndose
en cultivo cocos gram positivos como estreptococo grupo
A, Staphylococcus aureus; enterococos y enterobacterias gram
negativas. Tambin microorganismos como Clostridium, Vi- Imagen 3. ltima cura.

73
CAPTULO 5
INFECCIONES DE PIEL,
PARTES BLANDAS Y PI DIABTICO

brio vulnificus y Bacteroides fragilis, Klebsiella, E. coli han sido de aumentar la mortalidad un 35-53%, y un retraso de 3
implicados, as como Pseudomonas aeruginosa (1). Sin embar- das supone una mortalidad del 100%. En algunos casos
go, la infeccin monomicrobiana por Pseudomonas causante se ha sustituido el desbridamiento radical por drenaje e
de fascitis necrotizante es muy rara (2). incisin, siendo tambin efectivo y seguro, sobre todo en
Se describen como factores predisponentes estados de in- pacientes con situacin de shock sptico complicado y tras-
munosupresin, como neoplasia maligna en tratamiento tornos de la coagulacin (2).
con o sin neutropenia, muy frecuentemente hematolgica, La tasa de mortalidad reportada en la literatura es muy
diabetes mellitus, alcoholismo, desnutricin e infancia. (2) variable 9-70%, en funcin del diagnostico e instauracin
(3) de medidas teraputicas precoces.
Tambin se han visto otros factores como obesidad (4) y
ciruga o traumatismo previo como puerta de entrada del Bibliografa
microorganismo (a veces rasguos triviales) (5) (6). Se han
descrito casos de asociacin con toma de anti-inflamatorios 1. Zhao GJ, Hong GL, Liu JQ , Lu Y, Lu ZQ. Septic
no esteroideos, no como factor que aumente el riesgo, pero shock due to community-acquired Pseudomonas aeru-
s que su uso puede enmascarar los sntomas y retrasar el ginosa necrotizing fasciitis: A case report and literature
diagnostico (8). La adquisicin de Pseudomonas aeruginosa se review. Exp Ther Med 2014;7:1545-8.
ha relacionado con baos en agua contaminada, contacto 2. Emmett C, Kane G. Necrotising fasciitis caused by P
con ambientes hmedos como fregaderos, baeras, etc (7). aeruginosa in a male patient with chronic lymphocytic
Las manifestaciones clnicas incluyen fiebre, dolor, edema, leukaemia. BMJ Case Rep 2013;2013. doi: 10.1136/
eritema, ampollas hemorrgicas y finalmente necrosis de bcr-2012-008133
la piel y fascia que conduce a sepsis. La localizacin ms 3. Ahmed S, Ali SR, Samani ZA. Pseudomonas Ne-
frecuente fue la regin inguinal en el que la infeccin pro- crotizing Fasciitis in an Otherwise Healthy In-
paga a menudo hasta la pared abdominal y debajo de la fant. Case Rep Infect Dis 2012;2012:517135. doi:
regin femoral hasta el nivel de la rodilla (5). Se ha visto 10.1155/2012/517135
un aumento de incidencia de inflamacin retroperitoneal 4. Czymek R, Frank P, Limmer S, Schmidt A, Jungbluth
y cavidad abdominal predominantemente en mujeres, atri- T, Roblick U et al. Fournier's gangrene: is the fema-
buido a su anatoma genital (8). le gender a risk factor? Langenbecks Arch Surg 2010;
Las complicaciones graves de la fascitis necrotizante, in- 395: 173-80.
cluyendo shock sptico y fallo multiorgnico, requieren de 5. Arifi HM, Duci SB, Zatriqi VK, Ahmeti HR, Ismajli
medidas activas de reanimacin, terapia antimicrobiana VH, Gashi MM, et al. A retrospective study of 22 pa-
y desbridamiento quirrgico agresivo temprano (1). Para tients with necrotising fasciitis treated at the University
ello es necesario un diagnstico precoz del cuadro. La re- Clinical Center of Kosovo (20052010). Int Wound J
sonancia magntica es la tcnica que mayor sensibilidad 2013; 10: 461-5.
aporta para este tipo de infeccin (3), aunque de ninguna 6. Saw A, Kwan MK, Sengupta S. Necrotising fasciitis:
manera habra que esperar a la confirmacin del diagns- a life-threatening complication of acupuncture in a
tico para instaurar el tratamiento. patient with diabetes mellitus. Singapore Med J 2004;
En cuanto al tratamiento antibitico debe de establecerse 45: 180-2.
prontamente (menos de 1 hora) empezando por antibi- 7. Kappers MH, van der Klooster JM, Ouwendijk RJ,
ticos empricos de amplio espectro en combinacin (me- Dees A. Community-acquired necrotizing fasciitis
ropenem/imipenem, linezolid, piperacilina/tazobactam, due to Pseudomonas aeruginosa. Intensive Care Med.
cefalosporinas de tercera generacin, aminoglucsidos), 2006; 32: 1093-4.
ajustando posteriormente segn resultados del antibiogra- 8. Souyri C, Olivier P, Grolleau S, Lapeyre-Mestre M.
ma. Tener en cuenta si en las medidas para el fallo mul- Severe necrotizing soft-tissue infections and nonste-
tiorgnico se ha instaurado una terapia de reemplazo renal roidal anti-inflammatory drugs. Clin Exp Dermatol
continua, pues muchos frmacos antimicrobianos se elimi- 2008;33: 249-55.
nan por estas terapias, por lo tanto sera necesario en esos 9. Rodrguez-Gonzlez F, Marrero-Saavedra D, Rut-
casos monitorizar niveles (2). En algunos casos tambin se lln-Civit J, Cabrera-Vargas E, Martnez-Quintana E.
ha aadido al tratamiento dosis elevadas de corticoides (9). Ocular necrotizing fasciitis due to Pseudomonas aeru-
Tiene mucha importancia en esta infeccin el tratamiento ginosa in a non-neutropenic patient. Saudi J Ophthal-
quirrgico precoz, retrasar la ciruga ms de 24 horas pue- mol 2013; 27: 281-82.

74
Mujer de 69 aos con celulitis ampollosa
Snchez-Bern, I; Fernndez-Roldn, C;
Javier-Martnez, R; Lpez-Ruz, MA.
Hospital Universitario Virgen de las Nieves. Granada.

Caso clnico

Mujer de 69 aos de edad, con antecedentes personales de


hipertensin arterial, poliartrosis y tuberculosis pulmonar
en la infancia, no fumadora ni bebedora, y sin otros hbi-
tos txicos. En tratamiento habitual con furosemida 40 mg
cada 24 horas, atenolol 25 mg cada 24 horas, enalapril 10
mg cada 24 horas y analgsicos de forma ocasional. Vive
en rea urbana, sin antecedentes de viajes, contacto con
animales, consumo de alimentos caseros o lcteos no higie-
nizados ni relaciones sexuales de riesgo.
Refiere la aparicin hace 10 das de una placa numular
enrojecida, dolorosa y con aumento de temperatura local
de unos 2 centmetros de dimetro mayor en regin preti-
bial izquierda, motivo por el que consult en Atencin Pri-
maria donde le prescribieron tratamiento con cloxacilina
500 mg cada 6 horas por va oral durante 7 das. La evolu-
cin clnica, pese al tratamiento indicado, no fue satisfac-
toria, producindose un aumento progresivo de la lesin
hasta extenderse por toda la pierna (de tobillo a rodilla),
con aparicin de pequeas pstulas subcentimtricas sat-
lites dispersas por todo el contorno de la lesin principal.
En los 3 ltimos das, habiendo cumplido la antibioterapia
prescrita por su mdico, las pstulas aumentaron de ta-
mao y evolucionaron hasta hacerse vesculas a tensin de
aspecto necrohemorrgico, motivo por el que la paciente
decide consultar en el Servicio de Urgencias Hospitalarias.
En la anamnesis dirigida por rganos y aparatos que se
realiza a la paciente a su llegada al Servicio de Urgencias
negaba antecedente traumtico local, picadura o lesin Figura 1. Celulitis ampollosa en miembro inferior izquierdo. A: regin pretibial; B:
regin gemelar.
cutnea local con anterioridad al episodio actual. Por otra
parte, destacaba la asociacin en los 3 das previos de febr-
cula de hasta 37,5 centgrados, habindose detectado un del 96% sin oxigenoterapia, aunque con ligera taquipnea
pico febril aislado de 38 centgrados haca 48 horas. No de 22 respiraciones por minuto. Buena tolerancia al dec-
se obtuvo ningn otro sntoma de inters en el resto de la bito, sin ingurgitacin venosa yugular. En la auscultacin
anamnesis realizada. cardiorrespiratoria los ruidos cardiacos eran rtmicos y sin
En la exploracin fsica la paciente presentaba tendencia soplos, con murmullo vesicular conservado en todos los
a la hipotensin arterial (95/57 milmetros de mercurio), campos pulmonares con discretos crepitantes de despega-
con frecuencia cardiaca de 58 latidos por minuto, mante- miento bibasales. En la exploracin abdominal el abdo-
nindose afebril (36,7 centgrados). Mantena buen estado men era depresible, no doloroso y sin peritonismo, con rui-
general, consciente y orientada en las tres esferas, colabo- dos intestinales normales, y percusin normal. El miembro
radora. La paciente presentaba sobrepeso, adecuada hi- inferior izquierdo presentaba una placa de celulitis que se
dratacin y buena perfusin distal. Saturacin de oxgeno extenda desde el tobillo al tercio superior de la pierna, no

75
CAPTULO 5
INFECCIONES DE PIEL,
PARTES BLANDAS Y PI DIABTICO

alcanzando rodilla, con pstulas subcentimtricas satlites del germen causal de la celulitis no suele ser fcil, ya que la
dispersas, as como ampollas necrohemorrgicas a tensin rentabilidad de los cultivos es bastante escasa, siendo po-
de contenido purulento; no exista crepitacin local con sitivos en menos del 5% de los hemocultivos, del 5 al 40%
la palpacin, ni signos sugerentes de sndrome comparti- de los cultivos de aspirado, y del 20 al 30% de los cultivos
mental, con buenos pulsos perifricos (Figuras 1A y 1B). de biopsia punch.
El miembro inferior derecho no presentaba alteraciones.
En la inspeccin del resto de piel y faneras no se detect Evolucin
ninguna alteracin significativa.
En las pruebas complementarias realizadas a la paciente Durante los primeros das de ingreso hospitalario la pa-
en el Servicio de Urgencias destacaba la existencia de leu- ciente permaneci afebril, con estabilidad hemodinmica
cocitosis de 16690 clulas/uL con neutrofilia del 91,4%, y sin incidencias clnicas destacables. En la analtica gene-
con normalidad de las otras dos series del hemograma. En ral solicitada al ingreso se objetiv la persistencia de la leu-
la bioqumica presentaba una funcin renal e iones nor- cocitosis con descenso de la neutrofilia al 82,2% y descenso
males, perfil heptico y coagulacin normales, protenas de los reactantes de fase aguda, con PCR a las 48 horas de
totales de 6,1 g/dL, LDH 457 U/L, CPK 16 U/L, PCR 9,6 mg/dL, sin ninguna otra alteracin destacable en la
18,8 mg/dL, procalcitonina normal (<0,05 ng/mL) y ci- funcin hepatorrenal, iones ni perfil lipdico, y hemoglobi-
do lctico 2,2 mEq/L. Se realiz una tomografa axial na glicosilada de 5,9%.
computerizada (TAC) de la pierna izquierda, detectndose Al quinto da de ingreso en planta el Servicio de Microbio-
tumefaccin importante con aumento de densidad e infil- loga informa del aislamiento en los dos cultivos de exu-
tracin por lquido de la grasa subcutnea pero sin afecta- dado de las ampollas extrados en Urgencias de Nocardia
cin de estructuras profundas, no aprecindose tampoco la spp, notificando pocos das despus que se trataba de una
presencia de gas en los tejidos blandos. Nocardia abscessus sensible a claritromicina (CMI 1), moxi-
Una vez valorados todos los datos expuestos, se realizaron floxacino (CMI 0,75), amoxicilina-clavulnico (CMI 0,75),
sendos aspirados del contenido de 2 ampollas para reali- cefotaxima (CMI 2), tobramicina, amikacina (CMI 0,25),
zacin de cultivo microbiolgico, y se procedi al ingreso septrim (CMI 0,012), linezolid (CMI 0,5), con sensibilidad
de la paciente en planta para vigilar evolucin y completar intermedia a tetraciclina (CMI 2), y resistente a ciprofloxa-
estudio, inicindose tratamiento antibitico emprico con cino. Desde un punto de vista teraputico se decidi man-
meropenem 1g cada 8 horas y linezolid 600 mg cada 12h, tener en biterapia con linezolid 600 mg cada 12 horas y tri-
ambos administrados por va intravenosa, e indicndose metoprim 320 mg/ sulfametoxazol 1600 mg cada 8h por
curas locales y reposo relativo con elevacin del miembro. va intravenosa durante 15 das, pasando posteriormente a
trimetoprim/sulfametoxazol en monoterapia. La paciente
Diagnstico diferencial present intolerancia gastrointestinal a trimetoprim/sulfa-
metoxazol, refractaria a tratamiento mdico, por lo que se
La celulitis es una infeccin de la piel que se extiende hasta decidi continuar tratamiento con moxifloxacino 400 mg
el tejido celular subcutneo, manifestndose clnicamen- cada 24 horas. Se complet un mes de tratamiento antibi-
te como reas de edematizacin cutnea, eritema, dolor tico por va intravenosa, asociado a las curas locales corres-
y calor locales, pudiendo aparecer en algunos pacientes pondientes, con reduccin progresiva de los signos flogti-
vesculas o equimosis. Los agentes microbiolgicos impli- cos y sin complicaciones destacables locales ni sistmicas.
cados con mayor frecuencia en la etiologa de la celulitis Ante el aislamiento microbiolgico de Nocardia abscessus se
son el Staphylococcus aureus y el los estreptococos beta-hemo- realiz un amplio estudio para despistaje de inmunodepre-
lticos como el Streptococcus pyogenes. Ms infrecuentes son sin, solicitando serologas de VIH y virus hepatotropos,
otras etiologas como los bacilos gram negativos, Strepto- marcadores tumorales, inmunoglobulinas, proteinograma,
coccus pneumoniae, etc, pudiendo tambin considerar como beta-2-microglobulina, poblaciones linfocitarias, autoin-
posibles agentes causales en nuestra paciente ante la aso- munidad sistmica (ANAs, screening de ENAs y DNA),
ciacin de ampollas necrohemorrgicas, los clostridios u ECA y Mantoux, siendo todos ellos negativos.
otros anaerobios del tipo Bacteroides, Peptostreptococci, etc.1 Paralelamente se realiz un estudio de extensin de la no-
No obstante hay que tener presente que la mayora de los cardiasis, realizndose TAC torcico y RMN cerebral en
microorganismos pueden causar una celulitis si el inculo los que se descartaba afectacin pulmonar y neurolgica.
bacteriano es suficientemente alto. Una vez recibido el tratamiento antibitico especfico por
Es importante destacar el hecho de que la identificacin va intravenosa durante cuatro semanas y completado el

76
CAPTULO 5
INFECCIONES DE PIEL,
PARTES BLANDAS Y PI DIABTICO

5
estudio complementario, se procedi al alta hospitalaria Localizaciones ms frecuentes de la infeccin por Nocardia spp .
con el diagnstico de nocardiosis cutnea primaria, reco- LOCALIZADA PULMONAR 39%
mendndose continuacin del tratamiento con moxifloxa- CUTNEA 8%
cino 400 mg cada 24 horas por va oral durante tres meses Otra localizacin de forma Sistema 9%
aislada, con foco primario nervioso
ms y revisin en consultas externas. no identificable central
Otras: ojo, 12%
Diagnstico final hueso, etc.
DISEMINADA (al Afectacin pulmonar 75%
menos 2 localizaciones
Nocardiosis cutnea primaria. afectadas)
Afectacin cutnea 25%
Afectacin neurolgica 44%

Discusin Tabla 1. Localizaciones ms frecuentes de la infeccin por Nocardia spp5.

Nocardia spp es un actinomiceto, bacteria gram positiva ae- una afectacin cutnea, bien primaria por inoculacin di-
robia, de la que se han descrito unas 80 especies de las cua- recta o bien secundaria a diseminacin, con 4 posibles for-
les aproximadamente 30 son patgenas para el ser huma- mas de presentacin: cutnea primaria, consecuencia de la
no2-4. Son bacterias ubicuas, saprfitos ambientales, que se inoculacin bacteriana directa por lesin de continuidad
aslan en el suelo, materia vegetal en descomposicin o en de la piel, siendo clnicamente indistinguible de una infec-
el agua. cin pigena8; linfocutnea, donde se asocia una afecta-
Suelen penetrar en el ser humano por inhalacin o por cin linftica regional y ocasiona una linfangitis nodular;
inoculacin directa en piel, siendo muy infrecuente la in- afectacin cutnea secundaria a diseminacin hematge-
feccin secundaria a ingesta. La primera barrera que ofre- na, con lesiones cutneas indistinguibles a las de las formas
ce el organismo a la infeccin es la integridad de la piel primarias; o formacin de un micetoma, secundario a la
y mucosas, por lo que cualquier alteracin de las mismas produccin de una reaccin inflamatoria granulomatosa
justifica mayor susceptibilidad a la infeccin. La respuesta crnica alrededor del inculo bacteriano4.
inmune humoral desempea una proteccin limitada fren- La nocardiosis cerebral aparece de forma secundaria a
te a Nocardia spp, siendo la inmunidad celular el elemento una diseminacin hemtica desde un foco pulmonar o cu-
fundamental de defensa. De este modo, aunque hasta en tneo, y suele presentarse como uno o varios abscesos en
un tercio de los pacientes que sufren nocardiosis no se de- cualquier localizacin, con las manifestaciones propias del
tectan factores de riesgo predisponentes, en la mayora de mismo: fiebre, cefalea, meningismo, dficits neurolgicos
los afectados existe cierto grado de inmunodepresin ce- focales, convulsiones, etc. No obstante su evolucin suba-
lular: infeccin por VIH con inmunodepresin profunda guda o crnica hace que el paciente pueda permanecer
(menos de 100 CD4 totales/mL), receptores de trasplan- asintomtico durante aos, manifestndose finalmente
te de rgano slido durante el primer ao tras el mismo, como consecuencia del efecto masa que produce5.
neoplasia activa, enfermedades autoinmunes, enfermedad La afectacin de dos o ms localizaciones por Nocardia spp
granulomatosa crnica, corticoterpia crnica, enolismo o se denomina nocardiosis diseminada, y se produce como
malnutricin4-7. consecuencia de una diseminacin por va hematgena
Las manifestaciones clnicas de la infeccin por Nocardia desde un foco primario habitualmente cutneo o pulmo-
spp suelen tener un curso subagudo y ser bastante inespec- nar, pudiendo alcanzar cualquier rgano: pulmn, sistema
ficas, siendo muy frecuente la capacidad de diseminacin nervioso central, piel, ojo, rin, articulaciones, hueso, co-
del germen (Tabla 1), con especial tropismo neurolgico5. razn, etc. En ocasiones no se consigue identificar el foco
El rgano que se afecta con ms frecuencia es el pulmn, original de la infeccin, asumindose un origen pulmonar
hasta en un 75% de los casos, habitualmente de forma o cutneo resuelto2,9.
primaria por entrada directa del germen por inhalacin, Para el diagnstico de nocardiosis es fundamental el ais-
pudiendo presentar un curso clnico agudo, subagudo o lamiento microbiolgico de la bacteria, para lo que en un
crnico con fiebre, tos, expectoracin, dolor torcico, he- alto porcentaje de los pacientes no suele ser suficiente con
moptisis, etc. En ms de la mitad de los pacientes se pro- la toma de muestras habituales, requiriendo procedimien-
duce una diseminacin por va hemtica de la Nocardia spp tos invasivos para la toma de los mismos. El crecimiento
desde el pulmn a otras localizaciones, con predileccin de Nocardia spp oscila entre 48 horas y 14 das, aunque con
por el sistema nervioso central4,5,7. frecuencia se produce entre los das 3 y 5 del cultivo, pre-
Hasta en el 25% de los pacientes con nocardiosis existe sentando las colonias un tpico aspecto cerebriforme ce-

77
CAPTULO 5
INFECCIONES DE PIEL,
PARTES BLANDAS Y PI DIABTICO

roso, bien pigmentadas o bien de coloracin blanquecina Bibliografa


cuando producen hifas areas4. Las pruebas de biologa
molecular, como la reaccin en cadena de la polimerasa, 1. Carratal J, Rosn B, Fernndez-Sab N, Shaw E, del
que proporcionan resultados ms rpidos y precisos que Rio O, Rivera A, et al. Factors associated with com-
los mtodos convencionales en la identificacin de Nocardia plications and mortality in adult patients hospitalized
spp, pero no estn disponible en todos los laboratorios2. for infectious cellulitis. Eur J Clin Microbiol Infect Dis
Los antibiticos que suelen ser eficaces frente a Nocardia 2003;22:151-7.
spp son trimetoprim/sulfametoxazol, amikacina, imipe- 2. Brown-Elliot BA, Brown JM, Conville PS, Wallace RJ
nem, cefalosporinas de tercera generacin, quinolonas y Jr. Clinical and laboratory features of the Nocardia
linezolid10. La eleccin de la pauta teraputica depende de spp. based on current molecular taxonomy. Clin Mi-
la gravedad del cuadro clnico, el estado inmunitario del crobiol Rev 2006;19:259-82.
paciente y la localizacin de la infeccin, as como al re- 3. Minero MV, Marn M, Cercenado E, Rabadn PM,
sultado del antibiograma una vez dispongamos de l. De Bouza E, Muoz P. Nocardiosis at the turn of the cen-
este modo, el tratamiento recomendado en formas cut- tury. Medicine (Baltimore) 2009;88:250-61.
neas limitadas en inmunocompetentes suele ser monote- 4. Sorrel TC, Mitchell DH, Iredell JR, Chen SCA. No-
rapia con trimetoprim/sulfametoxazol. En inmunodepri- cardia Species. En: Mandell, Douglas, Bennetts, Edi-
midos o en cuadros diseminados graves se recomienda el tores. Principles and Practice of Infectious Diseases. 8
empleo de 2 o 3 agentes intravenosos, siendo de eleccin ed. Elsevier; 2015. P.2853-2863.
la combinacin de trimetoprim/ sulfametoxazol, amikaci- 5. Beaman BL, Beaman L. Nocardia species: host-para-
na y/o imipenem4. En aquellos pacientes con enfermedad site relationships. Clin Microbiol Rev 1994;7:213-64.
grave no neurolgica que mejoran tras 3 a 6 semanas de 6. Muoz J, Mirelis B, Aragn LM, Gutirrez N, Sn-
tratamiento por va intravenosa, se puede desescalar a un chez F, Espaol M, et al. Clinical and microbiological
rgimen por va oral en monoterapia. La duracin ptima features of nocardiosis 1997-2003. J Med Microbiol
del tratamiento no est bien establecida, recomendndose 2007;56:545-50.
en formas cutneas aisladas en inmunocompetentes una 7. Martnez Toms R, Menndez Villanueva R, Reyes
duracin de 3 a 6 meses, prolongndose de 6 a 12 meses Calzada S, Santos Durantez M, Valls Tarazona JM,
en caso de inmunodepresin. En pacientes con infeccin Modesto Alapont M, et al. Pulmonary nocardiosis: risk
pulmonar grave se prolonga el tratamiento durante 6 a 12 factors and outcomes. Respirology 2007;12:394-400.
meses. En todos los pacientes inmunodeprimidos, con no- 8. Satterwhite TK, Wallace RJ Jr. Primary cutaneous no-
cardiosis diseminada o con afectacin neurolgica se reco- cardiosis. JAMA 1979; 242:333-6.
mienda prolongar la pauta antibitica durante al menos 9. Corti ME, Villafae-Fioti MF. Nocardiosis: a review.
un ao4. Int J Infect Dis 2003;7:243-50.
La curacin de la nocardiosis cutnea o pulmonar prima- 10. Uhde KB, Pathak S, McCullum I Jr, Jannat-Khah DP,
ria se consigue hasta en el 90%, mientras que en el resto Shadomy SV, Dykewicz CA, et al. Antimicrobial-resis-
de formas clnicas la mortalidad asciente hasta el 50%4,9. tant nocardia isolates United States, 1995-2004. Clin
La tasa de recidivas es elevada, por lo que es necesaria una Infect Dis 2010;51:1445-8.
vigilancia estrecha del paciente en los 12 a 24 meses poste-
riores al diagnstico.
Como conclusin resulta fundamental resear la impor-
tancia de perseguir la identificacin microbiolgica en los
distintos cuadros clnicos infecciosos siempre que sea po-
sible ya que, adems de permitirnos desescalar la antibio-
terapia emprica iniciada con mayor seguridad en base al
resultado del antibiograma, puede condicionar, en funcin
del microorganismo aislado, un manejo clnico y diagns-
tico diferente as como modificaciones sustanciales en la
pauta teraputica.

78
Mujer de 58 aos con dolor, eritema
y aumento de volumen del pie izquierdo
Cornejo Saucedo, MA; Ruiz Estvez, B;
Brun Romero, FM.
Hospital Universitario Puerta del Mar. Cdiz.

Caso clnico En el momento de su llegada a Urgencias se realiz una


analtica bsica:
Mujer de 58 aos con el nico antecedente de diabetes
mellitus tipo 2 mal controlada, sin complicaciones macro Hemograma: hemoglobina 12,4 g/dl, 13.210 leucoci-
ni microvasculares conocidas y en tratamiento con anti- tos/mm3, plaquetas normales.
diabticos orales. Comienza 3 das antes de su consulta en Bioqumica: glucemia 413 mg/dl, urea 38 mg/dl,
el Servicio de Urgencias con fiebre de predominio vesper- creatinina 1,3 mg/dl, iones y enzimas hepticas nor-
tino, y la noche anterior con aparicin de dolor en el pie males, protena C reactiva 314,90 mg/l (valores nor-
izquierdo sin lesiones cutneas evidentes. Por la maana, males: 0-5 mg/l), procalcitonina 0,21 ng/ml (valores
el dolor se ha hecho ms intenso y presenta eritema y au- normales 0-0,5 ng/ml), lactato 16,7 mg/dl (valores
mento de volumen de dicho pie, con lesiones de coloracin normales <19 mg/dl).
negruzca en la regin dorsal del mismo. En las ltimas 3-4 Coagulacin: fibringeno 952 mg/dl, resto normal.
semanas haba tenido una lesin interdigital entre los de-
dos 4 y 5 del mismo pie, que haba tratado con curas Diagnstico diferencial
locales.
En la exploracin fsica destacaba la presencia de una le- La paciente consulta por la aparicin de signos inflamato-
sin eritematosa en el dorso del pie izquierdo a la altura rios y lesiones cutneas a nivel de pie izquierdo con fiebre
del 4 radio, con edema y dolor, extendindose dicha le- de varios das de evolucin, por lo que planteamos el diag-
sin por todo el dorso del pie con una ampolla necrtica/ nstico diferencial entre las distintas infecciones de piel y
hemorrgica hasta llegar a la regin posterior del malolo partes blandas:
tibial externo, donde se observa una coleccin hemtica
con signos inflamatorios, irradindose el dolor al resto de Erisipela: infeccin drmico-hipodrmica aguda (su
la pierna (imagen 1). Pulsos arteriales conservados y sim- extensin a tejidos ms profundos es inusual), no ne-
tricos. Tono muscular normal, sin alteraciones motoras o crosante, producida generalmente por el estreptococo
sensitivas. Ausencia de signos de enfermedad tromboem- beta-hemoltico del grupo A (Streptococcus pyogenes). Sue-
blica venosa. Resto de la exploracin fsica sin alteracio- le tener un inicio sbito, con fiebre y una placa so-
nes. breelevada, eritematosa, caliente y dolorosa de bordes
bien definidos. Son factores de riesgo para su apari-
cin el linfedema crnico, la solucin de continuidad
de la piel y la obesidad, pero no la diabetes mellitus.
En nuestro caso, no existan factores de riesgo y las
caractersticas de las lesiones eran diferentes.
Celulitis: infeccin aguda de la dermis profunda y el
tejido celular subcutneo, no necrosante, sin clara de-
limitacin entre la piel sana y la infectada, producida
generalmente por estreptococos o Staphylococcus aureus.
La superficie cutnea puede adoptar un aspecto en
piel de naranja debido al edema cutneo superficial
que rodea a los folculos pilosos. Suelen aparecer ves-
culas y ampollas de contenido lquido claro. Salvo en
los casos graves, las manifestaciones sistmicas suelen
ser leves. En nuestro caso, es uno de los diagnsticos
Imagen 1. Lesin de piel y partes blandas inicial. posibles, aunque la presencia de una ampolla necrti-

79
CAPTULO 5
INFECCIONES DE PIEL,
PARTES BLANDAS Y PI DIABTICO

ca/hemorrgica nos debe hacer pensar en otras posi-


bilidades.
Infecciones profundas necrotizantes: entre stas se
encuentran la miositis, la mionecrosis y la fascitis ne-
crotizante. Se trata de infecciones con afectacin de
cualquier estructura de la piel y partes blandas (der-
mis, tejido celular subcutneo, fascia superficial o pro-
funda y msculo) que se asocian a necrosis, y que pre-
sentan una progresin rpida. Adems de las lesiones
cutneas, son frecuentes la aparicin de fiebre, dolor e
hipersensibilidad o hipoestesia cutneas, la existencia
de necrosis y ampollas hemorrgicas, y la crepitacin
cuando hay produccin de aire. La diabetes mellitus
constituye un factor de riesgo importante para su ad-
quisicin. La disrupcin de la barrera cutnea y las
lceras crnicas del pie diabtico son factores favore-
cedores para la produccin de este tipo de infecciones.
ste sera, por tanto, el diagnstico ms probable en el
caso que describimos.

Otras infecciones como el ectima gangrenoso, las lceras


por presin del pie diabtico o las lesiones traumticas o
por mordedura quedaran inicialmente descartadas. La
primera est causada tpicamente por Pseudomonas aerugino-
sa y suele tratarse de una lesin redondeada y ovalada con Imagen 2. Radiografa del pie izquierdo que muestra la presencia de gas en el
4 radio.
halo eritematoso y centro necrtico que puede ser nica o
mltiple, siendo diferente a la lesin presentada por nues-
tra paciente. En cuanto a las lceras por presin, si bien la
paciente es una diabtica mal controlada, no presenta neu-
ropata diabtica y el lugar de presentacin no suele ser el
habitual para este tipo de lesiones. Por ltimo, las lesiones
traumticas o por mordedura quedaran descartadas por
no haberse producido dicho mecanismo lesional, tratndo-
se de una lesin de aparicin espontnea.

Evolucin

La paciente qued hospitalizada a cargo del Servicio de


Ciruga Ortopdica y Traumatologa. En el momento de
su ingreso se inici antibioterapia emprica de amplio es- Imagen 3. Lesin tras drenaje y des- Imagen 4. Lesin en fase
bridamiento quirrgico. de reepitelizacin.
pectro con meropenem ms daptomicina. Tras 48 horas
de tratamiento no se observaron cambios significativos en
la evolucin de la lesin. Ante la elevada sospecha clnica desbridamiento das ms tarde. Complet tratamiento du-
de infeccin necrosante de piel y partes blandas, se reali- rante 21 das con ertapenem ms linezolid. En el cultivo de
z radiografa del pie izquierdo, objetivndose la presen- dos muestras quirrgicas se aisl Actinomyces neuii. La evolu-
cia de burbujas de aire alrededor del 4 dedo (imagen 2), cin fue favorable, con normalizacin de los reactantes de
procedindose al desbridamiento quirrgico de la regin fase aguda y granulacin progresiva del tejido, alcanzando
anterior del dorso del pie al tercer da de ingreso. Tras esto dos meses despus una epitelizacin prcticamente com-
se produjo una mejora paulatina (imagen 3), persistiendo pleta (imagen 4).
una escara necrtica en la regin proximal que requiri

80
CAPTULO 5
INFECCIONES DE PIEL,
PARTES BLANDAS Y PI DIABTICO

Diagnstico final nos frecuentemente Staphylococcus aureus). Ms agresiva,


su mortalidad depende en gran medida de la presencia
La paciente fue diagnosticada de infeccin necrotizante de o no de shock txico.
piel y partes blandas (fascitis necrotizante) en el dorso del Tipo III: ms frecuente en Asia, en relacin a conta-
pie izquierdo por Actinomyces neuii. minacin de heridas o alimentos por microorganismos
que habitan en el agua, sobre todo Vibrio spp. Mortali-
Discusin dad del 30-40%.
Tipo IV: de origen fngico (Candida spp. en inmuno-
Se denomina infeccin necrotizante de piel y partes blan- comprometidos, Zygomycetos en inmunocompetentes).
das a aqulla que afecta a cualquiera de las estructuras que Curso agresivo con rpida extensin y alta mortalidad,
las componen (dermis, tejido celular subcutneo, fascia sobre todo en pacientes inmunocomprometidos.
superficial o profunda, msculo) y que, adems, se asocia
con necrosis tisular. Son procesos con el mismo abordaje Entre las comorbilidades asociadas con un mayor riesgo de
diagnstico y teraputico, y establecer su diagnstico supo- sufrir este tipo de infecciones se encuentran la enfermedad
ne el mayor desafo dentro de este tipo de infecciones, ya heptica crnica, la insuficiencia renal crnica y la terapia
que el retraso diagnstico y del desbridamiento quirrgico renal sustitutiva, la inmunosupresin (SIDA, cncer, tras-
conlleva un importante incremento de la morbimortalidad plante, uso de esteroides), la edad avanzada (>70 aos), el
al tratarse de infecciones con una rpida progresin. Su in- alcoholismo, el uso de drogas por va parenteral, la diabe-
cidencia es relativamente baja (0,4 casos por cada 100.000 tes mellitus, la obesidad, la malnutricin, la enfermedad
habitantes), aumentando progresivamente con la edad1. vascular perifrica, las lesiones cutneas crnicas y la insu-
La primera referencia a esta enfermedad se remonta a Hi- ficiencia cardiaca congestiva4.
pcrates en el siglo V a.C. como complicacin de la erisi- La prdida de la integridad de la piel es habitualmente el
pela. Su primera descripcin moderna se encuentra en el primer paso para su desarrollo, no siendo necesario que
ao 1871, y es realizada por Joseph Jones, un cirujano del sea demasiado extensa. Puede tratarse de heridas traum-
ejrcito confederado, durante la Guerra de Secesin, refi- ticas, quirrgicas, mordeduras de animales, picaduras de
rindose a la misma como gangrena hospitalaria ([] lo insectos, lceras crnicas o heridas de pie diabtico, identi-
primero que se percibe es una mancha de color prpura o ficables en torno al 80% de los casos con una exploracin
azul [] la piel de la zona se derrite en 24 horas mientras fsica minuciosa. Otra posibilidad sera la diseminacin he-
que una areola azul y morada, casi negra, en el fondo que matgena. Se produce una proliferacin bacteriana en el
rodea la masa muerta, se extiende en crculos []). Ms interior de la fascia, con produccin de toxinas y enzimas
tarde, en 1924, un cirujano misionero llamado Frank L. que permiten su difusin a travs de la misma, con una
Meleney, describi un brote de gangrena en adictos al opio primera fase de extensin horizontal. Esto, junto con la
en Pekn([] Un escalofro puede marcar el comienzo de respuesta inflamatoria, da lugar a la trombosis vascular y
los sntomas generales [] parches irregulares de tonali- oclusin de los vasos perforantes que ascienden hacia la
dad oscura desarrollan grandes ampollas que al romperse dermis y profundizan hacia los grupos musculares, produ-
desprenden un lquido seroso oscuro []). Finalmente, cindose la necrosis tisular y, por consiguiente, el dolor.
el trmino fascitis necrotizante fue acuado por Wilson En las fases iniciales, la distincin entre la celulitis y las in-
en 19522,3. feccin necrotizante puede resultar complicada. Las ma-
nifestaciones y hallazgos clnicos sugestivos de infeccin
Las infecciones necrotizantes de piel y partes blandas se necrotizante son: dolor intenso y persistente (habitualmente
clasifican en 4 tipos diferentes: desproporcionado), necrosis cutnea, presencia de gas o cre-
pitacin, edema que se extiende ms all de los mrgenes
Tipo I (70-80% de los casos): polimicrobiana, com- del eritema, anestesia cutnea, signos de toxicidad sistmica,
binando con frecuencia microorganismos aerobios y extensin rpida a pesar de tratamiento antibitico, y la pre-
anaerobios. Clnicamente ms fciles de reconocer, sencia de bullas, si bien este ltimo dato por s solo no es pa-
con un curso insidioso y mejor pronstico, con una tognomnico de infeccin profunda, ya que puede aparecer
mortalidad variable en funcin de la comorbilidad tambin en casos de erisipela, celulitis, sndrome de la piel
asociada. escaldada, etc. Un hallazgo caracterstico es la sensacin de
Tipo II (20-30% de los casos): habitualmente monomi- endurecimiento a la palpacin del tejido celular subcutneo,
crobiana (fundamentalmente Streptococcus pyogenes, me- a diferencia de la celulitis o la erisipela5.

81
CAPTULO 5
INFECCIONES DE PIEL,
PARTES BLANDAS Y PI DIABTICO

Con respecto al diagnstico, lo ms importante y funda- mortalidad cercana al 100%, siendo til la asociacin de
mental es la sospecha clnica. Las pruebas complementa- antibiticos que inhiben la sntesis proteica como clinda-
rias son accesorias para apoyar dicha sospecha clnica, con micina, ya que puede ayudar a reducir la produccin de
el inconveniente de que pueden retrasar el diagnstico y el toxinas y disminuir la respuesta inflamatoria, sobre todo
inicio del tratamiento. Existen varios scores basados en en las infecciones estreptoccicas, no vindose afectada su
pruebas de laboratorio y que clasifican a los pacientes se- efectividad por el tamao del inculo bacteriano. Por lti-
gn el riesgo de sufrir una fascitis necrotizante, destacando mo, el tratamiento especfico se basa en el uso de oxgeno
el LRINEC (Laboratory Risk Indicador For Necrotizing Fascitis) hiperbrico (disponibilidad muy escasa, no est justificada
y el score de Anaya, que utilizan valores analticos como la la demora del tratamiento quirrgico y desplazamiento del
protena C reactiva, el recuento de leucocitos, la cifra de enfermo a otro centro para su utilizacin) o inmunoglobu-
hemoglobina o los niveles de creatinina, sodio y glucosa (el linas (estudios limitados, podran ser tiles de forma precoz
primero) y criterios analticos y clnicos como la edad, la y sobre todo en casos de infeccin por Streptococcus pyogenes).
frecuencia cardiaca, la temperatura, el recuento de leuco- En cuanto a la identificacin de Actinomyces neuii como
citos, el hematocrito o el valor de la creatinina plasmtica agente etiolgico, decir que se trata de un microorganismo
(el segundo), siendo herramientas tiles en combinacin grampositivo aerobio y anaerobio facultativo descrito por
con los hallazgos clnicos y teniendo su principal utilidad primera vez en 1994 (aunque aislado previamente con otro
en el valor predictivo negativo6,7. Entre las pruebas de ima- nombre en 1985 en dos pacientes con endoftalmitis tras ser
gen, la radiografa simple tiene una utilidad muy limitada intervenidos de cataratas) que se ha visto involucrado en la
salvo que exista gas en los tejidos, la ecografa sirve para produccin de abscesos e infeccin de placas de ateroma,
descartar la presencia de colecciones y complicaciones vas- infeccin de lesiones cutneas (como lceras de pie diab-
culares, y la TC aporta la misma informacin que las ante- tico), endoftalmitis, endocarditis, bacteriemia, pericarditis,
riores pero con una mejor definicin anatmica, siendo la osteomielitis, corioamnionitis, infeccin de prtesis arti-
RNM la tcnica ms efectiva para documentar lesiones de culares y derivaciones ventriculoperitoneales, infecciones
partes blandas y evaluar su distribucin8. La biopsia debe del tracto urinario y prostatitis. Forma parte de la flora
obtenerse de la interfase entre el tejido sano y el necrtico, normal del ser humano, y habitualmente ha sido seala-
puede servir para el diagnstico de los casos dudosos, y su do como colonizador o contaminante, lo cual dificulta su
principal inconveniente es su escasa disponibilidad. Con identificacin como productor de infecciones. Crece en
respecto a los cultivos, debido a su bajo rendimiento los aproximadamente 48 horas en un medio de cultivo agar
hemocultivos no son tiles salvo que se trate de infecciones sangre incubado en condiciones aerobias o anaerobias. Es
graves o con estado txico y bacteriemia asociada, y en un microorganismo sensible a betalactmicos, macrlidos,
pacientes inmunocomprometidos; los cultivos de la zona rifampicina, vancomicina y clindamicina, con un perfil de
estn indicados ante la sospecha de un agente etiolgico sensibilidad variable a aminoglucsidos y quinolonas, y ha-
no habitual, ausencia de respuesta al tratamiento empri- bindose descrito casos de resistencia a tetraciclinas10.
co, pacientes inmunocomprometidos e infecciones necroti- Por tanto, como conclusin podemos decir que las infec-
zantes, y la mejor muestra es la biopsia, teniendo en cuenta ciones necrotizantes de piel y partes blandas son infeccio-
que aun as su rentabilidad es del 20-30%. nes graves cuyo diagnstico es fundamentalmente clnico
De cara al tratamiento, podemos establecer tres pasos: (que en sus fases iniciales pueden ser difciles de diferenciar
control de la fuente, tratamiento antibitico y tratamien- de las infecciones cutneas superficiales) y que requieren
to especfico. El control de la fuente consiste en todas las un tratamiento precoz consistente en el desbridamiento qui-
medidas fsicas dirigidas a eliminar la fuente de infeccin, rrgico asociado a antibioterapia emprica de amplio espec-
controlar la contaminacin bacteriana y restaurar la ana- tro, debiendo tomarse con cautela el resto de herramientas
toma y funcin previas, basndose en el drenaje, desbri- diagnsticas, siendo la demora en la realizacin del acto
damiento y descompresin quirrgicos, que debe hacerse quirrgico el principal factor pronstico de estos pacientes.
a la mayor brevedad posible y ser lo ms extenso posible, Actinomyces neuii es un microorganismo emergente potencial-
pudiendo ser necesario repetirlo en las primeras 24 horas, mente productor de infecciones de piel y partes blandas, so-
siendo su demora el mayor determinante de mortalidad bre todo en pacientes con lceras de pie diabtico.
en estos pacientes9. El tratamiento antibitico emprico de
amplio espectro (cubriendo grampositivos, gramnegativos
y anaerobios) debe iniciarse de forma precoz, aunque sin
un tratamiento quirrgico para desbridar la zona tiene una

82
CAPTULO 5
INFECCIONES DE PIEL,
PARTES BLANDAS Y PI DIABTICO

Bibliografa

1. Anaya DA, Dellinger EP. Necrotizing soft-tissue in-


fection: diagnosis and management. Clin Infect Dis.
2007;44:705-10.
2. Jones J. Investigation upon the nature, causes and
treatment of hospital gangrene as prevailed in the con-
federate armies. Surgical memoirs of the war of the
rebellion. New York: Hurd & Houghton for the U.S.
Sanitary Commission; 1871. p.257.
3. Meleney FL. Haemolytic streptococcal gangrene. Im-
portance of early diagnosis and early operation. J Am
Med Assoc. 1929;92:2009-12.
4. De Waele JJ. Management of necrotizing skin and
soft tissue infections. Expert Rev Anti Infect Ther.
2012;10:805-14.
5. Wang YS, Wong CH, Tay YK. Staging of necrotizing
fasciitis based on the evolving cutaneous features. Int J
Dermatol. 2007;46:1036-41.
6. Wong CH, Khin LW, Heng KS, Tan KC, Low CO.
The LRINEC (Laboratory Risk Indicator for Necro-
tizing Fasciitis) score: a tool for distinguishing necroti-
zing fasciitis from other soft-tissue infections. Crit Care
Med. 2004;32: 1535-41.
7. Anaya DA, Bulger EM, Kwon YS, Kao LS, Evans H,
Nathens AB. Predicting death in necrotizing soft tissue
infections: a clinical score. Surg Infect. 2009;10: 517-
22.
8. Malghem J, Lecouvet FE, Omoumi P, Maldague BE,
VandeBerg BC. Necrotizing fasciitis: contribution
and limitations of diagnostic imaging. J Bone Spine.
2013;80:146-54.
9. Kobayashi L, Konstantinidis A, Shackelford S, Chan
LS, Talving P, Inaba K et al. Necrotizing soft tissue in-
fections: delayed surgical treatment is associated with
increased number of surgical debridements and mor-
bidity. J Trauma. 2011;71:1400-5.
10. Von Graevenitz A. Actinomyces neuii: review of an un-
usual infectious agent. Infection. 2011;39:97-100.

83
CAPTULO 6
INFECCIONES OSTEOARTICULARES
Sndrome febril y tetraparesia aguda:
la importancia de un diagnstico precoz
Prez Medina, MI; Milln Garca, G;
Ruiz-Mesa, JD.
Hospital Universitario Regional de Mlaga

Caso clnico 12.900 /mm3 (N 12.100). En BQ: glucosa 100 mg/dl,


Creatinina 1.3 mg/dl; Na 128 mEq/l, K 3,66 mEq/l, Ca
Varn de 66 aos, natural de Reino Unido, sin alergias 9.3 mg/dl, AST 110 U/I, ALT 44 U/I, GGT 92 U/I, Bi-
medicamentosas, ex-fumador y consumidor habitual de lirrubina total 2.61 mg/dl (directa 1.74 mg/dl), PCR 267
cerveza y vino a diario. No presenta antecedentes mdi- mg/l; resto de bioqumica y gasometra venosa normales.
cos de inters. Como antecedentes quirrgicos destacan: En Rx de trax y abdomen no se objetivan hallazgos. TAC
vasectoma, varices, amigdalectoma. No realiza ningn craneal sin hallazgos significativos y ecografa abdominal
tratamiento habitual. con colelitiasis.
En el ltimo ao presenta dolor de intensidad leve-mode- Se le solicito RNM de columna vertebral y se extrajeron
rado a nivel de cadera izquierda, con incremento sbito hemocultivos y cultivos de orina
del dolor en las 2-3 ltimas semanas, incluso en reposo, ne-
cesitando de analgesia ambulatoria. Una semana despus, Diagnstico diferencial
acude a Urgencias de nuestro hospital al no existir mejo-
ra del cuadro y asociarse dolor intenso en columna dor- El diagnstico sindrmico de nuestro caso sera: tetrapare-
so-lumbar con dificultad para la deambulacin. Tras el ha- sia aguda y sndrome febril en varn de 68 aos. Los prin-
llazgo de artrosis severa en la cadera izquierda, es dado de cipales procesos a tener en cuenta ante este cuadro son:
alta a domicilio con tratamiento sintomtico (tapentadol 1) Infecciosos: las bacterias seran los grmenes ms rela-
y celecoxib). A pesar de controlarse el dolor parcialmente, cionados, destacando el Staphylococcus aureus o Mycobacterium
acude de nuevo a Urgencias por prdida de fuerza brusca tuberculosis (aunque sta suele dar un cuadro subagudo o
en las cuatro extremidades (de predominio izquierdo) de crnico). Pueden producir dao medular por una mielitis
72 horas de evolucin. Se acompaa de sndrome confu- directa o por diversos focos como osteomielitis vertebral,
sional agudo con amnesia antergrada y alucinaciones au- abscesos epidural, subdural o subaracnoideo. Otras bac-
ditivas y visuales. Adems se objetiva fibrilacin auricular terias a tener en cuenta son Treponema pallidum responsable
paroxstica con respuesta ventricular rpida no conocida. de la sfilis terciaria y las causantes de meningitis: Neisseria
y se realiza cardioversin farmacolgica efectiva en el rea meningitidis y Streptococcus pneumoniae. Dentro de los virus,
de Observacin. podemos destacar el enterovirus, VHS-2, VIH, aquellos
A la exploracin fsica se objetiva fiebre de hasta 38.5C, transmitidos por artrpodos, VVZ, VEB, etc. aunque se
con aceptable estado general, buena hidratacin de piel suelen manifestar con un curso ms subagudo. Una mi-
y mucosas con leve tinte ictrico de mucosa conjuntival. nora de casos se relaciona con infecciones por hongos o
Se visualizan telangiectasias sin eritema palmar. Tonos parsitos, que por las caractersticas de nuestro caso, se-
cardacos rtmicos sin soplos y murmullo vesicular conser- ran altamente improbables. 2) Los fenmenos vasculares
vado. Abdomen anodino. Miembros inferiores con ligero implicados pueden ser infartos o hemorragias medulares,
edema pretibial. En la exploracin neurolgica: est cons- dndose con mayor frecuencia en aquellos con ciertas mal-
ciente y orientado, sin alteraciones del lenguaje con pares formaciones congnitas. 3) Otro proceso que podra con-
craneales normales; tetraparesia de predominio crural e tribuir a la clnica de nuestro paciente sera la enfermedad
izquierda (MSD proximal y distal: 3/5, MSI proximal y osteoarticular degenerativa avanzada o hernias discales,
distal: 2/5, MID proximal: 3/5 y MID distal: 2/5, MII aunque no justificara la fiebre ni la rapidez del cuadro. 4)
proximal y distal 2/5); ROT se encuentran globalmente Las neoplasias como linfoma, leucemia o metstasis seas
dbiles; y reflejos cutneos plantares flexores bilaterales; no seran otras entidades a descartar, con una forma de pre-
presenta amiotrofias ni alteraciones en la sensibilidad. No sentacin atpica de las mismas. 5) Otras causas relacio-
se puede explorar la marcja. nadas con este cuadro son las enfermedades autoinmunes
En las pruebas complementarias realizadas al ingreso des- que dan lugar a mielitis transversa (Behcet, SAF, LES,..) o
tacan: Hb 13.8 g/dl; plaquetas 90.000 /mm3; leucocitos inflamatorias como sarcoidosis, aunque nuestro paciente

86
CAPTULO 6
INFECCIONES OSTEOARTICULARES

no parece cumplir criterios para ellas. Por ltimo, debemos generativos en la articulacin coxo-femoral izquierda, con
pensar en otras causas menos comunes (alteraciones me- derrame sinovial moderado y edema en psoas iliacos y
tablicas hereditarias, hipopotasemia peridica familiar) obturadores externos y glteo menor izquierdo. Adems
si despus de un amplio estudio, no hemos llegado a un de probable infarto lineal en tercio proximal de la difisis
diagnstico claro. femoral. Por lo que se decide prolongar el tratamiento an-
tibitico hasta completar 4 semanas ms con ceftriaxona y
Evolucin levofloxacino.
Al mes de finalizar antibioterapia se decide implantacin
Al ingreso tras la obtencin de cultivos, se inicia antibio- de PTC de cadera izquierda, que se realiza sin complica-
terapia emprica con ceftriaxona 2g/12h y levofloxacino ciones, estando el paciente actualmente deambulando y
500/24h, que posteriormente se modifica a linezolid 600 siendo independiente para sus actividades de la vida diaria
mg/12h tras identificarse Staphylococcus aureus por tcnica
de MALDI-TOF en hemocultivos. Diagnstico final
Se le realiza RNM de columna vertebral donde se iden-
tifican varias colecciones epidurales tanto a nivel cervical Abscesos epidurales mltiples (cervical, dorsal y lum-
(C5-C7) como dorsal (C7-T2, T6-T7, T10-T11), anterio- bar) por Staphylococcus aureus.
res y posteriores, con colapso del espacio subaracnoideo y Osteomielitis vertebral mltiple secundaria. Abscesos
compresin leve medular, as como alteracin de la seal del psoas bilateral.
de las vrtebras en los segmentos estudiados (figura 1), y a Artrosis severa de articulacin coxo-femoral izquierda.
nivel del segmento lumbar se visualiza un absceso epidural Tetraparesia aguda secundaria a abscesos epidurales.
lumbar de L2 a L5 con compresin de las races de la cola
de caballo, con edema seo en L3, L4 y L5 y foco de pio- Discusin
miositis paravertebral izquierdo en L5-S1 (figura 2) junto
con 2 pequeos abscesos del psoas bilateral (< 2 cm). La El absceso epidural espinal (AES) se caracteriza por el acu-
RNM craneal estaba dentro de los lmites de la normali- mulo de material purulento en el espacio epidural (entre
dad. duramadre y los lmites osteoligamentosos del canal espi-
Ante estos hallazgos en pruebas de imagen se decide inter- nal). Se localizan en el 80% de los casos a nivel posterior
vencin por Neurociruga (laminectoma mltiple a nivel (ms frecuente por va hematgena) y en el 20% a nivel
de C3-C4, T1 y L4-L5 y drenajes de colecciones epidura- anterior (ms frecuente por osteomielitis).
les). En cultivos de muestras de absceso epidural as como Su incidencia es baja, se encuentra alrededor del 0.2-10
en hemocultivos se aslan Staphyloccocus aureus MS, modifi- casos/10.000 ingresos hospitalarios. Sin embargo existe un
cndose antibioterapia a cloxacilina 2 gr/4h. aumento de incidencia en los ltimos aos debido al enve-
Se completa estudio con serologas a hepatitis, VIH, CMV, jecimiento de la poblacin, a un creciente nmero de fac-
VEB, herpes virus y sfilis que fueron negativas y una eco- tores como son la adiccin a drogas intravenosas y la infec-
cardiografa transtorcica con hallazgos de insuficiencia cin por VIH, mayor incidencia de lesiones traumticas, el
mitral y artica leve, FVI conservada y estudio negativo mayor nmero de tcnicas invasivas a nivel de raquis, ma-
para endocarditis yor sensibilidad de las pruebas de imagen y el cambio de
Once das despus de la ciruga, presenta empeoramiento patrones de referencia hospitalaria que concentra a estos
neurolgico progresivo realizndose nueva RNM crvi- pacientes en centros terciarios. Se presentan generalmente
co-dorsal donde se objetiva una fstula de LCR y coleccin sobre los 50 aos, con mayor prevalencia entre los 50-70
epidural dorsal (figura 3), por lo que es reintervenido (rea- aos, siendo ms frecuente en hombres que en mujeres.
pertura dorsal y drenaje de coleccin epidural). Dos sema- Entre las enfermedades sistmicas predisponentes desta-
nas despus del ingreso, se objetiva franca mejora clnica, can: la diabetes, la adiccin a drogas intravenosas, la en-
analtica y radiolgica de las colecciones a todos los niveles, fermedad renal terminal, el alcoholismo, la infeccin por
as como negativizacin de los hemocultivos. Tras comple- VIH, el cncer, la obesidad mrbida, la utilizacin crnica
tar 6 semanas con cloxacilina a dosis altas, se modifica a de corticoides y la septicemia. Los abscesos cutneos y
va oral con levofloxacino. fornculos son la fuente ms comn de infeccin. Entre
Durante su evolucin reaparece el dolor de caractersti- las circunstancias locales que predisponen a una infeccin
cas mecnicas a nivel de la cadera izquierda, solicitndose en el espacio epidural destacan: los traumatismos de co-
una RMN de dicha zona donde se aprecian cambios de- lumna recientes, la ciruga de columna, las inyecciones ex-

87
CAPTULO 6
INFECCIONES OSTEOARTICULARES

tratecales, y la insercin de catteres. En nuestro caso se


trataba de un paciente varn de 66 aos con historia de
abuso de alcohol.
El AES se debe sospechar ante pacientes que presenten
dolor raqudeo, fiebre y/o dficit neurolgico. El dolor
raqudeo est presente en el 70-90%. Podemos detectar
dolor a la percusin del rea afectada, contractura de la
musculatura paraespinal, limitacin de la movilidad de la
columna y dificultad para la deambulacin. La fiebre apa-
rece en el 33% % de los casos, pudiendo estar ausente en
algunos pacientes, lo cual favorece la demora del diagns-
tico. Hasta un tercio de los pacientes presentan dficit neu- Figura 1. RNM crvico-dorsal: Colecciones epidurales a nivel cervical (C5-C7),
dorsal (C7-T2, T6-T7, T10-T11), anteriores y posteriores.
rolgico significativo que puede progresar en horas o das.
Los sntomas y signos del absceso epidural espinal progre-
san en cuatro fases:Estadio 1: dolor a nivel del segmento
espinal afectado, Estadio 2: dolor radicular irradiando des-
de el rea espinal afectada.Estadio 3: debilidad motora y
dficit sensorial por debajo del nivel de la lesin, y afecta-
cin del esfnter vesical y/o rectal. Estadio 4: parlisis. En
el caso de nuestro paciente, el cuadro comenz con dolor
progresivo durante dos semanas al que asoci un dficit
neurolgico que le llev a tetraparesia en 72 horas, apare-
ciendo la fiebre como ltimo dato.
A nivel diagnstico, debemos sealar la importancia de la
precocidad del mismo. Es importante su diagnstico pre-
coz, pues en el caso de lesin neurolgica se debe realizar Figura 2. RNM lumbar: Absceso epidural lumbar de L2 a L5 con compresin de
las races de la cola de caballo, con edema seo en L3, L4 y L5.
descompresin y desbridamiento quirrgico urgente aso-
ciado a antibioterapia durante un tiempo prolongado. En
el caso de nuestro paciente, dicho diagnstico se demor
casi dos semanas en las cuales acudi en dos ocasiones al
servicio de urgencias. Las pruebas de laboratorio pueden
ayudar a dirigir nuestra sospecha clnica. Normalmente
aparece una elevacin de la VSG, as como de la PCR la
cual es ms precoz. Es frecuente la leucocitosis perifrica
con desviacin a la izquierda, hasta en un 60-77% de los
pacientes.
En esta patologa, las pruebas de imagen juegan un papel
fundamental siendo la clave para el diagnstico de la mis- Figura 3. RNM cervico-dorsal: Fstula de LCR y coleccin epidural dorsal
ma. En los casos de sospecha se debe realizar una RMN
con gadolinio, siempre procurando localizar el nivel teri- no es una buena herramienta para su diagnstico, pero es
co de la lesin para no cometer errores con el nivel estu- importante realizarla de inicio para detectar cuerpos ex-
diado.La sensibilidad de la RMN con gadolinio ronda el traos. El TAC con mielografa puede ser una opcin en
90-95%, por lo que se considera la prueba diagnstica y de pacientes con contraindicacin para RMN.
seguimiento de eleccin, siendo su especificidad tambin Por otra parte cabe sealar que siempre deben realizarse
elevada. Sus ventajas respecto a la RMN sin contraste son hemocultivos previos al inicio del tratamiento, pues en el
que permite visualizar la zona de separacin que hay entre 65-95% de los casos se asla el germen causante. Asimis-
el absceso y el saco tecal y los elementos nerviosos. Adems mo. el cultivo tomado de muestras quirrgicas es positivo
de identificar una posible osteomielitis asociada, aunque en el 82-90%. Los patgenos ms frecuentemente aislados
a veces sobrevalora la existencia y extensin de material son: Staphylococcus aureus 65% (de estos 20-40% SAMR), ba-
purulento en el canal espinal.La radiografa convencional cilos gram negativos 16%, Streptococcus <10%, Staphylococcus

88
CAPTULO 6
INFECCIONES OSTEOARTICULARES

coagulasa negativos <5%, anaerobios u otros. Bibliografa


Se debe comenzar el tratamiento antibitico en cuanto
se obtengan los correspondientes cultivos, con antibiti- 1. Darouiche RO. Spinal Epidural Abscess. N Engl J
cos de amplio espectro va intravenosa para cubrir cocos Med 2006; 355: 2012-20.
gram positivos (Staphylococcus y Streptococcus). En pacientes 2. Reihsaus E, Waldbaur H, Seeling W.Spinal epidural
inmunodeprimidos, en adictos a drogas intravenosas, en abscess: a meta-analysis of 915 patients. Neurosurg
los que hayan sufrido una infeccin reciente o hayan sido Rev 2000; 232:175204
sometidos a una manipulacin en el tracto urogenital, hay 3. Davis DP, Wold RM, Patel RJ, Tran AJ, Tokhi RN,
que aadir un antibitico eficaz contra bacterias gram ne- Chan TC et al. The clinical presentation and impact
gativas. El tratamiento debe administrarse un mnimo de of diagnostic delays on emergency department pa-
4 semanas. tients with spinal epidural abscess. J Emerg Med 2004;
El tratamiento de eleccin es la antibioterapia precoz, se- 26:285-91.
guida de laminectoma descompresiva y drenaje quirrgi- 4. Krishnamohan P, Berger JR. Spinal Epidural Abscess.
co amplio. Slo en pacientes seleccionados, el uso de an- Curr Infect Dis Rep 2014; 16:436
tibioterapia prolongada puede ser una alternativa, como 5. Roel JE, Fernndez L, Abraham L, Cuesta MC, Bou-
en ciertas situaciones como pacientes neurolgicamente za G, Boero A et al. Diagnstico y tratamiento del
estables con organismo patgeno identificado, acceso a absceso epidural espinal. Medicina (B Aires) 1999; 59:
imgenes de RM o TAC para el seguimiento y consulta 59-62
neuroquirrgica disponible, malos candidatos para la ci- 6. Bluman EM, Palumbo M, Lucas PR. Spinal epidu-
ruga, extensin del absceso que contraindique la ciruga o ral abscess in adults J Am Acad Orthop Surg 2004;
parlisis completa de tres o ms das de duracin. 12:155-63.
Una vez que se desarrolla la parlisis puede ser rpida- 7. Patel AR, Alton TB, Bransford RJ, Lee MJ, Bellabar-
mente irreversible, por lo cual el paciente requiere una ba CB, Chapman JR. Spinal epidural abscesses: risk
intervencin urgente si progresa la debilidad o aparece d- factors, medical versus surgical management, a retros-
ficit neurolgico.A nivel quirrgico lo ms importante es pective review of 128 cases. Spine J 2014; 14: 326-30
la descompresin urgente y el drenaje del absceso. El acto 8. Alton TB, Patel AR, Bransford RJ, Bellabarba CB,
quirrgico depende sobre todo de la localizacin y de la Lee MJ, Chapman JR. Is there a difference in neuro-
estabilidad de la columna. La estabilidad de la columna logic outcome in medical versus early operative mana-
suele preservarse tras la descompresin (ms en los absce- gement of cervical epidural abscesses? Spine J 2015;
sos posteriores). En los casos de inestabilidad ser necesaria 15:10-7
la artrodesis. Nuestro paciente sufra una tetraparesia de
menos de 72 horas de evolucin provocada por abscesos
mltiples y tcnicamente era posible la ciruga, por lo que
se opt por el tratamiento general de eleccin.
A nivel pronstico, entre un 4-22% de los pacientes perma-
necern con un dficit neurolgico irreversible, y la morta-
lidad se encuentra alrededor del 5-15%. En estos pacien-
tes pueden presentarse otras complicaciones como lceras
por decbito, infecciones del tracto urinario, neumonas y
trombosis venosa profunda.El predictor ms importante
de la evolucin neurolgica del paciente es el estado previo
a la ciruga.
Como conclusin podemos decir que el absceso epidural
es una patologa poco frecuente, pero grave, que presenta
una mortalidad y morbilidad muy elevadas. sobre todo de-
bido a secuelas neurolgicas y cuyo pronstico est relacio-
nado con la rapidez en su diagnstico y tratamiento. Por
eso hay que enfatizar en la importancia de mantener una
sospecha diagnstica elevada, teniendo en cuenta especial-
mente los factores de riesgo ms comunes.

89
Varn de 69 aos de edad
con dolor lumbar de larga evolucin
Fernndez lvarez, M; Navo Botas, M;
Martnez Egea, I; Arenas Miras, MM.
Hospital Universitario Virgen de Las Nieves. Granada.

Caso clnico a nivel perimaleolar, sin signos de trombosis venosa pro-


funda, estando los pulsos conservados y simtricos. Apor-
Varn de 69 aos de edad, que acude a Urgencias por ta una resonancia magntica nuclear (RMN) informada
dolor lumbar irradiado a ambos MMII de 10 meses de como espondilodiscartrosis lumbar, de predominio L4-L5
evolucin. Como Antecedentes Personales de inters des- y L5-S1. A este nivel existe una hernia de ncleo pulposo
taca hipertensin arterial con cardiopata hipertensiva, (HNP) extruda con compromiso de ambos recesos latera-
diabetes mellitus tipo 2 no insulin-dependiente, dislipemia les, ms el izquierdo.
mixta, EPOC tipo enfisema moderado (FEV1 76%), ar- Se solicita analtica bsica de sangre y orina en la que
tritis gotosa y apendicectomizado. Estaba en estudio por destaca leucopenia (2750 leucocitos/mm3 con 61,4% de
parte de Urologa por hematuria, diagnosticada por cito- polimorfonucleares, 21,5% de Linfocitos, 14,9% de mo-
loga de carcinoma urotelial. Ex-fumador desde hace 30 nocitos), Hemoglobina 9.7 g/dl, VCM 88,7, plaquetas
aos, no antecedentes epidemiolgicos de inters: niega 164000/mm3, glucosa 185 mg/dl, urea 31 mg/dl, creati-
relaciones sexuales de riesgo, no viajes recientes al extran- nina 1,15 mg/dl, ionograma normal, protenas totales 5,5
jero, no contacto con animales ni ingesta de productos no g/l, calcio 8,5 mg/dl, corregido 9,5, procalcitonina 0,05
pasteurizados. Vive en ambiente urbano. Se encuentra en ng/ml. Orina con sangre ++, resto negativo.
tratamiento con AAS 100mg, candesartan 32/ hidrocloro-
tiazida 12,5mg, simvastatina 20mg y metformina 850mg. Diagnstico diferencial
Alergia a paracetamol.
Paciente ingresado a cargo de Neurociruga por clnica Nos encontramos por tanto ante un paciente con una lum-
de 10 meses de evolucin de lumbalgia intensa, con do- balgia de larga data, refractaria a tratamiento analgsico,
lor irradiado por cara lateral del muslo y pierna izquierda asociando clnica neurolgica, atribuida a hernia discal
hasta cara externa del pie, con parestesias a dicho nivel y que se decide intervenir. Adems destaca el hallazgo de
en dorso del pie. Dos meses antes de su ingreso comienza una bicitopenia (anemia + leucopenia) en un paciente con
con dolor en la pierna contralateral, que le limita su vida sospecha de neoplasia de origen urotelial.
diaria, por lo que est en seguimiento por la Unidad del
Dolor sin mejora significativa. Asimismo refiere disminu- Tomando como sntoma gua la lumbalgia, debemos plan-
cin de la fuerza en ambos miembros inferiores de una tearnos el siguiente diagnstico diferencial, de mayor a
semana de evolucin. Presenta adems retencin urinaria menor frecuencia:
desde hace diez das por lo que ha requerido de sondaje
vesical. En la anamnesis dirigida destaca aparicin de fe- Enfermedades que originan estrechamiento del canal
brcula de manera ocasional de unos meses de evolucin, o de los agujeros de conjuncin, como la hernia discal,
sin perfil bacterimico. Niega otra clnica en la anamnesis la hipertrofia de las carillas articulares.
dirigida por rganos y aparatos. En la exploracin fsica Traumatismos, sobrecarga funcional/postural, con-
paciente hemodinmicamente estable, afebril. Buen es- tracturas musculares, fracturas vertebrales.
tado general, consciente y orientado. Balance muscular Enfermedades degenerativas como artrosis.
normal con claudicacin de MMII en maniobras antigra- Espondiloartropatas inflamatorias
vitatorias por dolor, no alteraciones de la sensibilidad, re- Tumores, tanto benignos como malignos, principal-
flejos osteotendinosos presentes y simtricos salvo aquleo mente mieloma, sarcoma y metstasis.
izquierdo que se encuentra abolido, Lassegue y Braggard Infecciones: espondilodiscitis pigenas, tuberculosa o
izquierdos positivos. Auscultacin cardiaca rtmica sin so- brucelar.
plos, respiratoria con murmullo vesicular conservado con Miscelnea: enfermedades del metabolismo seo (os-
crepitantes finos teleinspiratorios en bases. Abdomen ano- teoporosis, enfermedad de Paget...), anomalas congnitas
dino. Miembros inferiores con edemas con mnima fvea de la columna lumbar, dolor referido de origen visceral.

90
CAPTULO 6
INFECCIONES OSTEOARTICULARES

Figura 1. TAC que muestra lesin con destruccin sea.

Figura 3. RMN que muestra lisis a nivel L5-S1 junto con zonas abscesificadas.

L5-S1 (Figura 1), que podra estar en relacin con cambios


degenerativos avanzados, si bien no se puede descartar la
existencia de un proceso neoplsico o infeccioso en dicha
localizacin. Se realiza entonces interconsulta a Medicina
Interna por parte de Neurociruga para completar el estu-
dio. En analtica completa de sangre destaca una LDH de
845 U/l, siendo negativos o normales los marcadores tu-
morales, el proteinograma e inmunoglobulinas. La tomo-
Figura 2. PET que muestra aumento de captacin de contraste a nivel L5-S1. grafa por emisin de positrones (PET) que muestra focos
hipermetablicos que se correlacionan por TAC con lesio-
Teniendo en cuenta los antecedentes del paciente y la nes lticas localizadas en L5-S1, SUV 11.53 y 11.18 respec-
presencia de bicitopenia, una de las primeras posibilida- tivamente. En el resto del estudio no se aprecian otros focos
des diagnsticas es que se trate de una lesin degenerativa hipermetablicos que sugieran la presencia de actividad
como informa la RMN, y que el origen de la bicitopenia tumoral macroscpica (Figura 2). Ante estos hallazgos se
sea yatrgeno; pero debemos plantearnos tambin la posi- realiza biopsia con aguja gruesa sea de la lesin L5 guia-
bilidad de lesiones de origen metastsico con afectacin de da por TC y se enva una muestra para Anatoma Patol-
mdula sea. gica y Microbiologa que se cultiva en medios habituales
y medios para micobacterias. Se solicita adems IGRA y
Evolucin hemocultivo coincidiendo con un pico febril. En espera de
resultados, el paciente comienza con mayor deterioro del
Durante su estancia en la planta de Neurociruga el pa- estado general as como aumento de dolor en zona lumbar
ciente comienza con episodios de desorientacin tempo- por lo que se realiza RMN que muestra un patrn des-
roespacial autolimitados, asociando en alguna ocasin tructivo de las plataformas vertebrales con extenso edema
alucinaciones visuales, as como deterioro progresivo del seo a nivel de L5-S1 (Figura 3). Tras la administracin
estado general, asociando dolor abdominal y sensacin de contraste se delimitan colecciones abscesificadas en
nauseosa. Se realiza analtica de control donde se objetiva disco, espacio epidural anterior (donde comprime saco
empeoramiento de la funcin renal (cifras de creatinina en tecal) y paravertebral (psoas izquierdo). Los hallazgos son
3,19 mg/dl), y se realiza una tomografa axial computa- compatibles con un proceso infeccioso (espondilodiscitis).
rizada (TAC) abdominal donde informan lesin ltica en Ante este resultado, se programa puncin del absceso del

91
CAPTULO 6
INFECCIONES OSTEOARTICULARES

psoas izquierdo que se enva a microbiologa para tincin La forma ms comn es la enfermedad de Pott (aproxima-
de Gram, cultivo y PCR de tuberculosis. damente el 50% del total de casos). Afecta principalmente
Recibimos los resultados de los hemocultivos que fueron a la regin dorsal inferior y lumbar superior. La infeccin
negativos, IGRA positivo y la microbiologa de la biopsia normalmente comienza con inflamacin de zona ante-
sea que es negativa para bacterias habituales, con creci- rior de la articulacin intervertebral, propagndose bajo
miento de Mycobacterium tuberculosis complex sensible a anti- el ligamento anterior extendindose al cuerpo vertebral
tuberculosos de primera lnea, por lo que se diagnostica de adyacente. Una vez afectadas dos vrtebras adyacentes, la
espondilitis tuberculosa y se inicia tratamiento con Rim- infeccin se disemina al espacio discal intervertebral. Fi-
star (Etambutol 275mg, Isoniazida 75mg, Pirazinamida nalmente, se produce necrosis del tejido avascular discal y
400mg, Rifampicina 150mg). posterior colapso vertebral, dando lugar a cifosis lumbar
Se reciben resultados de las muestras obtenidas del absceso que distorsiona la anatoma del canal vertebral, con el pos-
del psoas con baciloscopia, PCR y cultivo positivos para terior riesgo de compresin que esto conlleva. Es tambin
Mycobacterium tuberculosis complex. comn la formacin de un absceso fro en el lugar de in-
Se completa estudio con la realizacin de baciloscopias en feccin. El sntoma ms comn es dolor local, que aumen-
esputo, que son negativas en 3 ocasiones con cultivo nega- ta progresivamente a lo largo de semanas y/o meses, en la
tivo para TB aunque la PCR del esputo para Mycobacterium mayora de los casos asociado a contractura paravertebral
tuberculosis complex fue positiva. Se realiza TAC trax infor- y rigidez. En menos del 40% de los casos, se puede asociar
mado como: Lesin de aspecto cicatricial apical izquierda fiebre y prdida ponderal. Con frecuencia el diagnstico
con calcificacin en su interior. Se observan microndulos de enfermedad de Pott se retrasa en el tiempo dado el cur-
de predominio subpleural en ambos lbulos superiores for- so subagudo de la infeccin.
mando un patrn en "pseudo-rbol en brote" Granulomas En la pruebas de imagen no existen hallazgos radiogrficos
calcificados bilaterales en probable relacin con TBC pre- patognomnicos, pero pueden ser tiles para identificar y
via. Ganglios mediastnicos de tamao no significativo. establecer la anatoma de la TBC esqueltica.
Tras el inicio del tratamiento antituberculoso y el drena- El diagnstico se establece con microscopa y cultivo de
je del absceso del psoas, el paciente evoluciona de forma material infectado. El tejido debe obtenerse con aspirado
favorable, con adecuada tolerancia a la medicacin, sin con aguja o con biopsia (guiada por Ecografa o TAC).
nuevas complicaciones, y con mejora de los parmetros El tratamiento consiste en terapia antituberculosa clsi-
analticos. ca. La duracin ptima de la misma se desconoce. Para
pacientes tratados con frmacos de primera lnea en au-
Diagnstico final sencia de afectacin extensa y enfermedad avanzada, se
recomienda 6 meses de tratamiento. Una mayor duracin
Espondilitis tuberculosa (Mal de Pott). Tuberculosis pul- del mismo (de 9 a 12 meses) se aconseja en pacientes cuyo
monar. tratamiento no incluye rifampicina o en aquellos donde la
afectacin sea ms extensa.
Discusin El tratamiento quirrgico est indicado en aquellos pa-
cientes que presentan afectacin vertebral y dficit neuro-
La tuberculosis esqueltica engloba la afectacin sea y lgicos o empeoramiento del dficit neurolgico a pesar de
articular. Supone un 10-35% de los casos de tuberculosis tratamiento correcto, cifosis mayor de 40 en el momento
extrapulmonar, y un 2% del total de casos. En pases desa- de presentacin, y en abscesos fros en pared torcica.
rrollados existe un mayor nmero de casos de tuberculosis Nuestro caso es interesante por la forma de presentacin,
extrapulmonar en paciente inmigrantes de reas endmi- con los resultados equvocos en pruebas de imagen, as
cas de TB. Durante la primoinfeccin por Mycobacterium como la localizacin de las lesiones vertebrales, ya que sue-
tuberculosis, se produce una siembra de microorganismos en len ser a niveles ms altos, y sin un antecedente epidemio-
hueso y/o tejido sinovial, permaneciendo de forma latente. lgico claro, tratndose seguramente de una reactivacin
En condiciones de malnutricin, edad avanzada, infeccin de una TB previa. En todo paciente con sospecha de una
por VIH o fallo renal, se puede producir una reactivacin espondilodiscitis de evolucin subaguda-crnica debemos
de la infeccin latente, dando lugar a la presentacin clni- tener presente la posibilidad de una tuberculosis, aunque
ca de la enfermedad. no se trate de pacientes inmunodeprimidos, ni con antece-
La tuberculosis esqueltica puede presentarse en forma de dentes epidemiolgicos de riesgo.
espondilitis (enfermedad de Pott), artritis y osteomielitis.

92
CAPTULO 6
INFECCIONES OSTEOARTICULARES

Bibliografa

1. Watts HG, Lifeso RM. Tuberculosis of bones and


joints. J Bone Joint Surg Am 1996; 78:288-98.
2. Sharma SK, Mohan A. Extrapulmonary tuberculosis.
Indian J Med Res 2004; 120:316-53.
3. Trecarichi EM, Di Meco E, Mazzotta V, Fantoni M.
Tuberculous spondylodiscitis: epidemiology, clinical
features, treatment, and outcome. Eur Rev Med Phar-
macol Sci 2012; 16 Suppl 2:58-72.
4. Turgut M. Spinal tuberculosis (Pott's disease): its clini-
cal presentation, surgical management, and outcome.
A survey study on 694 patients. Neurosurg Rev 2001;
24:8-13.
5. Weaver P, Lifeso RM. The radiological diagnosis of
tuberculosis of the adult spine. Skeletal Radiol 1984;
12:178-86.
6. Lifeso RM, Weaver P, Harder EH. Tuberculous spond-
ylitis in adults. J Bone Joint Surg Am 1985; 67:1405-
13.
7. Khoo LT, Mikawa K, Fessler RG. A surgical revisi-
tation of Pott distemper of the spine. Spine J 2003;
3:130-45.
8. Jain R, Sawhney S, Berry M. Computed tomography
of vertebral tuberculosis: patterns of bone destruction.
Clin Radiol 1993; 47:196-9.
9. Kim NH, Lee HM, Suh JS. Magnetic resonance ima-
ging for the diagnosis of tuberculous spondylitis. Spine
(Phila Pa 1976) 1994; 19:2451-5.
10. Colmenero JD, Ruiz-Mesa JD, Sanjuan-Jimenez R, et
al. Establishing the diagnosis of tuberculous vertebral
osteomyelitis. Eur Spine J 2013; 22 Suppl 4:579-86.
11. Merino P, Candel FJ, Gestoso I, et al. Microbiologi-
cal diagnosis of spinal tuberculosis. Int Orthop 2012;
36:233-8.
12. Upadhyay SS, Saji MJ, Yau AC. Duration of antitu-
berculosis chemotherapy in conjunction with radical
surgery in the management of spinal tuberculosis. Spi-
ne (Phila Pa 1976) 1996; 21:1898-903
13. Oguz E, Sehirlioglu A, Altinmakas M, et al. A new
classification and guide for surgical treatment of spinal
tuberculosis. Int Orthop 2008; 32:127-33.

93
CAPTULO 7
ENDOCARDITIS, INFECCIONES ASOCIADAS
A DISPOSITIVOS ENDOVASCULARES
Y OTRAS INFECCIONES CARDIOVASCULARES
Sndrome febril y dolor torcico:
no siempre es lo que parece
Jimnez Recio, A; Rivas Snchez, B;
Reguera Iglesia, JM.
Hospital Regional Universitario de Mlaga.

Caso clnico

Paciente varn de 15 aos, sin hbitos txicos ni alergias


medicamentosas conocidas, con antecedentes de rinitis
alrgica (gramneas y olivo). Estudiante de procedencia ru-
ral. Doce das antes de su ingreso present cuadro febril
con odinofagia, siendo diagnosticado por su mdico de fa-
milia de faringoamigdalitis aguda y recibiendo tratamiento
con amoxicilina-clavulnico durante tres das, con resolu-
cin de los sntomas.
Encontrndose previamente bien, ingresa en el hospital
por presentar en los ltimos dos das cuadro febril de hasta
39 con sudoracin profusa, sensacin de malestar general,
y dolor retroesternal irradiado a hombro izquierdo y espal-
da, que mejora en sedestacin y cuando se inclina hacia
delante, y empeora con la inspiracin. Figura 2.
En la exploracin fsica estaba consciente, orientado y con
moderada sensacin de enfermedad. No presentaba alte- Un ECG (figura 1) puso de manifiesto un ritmo sinusal
raciones en cabeza, cuello, boca y faringe, salvo hipertrofia junto a una elevacin del segmento ST con concavidad
amigdalar moderada. La auscultacin cardiaca era rtmica superior en mltiples derivaciones. La radiografa de trax
y sin soplos, con tonos apagados, y en la auscultacin res- (figura 2) mostraba un incremento de la silueta cardiaca y
piratoria presentaba hipoventilacin bibasal. El abdomen derrame pleural izquierdo de escasa cuanta. Se estableci
era blando y depresible sin dolor, masas u organomegalias. el diagnostico sindrmico de pericarditis aguda y el pa-
Extremidades normales. ciente quedo ingresado.
En las exploraciones complementarias presentaba: Hb:
13.6 g/dl, hematocrito: 40%, plaquetas: 236.000 mm3. Diagnstico diferencial
Leucocitos: 20.400 mm3 (N 84%, L 10%). Test de coagu-
lacin normales. Bioqumica: glucosa: 110 mg/dl, creati- La pericarditis aguda (PA) es un sndrome clnico plurietio-
nina: 0.91 mg/dl, iones normales. Perfil heptico: bilirru- lgico que se manifiesta por dolor torcico, roce pericrdi-
bina total: 0.48 mg/dl, GOT: 19 U/L, GPT: 16 U/L, FA: co y cambios evolutivos de la repolarizacin en el electro-
180 U/L, GGT: 155 U/L. CPK: 84 U/L, troponina 0.55 cardiograma (ECG). El diagnstico de pericarditis exige,
ng/ml, LDH: 192 U/L. PCR: 243 mg/L. Procalcitonina: al menos, 2 de estos 3 elementos y, aunque la auscultacin
2.23 ng/ml de un roce pericrdico permite, por s sola, establecer el
diagnstico, su ausencia nunca la descarta. La manifesta-
cin clnica fundamental de la pericarditis aguda es el do-
lor torcico, el cual se puede prestar a confusin con el de
un infarto agudo de miocardio (IAM). Sin embargo, a dife-
rencia del infarto, el dolor es de instauracin rpida, pero
no brusca, aunque intenso no es opresivo, se modifica con
la respiracin y los cambios posturales, y no se acompaa
de sintomatologa vegetativa. El ECG esta alterado en ms
del 80% de los casos de PA y consiste en una elevacin di-
Figura 1. fusa del segmento ST de concavidad superior (curva de le-

96
CAPTULO 7
ENDOCARDITIS, INFECCIONES ASOCIADAS
A DISPOSITIVOS ENDOVASCULARES
Y OTRAS INFECCIONES CARDIOVASCULARES

sin subepicrdica) con ondas T positivas. El segmento PR


puede estar descendido (curva de lesin auricular). Estos
cambios suelen durar horas o unos pocos das. A diferencia
del IAM, no se evidencia onda Q ni imagen especular. Los
marcadores de lesin miocrdica (CPK y troponina) suelen
estar normales o ligeramente elevados, debido a la infla-
macin epicrdica acompaante, pero nunca alcanzan las
cifras que se observan en el IAM.
El dolor pleurtico originado por un derrame pleural, una
neumona o un neumotrax, puede tener caractersticas
comunes a la pericarditis, pero el dolor se localiza ms la-
teralmente en el trax. Sin embargo, es frecuente la aso-
ciacin de pericarditis con pleuritis (pleuro-pericarditis)
por lo que el dolor puede tener caractersticas de ambos.
Patologas de la aorta, esfago y pared torcica presenta
caractersticas clnicas y radiolgicas que rara vez plantean
confusin con el diagnostico de PA.
Una vez establecido el diagnstico sindrmico de pericar- Figura 3.
ditis, hay que intentar establecer el diagnstico etiolgico.
La primera consideracin que hay que tener en cuenta es trminos relativos el taponamiento es ms frecuente en las
valorar si hay alguna enfermedad de base que pueda ser pericarditis especficas, no es raro en las pericarditis idiop-
la causa de la pericarditis. Los pacientes con insuficiencia ticas o virales y, en trminos absolutos, sta es la etiologa
renal, IAM reciente, ciruga cardaca, radioterapia en el ms frecuente. Otros aspectos clnicos de inters como la
trax, neoplasia conocida o enfermedades del colgeno, presencia de derrame pleural y el curso clnico prolongado
pueden desarrollar un cuadro de PA que siempre hay que son totalmente inespecficos, aunque una pericarditis que
considerar secundario a la enfermedad de base en primer se autolimita en pocos das es prcticamente siempre de
lugar. origen viral o idioptica.
Sin embargo, la situacin ms frecuente es que la pericar- El ultimo escaln en el diagnstico etiolgico de la peri-
ditis se presente como una enfermedad primaria. En es- carditis aguda seria la pericardiocentesis y la biopsia pe-
tos casos, la gran mayora (ms del 90%) de pericarditis ricrdica: ambas pueden aportar datos definitivos para el
son idiopticas o virales. A pesar del trmino idioptico, diagnostico de pericarditis especificas (purulenta, tubercu-
hay evidencia suficiente para suponer que la mayora de losa o neoplsica). Aunque su rendimiento no es muy ele-
los casos se deben a una infeccin viral o a una respuesta vado, 19% y 22% respectivamente, puede incrementarse
inmunolgica a sta. Intentar demostrar hasta el 35% cuando se practican con una finalidad no slo
esta infeccin viral representa un esfuerzo de complejidad diagnstica, sino teraputica, en pacientes que presentan
y coste tan elevado, que de ninguna manera son compen- taponamiento cardiaco.
sados, fundamentalmente, por el hecho de que la enferme-
dad se resuelve de manera espontnea, sin secuelas, en la Evolucin
gran mayora de los pacientes.
Existen una serie de datos clnicos que pueden ser muy A las 24 horas del ingreso comienza con signos clnicos de
tiles para orientar el diagnostico etiolgico de una PA. compromiso hemodinmico, hipotensin, taquicardia, ta-
La presencia de una infeccin intratorcica (neumona o quipnea, junto a ingurgitacin yugular y pulso paradjico.
empiema), obliga a descartar una pericarditis purulenta. Se realizo un ETT urgente objetivndose un derrame pe-
Otro aspecto muy importante a valorar ser el contexto ricrdico moderado-severo global con colapso de cavida-
geogrfico/epidemiolgico: la pericarditis tuberculosa es des derechas, variaciones significativas en el llenado mitral
muy poco frecuente en Espaa, alrededor del 4% del to- con la respiracin y vena cava inferior dilatada con colapso
tal de pericarditis, pero en algunas reas de frica puede parcial, compatibles con taponamiento ecocardiogrfico.
suponer el 80 % de las pericarditis. Por el contrario, otros Dada la situacin de taponamiento clnico y ecocardio-
datos clnicos son totalmente inespecficos, como la presen- grfico se realizo de forma urgente una pericardiocentesis
cia de derrame o de taponamiento cardiaco. Aunque en con salida manual de aproximadamente 500 cc de liquido

97
CAPTULO 7
ENDOCARDITIS, INFECCIONES ASOCIADAS
A DISPOSITIVOS ENDOVASCULARES
Y OTRAS INFECCIONES CARDIOVASCULARES

claramente purulento, y se coloc un drenaje subxifoideo. del total) fue una pericarditis purulenta3.
Tras dicha intervencin se restableci la estabilidad hemo- Clsicamente la PP era considerada una entidad que se
dinmica y se inicio tratamiento antibitico con ceftriaxo- presentaba en nios y adultos jvenes. Hasta 1945 la me-
na 1gr/12 horas y vancomicina a las mismas dosis. En un diana de edad de presentacin era en torno a 21 aos, sin
TAC de trax (figura 3) se evidenci la persistencia del de- embargo, en las series posteriores esta mediana de edad se
rrame pericrdico (aproximadamente 17 mm de espesor), ha incrementado en prcticamente 30 aos1,2.
junto a un derrame pleural bilateral de moderada cuanta, Se han descrito varios factores predisponentes para el de-
y mltiples adenopatas mediastnicas de hasta 15 mm de sarrollo de PP que han ido variando a lo largo del tiem-
dimetro. po. En una revisin de 200 autopsias de PP realizada por
A las 48 horas del drenaje pericrdico, el paciente persis- Klacsmann et al a largo de 86 aos, los pacientes fueron
ta febril y con dolor torcico. Una nueva analtica puso divididos en dos grupos. El primer grupo lo formaron los
de manifiesto una leucocitosis de 30.800 leucocitos/mm3 y pacientes diagnosticados en la era preantibitica (hasta
una elevacin de la PCR a 390 mg/L. 1943), y el segundo los diagnosticados en la era postan-
Un ETT de control mostr un derrame pericrdico de 8 tibitica (1943 hacia adelante). En la gran mayora de los
mm en cara infero-posterior y 4 mm en la antero-lateral, pacientes del primer grupo se encontr una infeccin,
sin signos de taponamiento y sin objetivarse ventana para fundamentalmente neumona, como factor de riesgo para
nueva puncin. Los hemocultivos realizados a su ingreso el desarrollo de PP, mientras que en el segundo grupo los
fueron negativos, y en el cultivo de lquido pericrdico se principales factores de riesgo fueron, la presencia de ciru-
aisl Streptococcus dysgalatiae spp. equisimilis. ga torcica reciente e insuficiencia renal crnica, adems
Ante la persistencia de los sntomas clnicos y el escaso dbi- del abuso de alcohol, la presencia de inmunosupresin y
to del drenaje, el paciente fue intervenido quirrgicamente enfermedad neoplsica4.
mediante estereotoma media. Se observ un pericardio La aparicin de la PP ocurre ms frecuentemente como
muy engrosado y se extrajo un lquido purulento muy es- consecuencia de la migracin de grmenes al saco peri-
peso y organizado que recubra el miocardio. Se realizo crdico provenientes de un foco sptico preexistente. Los
una pericardiectomia de frnico a frnico, y se conect el mecanismos para su desarrollo son mltiples: extensin
resto de la cavidad pericrdica a ambas pleuras. El estudio directa desde un foco infeccioso intratorcico o menos fre-
histolgico del pericardio fue compatible con pericarditis cuentemente subdiafragmtico, foco miocrdico, infeccin
fibrino-purulenta y un nuevo cultivo de lquido pericrdico directa por trauma o ciruga torcica, y por diseminacin
fue negativo. El postoperatorio transcurri sin incidencias, hematgena. La infeccin primaria es excepcional5.
complet 4 semanas de tratamiento antibitico parenteral, En la era preantibitica el principal organismo implicado
y fue dado de alta asintomtico. en el desarrollo de PP era el Streptococcus pneumoniae. Actual-
mente los organismos ms comnmente implicados en el
Diagnstico final desarrollo de PP son en orden de frecuencia: Staphylococcus
aureus, Haemophilus influenzae, Neisseria meningitidis y Streptococ-
Pericarditis purulenta complicada con taponamiento debi- cus pneumoniae5. Menos frecuentemente, aunque con ten-
da a Streptococcus dysgalatiae spp. equisimilis. dencia al incremento en los ltimos aos, se aslan bacilos
gram negativos entricos, Pseudomonas aeruginosa, Salmonella
Discusin spp, anaerobios y hongos, estos ltimos asociados a situa-
ciones de inmunosupresin, tratamientos prolongados con
La pericarditis purulenta (PP) podemos definirla como una corticoides y uso excesivo de antimicrobianos1.
infeccin localizada en el espacio pericrdico, con produc- En nuestro caso, nos encontramos ante PP por Streptococ-
cin de un exudado macro o microscpicamente purulen- cus dysgalactiae spp. equisimilis. El gnero Streptococcus es un
to. Es una entidad extremadamente rara en la actualidad, grupo muy heterogneo, formado por bacterias de forma
con un mal pronstico si no es diagnosticada y tratada redondeada, gram positivas, catalasa negativas, anaero-
de forma temprana. En la era preantibitica su inciden- bias facultativas, con tendencia a formar cadenas o pare-
cia era muy elevada, representando el 40% de todos los jas6. Existen numerosas especies, siendo la gran mayora
casos de pericarditis. Con la llegada de la antibioterapia de stas comensales, aunque un grupo importante pueden
su incidencia ha disminuido drsticamente1,2. Oliver et al llegar a ser patgenos para el ser humano. Han surgido
publicaron en el ao 2002 un estudio de 96 pacientes con diversas formas de clasificar a los estreptococos. Clasificar-
taponamiento cardaco en el cual solo uno de los casos (1% los en funcin de la hemlisis, es un buen marcador para

98
CAPTULO 7
ENDOCARDITIS, INFECCIONES ASOCIADAS
A DISPOSITIVOS ENDOVASCULARES
Y OTRAS INFECCIONES CARDIOVASCULARES

reconocer los aislamientos clnicos, ya que los principales descrito casos de PP secundaria a SDSE, siendo el nuestro
patgenos humanos son -hemolticos. An plenamente el primero en la literatura(7).
vigente tenemos la clasificacin Lancefield, descrita en La PP se suele presentar de forma aguda, con una gran
1933 y basada en las reacciones serolgicas de los carbo- afectacin del estado general y fiebre, predominando los
hidratos de la pared celular. Estos son denominados con sntomas de sepsis sobre los sntomas clsicos de pericar-
letras desde la A hasta la W, con la excepcin de la I y la ditis. La existencia de dolor torcico es poco frecuente,
J. La clasificacin ms reciente, se basa en la secuencia del sin embargo en nuestro caso fue uno de los sntomas gua.
gen 16SrRNA y divide los estreptococos en 5 grandes gru- Dada la inespecificidad de los sntomas, y que estos pueden
pos: grupo piognico, grupo mitis, grupo anginosus, grupo confundirse con el propio proceso infeccioso, en numerosas
salivaris y grupo bovis. ocasiones no se sospecha de forma temprana y no se diag-
Los aislamientos -hemolticos de los grupos A, B, C, F y G nostica hasta que el paciente presenta un taponamiento
pueden subdividirse en 2 grupos: formadores de colonias cardaco que obliga a realizar una pericardiocentesis. Por
grandes (> 0,5 mm de dimetro) y formadores de colonias tanto, es fundamental la sospecha clnica para el diagnsti-
pequeas (< 0,5 mm de dimetro). Los principales pat- co. El rendimiento de los cultivos del lquido pericrdico es
genos humanos son los -hemolticos, pertenecientes a los moderadamente elevado, con una tasa de determinaciones
grupos A y B, formadores de colonias grandes y el neumo- positivas que oscila entre el 50 y 60%5,9.
coco (alfahemoltico). Para el adecuado tratamiento es esencial el drenaje de la
Desde el punto de vista taxonmico, existen dos subespecies cavidad pericrdica de forma precoz y el inicio de antibio-
de la especie Streptococcus dysgalactiae: Streptococcus dysgalatiae terapia sistmica guiada posteriormente por los resultados
spp. equisimilis (-hemoltico, grupo C y G de Lancefield) y de los cultivos, hasta completar de 4 a 6 semanas aunque
dysgalactiae spp. dysgalactiae (-hemoltico o no hemoltico) no se conoce la duracin exacta del tratamiento1. El dre-
y de la que no se han documentado infecciones en huma- naje pericrdico se mantendr hasta que el dbito sea in-
nos7. Streptococcus dysgalatiae spp. equisimilis (SDSE) coloniza ferior a 25 ml/da5. En ocasiones, la formacin de fibrina
en humanos el tracto respiratorio superior, gastrointestinal, y loculacin del derrame puede hacer inefectivo el drenaje
genitourinario femenino y la piel. Se transmite de persona pericrdico, favoreciendo la aparicin de complicaciones
a persona y es capaz de producir una amplia variedad de como la pericarditis constrictiva. En estos casos ser ne-
enfermedades similares a las producidas por Streptococcus cesaria la realizacin de una pericardiectomia quirrgica
pyogenes. Algunos estudios moleculares han descrito que como ocurri en nuestro caso. Algunos autores han de-
los determinantes de virulencia de SDSE son parecidos a mostrado que la instilacin de fibrinolticos en la cavidad
los identificados en Streptococcus pyogenes y otros estreptoco- pericrdica puede evitar la formacin de fibrina o, una vez
cos del grupo piognico. A pesar de la existencias de nu- aparecida sta, ser una alternativa a la ciruga9, sin embar-
merosas similitudes entre ambas especies, Takahasi et al go, en la Gua de Practica Clnica de diagnstico y trata-
describieron algunas diferencias como mayor edad media miento de las enfermedades del pericardio de la Sociedad
de presentacin, mayor comorbilidad asociada a SDSE e Europea de Cardiologa recomiendan drenaje quirrgico
infecciones cutneas ms profundas cuando el organismo del derrame purulento10.
causal es S.pyogenes8. SDSE es causante de una variedad de La mortalidad del cuadro oscila entre el 2 y 20% en aque-
infecciones en humanos, superficiales o profundas, media- llos casos que reciben tratamiento adecuado. Son factores
das por toxinas o inmunolgicas. La faringitis es la presen- de mal pronstico: el retraso en el drenaje pericrdico, la
tacin primaria ms clsica en adultos y se ha relacionados existencia de taponamiento en el momento del diagnstico,
con brotes epidmicos en nios. Otras infecciones prima- la infeccin por Staphylococcus aureus o bacilos gram negativos,
rias afecta a la piel y a las partes blandas, incluyendo celu- la inmunosupresin y la malnutricin.
litis, pioderma, infecciones de heridas, erisipela y fascitis Podemos concluir que aunque la PP es relativamente rara
necrotizante, esta ltima, en ocasiones, pone en peligro la en la actualidad y est asociada con una elevada mortalidad,
vida del paciente al complicarse con sndrome del shock casi el 100% si el diagnstico se retrasa. En nuestro caso nos
txico estreptoccico. Desde zonas de colonizacin o de encontramos ante la particularidad que no se han descrito
infecciones primarias se puede producir enfermedad inva- otros casos de PP por SDSE, de ah el inters que aporta.
siva. La enfermedad invasiva incluye: artritis, osteomielitis,
infecciones pleuropulmonares, peritonitis, abscesos epidu-
rales, meningitis, endocarditis, septicemia puerperal, infec-
ciones neonatales y miositis. Hasta el momento no se han

99
CAPTULO 7
ENDOCARDITIS, INFECCIONES ASOCIADAS
A DISPOSITIVOS ENDOVASCULARES
Y OTRAS INFECCIONES CARDIOVASCULARES

Bibliografa

1. Parikh S, Memon N, Echols M, Shah J, McGuire DK,


Keeley EC. Purulent Pericarditis Report of 2 Cases
and Review of the Literature. Medicine (Baltimore)
2009; 88: 52-65.
2. Sagrista-Sauleda J, Barrabes JA, Permanyer-Miralda
G, Soler-Soler J. Purulent pericarditis: review of a 20-
year experience in a general hospital. J Am Coll Car-
diol 1993; 22:1661-5.
3. Oliver C, Marn F, Pineda J, Lujan J, Garca A, Cli-
ment VE, et al. Should we try to determine the specific
cause of cardiac tamponade?. Rev Esp Cardiol. 2002;
55: 493-8.
4. Klacsmann PG, Bulkley BH, Hutchins GM. The
changed spectrum of purulent pericarditis: an 86 year
autopsy experience in 200 patients. Am J Med 1977;
63: 666-73.
5. Suberviola Caas B, Rodrguez Borregan JC, Gon-
zlez Castro A, Miambres E, Burn Mediavilla FJ.
Pericarditis purulenta y empiema pleural por Strepto-
coccus pneumoniae. An Med Interna 2007; 24: 35-7.
6. Montes M, Garca-Arenzana JM. Gnero Streptococ-
cus: una revisin prctica para el laboratorio de micro-
biologa. Enferm Infecc Microbiol Clin 2007;24 Supl
3:14-20.
7. Takahashi T, Sunaoshi K, Sunakawa K, Fujishima S,
Watanabe H, Ubukata K. Clinical aspects of invasive
infections with Streptococcus dysgalactiae ssp. equisi-
milis in Japan: differences with respect to Streptococ-
cus pyogenes and Streptococcus agalactiae infections.
Clinical Microbiology and Infection. 2010: 16: 1097-
103.
8. Augustin P, Desmard M, Mordant P, Lasocki S, Maury
JM, Heming N. Clinical review: Intrapericardial fibri-
nolysis in management of purulent pericarditis. Criti-
cal Care 2011;15: 220. doi: 10.1186/cc10022
9. Maisch B, Seferovic PM, Ristic AD, Erbel R, Rienmu-
ller R, Adler Y, Tomkowski WZ, Thiene G, Yacoub
MH. Guidelines on the diagnosis and management of
pericardial diseases executive summary; The Task For-
ce on the Diagnosis and Management of Pericardial
Diseases of the European Society of Cardiology. Eur
Heart J. 2004; 25:587-610.

100
Sndrome consuntivo a estudio
Rubio-Marn, P; Anglada-Pintado, JC;
Gallego-Puerto, P; Rodrguez-Flix, L.
Hospital SAS de Jerez de la Frontera

Caso clnico

Se trata de un varn de 47 aos de edad, exfumador desde


hace 20 aos, (IPA de 21 paquetes/ao), bebedor excesivo
hasta los 7 meses previos al ingreso. Vive con sus padres en
un ambiente socio-familiar complicado. Como anteceden-
tes personales cabe resaltar hipertensin arterial en trata-
miento con enalapril 20mg con episodios frecuentes de hi-
potensin cumpliendo de manera errtica el tratamiento.
ICTUS lacunar en hemisferio izquierdo sin secuelas por lo
que tomaba AAS 300mg, sndrome vertiginoso como se-
cuela de un traumatismo acstico que trataba con sulpiri-
da 50mg y betahistina 16mg, adems bromazepam 1,5mg
para el control de ansiedad tras deshabituacin enlica. Imagen 1. Colonoscopia. leon distal normal.
Fue intervenido quirrgicamente de una hernia umbilical
y apendicectomizado.
En los meses previos fue estudiado por su mdico de aten-
cin primaria por prdida de ms de 15 kg de peso de 9
meses de evolucin y anemia que no mejoraba con hierro
oral, ante esto y dos determinaciones de sangre oculta en
heces positivas se decidi ingreso hospitalario para com-
pletar el estudio. Presentaba gran astenia, anorexia y dolor
abdominal periumbilical intermitente que se exacerbaba
con los movimientos. Como dato de inters relataba haber
tenido varios episodios de rectorragia de 1 mes de dura-
cin hace 2 aos por el que no consult. No tena snto-
mas de patologa digestiva y no se haba termometrado
fiebre. En la exploracin fsica destacaba su deterioro fsico
y mal estado nutricional, con IMC al ingreso de 16 kg/m2.
Estaba consciente, orientado y colaborador pero bradips-
quico, con tensin arterial 110/80 y temperatura 37,4C. Imagen 2. Endoscopia digestiva alta. Bulbo y segunda porcin duodenal con
mucosa blanquecina (A) que se biopsia, a descartar linfangiectasias difusas.
Presentaba aumento de la pigmentacin de la piel. En
cuello se palpaban pequeas adenopatas en cadena yu-
gular anterior derecha, sin ingurgitacin yugular, ni bocio, leucocitosis con un 82% de neutrfilos y 12,8% de linfoci-
la auscultacin cardiopulmonar era anodina, el abdomen tos, trombocitosis (620.000/ul) y una V.S.G de 58mm/h.
era depresible, no doloroso a la palpacin sin masas ni vs- El TTPa era de 40,60. El estudio de anemia evidenci
ceromegalias con peristaltismo conservado. Extremidades un dficit tanto de hierro srico como de transferrina y
inferiores sin edemas y los pulsos distales estaban conser- disminucin del ndice de saturacin siendo el resto del
vados. El resto de la anamnesis por rganos y aparatos no estudio, incluidos vitamina B12 y cido flico, normal.
presentaba datos relevantes, incluida la exploracin neuro- Se solicitaron tambin marcadores tumorales: CEA, CA
lgica que era normal. 19.9, antgeno de clulas escamosas que fueron negativos,
En el hemograma destacaba una hemoglobina de 9,1 g/dl enolasa-NSE positiva. Tanto la bioqumica, como la fun-
con un hematocrito del 30%, VCM 77,7fl, no presentaba cin heptica y tiroidea y examen de orina fueron norma-

101
CAPTULO 7
ENDOCARDITIS, INFECCIONES ASOCIADAS
A DISPOSITIVOS ENDOVASCULARES
Y OTRAS INFECCIONES CARDIOVASCULARES

les, la radiografa de trax y el electrocardiograma fueron


normales. En ecografa abdomino-plvica se apreciaban
signos inflamatorios en intestino delgado. La TAC tra-
co-abdominal presentaba adenopatas hiliares y mediast-
nicas en regin paratraqueal con un engrosamiento septal
de las paredes bronquiales y lesiones en vidrio deslustrado
as como algn engrosamiento pleural con calcificaciones.
El estudio del abdomen sin alteraciones. El trnsito con
bario presentaba hallazgos compatibles con un cuadro in-
flamatorio regional o componente malabsortivo. La pan-
colonoscopia e ileoscopia eran normales y en la gastros-
copia el bulbo y segunda porcin duodenal se evidenciaba
mucosa blanquecina de la que se tomaron biopsias para Imagen 3. Ecocardiografa transesofgica. Vlvula artica con imagen ecorrefrin-
gente en interior, imagen de alta sospecha de endocarditis sobre vlvula artica.
descartar linfangiectasias difusas. Se ampli la analtica
para descartar un sndrome malabsortivo presentando una
hipoproteinemia (5,8g/dl) con hipoalbuminemia (2,8g/dl)
y beta 2 microglobulina 4,40mg/l. Enzima convertidora
de angiotensina aumentada.
El paciente presentaba datos clnicos y analticos de desnu-
tricin calrico-proteica. Al mismo tiempo se el paciente
present durante su estancia en el hospital fiebre de bajo
grado, presentando un pico aislado de 39,2C, que no
era advertida por el paciente y que probablemente estuvo
presente en los ltimos meses. En este momento se le ex-
trajeron hemocultivos. El paciente refiri adems disnea y
mareos al caminar. Se realiz ecocardiografa transtorci-
ca para completar el estudio de la fiebre de origen desco-
nocido.
Imagen 4. La lmina propia de la mucosa duodenal aparece ocupada por
abundantes macrfagos de citoplasma amplio con estructuras granulares PAS
Diagnstico diferencial positivas y negativas en la tincin de Ziehl-Neelsen. Mucosa de ileon terminal
con hallazgos histolgicos compatibles con Enfermedad de Whipple.

Se plante la posibilidad de una neoplasia digestiva pero


en los estudios endoscpicos no mostraron hallazgos en la IgG para Chlamydophila pneumoniae con IgM negativa de
este sentido mientras que los anlisis indicaban hipopro- escaso significado. Establecindose como diagnstico en-
teinemia e hipoalbuminemia grave que se puso en relacin docarditis sobre vlvula artica por Tropheryma Whipplei
con una enteropata pierde protenas aun en ausencia de con insuficiencia artica moderada y Enfermedad de Whi-
diarrea. Como causa de la fiebre se plante una posible pple con afectacin cardiaca y digestiva con malabsorcin
endocarditis infecciosa. secundaria y malnutricin calrico-proteca se inici tra-
tamiento con ceftriaxona 2g al da intravenosa durante 4
Evolucin semanas a la que se continu con tratamiento durante un
ao con cotrimoxazol 800mg/160mg sin haber necesitado
Los informes histolgicos de la biopsia duodenal e ileal sustitucin valvular. Se le aadieron suplementos proteicos
coincidan en datos con Enfermedad de Whipple. La PCR a la dieta. Tras instauracin de tratamiento el paciente se
para Tropheryma whipplei fue positiva para dicho germen. encuentra asintomtico, con desaparicin de la verruga en
La ecocardiografa transtorcica y posteriormente transe- vlvula artica y una ganancia ponderal de 16 kg.
sofgica fueron muy sugestivas de endocarditis artica con
insuficiencia artica moderada. Los hemocultivos fueron Diagnstico final
repetidamente negativos antes de comenzar con antibio-
terapia y la serologa para grmenes productores de endo- Endocarditis sobre vlvula artica por Tropheryma whipplei
carditis tambin fue negativa salvo unos valores positivos con insuficiencia artica moderada. Enfermedad de Whi-

102
CAPTULO 7
ENDOCARDITIS, INFECCIONES ASOCIADAS
A DISPOSITIVOS ENDOVASCULARES
Y OTRAS INFECCIONES CARDIOVASCULARES

pple con afectacin cardiaca y digestiva con malabsorcin mediante la combinacin de la anatoma patolgica y PCR
secundaria y malnutricin calrico-proteca. de la vlvula obtenida quirrgicamente. Generalmente es
necesario el recambio quirrgico de la vlvula, indicado
Discusin si existe insuficiencia cardiaca, en presencia de infeccin
incontrolada y como prevencin de eventos emblicos.
La Enfermedad de Whipple es una enfermedad multisis- En el tratamiento de la Enfermedad de Whipple es ne-
tmica crnica causada por un bacilo intracelular gram cesario usar antibiticos que atraviesen la barrera hema-
positivo del grupo de los actinomicetos, Tropheryma whipplei. to-enceflica para tratar las manifestaciones neurolgicas.
Esta bacteria comensal se detecta en la saliva, jugo gstri- Generalmente se utiliza ceftriaxona 2g al da va intrave-
co, heces y mucosa digestiva en el 7-35% de los individuos nosa durante 2 semanas ms cotrimoxazol 800mg/160mg
sanos, produciendo enfermedad slo a aquellos que pre- durante al menos 1 ao. Con respecto al tratamiento de
sentan alguna caracterstica an no definida en su sistema la endocarditis infecciosa por Tropheryma whipplei adems
inmunolgico. del recambio de la vlvula en los supuestos anteriormente
Su incidencia es de 1 caso por milln de personas ao. En descritos se debe ampliar a 4 semanas el tratamiento con
el ao 1907 se describi el primer caso de esta enfermedad ceftriaxona 2 g al da. Resto del tratamiento igual.
que es ms frecuente en hombres de mediana edad. En un El seguimiento de le Enfermedad de Whipple debe hacer-
primer momento se denomin a esta enfermedad lipodis- se tras el diagnstico durante al menos 10 aos. En los pri-
trofia intestinal por su aspecto macroscpico en intestino mero 5 aos se deben tomar nuevas biopsias de intestino
delgado y no fue hasta el ao 2000, tras aislarse la Trophery- delgado. En el caso de la endocarditis nosotros realizamos
ma whipplei, que recibi su actual nombre. una ecocardiografa cada 6 meses. En caso de recada se
La triada tpica de esta enfermedad, presente en la mayora recomienda penicilina 1.200.000UI al da ms estreptomi-
de los pacientes en el momento del diagnstico, engloba la cina 1g al da intravenosa 15 das continuando tratamiento
prdida de peso, diarrea intermitente y artralgias migra- con doxiciclina 200mg al da e hidroxicloroquina 200mg 3
torias de grandes articulaciones. Adems de esta clnica veces al da o cefalosporinas 1ao.
puede dar lugar a afectacin cutnea con hiperpigmenta-
cin, afectacin del sistema nervioso central presentando
disminucin del nivel de conciencia, con hemiparesia y
ataxia en algunos casos, y afectacin cardiaca pudiendo
involucrar a todas las capas del corazn tanto pericardio,
miocardio y/o endocardio como ocurra en este caso. La
enfermedad debe ser diagnosticada y tratada precozmente
ya que sin diagnstico ni tratamiento es mortal.
Tradicionalmente su diagnstico se ha realizado por la cl-
nica y por la evidencia de inclusiones PAS positivas en los
macrfagos espumosos de la lmina propia de la mucosa
intestinal tras obtencin de una muestra mediante gastros-
copia. Su cultivo siempre ha sido negativo por lo que en la
actualidad la PCR rARN 16s para la confirmacin diag-
nstica es junto con la inmunohistoqumica con anticuer-
pos anti Tropheryma whipplei esenciales para su diagnstico.
La Endocarditis Infecciosa por Tropheryma whipplei es una
entidad poco frecuente de la que existen pocas referencias
bibliogrficas, menos de 100 casos descritos. Por este mo-
tivo y aunque la afectacin cardiaca no es infrecuente, no
suele tenerse en cuenta a la hora de definir la etiologa ante
una endocarditis con hemocultivos negativos, con el agra-
vante de que la causa principal de negatividad de stos es
la administracin previa de tratamiento antibitico. Pue-
de afectar tanto a vlvulas nativas, ms frecuentemente la
artica, como protsicas. Su diagnstico definitivo se hace

103
CAPTULO 7
ENDOCARDITIS, INFECCIONES ASOCIADAS
A DISPOSITIVOS ENDOVASCULARES
Y OTRAS INFECCIONES CARDIOVASCULARES

Bibliografa

1. Chan V, Wang B, Veinot JP, Suh KN, Rose G, Des-


jardins M, Mesana TG. Tropheryma whipplei aortic
valve endocarditis without systemic Whipples disease.
Int J Infect Dis. 2011; 15: e804-6.
2. Escher R, Roth S. Endocarditis due to Tropheryma
whipplei: rapid detection, limited diversity, and long-
term clinical outcome in a local experiece. Clin Micro-
biol Infect. 2010; 16:1213-22.
3. Lagier JC, Lepidi H. Systemic Tropheryma whipplei:
clinical presentation of 142 patients with inections
diagnosed or confirmed in a reference center. Medici-
ne (Baltimore). 2010; 89:337-45.
4. Puchal X. Whipples disease. Rev. Med Intern. 2009;
30:233-41.
5. Marn M, Muoz P, Snchez M, del Rosal M, Rodr-
guez-Crixems M, Bouza E. Tropheryma whipplei In-
fective Endocarditis as the only manifestation of Whi-
pples disease. J Clin Microbiol. 2007; 45:2078-81.
6. Miguelena J, Muoz R, Maseda R, Epeldegui A. En-
docarditis por Tropheryma Whipplei. Rev Esp Car-
diol. 2010; 63:250-1.
7. Keita AK, Raoult D, Fenollar F. Tropheryma whipplei as
a commensal bacterium. Future Microbiol. 2013; 8:57-71.
8. Cosme A, Bujanda L. Enfermedad de Whipple. Rev
Esp Enferm Dig. 2012; 104:276.
9. Montejo M, Goikoetxea J, Mularoni A, Blanco MJ,
Gaafar A, Boado MV, Marn M, Aramendi JI. Endo-
carditis infecciosa por Tropheryma whipplei: a prop-
sito de 3 casos. Rev Esp Quimioter. 2013; 26:168-70

104
Fiebre y disnea en paciente con endocarditis
protsica Qu no debemos olvidar?
Gmez Carrillo, V; Snchez Montes, M;
Plata Ciezar, A.
Hospital Regional Universitario. Mlaga.

Caso clnico

Varn de 82 aos con antecedentes personales de alergia


al contraste yodado, hipertensin arterial, hipercolestero-
lemia y hemorragia digestiva en relacin con la toma de
antiinflamatorios no esteroideos (AINES) hace 14 aos.
Se le realiz un recambio valvular artico hace 10 aos
por estenosis valvular artica, con implantacin de vlvula
biolgica y reintervinindose hace 2 meses por degenera-
cin de dicha vlvula con implantacin de nueva prtesis
biolgica Edwards Magna 23. Durante este ltimo ingre-
so debut con una fibrilacin auricular de novo con un
CHADS2 de 2, por lo que requiri anticoagulacin oral
al alta.
Ingres en Enfermedades Infecciosas hace un mes por en-
docarditis protsica precoz posible por Staphylococcus epider-
midis resistente a meticilina. El paciente present una buena
evolucin, dndose de alta con daptomicina 750 mg/24h
que se administraba diariamente en Hospital de Da. Imagen 1. Radiografa posteroanterior de trax donde se aprecia infiltrado alveo-
lointersticial de predominio en lbulo inferior y medio derecho.
Su tratamiento domiciliario habitual en este momento era
enalapril 20 mg/12 horas, simvastatina 20 mg/24 horas,
furosemida 20 mg/24 horas, acenocumarol segn contro-
les, omeprazol 20 mg/24 horas y daptomicina 750 mg/24
horas.
Al sptimo da del alta, durante su visita al Hospital de Da
para la infusin del antibitico, refiere febrcula de pre-
dominio vespertino as como ligero aumento de disnea y
mal estado general pero sin focalidad digestiva, urinaria o
abdominal. Dado sus antecedentes se decide ingreso para
estudio.
A la exploracin, el paciente presentaba regular estado
general con presin arterial de 120/75 mmHg, una fre-
cuencia cardiaca de 75 lpm, y 38,5 C de temperatura.
Saturacin basal de oxgeno 92% y 75 Kg de peso. A la
auscultacin se encontraba arrtmico y destacaba un soplo
eyectivo de 1/4 en foco artico. Adems presentaba mni- Imagen 2. TAC de trax donde se aprecia enfisema paraseptal y mltiples focos
de consolidacin con broncograma areo, vidrio deslustrado y algunas reas
mos crepitantes en ambas bases. A nivel abdominal y en con empedrado de predominio subpleural.
extremidades inferiores la exploracin era anodina y no
presentaba estigmas de endocarditis. Al ingreso destacaba grafa de trax pstero-anterior (Imagen 1) se visualizaba
en la analtica de sangre 9950 leucocitos/mm3 con 86.3% una condensacin en hemitrax derecho; amplindose el
de neutrfilos y un aumento de enzimas hepticas (GOT estudio con un TC de trax dos das despus (Imagen 2),
39 UI/L, GPT 40 UI/L, GGT 405 UI/L, FA 281 UI/L) dnde describan adenopatas paratraqueales, precarnicas
con Protena C Reactiva (PCR) 312 mg/L. En la radio- y en ventana aortopulmonar en torno al centmetro, un

105
CAPTULO 7
ENDOCARDITIS, INFECCIONES ASOCIADAS
A DISPOSITIVOS ENDOVASCULARES
Y OTRAS INFECCIONES CARDIOVASCULARES

derrame pleural de escasa cuanta derecho, enfisema para- la neumona eosinfila aguda por la ausencia de eosi-
septal y mltiples focos de consolidacin con broncograma nfilos en el lavado bronquioloalveolar. Generalmente
areo, vidrio deslustrado y algunas reas con empedrado se manifiesta de una manera ms aguda y fulminante
de predominio subpleural ms en hemitrax derecho que que la clnica que presentaba nuestro paciente.
izquierdo
Los hemocultivos seriados extrados en tres ocasiones con Otras posibilidades aunque ms remotas por la evolucin y
3 das de diferencia entre ellos y el urocultivo fueron nega- porque seran independientes del episodio inicial pero que
tivos y en la ecocardiografa transtorcica realizada a los 3 habra que considerar una vez descartadas las anteriores
y 14 das del ingreso se mostraba una bioprtesis artica seran la etiologa tumoral o la inflamatoria: Carcinoma
normofuncionante y una hipertrofia ventricular izquierda bronquioloalveolar, afectacin pulmonar por una vasculitis
con funcin sistlica ventricular conservada sin datos suge- (Sndrome de Churg-Strauss, Sndrome de Goodpasture,
rentes de endocarditis. neumonitis lpica) o aspergilosis pulmonar alrgica.

Diagnstico diferencial Evolucin

En este caso tendramos que hacer un diagnostico diferen- Durante el ingreso se inici antibioterapia emprica de am-
cial de fiebre y condensacin pulmonar en un paciente plio espectro ante la posibilidad de una neumona nosoco-
con un ingreso reciente. Como causas principales podemos mial pero el paciente persisti con fiebre y elevacin de las
distinguir: infecciosas, neoplsicas e inflamatorias/ reuma- enzimas hepticas as como de la PCR.
tolgica o autoinmune. Estos datos clnicos junto con los hallazgos encontrados en
el TC de trax apoyaban el diagnstico de una neumona
Dado los antecedentes que presenta de endocarditis y el eosinoflica por frmacos sospechando como agente res-
ingreso hospitalario reciente, podramos inclinarnos ms, ponsable la daptomicina, motivo por el cual se decidi la
en primer lugar, hacia una causa infecciosa de origen no- interrupcin de este antibitico, instauracin de corticoi-
socomial aunque sin descartar otras posibilidades: des a dosis iniciales de 40 mg de prednisolona cada 12 ho-
ras y se modific la antibioterapia a vancomicina los das
1. Endocarditis infecciosa recidivada con edema agudo que quedaban hasta completar las seis semanas que tena
de pulmn: se realiz una ecografa transtorcica sin programadas por la endocarditis con muy buena respuesta
datos de insuficiencia cardiaca o malfuncionamiento clnica.
de prtesis y hemocultivos seriados para descartarla. Al alta el paciente se encontraba afebril y sin disnea, he-
2. Neumona de origen nosocomial para lo que se instau- modinmicamente estable y con buenas saturaciones ba-
r tratamiento antibitico emprico de amplio espec- sales. Se normalizaron las enzimas hepticas y PCR. En
tro en espera de resultados por posibilidad de grme- la revisin en consulta un mes tras el alta con disminucin
nes resistentes. Los antgenos en orina a Neumococo progresiva de esteroides, el paciente estaba estable y sin
y a Legionella resultaron negativos, no se pudo realizar sintomatologa respiratoria, la TC de trax de control pre-
esputos por escasa expectoracin del paciente. sentaba mltiples focos de consolidacin con broncograma
3. Neumona organizativa criptogentica: patrn radio- areo, vidrio deslustrado y algunas reas de empedrado de
grfico similar al de nuestro paciente pero curso gene- predominio subpleural derecho pero con importante me-
ralmente ms subagudo, buena respuesta a esteroides jora con respecto a la TC del ingreso. En la TC de con-
pero frecuentes recidivas ya que no existe un desenca- trol realizada tres meses despus del alta no presentaba ya
denante conocido. reas de vidrio deslustrado ni de empedrado, siendo la TC
4. Neumona eosinoflica asociada a frmacos: similar a normal salvo por el enfisema paraseptal.
la neumona organizativa criptogentica pero con un
desencadenante claro, un frmaco, y con eosinfilos Diagnstico final
en el lavado bonquioloalveolar. Los frmacos ms fre-
cuentemente relacionados con esta entidad son meto- Toxicidad pulmonar severa por Daptomicina (probable
trexate, nitrofurantona, fenitona y sulfamidas. neumona eosinoflica) en paciente con endocarditis arti-
5. Neumonitis intersticial aguda (Sndrome de Ham- ca precoz por S. epidermidis multirresistente.
man-Rich): enfermedad con afectacin pulmonar di-
fusa e insuficiencia respiratoria que se distinguira de

106
CAPTULO 7
ENDOCARDITIS, INFECCIONES ASOCIADAS
A DISPOSITIVOS ENDOVASCULARES
Y OTRAS INFECCIONES CARDIOVASCULARES

Discusin Table 1. Criteria for inclusion as definite, probable, possible and unlikeky in
terms of the diagnosis of eosinophilic pneumonia and the potential association
with daptomycin exposure
La incidencia de endocarditis infecciosa en estudios pobla- Definite
Concurrent exposure to daptomycin
cionales se estima entre 1,5 y 9,2 casos por 100.000 habi- Fever
tantes, pero las cifras son difciles de precisar debido a los Dyspnoea with increased oxygen requirement or requiring mechanical venti-
lation
cambios de las definiciones que se han experimentado en New infiltrates on chest x-ray or CT scan
los ltimos aos y a las variaciones de las enfermedades de Bronchoalveolar lavage with >25% eosinophils
Clinical improvement following daptomycin withdrawal
base predisponentes a endocarditis en distintos territorios y
con el paso del tiempo. En un estudio poblacional realizado Probable
Concurrent exposure to daptomycin
en Madrid, la incidencia de endocarditis fue de 6,4 casos Dyspnoea with increased oxygen requirement or requiring mechanical venti-
lation
por 100.000 habitantes y ao. En este ltimo estudio, la en- New infiltrates on chest x-ray or CT scan
docarditis ocurri en 0,8 pacientes de cada 1.000 ingresos Bronchoalvelolar lavage with 25% eosinophils OR peripheral eosinophilia
Clinical improvement following daptomycin withdrawal
y estuvo presente en el 3,5% de todos los enfermos con bac-
teriemia (1). Predomina la incidencia en el sexo masculino Possible
Concurrent exposure to daptomycin
sobre el femenino en la mayora de las series publicadas en New infiltrates on chest x-ray or CT scan
adultos con proporciones que oscilan entre 3:2 y 9:1 (2). Clinical improvement following daptomycin withdrawal OR the patient died

La endocarditis infecciosa, al ser relativamente infrecuente, Unlikely


All other cases that did not meet the above criteria
es una enfermedad compleja a la hora de validar su trata-
a Peripheral eosinophilia is defined as a peripheral blood eosinophil level
miento en estudios aleatorizados. La mayora de la informa- greater that the upper limit of normal for the reporting laboratory, or the
cin se obtiene de los estudios in vitro y en modelos anima- reporter noted an elevated blood eosinophil level but did not provide the
actual laboratory value.
les, junto con los estudios observacionales publicados.
La cobertura emprica de la endocarditis debe cubrir Tabla 1. Caso definitivo, probable y posible (7).
obligatoriamente los microorganismos grampositivos,
especialmente Staphylococcus spp., ya que stos son los la gentamicina dado lo avanzado de su edad. Inicialmente
responsables etiolgicos en la mayora de los casos, te- present buena respuesta, sin embargo al comenzar nue-
niendo adems en cuenta la posibilidad de resistencia a vamente con fiebre acompaada de afectacin pulmonar
meticilina. Vancomicina ha sido el antibitico de referen- se decidi ingreso para estudio, diagnosticndose de neu-
cia para Staphylococcus spp. resistentes a meticilina, pero su mona eosinoflica intersticial asociada a la toma de dap-
toxicidad renal y ms recientemente los fracasos descritos tomicina. El primer caso fue publicado en el ao 2007 tras
en bacteriemias por Staphylococcus aureus meticilin-resistente la administracin de daptomicina (5) con buena respuesta
(SARM) con CMI a vancomicina 1,5 g/ml, desaconse- tras la suspensin del mismo existiendo actualmente 29 ca-
jan su uso en la mayora de las ocasiones (3). sos en la literatura con esta patologa.
Daptomicina es una alternativa adecuada en determina- La neumona eosinoflica es una entidad infrecuente. Se
das ocasiones. Su accin rpidamente bactericida frente a manifiesta como infiltrados pulmonares con eosinofilia
clulas bacterianas en todos los estadios de crecimiento y asociado a la toma de medicamentos, como antibiticos,
su actividad en biofilms hace de daptomicina un antibiti- antiepilpticos y antiinflamatorios no esteroides.
co muy atractivo. La dosis recomendada de 6 mg/Kg/da La neumona eosinoflica inducida por medicamentos
en bacteriemias y endocarditis puede resultar insuficiente. puede presentarse como un sndrome agudo o crnico
Actualmente, la dosis recomendada en estas entidades es que puede ocurrir en cuestin de das hasta semanas de
10 mg/Kg/da que se administrara cada 48 horas si el la toma del frmaco. La eosinofilia perifrica no siempre
aclaramiento de creatinina es <30 ml/min. Existe la po- tiene que estar presente en el curso de la enfermedad, de
sibilidad de asociacin con gentamicina y rifampicina, as hecho la ausencia de eosinofilia no excluye el diagnstico
como con cloxacilina y fosfomicina por la sinergia existen- de neumona eosinoflica inducida por frmacos. Para su
te. Otras alternativas a vancomicina como linezolid y tige- diagnstico se debe demostrar evidencia de neumona con
ciclina deben desaconsejarse inicialmente en endocarditis una eosinofilia significativa en el lavado broncoalveolar y
por su accin bacteriosttica (4). una asociacin temporal con un frmaco. Por supuesto, las
Nuestro paciente presentaba una endocarditis sobre vlvu- causas infecciosas de eosinofilia e infiltrados pulmonares,
la protsica precoz, con crecimiento de S. epidermidis mul- tales como infecciones por hongos o parsitos, deben ser
tirresistente por lo que se opt por tratamiento con dapto- excluidas. La mejora clnica debe ocurrir con el cese del
micina para evitar la toxicidad renal de la vancomicina y agente agresor y sus sntomas deben recurrir despus de

107
CAPTULO 7
ENDOCARDITIS, INFECCIONES ASOCIADAS
A DISPOSITIVOS ENDOVASCULARES
Y OTRAS INFECCIONES CARDIOVASCULARES

una nueva provocacin. Sin embargo, no se recomienda la Bibliografa


reexposicin con el agente causal (6).
La base del tratamiento de la neumona eosinoflica es la 1. Bouza E, Menasalvas A, Munoz P, Vasallo FJ, del Mar
interrupcin del agente causal y a la administracin de Moreno M, Garcia Fernandez MA. Infective endocar-
esteroides. Los pacientes pueden requerir soporte respira- ditis--a prospective study at the end of the twentieth
torio con oxgeno suplementario y ventilacin asistida sin century: new predisposing conditions, new etiologic
embargo la mayora de los pacientes responden a las pocas agents, and still a high mortality. Medicine (Baltimore)
horas del inicio de la terapia con esteroides. 2001; 80: 298-307.
En un estudio publicado en el ao 2012 (7) pretende ana- 2. Hill EE, Herijgers P, Claus P, Vanderschueren S, He-
lizar los informes presentados en la FDA como efectos rregods MC, Peetermans WE. Infective endocarditis:
adversos de los casos recogidos tras la administracin de changing epidemiology and predictors of 6-mon-
daptomicina desde el ao 2004-2010. Los casos se defi- th mortality: a prospective cohort study. Eur Heart J
nieron como definitivos, probables y posibles (Tabla 1) en 2007; 28: 196-203.
trminos de diagnstico de la neumona eosinoflica y la 3. Kanafani Z, Boucher H, Fowler V, Cabell C, Hoen
posible asociacin con la exposicin a daptomicina. B, Miro JM, et al. Daptomycin compared to standard
Los casos definitivos tenan exposicin a la daptomicina therapy for the treatment of native valve endocarditis.
asociados a fiebre, disnea con necesidad de oxgeno o ven- Enferm Infecc Microbiol Clin 2010; 28: 498-503.
tilacin mecnica, y nuevos infiltrados en la radiografa de 4. Mensa J, Barberan J, Llinares P, Picazo JJ, Bouza E, Al-
trax as como presentar en el lavado broncoalveolar ms varez-Lerma F, et al. Gua de tratamiento de la infec-
del 25% de eosinfilos todo esto acompaado de la me- cin producida por Staphylococcus aureus resistente a
jora clnica tras la retirada daptomicina. De los 58 casos meticilina. Rev Esp Quimioter 2008; 21: 234-58.
recogidos finalmente 7 fueron definitivos, 13 probables y 5. Hayes D Jr,Anstead MI,Kuhn RJ. Eosinophilic pneu-
38 posibles considerando por tanto este tipo de reaccin monia induced by daptomycin. J Infect2007; 54: 211-
un efecto adverso a tener muy en cuenta en los pacientes 3.
en tratamiento con daptomicina ya que el uso prolongado 6. SolomonJ,SchwartzM. Drug-, toxin-, and radiation
del mismo y la no interrupcin del tratamiento aumenta la therapy induced eosinophilic pneumonia.Semin Res-
morbimortalidad de los pacientes. (7) pir Crit Care Med2006; 27: 192-7.
Por tanto podemos concluir que la neumona eosinoflica 7. Kim PW,Sorbello AF,Wassel RT,Pham TM,Tonning
inducida por daptomicina es un evento poco comn, por JM,Nambiar S. Eosinophilic pneumonia in patients
lo que es fundamental el reconocimiento precoz de esta treated with daptomycin: review of the literature and
afeccin ya que a menudo es reversible. Una revisin de la US FDA adverse event reporting system reports. Drug
medicacin puede ayudar a reconocer esta toxicidad pul- Saf 2012; 35: 447-57.
monar inducida por frmacos potencialmente fatal. Los
corticosteroides, incluso a dosis medias, deben considerar-
se en cualquier paciente que se sospeche de neumona eo-
sinofilica inducida por daptomicina ya que la respuesta a
los esteroides puede ayudar en el diagnstico, as como la
mejora clnica a la retirada del mismo, como fue el caso de
nuestro paciente, quien siguiendo los criterios de Kim et al
se tratara de una neumona eosinoflica posible asociada
a la toma de daptomicina ya que no presentaba eosinofilia
ni se realiz broncoscopia por excelente evolucin tras la
retirada del frmaco e inicio de corticoterapia.
Consideramos por tanto de inters nuestro caso por el es-
caso nmero de casos descritos hasta la fecha, as como
la importancia remarcada de considerar este tipo de afec-
tacin en todo aquel paciente que tras la administracin
de daptomicina persiste febril, con disnea y con nuevos
infiltrados radiogrficos ya que de ello va a depender su
evolucin y pronstico.

108
Varn de 64 aos con rotura
de aneurisma de aorta abdominal
Ros Holgado, IM; Martn Aspas, A;
Doz Artazcoz, E.
Hospital Universitario Puerta del Mar. Cdiz.

Caso clnico

Presentamos el caso de un paciente varn de 64 aos, fu-


mador activo de 20 cigarrillos/da desde la juventud, y
antecedentes de hipertensin arterial y dislipemia en trata-
miento con losartan y simvastatina. Acude a urgencias de
otro centro refiriendo dolor lumbar derecho de dos meses
de evolucin que no interfiere con su actividad habitual,
hasta que se hace de forma brusca ms intenso y no cede
con analgesia de primer escaln.
A su llegada estaba consciente, orientado y colabora-
dor, eupneico con saturacin basal de oxgeno 98%, TA
190/112 mm/Hg, FC 83 lpm, y con sudoracin profusa.
Auscultacin cardiopulmonar normal. El abdomen era
blando y depresible pero doloroso a la palpacin en epi-
gastrio e hipocondrio derecho con defensa abdominal. En
el servicio de Urgencias se realiza una analtica en la que
se objetiva hemoglobina 13 g/dl, 10180 leucocitos/mm3
con 54.8% neutrfilos, creatinina 0.99 mg/dl, Protena
C Reactiva 29 mg/L, troponina ultrasensible 7,74 ng/L Figura 1. Aneurisma de aorta abdominal (imagen superior izquierda corte trans-
versal, imagen superior derecha corte sagital). Aneurisma de aorta torcico (ima-
(VN: 0-14) y una radiografa de trax sin alteraciones re- gen inferior izquierda corte transversal, imagen inferior derecha corte sagital).
levantes. Se administra tratamiento analgsico intraveno-
so sin mejora sintomtica, por lo que se solicita ecografa
abdominal en la que se objetiva un aneurisma de aorta
abdominal de 6,6 cm en el segmento proximal sin visuali-
zar datos de rotura. Se decide completar el estudio con un
TAC toraco-abdominal, donde se observa un aneurisma
de aorta torcica descendente con marcado engrosamien-
to de la pared artica, desplazamiento interno de calcifica-
ciones intimales, hematoma intramural y trombo asociado
al aneurisma; y otro aneurisma, en aorta abdominal in-
frarrenal que se extiende desde inmediatamente el nivel
de las arterias renales hasta aproximadamente 1 cm de la Figura 2. Aneurisma de aorta abdominal con material inflamatorio periartico
(imagen izquierda). By-pass aorto-artico y de arteria ilaca comn derecha a
bifurcacin ilaca con una imagen de defecto de replecin tronco celaco (imagen derecha).
compatible con diseccin artica en su regin ms inferior,
posterior y externa (figura 1). toma muestra de pared artica para cultivo por la presen-
Dados estos hallazgos, se traslada a nuestro hospital para cia de tejido anormal inflamatorio periartico (figura 2).
valoracin por Ciruga Cardaca y Ciruga Vascular. In- El paciente presenta un postoperatorio complicado con
gresa en la Unidad de Cuidados Intensivos (UCI) y tras inestabilidad hemodinmica pese a tratamiento con dro-
seis das de ingreso se realiza intervencin quirrgica gas vasoactivas y, por anuria y sospecha de lesin vesical
programada del aneurisma abdominal mediante by pass se realiza una ecografa y posteriormente un TAC de ab-
aorto-artico y de ilaca comn derecha a tronco celaco. domen que mostraba, adems de los cambios postquirr-
Como complicacin se resecan ambas arterias renales, y se gicos, un defecto de replecin intraartico a nivel de ar-

109
CAPTULO 7
ENDOCARDITIS, INFECCIONES ASOCIADAS
A DISPOSITIVOS ENDOVASCULARES
Y OTRAS INFECCIONES CARDIOVASCULARES

terias renales y una coleccin periartica que englobaba CLASIFICACION DE LAS AORTITIS
NO INFECCIOSAS 1. VASCULITIS DE arteritis de clulas
a ambas arterias renales produciendo una hipoperfusin GRANDES VASOS gigantes, arteritis de
renal bilateral. El paciente comienza con hemofiltracin Takayasu, arteritis
reumatoide, lupus
veno-venosa continua y tras recuperar la estabilidad he- eritematoso sistmico,
modinmica se inici hemodilisis intermitente. Se reali- espondilitis anqui-
losante, sndrome de
z un ecocardiograma transtorcico que evidenciaba una Reiter.
fraccin de eyeccin del 68%, sin anomalas en la contrac- 2. VASCULITIS DE arteritis de Wegener,
tilidad, con desplazamiento de la aurcula izquierda por PEQUEO Y poliarteritis nodosa,
MEDIANO VASO enfermedad de Behet,
el aneurisma de aorta descendente sin otras alteraciones. policondritis recidivante
Analticamente se produce un aumento de reactantes de 3. IDIOPATICA aortitis idioptica, aneu-
risma artico inflamato-
fase aguda y signos clnicos de infeccin respiratoria por lo rio, periaortitis
que se extraen muestras para microbiologa (hemocultivos, 4. POR RADIACION
cultivo de aspirado traqueal y urocultivo) y se inicia trata- INFECCIOSAS 1. BACTERIAS Ej. Salmonella ssp,
miento antibitico emprico de amplio espectro con buena Staphylococcus spp
respuesta. Todos los cultivos extrados fueron negativos, 2. MICOBACTERIAS Mycobacterium tuber-
culosis
salvo el cultivo de la pared artica tomada por el cirujano
3. LUETICA Sfilis
en el que se aisl Staphylococcus lugdunensis, por lo que se ini-
4. VIRAL VIH o SIDA
cia tratamiento con cloxacilina i.v. (2 g cada 8 horas) segn
antibiograma. Tabla 3. Clasificacin de las aortitis. Tomada de Restrepo, et al (4).

Diagnstico diferencial aneurisma artico torcico como para el abdominal, aun-


que en el primero es ms frecuente la presencia de factores
En resumen, se trata de un varn de 64 aos con aneuris- genticos (enfermedad de Marfan), infecciosos o inflama-
ma de aorta torcica y aneurisma disecante de aorta ab- torios. La localizacin ms frecuente es infrarrenal, y no
dominal, con aislamiento en cultivo de pared artica de S. asocia hallazgos inflamatorios en el TAC.
lugdunensis. Los hallazgos que sugieren infeccin del aneurisma son:
Este microorganismo se encuentra en la piel humana y aneurisma sacular o multilobulado, inflamacin del tejido
es una especie de estafilococo coagulasa negativo, pero periartico, presencia de aire intramural o en una colec-
comparte semejanza con Staphylococcus aureus por su gran cin periartica o una coleccin perivascular. Cuando nos
virulencia y capacidad de producir infecciones supuradas. encontramos ante estos hallazgos, debemos pensar que se
Es un raro contaminante (<20%), y normalmente lo pre- ha producido una infeccin sobre un aneurisma degenera-
sentan los pacientes con inmunodepresin o alguna enfer- tivo preexistente. El aneurisma de aorta inflamatorio se ca-
medad de base como diabetes mellitus, insuficiencia renal racteriza por engrosamiento de la adventicia presentando
crnica, cirugas o traumatismos previos. Clnicamente, en el TAC un anillo inflamatorio mayor a 1 cm alrededor
se presentan con fiebre persistente (>80%), bacteriemia de la aorta cuya intensidad puede aumentar con el con-
(85%), anemia (81%), y sepsis (en ms de un tercio de los traste intravenoso y se extiende a retroperitoneo; tambin
casos) (1). Es un patgeno responsable tanto de infecciones puede aparecer fibrosis retroperitoneal, o adherencia del
nosocomiales como de la comunidad, y se ha relacionado tejido inflamatorio a estructuras ventrales a la aorta (duo-
con endocarditis, osteomielitis, infecciones del tracto uri- deno, urteres). Por lo tanto, ante la presencia de cualquie-
nario, artritis sptica, abscesos del sistema nervioso central ra de estas caractersticas, se debe considerar una etiologa
e infecciones de partes blandas. En endocarditis infecciosa, distinta a la degenerativa (3).
la mortalidad global se encuentra entre el 18 y el 38.8%, La aortitis es la afectacin de la pared de la aorta inde-
ms alta que las ocasionadas por S. aureus (22%) y el resto pendientemente de su causa. La clnica de presentacin es
de estafilococos coagulasa negativos (16%) (2). inespecfica, con sntomas como fiebre, prdida de peso,
En la valoracin etiolgica del aneurisma artico, la histo- sndrome coronario agudo o dolor abdominal. Bsicamen-
ria clnica y las imgenes radiolgicas son fundamentales. te se diferencian en dos tipos: inflamatorias (infecciosas y
El aneurisma degenerativo sin rotura suele ser asintom- no infecciosas) y no inflamatorias (tabla 3).
tico y sus factores de riesgo son principalmente la edad Dentro de las causas infecciosas, se encuentran las produci-
avanzada, el sexo masculino, la hipertensin arterial y el das por bacterias (Salmonella spp. y S. aureus, los ms frecuen-
tabaquismo. Estos factores son compartidos tanto para el tes, ocasionan el 40% de casos; Listeria, Bacteroides fragilis,

110
CAPTULO 7
ENDOCARDITIS, INFECCIONES ASOCIADAS
A DISPOSITIVOS ENDOVASCULARES
Y OTRAS INFECCIONES CARDIOVASCULARES

Clostridium septicum y Campylobacter jejuni), tuberculosis, sfilis, Criterios de grupo de D. Raoult para el diagnstico de infeccin vascular por
fiebre Q (7)
o por virus como el VIH. Suelen afectar habitualmente a A. DEFINITIVO
un segmento ya lesionado con placas de arterioesclerosis o Cultivo positivo, PCR o inmunohistoqumica de muestra arterial (prtesis
o aneurisma) o de absceso periarterial o espondilitis adyacente a la
un aneurisma y se diseminan por contigidad o por mbo- aorta
los spticos (en sepsis o endocarditis infecciosa) (4). B. CRITERIOS MAYORES
Microbiolgicos: cultivo positivo, PCR de sangre o mbolo, o serologa
con ttulos en fase I 6400
Evolucin Evidencia de afectacin vascular:
TC: aneurisma o prtesis vascular + absceso periarterial, fstula
o espondilodiscitis
Durante el ingreso en UCI se consigue estabilizar hemo- PET-scan: captacin en aneurisma o prtesis vascular
dinmicamente al paciente, se logra la extubacin orotra- C. CRITERIOS MENORES
queal y tras tres semanas de antibioterapia dirigida con Serologa con ttulos IgG fase I 800 < 6400
Temperatura 38C
cloxacilina, se da de alta a la planta de Ciruga Cardaca. Embolos
A su llegada nos consultan para valorar la duracin del Presencia de aneurisma o prtesis vascular
Diagnstico definitivo si: 1) criterio A; 2) dos criterios B; 3) un criterio B + dos
tratamiento antibitico. criterios C (que incluya microbiolgico y vascular)
Cuando realizamos la primera valoracin, nos encontra- Diagnstico probable si: predisposicin vascular, evidencia serolgica y fiebre
o mbolo
mos con un paciente con buen estado general, afebril, es-
Clasificacin de la fiebre Q crnica segn el Grupo de Consenso Holands de
table hemodinmicamente, en respiracin espontnea sin Fiebre Q (6)
precisar oxigenoterapia, sin secuelas tras su paso por UCI, A. PROBADA (si cualquiera de los siguientes):
y sin datos de infeccin en herida quirrgica, as como nor- PCR positiva para Coxiella burnetti en suero, plasma o tejido en ausen-
cia de fiebre Q aguda
malizacin de parmetros analticos inflamatorios. Reha- Serologa con ttulos IgG en fase I 1024 con criterios definitivos de
endocarditis segn los criterios de Duke revisados
cemos la historia clnica, y el paciente refiere cuadro de Datos de infeccin vascular en PET, TC, RM o ecografa
dolor lumbar de dos meses de evolucin, sin interferir en B. PROBABLE si la serologa con ttulos IgG fase I 1024 y cualquiera de las
su vida diaria que ceda con paracetamol, hasta el da que siguientes manifestaciones clnicas:
aparece un dolor brusco por lo que acudi a urgencias. No Valvulopata que no cumpla los criterios mayores de los criterios de Duke
Aneurisma, prtesis vascular o valvulopata protsica sin signos de
haba tenido fiebre en ningn momento, tiritonas, prdida infeccin en PET,TC, RMN o ecografa
Signos de probable infeccin crnica por fiebre Q en tejido no cardiaco
de peso, ni otros sntomas. En la analtica inicial no ha- o vascular en PET, TC, RMN o ecografa.
ba elevacin de reactantes de fase aguda y al no presentar Embarazo
Sntomas de infeccin crnica (fiebre, sudor nocturno, prdida de peso,
fiebre, no hubo hemocultivos previos a las complicaciones hepatoesplenomegalia)
infecciosas postquirrgicas. Inflamacin ganulomatosa en estudio anatomo-patolgico
Enfermedad inmune
En este momento nos planteamos una pregunta, Es real- C. POSIBLE
mente S. lugdunensis el causante de la rotura del aneurisma, Serologa con ttulos IgG fase I I 1024 sin las manifestaciones descri-
tas previamente
o forma parte del 20% del porcentaje de contaminantes?
En cualquier caso, ya que el germen fue aislado en mate- PCR: reaccin en cadena de polimerasa. PET: tomografa por emisin de positro-
nes. TAC: tomografa axial computerizada. RMN: resonancia magntica nuclear.
rial biolgico estril, decidimos completar hasta seis sema-
nas de tratamiento antibitico con cefazolina 1 gr diario Tabla 4. Criterios diagnsticos de infeccin crnica por Coxiella burnetii (6,7).
i.v., dada su eficacia y comodidad de administracin en
pacientes en hemodilisis (5). cin y deteccin del ADN bacteriano del microorganismo
en sangre mediante la reaccin en cadena de la polimerasa
Diagnstico final (PCR) que result negativa (realizada en Centro de Refe-
rencia Instituto de Salud Carlos III de Majadahonda). No
Objetivamente nos encontramos ante un paciente con una se dispuso de tejido de aorta para poder realizar estudio
complicacin cardiovascular y una presentacin clnica de PCR o inmunohistoqumica sobre el mismo. Por lo que
inespecfica, sospecha de naturaleza infecciosa segn los el resultado final fue: rotura de aneurisma abdominal con
hallazgos radiolgicos y operatorios, pero sin fiebre, por lo infeccin crnica por fiebre Q.
que solicitamos una serologa de endocarditis con hemo- Se inici tratamiento con doxiciclina (100 mg/12 ho-
cultivos negativos. ras) ms hidroxicloroquina ajustado a funcin renal (200
Los resultados obtenidos fueron los siguientes: Bartonella (Ig mg/12 horas) tras comprobar normalidad en el examen
G e Ig M negativo), sfilis (EIA) negativo, Coxiella burnetii (Ig de fondo de ojo. Por parte de Ciruga Cardiaca se decidi
G fase I > 1:32768, Ig G fase II 1:32768, Ig M fase I posi- posponer la intervencin sobre el aneurisma torcico a fin
tivo dbil, Ig M fase II negativo). Solicitamos la amplifica- de ver evolucin de la funcin renal y planear la tcnica

111
CAPTULO 7
ENDOCARDITIS, INFECCIONES ASOCIADAS
A DISPOSITIVOS ENDOVASCULARES
Y OTRAS INFECCIONES CARDIOVASCULARES

ms adecuada. Dos meses despus el paciente falleci en Puntos clave:


otro centro por un cuadro de shock, sin haberse realizado
estudio de TAC ni necropsia. 1. Una adecuada historia clnica y evaluacin de las
pruebas complementarias son esenciales para inter-
Discusin pretar los resultados microbiolgicos obtenidos.
2. Es necesario conocer los signos radiolgicos del aneu-
Al no realizarse de rutina el cultivo de C. burnetii ni dis- risma de aorta que deben hacernos sospechar e inves-
poner de mtodos comerciales de PCR, el diagnstico de tigar una probable etiologa infecciosa o inflamatoria
fiebre Q crnica se hace fundamentalmente a travs de la para un manejo teraputico adecuado.
determinacin de anticuerpos por inmunofluorescencia, si 3. Debido a la inespecfica presentacin clnica de la
bien en centros especializados, se puede realizar PCR en fiebre Q crnica, es necesario tener un alto nivel de
sangre o tejido biolgico o inmunohistoqumica en tejido sospecha para diagnosticar adecuadamente estos pa-
biolgico. Hay discrepancias entre los criterios que se han cientes.
de cumplir para el diagnstico de fiebre Q crnica segn
el grupo francs y holands de expertos en fiebre Q (tabla
4), cumpliendo nuestro paciente criterios de infeccin de-
finitiva para ambos grupos. La PCR en suero en nuestro
caso (determinada antes del inicio de antibioterapia espe-
cfica) fue negativa, algo explicado dada la variable sensi-
bilidad descrita para esta tcnica en la fiebre Q crnica (33
al 64%) (8).
Del 1 al 11% de infecciones agudas por fiebre Q van a cro-
nificarse tras varios meses o aos despus de la infeccin
aguda. En la historia clnica dirigida el paciente no recor-
daba ningn episodio compatible con fiebre Q aguda. Slo
en el 16% de los pacientes con endocarditis por fiebre Q
est documentado un episodio agudo previo (9). Los fac-
tores de riesgo para adquirirla son valvulopata precoz, ci-
ruga y prtesis valvulares, aneurismas, insuficiencia renal,
edad avanzada y elevacin precoz de ttulos de Ig G fase I
segn algunos autores (6,7). La manifestacin cardiovascu-
lar principal es la endocarditis infecciosa (60-70%), (que en
nuestro paciente no se pudo descartar completamente al
no haberse realizado un ecocardiograma transesofgico),
seguido de la infeccin vascular (7%), ya sea infeccin de
prtesis vascular o de aneurisma, como es nuestro caso. La
mortalidad descrita alcanza el 25% en pacientes con aneu-
risma de aorta abdominal y/o enfermedad ilaca, y presen-
tan un 30% ms de complicaciones agudas del aneurisma
respecto a pacientes sin infeccin (10). El tratamiento es
prolongado, con Doxiciclina (100 mg/12 horas) asociado
a Hidroxicloroquina (200 mg/8 horas) durante 18 meses
en infecciones de vlvula nativa y 24 meses en infeccio-
nes de vlvula protsica en endocarditis (9). En infeccin
vascular el tratamiento mnimo son 18 meses, pero dada
la baja prevalencia de casos, la duracin es ms controver-
tida y debe individualizarse. En cualquiera de los casos, la
intervencin quirrgica temprana es fundamental porque
mejora la supervivencia (8).

112
CAPTULO 7
ENDOCARDITIS, INFECCIONES ASOCIADAS
A DISPOSITIVOS ENDOVASCULARES
Y OTRAS INFECCIONES CARDIOVASCULARES

Bibliografa

1. Lin JF, Cheng CW,Kuo AJ, Liu TP,Yang CC,Huang


CT. Clinical experience and microbiologic characte-
ristics of invasive Staphylococcus lugdunensis infection in a
tertiary center in northern Taiwan. J Microbiol Immu-
nol Infect. 2014; 20: 1-7.
2. Liu PY, Huang YF, Tang CW, Chen YY, Hsieh KS,
Ger LP. Staphylococcus lugdunensis Infective Endocarditis:
A Literature Review and Analysis of Risk Factors. J
Microbiol Immunol Infect. 2010; 43: 478-84.
3. Jeffrey J, Robert W T. Clinical features and diagnosis
of abdominal aortic aneurysm. In: UpToDate. [Con-
sultado 14 Abril 2015]. Disponible en: http//uptoda-
te.com
4. Restrepo CS,Ocazionez D,Suri R, Vargas D. Aorti-
tis: Imaging Spectrum of the Infectious and Inflam-
matory Conditions of the Aorta. Radiographics. 2011;
31: 435-51.
5. Chan KE, Warren HS, Thadhani RI, David JR, Steele
JL, Hymes FW, et al. Prevalence and Outcomes of An-
timicrobial Treatment for Staphylococcus aureus Bactere-
mia in Outpatients with ESRD. J Am Soc Nephrol.
2012; 23: 15519.
6. Kampschreur LM, Dekker S, Hagenaars J, Lestrade P,
Renders N, de Jager-Leclercq M. Identification of risk
factors for chronic Q fever, the Netherlands. Emerg In-
fec Dis. 2012; 18: 563-70.
7. Raoult D. Chronic Q fever: expert opinin versus lite-
rature analysis and consensus. J Infect. 2012; 65:102-8.
8. Centers for Disease Control and Prevention. Diagno-
sis and Management of Q Fever. MMWR. 2013; 62:
1-29.
9. Million M, Thuny F, Richet H, Raoult D. Long-term
out come of Q fever endocarditis: A 26-year personal
survey. Lancet Infect Dis. 2010; 10: 527-35
10. Fenollar F,Fournier PE,Raoult D. Molecular detec-
tion of Coxiella burnetii in the sera of patients with Q
fever endocarditis or vascular infection. J Clin Micro-
biol. 2004; 42: 491924.

113
Mujer de 48 aos con sndrome febril
tras sustitucin valvular artica
Escobar Sevilla, J; Cantero Nieto, L;
Contreras Bolvar, V; Hidalgo Tenorio, C.
Complejo Hospitalario Granada Virgen de las Nieves de Granada.
Hospital Regional Universitario de Mlaga.

Caso Clnico Infeccin de la herida quirrgica


Asociada al acceso vascular (perifrico o central)
Mujer de 48 aos con antecedentes personales de Este- Asociada a material protsico
nosis artica moderada por Vlvula bicspide calcificada,
Hipertensin arterial en tratamiento con betabloqueantes, Causas no infecciosas
Ex tabaquismo, Dislipemia, Migraa, Hernia de Hiato y
Alergia a betalactmicos. Problemas trombticos: Tromboembolismo pulmonar,
La paciente ingres en el Servicio de Ciruga Cardiaca de Tromboflebitis.
forma programada para recambio valvular artico e im- Reaccin transfusional
plantacin de una prtesis artica St. Jude n 21. La inter- Sndrome postpericardiectoma.
vencin quirrgica fue rpida, con tiempos de circulacin Frmacos sospechosos (Inhibidores de la recepta-
extracorprea y clampaje artico de 70 y 60 minutos, res- cin de serotonina, heparina de Bajo peso molecular,
pectivamente, y sin complicaciones intraoperatorias. En las AINE, diurticos, betalactmicos, fluorquinolonas,
primeras 24 horas postciruga la paciente present hipoten- vancomicina)
sin por disfuncin biventricular leve-moderada y fracaso Alteraciones endocrinas como hipertiroidismo o crisis
renal agudo que se control con frmacos vasoactivos en la addisoniana.
Unidad de Cuidados Crticos (UCI). El da +7 postopera-
torio comenz con fiebre de hasta 38 C y tiritona sin otra Evolucin Clnica
sintomatologa; se paut de forma emprica Daptomicina
10 mg/kg/d. En el hemocultivo se aisl en un frasco de Se le realiz ETE en el da + 13 de administracin del
dos un Staphylococcus coagulasa negativo (SCN); urocultivo antibitico, en la que se observ buen funcionamiento de
negativo; y ecografa transtorcica (ETT) normal a las 72 la vlvula protsica sin signos de vegetacin.
horas del inicio de la fiebre; los cirujanos realizaron una in- Los HC al da + 16 fueron negativos. El da + 20 present
terconsulta al Servicio de Enfermedades Infecciosas. En el nuevo pico febril de 378 C y una flebitis de una va perif-
momento de la evaluacin la paciente estaba afebril y asin- rica, aislndose nuevamente en hemocultivos un SCN. Por
tomtica. En la anamnesis destacaba Fenmeno de Ray- este motivo se repiti la ETE en el da + 23 postoperatorio
naud de aos de evolucin, Ictus en la juventud, aborto de donde se observaba una masa vibrtil de aspecto filamen-
primer trimestre y familiar de primer grado fallecido por toso dependiente de la pared artica de unos 7 mm de di-
Ictus isqumico. En la exploracin fsica intensa livedo re- metro sobre una zona aneurismtica de 24 cm a 36 cm en
ticularis de miembros inferiores; y en la analtica slo PCR la aorta ascendente (Imagen 1 y 2), compatible con endar-
de 123 mg/dL. En ese momento (da + 12 postoperatorio) teritis. El Angio - TAC torcico a da + 24 postoperatorio
se aade Fosfomicina 24 gramos al da repartidos en 3 do- (+ 12 de terapia antimicrobiana) descart complicacin a
sis e Ibuprofeno 600 mg cada 12 horas va oral. nivel de aorta ascendente y detecta cambios postquirrgi-
cos post esternotoma y atelectasias laminares bibasales
Diagnstico diferencial con mnimas condensacin bilaterales. El hemocultivo de
control del da + 25 fue negativo. Se decidi mantener el
El Diagnostico diferencial en el caso de nuestra paciente tratamiento antibitico hasta completar 6 semanas ante
sera el de una fiebre postquirrgica en la primera semana el diagnstico de posible endarteritis infecciosa. Se llev
postoperatoria. a cabo un estudio de coagulacin especial encontrndo-
se Heterocigosis para Factor XII (posicin 46), Anticoa-
Causas infecciosas gulante Lpico positivo, IgM IgG Beta2 Glicoprotena
negativas. Tras esto se inicia terapia con anticoagulantes
Infeccin del tracto urinario asociada a catter urinario dicumarnicos con objetivo de INR 25 35.

114
CAPTULO 7
ENDOCARDITIS, INFECCIONES ASOCIADAS
A DISPOSITIVOS ENDOVASCULARES
Y OTRAS INFECCIONES CARDIOVASCULARES

La paciente permaneci afebril y sin incidencias posterio- percutneo10. La TC multicorte ha demostrado buenos
res; la ETE al alta no encontr nuevos hallazgos. resultado para valoracin del alcance perivalvular y en la
existencia de pseudoaneurismas en pacientes postoperados
Diagnostico final cercanos al 100% de Sensibilidad(SN) y Especificidad(EP)
y mejores que ETE que obtuvo resultados de SN 97 % y
Sndrome febril postciruga cardiaca, masa intraartica EP 88 % en la valoracin de la extensin extravalvular11.
en la que no se puede descartar endarteritis por SCN o La rpida actuacin en contra de los patgenos implicados
trombo intraarterial en el contexto de Sndrome antifosfo- puede modificar el curso de la enfermedad y mejorar el
lipdico primario en espera de confirmacin 2 marcadores pronstico, esto justific en nuestro caso la terapia precoz
inmunolgicos a los 6 meses. y prolongada hasta las 6 semanas con el fin de esterilizar
totalmente la situacin clnica de nuestra paciente.
Discusin Con respecto al Sndrome antifosfolipdico (SAF), se tra-
ta de una enfermedad autoinmune sistmica, importante
El sndrome postpericardiectoma se presenta entre un 89 causa de hipercoagulabilidad adquirida, que afecta hasta
% y 15 % de los pacientes sometidos a ciruga cardiaca, y al 2 % de la poblacin general, y se define por criterios
la mayora de casos requieren hasta 21 das para llegar al clnicos que incluye a trombosis arteriales o venosas, his-
diagnstico1,2. La sintomatologa de este sndrome incluye torial de abortos, prdidas embrionales o fetales; y marca-
derrame pericrdico, PCR elevada, fiebre, y mejora con dores serolgicos positivos en dos ocasiones separado por
antiinflamatorios no esteroideos3. 12 semanas como la beta2 glicoprotena, el anticoagulante
La endocarditis sobre vlvula protsica (EVP) es la forma lpico o anticuerpos anticardiolipina12. Es necesario la sos-
ms grave de EI y ocurre en el 1-6 % de los paciente con pecha para poder llegar al diagnstico, como en nuestra
prtesis valvulares4, con una incidencia de un 03-12 % paciente, as como la confirmacin serolgica para reunir
por paciente-ao5, equivale a un 10-30 % de todos los casos los criterios vigentes13 para su diagnostico, ya que existe
de EI; lo que supone entre un 16% y 26% segn cohortes5 un porcentaje de pacientes con marcadores serolgicos sin
y, su incidencia est al alza en pases desarrollados6. La EI enfermedad, (hasta un 5%)11.
sobre vlvula protsica precoz (EIVPP) es aquella que ocu- En conclusin en el caso de nuestra paciente decidimos
rre en el primer ao tras la ciruga, la mortalidad de este tratar en primera instancia como una sndrome post-pe-
proceso oscila entre el 20% y 40%4. La importancia reside ricardiectoma y de una bacteriemia aislada en probable
en la adquisicin de la infeccin en el perioperatorio, se ha relacin a acceso venoso. Al encontrar nuevo episodio de
demostrado que hasta un 37 % de la EIVPP estaba asocia- bacteriemia y considerar una paciente de alto riesgo, deci-
da a infeccin nosocomial o cuidados sanitarias7. La EIV- dimos minimizar el riesgo de implantacin de este germen
PP se debe con mayor frecuencia a estafilococo coagulasa con un tratamiento completo y precoz. Los hallazgos de las
negativo (SCN), y se asocia a un alto riesgo de mortalidad pruebas complementarias nos orientaron hacia una afecta-
(>45 %), a menudo requiriendo sustitucin valvular8. Estos cin de la aorta ascendente que si bien no existe certeza de
SCN presentan una importantes tasa de resistencia a oxa- afectacin bacteriana, decidimos tratar de manera agresi-
cilina5. Existe un alto riesgo de contaminacin en la extrac- va para evitar las connotaciones pronsticas negativas de
cin de hemocultivos y los microorganismos implicados en no tratar una endarteritis infecciosa postquirrgica, pero
la endocarditis infecciosas sobre vlvula protsica precoz no olvidamos las otra patologas subyacentes en nuestro
(EIVPP) suelen ser saprofitos cutneos lo que dificulta la caso un Sndrome antifosfolipdico que pudiera ser causan-
interpretacin de los resultados de los hemocultivos. El te de este tipo de afectaciones vasculares, situndonos ante
porcentaje de bacteriemia asociada a catter que desarro- una situacin de difcil manejo clnico.
llan endocarditis esta bien reportado en Staphylococcus aureus
(sobre un 22 %) o S. lugdunensis pero no as para el resto de
especies de Staphylococcus coagulasa-negativos9. Por tanto
el diagnstico suele ser ms difcil por falsos negativos de la
ecocardiografa2 y los criterios de Duke reducen su sensibi-
lidad en este marco5.
La etiologa microbiolgica de la Endarteritis es similar
a la de la EIVPP9, aunque es ms frecuente en pacientes
con cardiopatas congnitas o recambio valvular artico

115
CAPTULO 7
ENDOCARDITIS, INFECCIONES ASOCIADAS
A DISPOSITIVOS ENDOVASCULARES
Y OTRAS INFECCIONES CARDIOVASCULARES

Bibliografa the International Society on Thrombosis and Hae-


mostasis. Subcommittee on Lupus Anticoagulant/
1. Lehto J, Gunn J, Karjalainen P, Airaksinen J, Kivinie- Antiphospholipid Antibody of the Scientific and Stan-
mi T. Incidence and risk factors of pericardiotomy sn- dardisation Committee of the International Society
drome requiring medical attention: The Finland pos- on Thrombosis and Haemostasis. J Thromb Haemost.
tpericardiotomy syndrome study. J Thorac Cardiovasc 2009;7:1737-40.
Surg 2015 Jan 21 pii: S0022-5223(15)00051-3. 13. Ruiz-Irastorza G, Crowther M, Branch W, Kha-
2. Imazio M, Brucato A, Rovere ME et al. Contemporary mashta MA. Antiphospholipid syndrome. Lancet
features, risk factors, and prognosis of the post-pericar- 2010;376:1498-509
diotomy syndrome. Am J Cardiol 2011;108: 1183-7.
3. Imazio M, Hoit BD . Post-cardiac injury syndromes.
An emerging cause of pericardial diseases. Int J Car-
diol. 2013;168:648-52.
4. Vongpatasin W, HIllis LD, Lange RA. Prosthetic heart
valves. N Engl J Med 1996; 335: 407-16.
5. Habib G, Hoen B, Tornos P et al. Gua de Prctica
clnica para prevencin, diagnstico y tratamien-
to de la endocarditis infecciosa. Version corregida
03/10/2013. 2009;62:1465.e1-.e54
6. Tleyjeh IM, Abdel-Latif A, Rahbi H et al. A syste-
matic review of population-based studies of infective
edncarditis. Chest 2007; 132:1025-35.
7. Wang A, Athan E, Pappas PA et al. Contemporany
clincal profile and outcome of prosthetic valve endo-
carditis. JAMA 2007;297:1354-61.
8. Chirouze C, Cabell CH, Fowler VG Jr, Khayat N,
Olaison L, Mir JM, et al. Prognostic factors in 61 ca-
ses of Staphylococcus aureus prosthetic valve infecti-
ve endocarditis from the International Collaboration
on Endocarditis merged database. Clin Infect Dis.
2004;38:1323-7.
9. Sabe MA, Shrestha NK, Gordon. Staphylococcus
lugdunensis: a rare but destructive cause of coagula-
se-negative staphylococcus infective endocarditis. Eur
Heart J Acute Cardiovasc Care. 2014;3: 275-80.
10. Amat-Santos IJ, Messika-Zeitoun D, Eltchaninoff H.
Infective Endocarditis Following Transcatheter Aortic
Valve Implantation: Results from a Large Multicen-
ter Registry. Circulation. 2015;131:1566-74. DOI:
10.1161/CIRCULATIONAHA.114.014089.
11. Feutchtner GM, Stolzmann P, Dichtl W Multisli-
ce computed tomography in infective endocarditis:
comparison with transesophageal echocardiogra-
phy and intraoperative findings. J Am Coll Cardiol.
2009;53:436-44.
12. Pengo V1, Tripodi A, Reber G, Rand JH, Ortel TL,
Galli M, De Groot PG et al. Update of the guideli-
nes for lupus anticoagulant detection. Subcommittee
on Lupus anticoagulant/Antiphospholipid Antibody
of the Scientific and Standardisation Committee of

116
Fiebre, lesiones cutneas y bajo nivel de consciencia
tras 20 das de ingreso en paciente en hemodilisis
Ruiz Ruiz, F; Navarro Amuedo, MD;
Praena Segovia, J; Martn Bermdez, R.
Hospital Universitario Virgen del Roco. Sevilla.

Caso Clnico se correspondan a las lesiones vasculticas ya conocidas y


no presentaba aftas orales. Los pulsos pedios estaban pre-
Presentamos el caso de un varn de 48 aos con mltiples sentes de manera bilateral.
antecedentes personales entre los que destacan hiperten- Al ingreso se dispona de una analtica sangunea urgente
sin arterial de larga evolucin con hipertrofia ventricu- en la que nicamente destacaba una PCR de 142 mg/l,
lar izquierda ligera y contractilidad conservada. En 2010 creatinina de 3,25 mg/dl (similar a la basal) y hemoglo-
presenta cuadro de prpura cutnea, dolor articular, clau- bina de 100 g/l y una Rx de trax sin ningn hallazgo
dicacin mandibular y mononeuritis mltiple. Mediante patolgico.
biopsia cutnea y perianal es diagnosticado de vasculitis
sistmica tipo Poliangeitis Microscpica con ANCA nega- Evolucin y diagnsticos diferenciales
tivos. Desde el diagnstico presenta en los sucesivos aos
numerosos brotes de actividad de vasculitis con episodios Ante sndrome febril comunitario agudo en paciente in-
de afectacin cutnea, pulmonar con aparicin de hemo- munodeprimido en hemodilisis, el diagnstico diferencial
rragia alveolar y afectacin renal con deterioro del filtrado inicial se estableci entre infecciones bacteriana: bacterie-
glomerular progresivo. A lo largo de la evolucin precisa mia relacionada con catter (BRC) vs infeccin de piel y
pulsos de glucocorticoide, ciclofosfamida, plasmafresis, partes blandas (lcera necrtica del dedo del pie). Otros
micofenolato de mofetilo y dos ciclos de rituximab. Final- diagnsticos contemplados fueron enfermedad digestiva
mente en el ao 2013, tras nuevo brote con afectacin re- por CMV/herpes virus dada la clnica de disfagia y en-
nal, presenta deterioro de la funcin renal con necesidad fermedad sistmica por poliangeitis debido a la progre-
de terapia de sustitucin renal (TRS) inicialmente a travs sin de las lesiones cutneas. Se mantuvo el tratamiento
de catter tunelizado y luego a travs de fstula AV radial inmunosupresor y se solicitaron: hemocultivos diferencia-
izquierda, pero ante la trombosis de la misma, se canaliz les, cultivo de la punta del catter (finalmente se recambi
nuevo catter tunelizado en yugular derecha que mantena el catter para dilisis a femoral derecha), cultivo de frotis
en el momento del ingreso. Tratamiento actual: micofeno- de la lesin necrtica del 4 dedo del pie izquierdo, PCR
lato de mofetilo y deflazacort (15 mg al da). herpes virus en biopsia, PCR citomegalovirus en sangre,
En el episodio actual el paciente ingresa en planta de Ne- cultivo de las lesiones cutneas, serologas (hepatitis, sfi-
frologa derivado desde la Unidad de Hemodilisis por lis y VIH); y endoscopia digestiva oral (informada como:
presentar fiebre (de 38,5) junto con tiritona durante la se- gastritis edematosa/eritematosa con infiltrado petequial
sin de hemodilisis. Niega clnica respiratoria ni gastroin- en la submucosa con afectacin corporal de etiologa ines-
testinal, refiriendo nicamente una lesin ulcerada en el 4 pecfica, no pudindose descartar la infecciosa como CMV
dedo del pie izquierdo junto con leve disfagia a slidos y o herpes. Las muestras para CMV fueron negativas y la
lquidos, ambas de varias semanas de evolucin. biopsia aport el diagnstico final de gastritis erosiva sin
A la exploracin fsica se encontraba consciente, orientado otros datos.
y colaborador. No presentaba aftas orales. Bien hidratado Tras la toma de cultivos se inici antibioterapia empri-
y perfundido. Eupneico en reposo. Hemodinmicamente ca con vancomicina y meropenem. Los resultados de to-
estable con cifras tensionales de 115/65 mmHg y FC 75 dos ellos fueron negativos salvo el cultivo del dedo donde
lpm. En ese momento estaba afebril (se haba administra- se aisl Enterobacter cloacae y Pseudomonas aeruginosa, por lo
do antitrmico) y tanto la auscultacin cardiorrespiratoria que se simplific antibioterapia a ciprofloxacino asociado
como la exploracin abdominal no arrojaron datos de pa- a gentamicina. Finalmente se decidi la amputacin del
tologa. Presentaba una lesin necrtica en el 4 dedo del dedo. La evolucin clnica fue favorable, todos los estudios
pie izquierdo con un reborde de celulitis y con exudacin; microbiolgicos solicitados fueron negativos y el pacien-
tambin llamaba la atencin la presencia de lesiones pun- te qued afebril con normalizacin de los reactantes de
tiformes y purpricas en ambos miembros inferiores, que fase aguda (RFA). Sin embargo, tras finalizar tratamiento

117
CAPTULO 7
ENDOCARDITIS, INFECCIONES ASOCIADAS
A DISPOSITIVOS ENDOVASCULARES
Y OTRAS INFECCIONES CARDIOVASCULARES

antimicrobiano el paciente comienza de nuevo con fiebre


asociada a tiritona, sin foco aparente presentando el lecho
quirrgico buen aspecto sin datos de complicacin o infec-
cin en el mismo. De nuevo presenta ascenso de los RFA
(PCR 85 mg/l y leucocitosis de 13500 clulas con predo-
minio de PMN). Ante la reaparicin de la fiebre, en esta
ocasin de origen nosocomial, se realiz el diagnstico di-
ferencial entre BRC, gripe nosocomial, infeccin del tracto
urinario, fiebre medicamentosa o brote de poliangetis. Se Verruga a nivel de vlvula mitral. Endocarditis de vlvula artica
solicit TC de trax y abdomen, que no evidenci hallaz-
gos patolgicos. Se repitieron los hemocultivos que resulta-
ron, de forma repetida, negativos y se solicit PCR gripe y
VRS en frotis nasofarngeo resultando negativos. Se inici
antibioterapia emprica en esta ocasin con ceftazidima y
vancomicina.
Tras varios das de evolucin se comprob que el trata-
miento antimicrobiano no supuso ninguna mejora en
el paciente, ya que segua persistiendo la fiebre prctica- Endocarditis de velo anterior Ndulos de Osler
de vlvula tricspide
mente a diario (en torno a 38,5). Se realiz rastreo mi-
crobiolgico de forma repetida sin aislamiento de ningn
microorganismo. Simultneamente presenta progresin de con tincin de Gram de lquido cefalorraqudeo (LCR)
las lesiones vasculticas de los miembros inferiores que lle- negativa para microorganismos. La bioqumica del LCR
garon a ocupar la prctica totalidad de los miembros infe- presentaba 93 clulas (86% PMN); 300 hemates; prote-
riores y el perin. Dichas lesiones se biopsiaron obteniendo nas 0,34 g/l y glucosa 0,37 g/l. La PCR de virus herpes en
el resultado de vasculitis leucocitoclstica. LCR fue negativa. Se solicit cultivo aerobio, tinta china y
Tras 2 semanas con persistencia de la fiebre a pesar de cultivo de hongos.
la antibioterapia y tras descartar otros focos infecciosos se En la analtica destacaba una hemoglobina de 6,9 g/l;
decidi suspender antibioterapia y repetir los hemoculti- PCR 250 mg/l; 14900 leucocitos (88% PMN); creatinina
vos sin fiebre a las 72h sin antibioterapia que, de nuevo, 6,7 g/l (en hemodilisis); urea 150 mg/dl; procalcitoni-
fueron negativos. Una vez descartada la infeccin como na 100 ng/l; resto bien. Se realiz electroencefalograma
causa de la fiebre y habindose demostrado la presencia (EEG) que mostr datos de afectacin cerebral compatible
de un brote de vasculitis cutnea por la progresin de la con encefalopata vrica o posible estatus estando los resul-
lesiones y confirmacin por biopsia se inici tratamiento tados artefactados por sedantes.
inmunosupresor con bolos de 1g de metilprednisolona con
desaparicin de la fiebre durante 48h. Tras dos das asin- Simultneamente se solicitaron hemocultivos diferenciales,
tomtico, presenta empeoramiento agudo con evolucin se retir el catter central, se cultiv la punta del mismo y
trpida y reaparicin de fiebre de hasta 38,6C asociado se extrajo muestra de broncoaspirado para cultivo, siendo
a bajo nivel de consciencia con Glasgow de 8 obligando stos los resultados:
a la intubacin orotraqueal por depresin del centro res-
piratorio y traslado a UCI. Ante la evolucin trpida tras Hemocultivos diferenciales positivos para Staphylococcus
intensificar tratamiento inmunosupresor se plante el diag- aureus sensible a oxacilina (2 de 2) con tiempo de creci-
nstico diferencial entre vasculitis del SNC que pareca miento a favor de catter.
menos probable debido al empeoramiento tras el bolo de En cultivo de la punta del catter y en cultivo de bron-
esteroides y la presencia de una infeccin nosocomial del coaspirado y se aisl S. aureus sensible a oxacilina.
SNC en paciente inmunodeprimido (meningitis por Listeria En este momento se dispone de resultado de cultivo de
monocytogenes, Criptococcus spp., etc) vs sepsis grave con afec- LCR donde tambin se asla S. aureus, siendo negativo
tacin neurolgica secundaria a infeccin nosocomial de el cultivo de hongos.
foco en catter y/o respiratorio.
A su ingreso en UCI se realiza radiografa de trax y TC Se inici antibioterapia con cloxacilina 2g cada 4h iv. Tras
crneo normales. Tambin se realiz una puncin lumbar 72h de antibioterapia ptima y desde la retirada del cat-

118
CAPTULO 7
ENDOCARDITIS, INFECCIONES ASOCIADAS
A DISPOSITIVOS ENDOVASCULARES
Y OTRAS INFECCIONES CARDIOVASCULARES

ter el paciente segua febril y los hemocultivos de control brote de actividad de la enfermedad autoinmune siendo
persistieron positivos de forma repetida (en 6 tandas de menos frecuente la aparicin de sndrome linfoprolifera-
hemocultivos en das alternos), adems se produjo la apa- tivo o neoplasias. En los pacientes inmunodeprimidos el
ricin de pequeas hemorragias en las yemas de los dedos reconocimiento de un proceso infeccioso intercurrente
de las manos. Ante la bacteriemia persistente con apari- puede ser difcil debido a que los signos pueden solaparse
cin de posibles eventos embolgenos a distancia se aadi con los propios de la enfermedad sistmica y por el propio
gentamicina a la cloxacilina y se realiz ecocardiografa estado de inmunosupresin. Adems, en la prctica clnica
transesofgica con los siguientes hallazgos: verrugas en habitual, no es raro encontrarnos con la presentacin de
todo el velo posterior de la vlvula mitral con insuficiencia forma simultnea de algn foco sptico (en nuestro caso la
mitral ligera (2 jets de regurgitacin); vlvula artica bi- lcera del pie y/o CVC) y de sntomas sugestivos de brote
cspide con endocarditis e insuficiencia moderada; vlvula agudo de enfermedad (progresin de lesiones cutneas).
tricspide con endocarditis en velo anterior e insuficiencia Existen pruebas complementarias encaminadas a valorar
ligera; vlvula pulmonar con endocarditis sin insuficien- la reactivacin de la enfermedad (anticuerpos anti-ADN,
cia, fraccin de eyeccin del 40%. Tras ello se mantuvo el activacin del complemento, anticuerpos anticitoplasma
tratamiento ya iniciado, pero a pesar del mismo, los hemo- de neutrfilo, etc) pero no suelen estar disponibles para la
cultivos se mantenan positivos y la evolucin clnica fue in- prctica clnica diaria demorndose semanas y, sobre todo,
satisfactoria. Se realizaba hemodilisis a diario por signos pueden modificarse con la infeccin. Por ello, es muy im-
congestivos en progresin (edema intersticial pulmonar y portante mantener siempre un elevado ndice de sospecha
situacin de anasarca), permaneca intubado con VMI y de infecciones oportunistas y/o infecciones relacionadas
present varios episodios de hemorragia digestiva alta (con con el mbito sanitario y nosocomiales.
origen en la gastritis erosiva puesta de manifiesto median- La BRC es responsable del 10-20% de las infecciones no-
te nueva endoscopia oral) con importante anemizacin a socomiales y supone el 12% de las bacteriemias nosoco-
pesar de las transfusiones sanguneas. El paciente fallece a miales. El riesgo de BRC va a depender del tipo de catter
las dos semanas tras evolucin trpida con mltiples com- siendo la incidencia de bacteriemias de catteres tuneliza-
plicaciones. dos de 1.6 por 1000 das de catter, menor que en catter
central no tunelizado y catter central de insercin peri-
Diagnsticos finales frica (riesgo de bacteriemia de 2,6 y 2,1 por 1000 das
catter, respectivamente). Las infecciones por Staphylococcus
Bacteriemia complicada por S. aureus relacionada con coagulasa negativo y S. aureus son las causas ms frecuen-
catter tunelizado de dilisis. tes de BRC. Se recomienda retirada del catter de forma
Endocarditis definida por S. aureus de las 4 vlvulas inmediata (AII) siendo el tratamiento antimicrobiano de
cardacas nativas secundaria a BRC. eleccin para S. aureus meticiln sensible (SAMS) la cloxa-
Meningitis por S. aureus de origen hematgeno secun- cilina (AI). En pacientes en hemodilisis cefazolina es el
dario a endocarditis. tratamiento de eleccin para la BRC por SAMS (AII). La
Vasculitis sistmica (ANCA negativos) con tratamiento duracin recomendada de tratamiento es de 14 das si se
inmunosupresor. cumplen los siguientes criterios: a) situacin clnica estable
y hemocultivos negativos en las primeras 72 horas de inicio
Discusin del tratamiento eficaz, (2) retirada del catter en las prime-
ras 72h, (3) ausencia de signos clnicos y radiolgicos de
Sin duda, lo didctico y docente del caso presentado, ade- infeccin profunda, (4) ecocardiografa sin signos de endo-
ms de la excepcionalidad de la endocarditis infecciosa carditis, y (5) ausencia de material protsico intravascular
(EI) de las 4 vlvulas cardacas y meningitis secundaria por o articular. Si no se cumplen estos criterios el tratamiento
S. aureus es conocer la dificultad que conlleva establecer debe prolongarse hasta cumplir 4 semanas de tratamiento.
un diagnstico acertado ante un sndrome febril de ori- La persistencia de fiebre y bacteriemia pasadas 72 horas
gen desconocido en un paciente con enfermedad sistmica desde la retirada del catter e inicio de tratamiento deben
bajo tratamiento inmunosupresor. La fiebre es muy comn hacer sospechar la existencia de una endocarditis.
en pacientes con una enfermedad autoinmune sistmica En nuestro caso el paciente present BRC por SAMS com-
(puede ser la manifestacin inicial de la misma) por lo plicada con endocarditis con afectacin de las 4 vlvulas
que ante dicho sntoma se plantea de manera habitual el cardacas. No hemos encontrado ningn caso descrito en
diagnstico diferencial entre etiologa infecciosa frente a la literatura. Se sabe que la inmunosupresin supone un

119
CAPTULO 7
ENDOCARDITIS, INFECCIONES ASOCIADAS
A DISPOSITIVOS ENDOVASCULARES
Y OTRAS INFECCIONES CARDIOVASCULARES

factor de riesgo para la adquisicin de una endocarditis


infecciosa. El tratamiento de EI por S. aureus se basa en
cloxacilina en combinacin con gentamicina. La sustitu-
cin valvular, se recomienda cuando se asocia insuficiencia
cardaca, infeccin incontrolada o para la prevencin de
embolias. En el caso de afectacin por S. aureus, dada la
alta tasa de mortalidad (>45%), se recomienda valorar la
ciruga de manera precoz.
La meningitis por SAMS es una entidad poco frecuente
fuera de los servicios de neurociruga en relacin con me-
ningitis postquirrgicas. En un estudio retrospectivo rea-
lizado en un periodo de 10 aos en un hospital de tercer
nivel se identificaron 668 casos de meningitis bacteriana
siendo 33 (4,9%) causadas por S. aureus, de ellas el 36%
fueron meningitis postoperatorios frente a 21 casos (64%)
por diseminacin hematgena secundarias a endocarditis,
infeccin de piel y partes blandas y neumona. Los factores
de riesgo asociados para presentar meningitis por S. aureus
hematgena fueron presentar edad avanzada, comorbili-
dad y consumo de drogas va parenteral. No hubo diferen-
cia en la tasa de S. aureus resistente a meticilina entre ambos
grupos. La tasa de mortalidad fue mayor en los pacientes
con meningitis por diseminacin hematgena.
Este caso nos ensea la dificultad en el manejo diagnsti-
co de la aparicin de fiebre en pacientes con enfermedad
sistmica inmunodeprimidos, sobre todo en los casos don-
de se solapa fiebre comunitaria con fiebre de adquisicin
de nosocomial. Es muy importante considerar dentro del
diagnstico diferencial la actividad de la enfermedad de
base y mantener un elevado ndice de sospecha de infec-
ciones oportunistas y complicaciones relacionadas con la
hospitalizacin durante todo el ingreso, como sucedi con
nuestro paciente.

Bibliografa

1. Font Franco J, Cervera Segura R. Valoracin diagns-


tica de la fiebre en pacientes con enfermedad autoin-
mune sistmica. Infeccin frente a actividad inflama-
toria. Rev Clin Esp 2001;201:648-9.
2. Vincent JL, Bihari DJ, Suter PM, et al. The prevalence
of nosocomial infection in intensive care units in Euro-
pe. Results of the European Prevalence of Infection in
Intensive Care (EPIC) Study. EPIC International Ad-
visory Committee. JAMA 1995; 274:639-44.
3. Eggimann P. Prevention of intravascular catheter in-
fection. Curr Opin Infect Dis 2007; 20:360-9.
4. Aguilar J, Urday-Cornejo V, Donabedian S et al. Sta-
phylococcus aureus Meningitis. Case Series and Lite-
rature Review. Medicine 2010; 89: 117-25

120
CAPTULO 8
INFECCIN VIH
Y ENFERMEDADES ASOCIADAS
Inmunosuprimir al inmunodeprimido?
Coinfeccin VIH/VHC con sospecha
de crioglobulinemia
Martnez Prez-Crespo, PM; Lpez Montesinos, I;
Domnguez Castellano, A; Ros Villegas, MJ.
Hospital Universitario Virgen Macarena. Sevilla.

Caso clnico

Presentamos el caso de un varn de 52 aos de edad, sin


alergias medicamentosas conocidas, fumador de 60 paque-
tes/ao, bebedor de ms de 70 g de alcohol al da y usua-
rio de drogas por va parenteral (UDVP) en tratamiento
deshabituador con metadona. Es diagnosticado de infec-
cin por virus de la inmunodeficiencia humana (VIH) en
1993, con ltimo estadiaje A2, iniciando tratamiento anti-
rretroviral con estavudina (d4T), didanosina (ddI) y nevira-
pina (NVP) con nula adhesin al mismo, abandono poste-
rior, y escasa asistencia a citas de revisiones peridicas, sin
presentar infecciones oportunistas ni otras enfermedades
definitorias de sndrome de inmunodeficiencia adquirida
(SIDA). En su ltima revisin (octubre de 2014), presenta- Imagen 1. Mano derecha del paciente doce das despus del comienzo de la
clnica. Se aprecia predominio de lesiones ampollosas con rotura de las mismas,
ba un recuento de linfocitos CD4 de 500-749 clulas/mm3 prdida de sustancia y puntos de sangrado.
y carga viral inferior a 10.000 copias/ml. Diagnosticado
tambin de cirrosis heptica (Child A) por infeccin crni-
ca por virus de la hepatitis C (VHC) genotipo 1-A para el
que inicialmente realiz tratamiento con interfern pegila-
do y ribavirina con buena respuesta, suspendido finalmen-
te en mayo de 2014 por mal cumplimiento del mismo. Se
solicitaron elastometra, estadio F4 (49 Kpa), y ecografa
(esta ltima no realizada por el paciente), con ltimo con-
trol de carga virolgica en octubre de 2014 cuantificada
en 4.000.000 UI/ml. Presenta adems antecedentes de
episodios autolimitados de lesiones purpricas palpables
vasculticas en partes acras con presencia de crioglobulinas
positivas, compatible con crioglobulinemia mixta, en estu-
dio y seguimiento por Dermatologa.
El paciente acude al Servicio de Urgencias de nuestro cen-
tro trado por su familiar en enero de 2015, refiriendo ste Imagen 2. Miembro inferior izquierdo del paciente doce das despus del inicio
de la clnica. Se aprecian lesiones vasculticas purpricas, as como otras ampo-
un cuadro de deterioro importante de su estado general, llosas y ulcerosas con prdida de sustancia en estado cicatricial.
bradipsiquia, bradilalia y desorientacin de unos tres das
de evolucin, con la aparicin de lesiones dermatolgicas (tensin arterial 111/80 mmHg y frecuencia cardaca 62
purpricas con edema y eritema acompaante en manos, lpm), y permanece afebril con tendencia a la hipotermia
codos y miembros inferiores en las ltimas 24 horas. A (35.4C). Neurolgicamente no se objetiva rigidez de nuca
su llegada a Urgencias, presenta mal estado general, con ni otros signos menngeos, con pupilas isocricas y normo-
desorientacin temporoespacial, lenguaje incoherente y rreactivas y movimientos oculares extrnsecos conservados,
somnolencia, con 12 puntos en la escala Glasgow, respues- sin otros dficits neurolgicos apreciables y reflejos cut-
ta ocular a estmulos verbales y sin obedecer rdenes ni neo-plantares flexores. A la exploracin cardiopulmonar se
capacidad de colaboracin. Muestra signos de deshidra- auscultan tonos cardacos rtmicos y puros, sin soplos au-
tacin en piel y mucosas, con estabilidad hemodinmica dibles, y murmullo vesicular conservado sin ruidos patol-

122
CAPTULO 8
INFECCIN VIH
Y ENFERMEDADES ASOCIADAS

Clnica Frecuencia cina, manteniendo cefotaxima y corticoterapia. En cuanto


Prpura palpable 80-100%
a la encefalopata, sea cual sea la afeccin primaria en este
Artralgias/ artritis 80-100%
cuadro parece sensato plantear un componente multifacto-
Astenia 80-100%
rial del mismo, y dada la infeccin crnica por VHC que
Neuropata perifrica 50-70%
padece el paciente se puede considerar incluso un predo-
Afectacin heptica 40-60%
minio heptico del proceso con desarrollo de encefalopata
Otras manifestaciones dermatolgicas 10-40%
(lceras cutneas, lesiones necrticas, acrocianosis, fenmeno
heptica. Tampoco se puede descartar en este momento la
de Ryanaud) presencia de una infeccin oportunista, una entidad aso-
Afectacin renal 10-30% ciada a VIH o incluso enfermedad definitoria de SIDA, y
recordemos que el enfermo es consumidor activo de txi-
Afectacin pulmonar severa <1% cos por va intravenosa. Por ltimo, no podemos olvidar
5, 6, 7
Tabla 1. Manifestaciones clnicas de la vasculitis crioglobulinmica . la posibilidad de que el cuadro actual sea secundario a la
crioglobulinemia mixta sospechada en este paciente.
gicos sobreaadidos, eupneico con pulsioximetra del 98%
a aire ambiente. El abdomen es blando y depresible, sin Evolucin
puntos dolorosos aparentes ni signos de peritonismo, con
palpacin de hepatomegalia de unos 3 cm bajo el reborde En las horas siguientes a su ingreso en observacin el pa-
costal y ruidos hidroareos conservados. Se aprecian lesio- ciente desarrolla deterioro progresivo de su estado general
nes dermatolgicas maculopapulares y ampollosas viol- y analtico, con tendencia a la hipotensin, taquicardia, ta-
ceas en manos y pies con edema y eritema acompaante. quipnea y empeoramiento del cuadro encefaloptico y las
Se solicitan como pruebas complementarias hemograma, lesiones dermatolgicas, que pasan a ser de distribucin
coagulacin, bioqumica, gasometra venosa, electrocar- generalizada en extremidades e intensamente dolorosas,
diograma, radiografa de trax y hemocultivos seriados. presentndose en distinto estado de evolucin y siendo
Analticamente destaca leucocitosis de 25050 clulas/ la mayora ampollosas sobre fondo eritematoviolceo, y
mm3 con 17443 neutrfilos/mm3, alargamiento de los otras de contenido necrtico, algunas mostrando solucin
tiempos de coagulacin con INR en 3.6, discreta acidosis de continuidad y prdida de sustancia con ligero sangrado,
(pH 7.33, pCO2 36.9 mmHg y HCO3 19.5 mmol/l), hi- junto a otras de aspecto cicatricial de coloracin roscea
poglucemia (39 mg/dl) y elevacin de los niveles plasm- y lesiones purpricas palpables hasta muslos y brazos. Se
ticos de creatinina (2.94 mg/dl), urea (198 mg/dl), potasio solicita valoracin por el Servicio de Cuidados Intensivos,
(7.2 mEq/l), CPK (1108 U/L), LDH (1150 U/L), GOT que indica ingreso su unidad.
(244 U/L), lactato (4.1 mmol/l), procalcitonina (2.2 ng/ Durante las primeras horas de su ingreso en la Unidad
ml) y PCR (132.22 mg/l). En la radiografa de trax y el de Cuidados Intensivos (UCI) se objetiva el desarrollo de
ECG no se aprecian hallazgos patolgicos. trombopenia (76000 plaquetas) asociada al alargamiento
de los tiempos de coagulacin detectado en Urgencias y
Diagnstico diferencial que se mantendr durante el ingreso, por lo que se opta
por no practicar puncin lumbar u otras tcnicas diagns-
Nos encontramos ante un paciente coinfectado VIH/VHC ticas invasivas. Es valorado conjuntamente por los servicios
con nula adherencia al tratamiento que presenta un cua- de Enfermedades Infecciosas, Dermatologa y Cuidados
dro encefaloptico agudo asociado a deterioro severo de Intensivos para coordinar una estrategia teraputica, deci-
su estado general, lesiones cutneas purpricas palpables diendo mantener la antibioterapia emprica mencionada,
y ampollosas, eritematoviolceas en extremidades, una ele- aadiendo terapia antirretroviral con doluteglavir, teno-
vacin importante de reactantes de fase aguda (neutrofilia, fovir y emtricitabina y posponiendo inicio de tratamiento
PCR y procalcitonina) y fracaso multiorgnico por insufi- especfico para VHC en funcin de la recuperacin del
ciencia renal y heptica aguda o crnica reagudizada. En paciente. Se decide iniciar adems tratamiento con ciclo-
primer lugar, ante la posibilidad de meningitis aguda se ad- fosfamida ante la alta sospecha de vasculitis crioglobulin-
ministra antibioterapia emprica precoz con cefotaxima 2g mica. Del mismo modo se comienza profilaxis antibitica
va intravenosa (iv) y dexametasona. Ante la posibilidad de dada la patologa basal del paciente en ausencia de trata-
sepsis grave con foco probable en partes blandas o sistema miento estable. A las 72 horas de su ingreso hospitalario,
nervioso central, asociado a fracaso multiorgnico, se am- Microbiologa informa del crecimiento de levaduras en los
pla la antibioterapia emprica con ampicilina y vancomi- hemocultivos extrados al ingreso, por lo que se inicia trata-

123
CAPTULO 8
INFECCIN VIH
Y ENFERMEDADES ASOCIADAS

miento emprico con caspofungina. Finalmente se identifi- creto la prevalencia puede ser mucho mayor, llegando a
car el hongo como Candida albicans, no logrando dilucidar ms del 65% 2.
el foco del mismo, desescalando el tratamiento antifngico Existe controversia entre la influencia que tienen el VIH
a fluconazol. Al sexto da de ingreso Micobiologa infor- y la coinfeccin VIH/VHC en la crioglobulinemia mix-
ma de nuevo del crecimiento de Staphilococcus aureus me- ta. Parece jugar un papel importante, aunque menor que
ticilin-sensible en los hemocultivos extrados inicialmente. la infeccin por VHC, en la prevalencia de esta enferme-
El paciente fue valorado por los servicios de Oftalmologa, dad: pueden detectarse crioglobulinas en el 17-26% de los
sin hallar alteraciones en la exploracin del fondo de ojo; pacientes VIH, aunque este aumento no se correlaciona
Cardiologa, que realiza ecocardiografa transtorcica, de- con mayor presencia de enfermedad vascultica3. Por otro
tectando hipertensin pulmonar importante (80mmHg) lado, diversos estudios han reportado una mayor preva-
con dilatacin de cavidades derechas, aurcula izquierda lencia de crioglobulinemia en pacientes coinfectados por
dilatada, funcin sistlica conservada y ausencia de le- VIH/VHC que en monoinfeccin por VHC, aunque
siones intracavitarias; y Reumatologa, impresionndoles otros investigadores no han encontrado dicha asociacin3.
el cuadro de vasculitis crioglobulinmica y aconsejando La forma de presentacin ms frecuente en pacientes co-
mantener corticoterapia y tratamiento con ciclofosfamida. infectados por ambos virus es la polineuropata, seguido
Se realiza adems TAC craneal, que resulta normal, carga de lesiones cutneas y dao renal4. Los pacientes que de-
viral de VIH (373000 copias/ml) y recuento celular (200 sarrollan crioglobulinemia en la coinfeccin VIH/VHC
CD4/mm3). tienden a ser hombres ms jvenes y con mayor historial
El paciente comienza a evolucionar favorablemente, con de consumo de drogas por va parenteral que los monoin-
mejora de la funcin renal, heptica y neurolgica, de sus fectados por VHC4, que no obstante es la causa primaria
lesiones dermatolgicas, as como disminucin de sus re- ms frecuente de crioglobulinemia.
actantes de fase aguda, por lo que una vez estabilizado, se La vasculitis crioglobulinmica o crioglobulinemia mixta
traslada a planta de Enfermedades Infecciosas, continuan- es una entidad clnica perteneciente a la familia de las vas-
do con una evolucin favorable hasta el alta hospitalaria a culitis: enfermedades de carcter autoinmune caracteriza-
los 21 das de su ingreso. das por la presencia de inflamacin y necrosis de los vasos
sanguneos, lo que acaba desencadenando dao tisular.
Diagnstico definitivo Clasificada segn la nomeclatura de Chapel-Hill de 2012
dentro de las vasculitis por hipersensibilidad o leucocito-
El paciente fue dado de alta con los diagnsticos de crio- clsticas, afecta predominantemente a pequeos vasos (ar-
globulinemia mixta secundaria a coinfeccin VIH/VHC teriolas, capilares y vnulas poscapilares) y se caracteriza
con desarrollo de sepsis grave de probable origen en partes por la presencia de crioglobulinas circulantes, que son in-
blandas con candidemia por Candida albicans y bacteriemia munoglobulinas monoclonales (crioglobulinas de tipo I) o
por Staphiloccocus aureus meticilin sensible, cuadro encefalo- policlonales (crioglobulinas de tipo II y III, tambin llama-
ptico agudo de etiologa multifactorial (encefalopata he- das crioglobulinas mixtas) que precipitan con el fro. Sus
ptica y/o encefalopata asociada a sepsis (EAS). manifestaciones clnicas aparecen como consecuencia de
la oclusin vascular por crioprecipitados o por fenmenos
Discusin inflamatorios derivados de la formacin de inmunocom-
plejos por las crioglobulinas. Este fenmeno puede obser-
A pesar de la progresiva disminucin de la incidencia de varse en gran cantidad de entidades patolgicas de fondo:
VHC, la coinfeccin de VIH/VHC es un problema de infecciones agudas o crnicas como VHB, VIH o virus de
salud creciente, aportando en Espaa una alta carga de Epstein-Barr; hepatopatas crnicas; enfermedades he-
enfermedad asociada que tiende a empeorar. La presencia matolgicas como mieloma mltiple, macroglobulinemia
de VHC empeora la evolucin del paciente VIH, aumen- de Waldestrom o sndromes linfoproliferativos; conectivo-
tando su morbimortalidad, reduciendo la reconstitucin patas o causas idiopticas. Sus manifestaciones clnicas
inmunolgica e incrementando la activacin inmune cr- clsicas consisten en prpura palpable, artralgias o artritis
nica. Su prevalencia est especialmente ligada al uso de y astenia, que constituyen la trada de Meltzer, y ocurren
drogas por va parenteral, con una prevalencia del 50-80% en el 80-100% de los pacientes. Otras manifestaciones
en esta poblacin1. En Andaluca, en la actualidad la pre- dermatolgicas menos frecuentes son lceras cutneas,
valencia de coinfeccin VIH/VHC es aproximadamente fenmeno de Ryanaud, acrocianosis y lesiones necrticas
del 40% (1), aunque se estima que en nuestro medio con- en zonas acras. Puede desarrollarse neuropata perifri-

124
CAPTULO 8
INFECCIN VIH
Y ENFERMEDADES ASOCIADAS

ca en el 50-70% de los pacientes, afectacin heptica en En nuestro caso, es difcil conocer con exactitud hasta qu
un 40-60% de casos, y en el 10-30% de ocasiones existe punto cul de las entidades descritas jug un papel deter-
afectacin renal, que en el 80% ser una glomerulonefritis minante, aunque destaca la rpida mejora que el pacien-
membranoproliferativa difusa, y de las que un 15% evolu- te present tras el inicio del tratamiento inmunosupresor.
cionar a nefropata aguda terminal. En raras ocasiones se Una biopsia renal podra haber ayudado a aclarar esta
produce afectacin vascultica del sistema nervioso central, cuestin, pero no pudo ser practicada por las alteraciones
digestivo o cardiopulmonar. Aunque las vasculitis por hi- de la coagulacin que presentaba el paciente.
persensibilidad suelen tener un pronstico benigno y auto-
limitado, en la crioglobulinemia (y en menor medida, en la Bibliografa
prpura de Schonlein-Henoch) puede ocurrir afectacin
visceral grave con desenlace fatal5, 6, 7. 1. Serrano-Villar S,Sobrino-Vegas P,Monge S,Dronda
Su diagnstico, ms all de la demostracin serolgica de F,Hernando A,Montero M, et al. Decreasing preva-
crioglobulinas o histolgica de vasculitis leucocitoclstica, lence of HCV virus coinfection in all risk groups for
es puramente clnico y se basa en la presencia de sntomas HIV infection between 2004 and 2011 in Spain. J Vi-
compatibles: prpura palpable y alguna de las lesiones des- ral Hepat.2015;22:496-503.
critas8. En nuestro caso exista una fuerte sospecha clnica 2. Cifuentes C, Mira JA, Vargas J, Neukam K, Escassi
de la misma, apoyada por la presencia de crioglobulinas y C, Garca-Rey S, et al. Prevalencia de los marcadores
lesiones anatomopatolgicas compatibles, por lo que se de- de infeccin de los virus de las hepatitis en pacientes
cidi tratar enrgicamente, considerando que no hacerlo portadores de VIH en el sur de Espaa. Enferm Infecc
pondra en grave riesgo la vida del enfermo. Microbiol Clin. 2012;30:4527.
Actualmente, la primera lnea de tratamiento de la crio- 3. Saadoun D, Aaron L, Resche-Rigon M, Pialoux G,
globulinemia en casos leves o moderados es el control de la Pette JC, Cacoub P. Cryoglobilinaemia vasculitis in
enfermedad de base junto al uso de corticoides en la fase patients coinfected with HIV and hepatitis C virus.
aguda para el control sintomtico. Sin embargo, en casos AIDS. 2006; 20:8717.
rpidamente progresivos o de mal pronstico se postula el 4. Ojaimi S, Lin MW, Singh KP, Woolley I. The two-ed-
uso precoz de inmunosupresores. En caso de precisarlos, ged sword: vasculitis associated with HIV and hepatitis
la primera eleccin ha de ser el rituximab asociado a ciclo C coinfection. International Journal of STD & AIDS.
agudo de corticoides. De segunda lnea se consideran la 2014; 25:77-88.
corticoterapia a altas dosis de manera mantenida, la ciclo- 5. Longo DL, Kasper DL, Jameson JL, Fauci AS, Hauser
fosfamida, o en ltimo lugar, la plasmafresis. En nuestro SL, Loscalzo JL, editors.Harrison's Principles of Internal
caso, no pudo usarse rituximab, debido a las contraindi- Medicine. 18th ed. New York: McGraw-Hill; 2012.
caciones que mantiene con el VIH: no debe usarse hasta 6. Farreras Valenti P. y Rozman C.Medicina Interna. 16
conseguir la ausencia de replicacin viral o un recuento ed. Barcelona: Elsevier; 2008.
celular mayor a 250 CD4/mm3. En el momento de iniciar 7. Ferri C, Sebastiani M, Giuggioli D, Cazzato M, Lon-
el tratamiento el paciente llevaba ya tres das con cortico- gombardo G, Antonelli A, el al. Mixed cryoglobu-
terapia sin mostrar gran mejora, y la plasmafresis, opcin linemia: demografic, clinical and serologic features
que tambin se plante, no goza de suficiente evidencia and survival in 231 patients. Semin Arthtitis Rheum.
aplicable a nuestro caso y adems no previene la forma- 2004;33:355-74.
cin de nuevas crioglobulinas, debiendo asociarse a su uso 8. Damoiseaux J. The diagnosis and classification
un inmunosupresor, por lo que se plante como ltima of the cryoglobulinemic syndrome. Autoimmun
opcin. Se decidi la administracin de ciclofosfamida, a Rev.2014;13:359-62.
pesar del riesgo de descontrol de replicacin viral y yatro- 9. Pietogrande M, De Vita S, Zignengo AL, Pioltelli P,
genia, junto a la terapia antirretroviral precoz, tal y como Sansonno D, Sollima S, et al. Recommendations for
est indicada en la vasculitis crioglobulinmica asociada the management of mixed crioglobulinemia sindrome
a VIH. En cambio, la indicacin de tratamiento para el in hepatitis C virus-infected patients. Autoinmun Rev.
VHC debe posponerse entre 1 y 4 meses ante la presencia 2011; 10:444-54.
de cirrosis descompensada o el uso de inmunosupresores4,
9
. No obstante, creemos que estos conceptos habrn de ser
revisados prximamente ante la llegada de los nuevos tra-
tamientos para la infeccin por VHC.

125
Varn VIH de 39 aos con FOD,
astenia y prdida de peso
Alarcn Garca, JC; Gonzlez Estrada, A;
Praena Segovia, J; Luque Mrquez, R.
Hospital Universitario Virgen del Roco. Sevilla.

Caso clnico

Paciente varn de 39 aos diagnosticado de infeccin por


VIH en diciembre de 2012 por transmisin heterosexual.
Inici terapia antiretroviral (TAR) con Atripla con 16
CD4/ul % y una carga viral de 49400 copias/ml. No otros
antecedentes personales de mencin.
Ingresa tras un ao de TAR para estudio por fiebre, astenia
y prdida de 15 kg de peso de tres semanas de evolucin.
Afebril al momento de la exploracin donde destaca he-
patoesplenomegalia masiva sin adenopatas en territorios
perifricos. No se apreciaban otros hallazgos relevantes en
el resto de la exploracin.
En bioqumica destacan: Fosfatasa alcalina 616 U/L,
GGT 423 UI/L, Bilirrubina total 0.53 mg/dl, Albmi- Figura 1. Hepatoesplenomegalia.
na 2.5 g/dl. Inmunoglobulina G 3203 mg/dl, Inmuno-
globulina A 683.80 mg/dl. VSG 113 mm/h, Protena C
Reactiva 73.1 mg/l. En hemograma se objetiva: Leuco-
citos 3.30 x10e9/L; Neutrfilos 2.10 x10e9/L; Linfocitos
0.7 x10e9/L; Hemoglobina 90 g/L de perfil normocti-
co normocrmico; Hematocrito 0.26 L/L; Plaquetas 246
x10e9/L. Beta-2-microglobulina 14.30 mg/l. El recuento
de linfocitos CD4+ fue de 110/ul (% CD4+ 14.78). Car-
ga viral 849 copias/ml. Los hemocultivos, baciloscopia y
cultivo de esputo para micobacterias resultaron negativos.
Una radiografa de trax no mostr alteraciones.

Diagnstico diferencial y evolucin

El diagnstico de fiebre de origen desconocido (FOD) en Figura 2. Captacin adenoptica en PET-TC.


un paciente VIH siempre es una tarea ardua y comple-
ja. Presenta un diagnstico diferencial amplio entre cuyas mesentrico como retroperitoneal, y una marcada hepa-
etiologas ms frecuentes se encuentran las infecciones, toesplenomegalia homognea (bazo de 23 cm) con una
en concreto Mycobaterium tuberculosis y Mycobacterium avium mnima cantidad de lquido libre en pelvis menor (figura
complex (MAC), seguidas por toxoplasmosis, leishmaniasis 1). Basndonos en estos datos, se plante como primera
o infeccin por citomegalovirus. Entre los procesos no in- opcin un sndrome linfoproliferativo, y menos probable
fecciosos destacan las neoplasias, sobre todo los sndromes una infeccin diseminada por micobaterias y una leishma-
linfoproliferativos (Linfoma no Hodgkin y Hodgkin) [1] niasis.
Una de las pruebas ms tiles para iniciar el estudio de la El estudio de anemia la catalog de trastornos crnicos
FOD en VIH es la TC de trax y abdomen. En nuestro y la serologa para fiebre de duracin intermedia result
caso objetiv mltiples adenopatas mesentricas y retro- negativa. Para seleccionar la adenopata ptima para la
peritoneales con tendencia a la formacin de conglomera- biopsia retroperitoneal se solicit una PET-TAC (figura 2)
dos, que alcanzaban los 21 mm de dimetro tanto a nivel que demostr afectacin linftica infradiafragmtica (me-

126
CAPTULO 8
INFECCIN VIH
Y ENFERMEDADES ASOCIADAS

que mostr una hepatitis granulomatosa identificndose


bacilos intracelulares ZN positivos hallazgos sugestivos de
infeccin por Mycobaterium avium intracelullare (MAI). Ante
los hallazgos se inici, con buena tolerancia, tratamiento
con Isoniacida (H), Rifampicina (R), Etambutol (E), Azi-
tromicina y Moxifloxacino. Se remiti un cilindro heptico
para cultivo de micobacterias que result negativo.
El diagnstico final se alcanz mediante tcnicas molecu-
lares. Se realiz extraccin de DNA (Qiagen QIAamp
DNA FFPEkit) de biopsias de adenopata y heptica a la
que se aplicaron una PCR en tiempo real para Mycobac-
terium tuberculosis complex (artus M. tuberculosis RG PCR
Kit v. 1.0 de Qiagen) que result negativa. A una alcuo-
ta del DNA se le realiz una tcnica de hibridacin para
Figura 3. Biopsia ganglionar con Zielh-Neelsen positivo. micobacterias no tuberculosas (kit comercial GenoType
HAINE) que identific una Mycobacterium genavense en am-
bas muestras (los cultivos de micobacterias siempre fueron
negativos).
Se modific el tratamiento a Rifabutina 150 mg/d (3 ve-
ces por semana), Etambutol 400 mg, Claritromicina 500
mg/12h, Amikacina 1000 mg/d y cotrimoxazol a dosis
profilcticas. Se modifica tambin TAR, cambiando Atri-
pla por ATV-3TC-DRV/r. Fue necesario aadir Pred-
nisona 1 mg/Kg/d por persistencia de la fiebre a pesar
de la pauta antimicrobiana, continuando con una dosis
lentamente decreciente. Al alta el paciente estaba afebril,
con mejora del estado general y con un control analtico
que mostr un descenso del nivel de Protena C Reactiva
a 29 mg/l.
La evolucin del paciente es lenta y trpida. Ingresa pos-
Figura 4. Ascitis quilosa. teriormente por fracaso renal agudo de origen multifac-
torial, debido a nefrotoxicidad por frmacos y deplecin
sentrica y retroperitoneal con un SUV mximo de 9,3) y de volumen por cuadro emtico. Se sustituy Claritromi-
esplnica, objetivando una lesin hipermetablica en me- cina por Azitromicina 500 mg, se suspendi Amikacina y
diastino posterior. se aadi Moxifloxacino 400 mg. Posteriormente present
Se realiz aspirado de mdula sea (MO) que no evidenci varios ingresos por dolor abdominal y sndrome constitu-
bacilos alcohol resistentes (B.A.A.R.) y biopsia que mostr cional, realizndose endoscopia oral con toma de biopsia
un infiltrado granulomatoso no necrotizante con Zielh-Ne- duodenal informada como duodenitis granulomatosa con
elsen (ZN), Grocott y Giemsa negativos. La citometra de ZN negativa.
flujo de la MO no evidenci infiltracin por linfoma. Se Tras un ao de tratamiento, con un recuento de CD4+ de
enviaron muestras para cultivo de micobacterias. La PCR 72 c/ul y una carga viral de < 20 copias/ml, reingresa por
para Leishmania spp fue negativa. Mediante laparoscopia se dolor y distensin abdominal asociados a deposiciones dia-
biopsi una adenopata mesentrica que se inform como: rricas amarillentas y fiebre intermitente. A la exploracin
infiltrado histiocitario con ZN positivo en relacin a in- se evidencia desnutricin protico-calrica y presencia de
feccin por micobacterias atpicas (figura 3) y ausencia ascitis grado 3 junto a edemas en miembros inferiores. En
de infiltracin por proceso linfoproliferativo. No se remi- la analtica persiste el patrn de colestasis disociada con FA
ti muestra a microbiologa para cultivo. La muestra de 1063 U/L, GGT 358 UI/L y Bilirrubina total en 0.57 mg/
biopsia fue incluida en su totalidad en parafina, solicitn- dl. Protena C Reactiva 46.4 mg/L, Albumina 2.1 g/dl.
dose extraccin de DNA de la muestra parafinada para Beta-2-microglobulina 10.60 mg/l. En hemograma desta-
amplificacin mediante PCR. Se realiz biopsia heptica can Leucocitos 1.97 x10e9/L, Neutrfilos 1.38 x10e9/L,

127
CAPTULO 8
INFECCIN VIH
Y ENFERMEDADES ASOCIADAS

Linfocitos 0.2x10e9/L, Hb 86 g/L, Hematocrito 0.27 patognesis continua siendo desconocida. Su distribucin
L/L. VSG 70 mm/h Los hemocultivos fueron negativos. geogrfica es amplia, siendo Europa y Norte Amrica las
En la TC de abdomen de control previo al ingreso persis- zonas donde se han reportado la mayora de aislamientos.
ten las adenopatas retroperitoneales y mesentricas con En un estudio restrospectivo multicntrico se observ que
tendencia a la formacin de conglomerados, objetivndo- los sntomas y signos ms frecuentes eran astenia, prdida
se ascitis periheptica, periesplnica, entre asas y en pelvis de peso, fiebre, esplenomegalia, diarrea y hepatomegalia,
menor. Persistencia as mismo de hepatoesplenomegalia. presentando el 62.5% adenopatas, sobre todo abdomina-
Se realiz parecentesis evacuadora con salida de 3,5 litros les [5], siendo indistinguibles de una infeccin diseminada
de lquido de aspecto quiloso (figura 4), hallazgos que se por otras micobacterias. Sin embargo parece que el dolor
confirmaron posteriormente mediante el anlisis bioqu- abdominal es un sntoma que podra asociarse con ms
mico del lquido asctico: Trigliceridos 217.6 mg/dl, c- frecuencia a MG. [6, 7]
lulas 106 cel/mm3, polimorfonucleares (PMN) 13.2 %, Los hallazgos microbiolgicos e histolgicos son similares
mononucleares 86.8%, protenas 26.7 g/L, glucosa 0.99 a aquellos encontrados en una infeccin por MAC. En un
g/L, ADA 17.9 mU/ml. No se evidenciaron B.A.A.R y estudio retrospectivo de una cohorte valenciana el tiem-
el cultivo de micobacterias result negativo. El paciente po medio de crecimiento fue de 3-4 meses [6]. Una ba-
precis alimentacin por sonda nasogstrica y nutricin ciloscopia intensamente positiva junto a tal retraso en el
parenteral ajustada a las necesidades nutricionales por as- cultivo debera hacernos sospechar de una MG. La san-
citis quilosa y enteropata pierde-protenas atribuida a lin- gre, el tracto digestivo y las adenopatas parecen los sitios
fangiectasia intestinal por bloqueo linftico. La etiologa ms rentables para visualizar esta micobacteria [4, 5, 6].
de la ascitis quilosa es amplia, siendo las neoplasias como Su cultivo en medios de micobacterias es lento y requiere
linfoma una de las causas ms frecuentes. En nuestro caso enriquecimiento con micobactina por lo que su rentabili-
y gracias a los hallazgos de la TC abdominal, se atribuy dad es escasa. La introduccin desde hace algunos aos
al bloqueo linftico retroperitoneal producido por los con- de nuevos mtodos diagnsticos mediante identificacin
glomerados adenopticos. genotpica, como la amplificacin de secuencias de ADN
especficas, ha proporcionado un diagnstico rpido y pre-
Diagnsticos finales ciso. En nuestro caso se realiz un anlisis mediante hibri-
dacin en fase slida en tiras de nitrocelulosa mediante el
Infeccin diseminada por Mycobacterium genavense. kit comercial GenoType a partir de una muestra de DNA
Ascitis quilosa por bloqueo linftico. Mesenteritis retrctil. extrada de la muestra parafinada de biopsia que permiti
Desnutricin metablico-proteica. Enteropata pierde-pro- la identificacin precisa de la especie [8].
teinas (probable) El tratamiento ptimo de MG continua estando poco cla-
Infeccin VIH categora C3. ro. En un estudio en ratones se demostr la eficacia del
tratamiento con claritromicina y rifabutina. El ciprofloxa-
Discusin cino no result efectivo [9]. El rgimen teraputico incluy
un macrlido, etambutol y rifampicina. En las anteriores
Mycobacterium genavense (MG) es una micobateria no tuber- series expuestas, los frmacos ms usados fueron claritro-
culosa de lento crecimiento descubierta a principios de la micina, etambutol, rifabutina y amikacina. Entre otros
dcada de los 90, causante de una infeccin diseminada en frmacos utilizados se encontraban quinolonas como ci-
pacientes con infeccin por VIH [2]. As mismo se ha visto profloxacino o moxifloxacino, isoniazida y rifampicina [4,
que puede afectar a pacientes inmunodeprimidos no-VIH 5, 6]. La duracin del tratamiento es similar al del resto de
como transplantados [3], pacientes con leucemia linftica infecciones por MAC.
crnica, sarcoidosis y LES La mayora de los casos descri- Las complicaciones de una infeccin por MG pueden ser
tos pertenecen a la era pre-TARGA, presentes en pacientes mltiples y variadas. Recientemente se ha descrito el desa-
con un recuento de CD4+ habitualmente menor de 100 rrollo de ascitis, hipoalbuminemia, transformacin retrc-
c/ul, como pudo comprobarse en una serie de 24 casos en til del mesenterio con oclusin trombtica de vena cava
Italia recogidos por Tortoli et al [4]. Sin embargo se han infrarrenal y mesentrica superior en un paciente VIH y
documentado casos en pacientes no VIH con un contaje diseminacin por M.genavense [7]. En otro caso similar se
de CD4+ mucho mayor. Ha sido aislada en aves, perros, document as mismo mesenteritis retrctil y oclusin par-
conejos y en el tracto gastrointestinal de personas inmu- cial de la vena mesentrica superior. En nuestro caso, el pa-
nocompetentes, aunque su prevalencia actual as como su ciente desarroll adems de la ascitis quilosa mencionada

128
CAPTULO 8
INFECCIN VIH
Y ENFERMEDADES ASOCIADAS

previamente, infartos esplnicos y fibrosis retroperitoneal cin, amikacin, ethambutol, clarithromycin and rifabu-
an en estudio, atribuida en principio a mesenteritis re- tin. Journal of Antimicrobial Chemotherapy 1998; 42, 483-
tractil con buena respuesta al tratamiento con corticoides. 487.
En conclusin, MG es una micobacteria no tuberculosa de
crecimiento fastidioso con una incidencia en aumento no
solo en pacientes VIH, sino en pacientes inmunosuprimidos
por diferentes etiologas. Es potencialmente capaz de pro-
ducir multitud de complicaciones que ensombrecen el pro-
nstico de estos pacientes. Los nuevos mtodos moleculares
permiten un diagnstico rpido y preciso, siendo necesaria
una alta sospecha diagnstica, sobre todo cuando nos ha-
llamos ante baciloscopias intensamente positivas y ausencia
de crecimiento en cultivo, debiendo establecer una estrecha
relacin clnico-microbilogo para su deteccin.

Bibliografa

1. Hot J, Schmulewitz L, Viard JP et al. Fever of Unk-


nown Origin in HIV/AIDS Patients. Infect Dis Clin
North Am 2007; 21: 1013-32.
2. Bttger EC,Teske A,Kirschner P,Bost S,Chang
HR,Beer V,Hirschel B. Disseminated Mycobacte-
riumgenavense infection in patients with AIDS. Lan-
cet1992; 340:76-80.
3. Doggett JS, Strasfeld L. Disseminated Mycobacterium
genavense with pulmonary nodules in a kidney trans-
plant recipient: case report and rewiew of the literatu-
re. Transpl Infect Dis 2011; 13: 38-43.
4. Tortoli E, Brunello F, Cagni AE et al. Mycobacterium
genavense in AIDS patients, report of 24 cases in Italy
and review of the literature. Eur J Epidemiol 1998; 14:
219224.
5. Pierre Charles, Lortholary O, Dechartres A et al. My-
cobacterium genavense Infections. A Retrospective Multi-
center Study in France, 1996-2007. Medicine 2011;
90: 223-30.
6. Santos M, Gil-Brusola A, Escandell A, Blanes M and
Gobernado.M. Mycobacterium genavense Infections in a
Tertiary Hospital and Reviewed Cases in Non-HIV
Patients. Patholog Res Int 2014; 2014:371370.
7. Borde JP, Offensperger WB, Kern WV and Wagner D.
Mycobacterium genavense specific mesenteritic syndrome
in HIV-infected patients: a new entity of retractile me-
senteritis? AIDS 2013; 27: 281724.
8. Alcaide Fernndez de Vega A. Nuevos mtodos de
identificacin de micobacterias. Enferm Infecc Microbiol
Clin 2006; 24 (Supl 1):53-7.
9. Vrioni G, Nauciel C, Kerharo G and Matsiota-Ber-
nard P. Treatment of disseminated Mycobacterium
genavense infection in a murine model with ciprofloxa-

129
Fiebre, adenopatas y lesiones nodulares
esplnicas en un paciente con infeccin
por el virus de la inmunodeficiencia humana
Gmez Rubio, J; Flores Alvarez, F;
Brcena Atalaya, AB; Mira Escarti, JA.
Hospital Universitario de Valme. Sevilla.

Caso clnico

Varn de 34 aos diagnosticado dos meses antes de in-


feccin por VIH que en el momento del diagnstico pre-
sentaba una cifra de linfocitos CD4+ de 3 clulasl/L y
una carga viral plasmtica (CVP) de 31.000 copias/ml.
Inici tratamiento antirretroviral (TAR) con efavirenz 600
mg/24 h v.o., emtricitabina 200 mg/24 h v.o. y tenofo-
vir 200 mg/24 h v.o., coformulados a dosis fijas, y sulfa-
metoxazol-trimetoprim 160/800 mg 3 veces por semana
v.o. Un mes y medio despus del inicio del TAR ingres
por fiebre (38-39 C) de predominio vespertino, con esca-
lofros, sudoracin, astenia, emaciacin grave y malestar
general de tres semanas de evolucin. En la exploracin
fsica destacaba delgadez extrema (IMC: 17,6), diversas le-
siones cutneo-mucosas de color rojo-vinoso, ligeramente
sobreelevadas en el paladar, el hombro izquierdo y la re-
gin dorsal del pie sugestivas de sarcoma de Kaposi y que
el paciente se neg a biopsia. Su temperatura era de 38,7
C y estaba discretamente taquipnico en reposo, con una
saturacin de oxgeno de 96% respirando aire ambiente.
Se palpaba una esplenomegalia de 6 centmetros, indolora
y de consistencia blanda, pequeas adenopatas, rodaderas
e indoloras, en ambas regiones inguinales, as como en las
regiones supraclavicular izquierda y submandibular dere-
cha. En la analtica destacaba: pancitopenia (Hb 9,1 mg/

Figura 2. Corte transversal de la TC abdominal, se aprecia bazo de gran tamao


con lesiones focales de densidad lquido (arriba). Macroscopa del bazo con su-
perficie polilobulada y lesiones nodulares de aspecto qustico y grumoso (abajo).

dL, leucocitos 2.100 clulas/l y plaquetas 106.000 clu-


las/ul), hipoproteinemia (6,1 mg/dL), LDH: 384 U/L, fos-
fatasa alcalina: 856 U/L y protena C reactiva: 73 mg/L.
La cifra de linfocitos CD4+ era 156 CD4+ clulas/L y la
CVP 456 copias/ml. Los hemocultivos fueron negativos.
La prueba de Mantoux fue negativa. En la radiografa de
trax se observ un infiltrado reticulonodular bilateral de
predominio apical derecho. La tomografa computadori-
zada (TC) toraco-abdominal mostr mltiples adenopatas
mediastnicas (Figura 1), hiliares, supraclaviculares e infra-
diafragmticas y un bazo de gran tamao, con abundantes
Figura 1. Corte transversal de la TC de trax en el que se aprecian adenopatas lesiones focales, las mayores de tres centmetros de dimetro,
mediastnicas de gran tamao.
de densidad lquido o discretamente superior (Figura 2).

130
CAPTULO 8
INFECCIN VIH
Y ENFERMEDADES ASOCIADAS

Diagnstico diferencial

Al tratarse de un paciente profundamente inmunodepri-


mido, con adenopatas generalizadas y lesiones qusticas
esplnicas, que haba iniciado recientemente TAR, se sos-
pech la existencia de un sndrome inflamatorio de recons-
titucin inmune (SIRI) y se plante un amplio diagnstico
diferencial, que inclua infecciones oportunistas y neopla-
sias como etiologas ms probables.

Evolucin

Se inici tratamiento con antiinflamatorios no esteroideos


y corticoides (prednisona a dosis de 1 mg/kg/da) durante
10 das y se mantuvo el TAR, con lo que remiti par-
cialmente la fiebre. Los corticoides fueron administrados
por la situacin clnica que presentaba pese a estar con-
traindicados por presentar lesiones sugestivas de sarcoma
de Kaposi. Tras los hallazgos de la TC se realiz una la-
parotoma con esplenectoma y biopsias heptica y espl-
nica intraoperatorias. Macroscpicamente, el bazo media
22x16x6 centmetros, pesaba 1.026 g, presentaba una su-
perficie polilobulada de coloracin violcea y a los cortes se-
riados mostraba lesiones nodulares de aspecto qustico con
reas de coloracin blanquecina y apariencia grumosa, de
2 a 4 centmetros (Figura 2). En una muestra del bazo re-
mitida desde quirfano y en otra de esputo recogida cinco Figura 3. Tincin de quistes de Pneumocystis jirovecii con metenamina plata de
Grocott en la muestra histolgica de bazo.
das antes se visualizaron bacilos acido-alcohol resistentes
mediante la tincin de Ziehl-Nielsen. En la muestra de es-
puto, el cultivo de micobacterias en medio de Lowenstein 160/800 mg 3 veces por semana v.o. El paciente presen-
fue positivo y la amplificacin de cidos nucleicos (PCR) t una evolucin favorable con desaparicin de la fiebre y
para Mycobacterium tuberculosis fue negativa. El estudio ana- mejora del estado general.
tomopatolgico de las muestras del bazo y del hgado mos-
traron extensas reas de necrosis con calcificaciones y una Diagnstico final
inflamacin granulomatosa difusa, visualizndose adems
Pneumocystis jirovecii con la tincin de plata metenamina de Pneumocistosis extrapulmonar (hepatoesplnica).
Grocott (Figura 3) y bacilos acido-alcohol resistentes con Infeccin diseminada por Mycobacterium avium complex.
la tincin de Ziehl-Nielsen. Ante la sospecha de una infec- Sindrome de reconstitucin inmune (SIRI).
cin diseminada por una micobacteria no tuberculosa se
decidi tratar como Mycobacterium avium complex (MAC), al Discusin
ser la ms frecuente y la que presenta mayor morbimorta-
lidad en pacientes infectados por el VIH y se administr El SIRI se define como la aparicin o empeoramiento de
rifabutina, 300 mg/24 h v.o., azitromicina, 500 mg/24 h infecciones preexistentes, enfermedades autoinmunes o
v.o., (no se administr claritromicina porque interacciona neoplasias, a consecuencia de la restauracin de la respues-
con efavirenz) y etambutol, 1200 mg/24 h v.o. Das des- ta inmunolgica antgeno-especfica inducida tras iniciar el
pus se identific mediante hibridacin de cidos nucleicos TAR en algunos pacientes infectados por el VIH, pese a la
MAC en las muestras de esputo y bazo. Simultneamente respuesta eficaz al mismo (descenso de la CVP y aumento
se inici tratamiento de la infeccin hepatoesplnica por P. del nmero de linfocitos CD4+)1,2. La incidencia del SIRI
jirovecii con sulfametoxazol-trimetoprim, 800/160 mg/12 es difcil de precisar, aunque en la mayora de las series pu-
horas i.v. durante 21 das, seguido de forma profilctica blicadas vara entre el 22,7% y el 31,7%. Se han definido

131
CAPTULO 8
INFECCIN VIH
Y ENFERMEDADES ASOCIADAS

dos tipos de SIRI, el paradjico (empeoramiento o recada tes con infeccin por VIH profundamente inmunodepri-
de una infeccin oportunista u otra etiologa, tratada o en midos, pues su reconocimiento precoz es prioritario para
tratamiento) y el desenmascarado (presentacin acelerada iniciar una terapia potencialmente curativa.
de una infeccin oportunista u otra etiologa que se encon-
traba latente o subclnica). Las infecciones ms frecuentes Bibliografa
son las producidas por M. tuberculosis, MAC y C. neoformans,
aunque tambin se ha relacionado con P. jirovecii1,3. Wislez 1. Shelburne S, Visnegarwala F, Darcourt J, Gravis E,
et al. y Dean et al., han publicado los trabajos ms impor- Thomas P, White A, et al. Incidence and risk factors
tantes de SIRI asociado a P. jiroveci y ambos autores des- for immune reconstitution inflammatory syndro-
cribieron respuestas paradjicas que se presentaron como me during highly active antiretroviral therapy. AID
una neumona a las 2 semanas de iniciado el TAR en pa- 2005;9:399-406.
cientes que haban mejorado con tratamiento frente al P. 2. Shelburne SA, Hamill RJ. The immune reconstitution
jirovecii y suspendieron los corticoides4,5. inflammatory syndrome. AIDS Rev 2003;5: 6779.
P. jirovecii se considera un patgeno del tracto respiratorio 3. Lipman M, Breen R. Immune reconstitution inflam-
y se asocia comnmente a neumona potencialmente mor- matory syndrome in HIV. Current Opinion Infect Dis
tal en pacientes con infeccin por el VIH profundamente 2006;19:205.
inmunodeprimidos. Sin embargo, aunque continua siendo 4. Wislez M, Bergot E, Antoine M, Parrot A, Carette MF,
poco frecuente, se han comunicado cada vez con ms fre- Mayaud C, et al. Acute respiratory failure following
cuencia en los ltimos aos casos de infeccin extrapulmo- HAART introduction in patients treated for Pneumo-
nar por P. jirovecii en pacientes inmunodeprimidos que ha- cystis carinii pneumonia. Am J Respir Crit Care Med
ban recibido previamente profilaxis con pentamidina en 2001;164:84751.
aerosol debido a la mala distribucin sistmica del frma- 5. Dean GL, Williams DI, Churchill DR, Fisher MJ.
co6,7. La patognesis de la infeccin extrapulmonar no est Transient clinical deterioration in HIV patients with
clara y probablemente se deba a una diseminacin hema- Pneumocystis carinii pneumonia after starting highly
tgena o linftica desde el pulmn7. P. jirovecii puede inva- active antiretroviral therapy: Another case of immune
dir numerosos rganos, siendo los ms frecuentes: hgado, restoration inflammatory syndrome. Am J Respir Crit
bazo y mdula sea8. En estos casos, los hallazgos anato- Care Med 2002;165:1670.
mopatolgicos demuestran granulomas necrotizantes con 6. Galimberti A, Morandi E, Giani G, Vitri P, Piraneo S,
calcificaciones centrales. Un alto ndice de sospecha en De Pasquale L, Bastagli A. Pneumocystosis of splenic
estos pacientes es fundamental para el reconocimiento y localization in the course of HIV infection: a new indi-
tratamiento precoz de la infeccin9. cation for splenectomy. Description of a case. Ann Ital
Nuestro paciente present un SIRI, al existir una relacin Chir. 1997;68: 559-62; discussion 562-3.
temporal con el inicio del TAR, el descenso brusco de la 7. Valds E, Borzoni F, Onnis D, Piras S, Vespa A, Varsi
CVP y el aumento rpido de la cifra de linfocitos CD4+ C. Disseminated Pneumocystis carinii infection: clini-
(se aplicaron los criterios de French). Se ha de aclarar que co-pathologic findings in an AIDS patient. Pathologica
el SIRI pudo ser debido a cualquiera de las dos infecciones 1994;86:659-64.
oportunistas que presentaba (P. jirovecii / MAC) y ambas 8. Anuradha, Sinha A. Extrapulmonary Pneumocystis
probablemente jugaron un papel primordial en el desa- carinii infection in an AIDS patient: a case report.
rrollo del mismo. Lo excepcional del caso presentado es Acta Cytol 2007;51:599-601.
que se trata de una nueva forma, bien documentada, de 9. O'Neal CB, Ball SC. Splenic pneumocystosis: an
SIRI asociado a infeccin por P. jirovecii de presentacin atypical presentation of extrapulmonary Pneumocys-
atpica (con afectacin hepatoesplnica), en un paciente tis infection. AIDS Read. 2008; 18:503-8.
que no haba recibido previamente profilaxis con pentami-
dina inhalada. En la literatura mdica, la totalidad de ca-
sos de pneumocistosis extrapulmonar que se han descrito
no tienen relacin con el SIRI y ocurren en pacientes que
recibieron previamente profilaxis con pentamidina inha-
lada. Adems, nuestro caso pone de relieve la necesidad
de incluir la infeccin extrapulmonar por P. jirovecii en el
diagnstico diferencial de las lesiones esplnicas en pacien-

132
Lesiones ocupantes de espacio cerebrales
en mujer con infeccin por el virus
de la inmunodeficiencia humana
Pacheco Yepes, R; Ropero Luis, G;
Valiente De Santis, L.
Hospital Regional Universitario de Mlaga.

Caso clnico tamol, prednisona, metadona, y cotrimoxazol como profi-


laxis en das alternos.
Se trata de una mujer de 45 aos con problemtica social,
indigente. Sin alergias medicamentosas conocidas. Fuma- Enfermedad actual
dora importante (tabaquismo acumulado 40 paquetes-ao),
con hbito enlico considerable y exadicta a drogas por va La paciente acude a Urgencias en mayo de 2008 por cua-
inhalatoria. Como antecedentes personales destacan: in- dro de alteracin del comportamiento y del lenguaje aso-
feccin por el virus de la inmunodeficiencia humana (VIH) ciado a prdida de fuerza en hemicuerpo derecho desde el
diagnosticada en 1981 con 18 aos de edad, hepatopata da anterior. Refiere sensacin distrmica no termometra-
crnica por virus de la hepatitis C (VHC), infeccin por vi- da en los ltimos das. Niega cefalea, nauseas o vmitos, y
rus de la hepatitis B (VHB) pasada, asma bronquial grave, no presenta otra sintomatologa asociada en la anamnesis
colelitiasis y coledocolitiasis con pancreatitis secundaria, por rganos y aparatos. En la exploracin fsica se encuen-
tuberculosis pulmonar en la infancia, y fue intervenida de tra con tendencia a la hipertensin (155/88 mmHg), taqui-
un fibroma uterino. crdica a 123 latidos por minuto, febricular con 37,8C, y
eupneica con una saturacin arterial de oxgeno del 96%
Historia previa con gafas nasales a 2 litros por minuto. Se encuentra cons-
ciente, desorientada en tiempo, espacio y persona, escasa-
Desde el diagnstico de infeccin por VIH hasta 2005 la mente colaboradora. Regular estado general. Bien hidra-
paciente no realiza seguimiento regular en consulta, no re- tada y perfundida, normocoloreada. Hbito cushingoide.
cibiendo tratamiento antirretroviral (TARGA). En el ao No presenta alteraciones en cabeza y cuello. Los tonos
2005 (tres aos antes del episodio referido) se encontra- cardacos son rtmicos y regulares sin soplos, presentando
ba en estadio B3 con criterios definitorios de SIDA (her- hipoventilacin generalizada y sibilantes espiratorios dis-
pes zster, candidiasis orofarngea) y recuento de CD4 de persos bilaterales a la auscultacin pulmonar. El abdomen
79 clulas/l con carga viral de VIH 77.448 copias/ml, es blando y depresible, sin dolor a la palpacin, masas ni
decidindose iniciar TARGA con Emtricitabina (FTC) + organomegalias. En las extremidades inferiores los pulsos
Tenofovir (TDF) + Lopinavir/Ritonavir (LPV/r). A los pedios estn presentes y no existen edemas. En cuanto a
seis meses se modific a FTC + TDF + Saquinavir/Rito- la exploracin neurolgica, se encuentra desorientada en
navir (SQV/r) por fracaso teraputico secundario a mala las tres esferas, con pupilas midriticas bilaterales reacti-
adherencia y discontinuidad del seguimiento. Solo en una vas a la luz, afasia motora, movimientos oculares exter-
ocasin entre 2005 y 2008 present carga viral inferior a nos conservados, parlisis facial derecha central sin otra
50 copias/ml, con recuento de CD4 siempre por debajo focalidad de pares craneales, hemiparesia derecha 4/5, no
de 200 clulas/mm3. En este intervalo la paciente requiri colaboracin en la prueba dedo-nariz, reflejos osteotendi-
mltiples ingresos hospitalarios en el Servicio de Enferme- nosos no exaltados, reflejo cutneo plantar flexor bilateral,
dades Infecciosas por reagudizacin asmtica con hiperre- Romberg negativo y marcha inexplorable.
actividad bronquial en el contexto de infeccin respiratoria
no condensante, recibiendo tratamiento con antibiotera- Pruebas complementarias
pia, corticoterapia intravenosa y posteriormente oral en
pauta descendente de forma permanente. A pesar de esto En la analtica extrada en el rea de Urgencias no presen-
la paciente segua fumando de forma activa y persista en ta alteraciones significativas en el hemograma ni en la bio-
el consumo de alcohol y ocasionalmente de otros txicos qumica, salvo una hiponatremia leve (sodio 131 mmol/l)
va inhalatoria. En el momento del episodio actual la me- y niveles de etanol de 10 mg/dl. La gasometra venosa y
dicacin habitual de la paciente inclua TDF + FTC + la coagulacin son normales. El anlisis rpido de la orina
SQV/r, inhaladores de budesonida + formoterol y salbu- da resultado positivo para cocana y opiceos, presentan-

133
CAPTULO 8
INFECCIN VIH
Y ENFERMEDADES ASOCIADAS

do adems en la tira visual positividad para hemoglobi-


na (5+), urobilingeno (3+) y leucocitos (2+). Se extraen
hemocultivos que posteriormente resultaron negativos. El
electrocardiograma presenta ritmo sinusal con eje a 100
sin bloqueos ni alteraciones de la repolarizacin. En la ra-
diografa de trax slo se aprecia un pequeo infiltrado
residual a nivel del lbulo superior izquierdo. Se solicita
una tomografa computarizada (TC) de crneo (Imagen 1),
que evidencia una lesin frontal izquierda de morfologa
nodular, mal definida, de unos 2,5 cm de dimetro y 4 cm
en el siguiente corte, sin poder distinguir si se trata de una
nica lesin o dos adyacentes, con una zona hiperdensa
perifrica y central ms hipodensa, y que provoca un efec-
to masa con desplazamiento de la lnea media, as como
edema perilesional que produce borramiento de surcos
homolateral.
Con los datos obtenidos de la exploracin y de las pruebas
complementarias se ingresa en planta de hospitalizacin Imagen 1. Tomografa computarizada de crneo sin contraste realizada en
Urgencias. Se observa una lesin frontal izquierda de morfologa nodular mal
de Enfermedades Infecciosas con el diagnstico sindr- definida con efecto masa y edema perilesional.
mico de Lesin ocupante de espacio (LOE) mltiple con
efecto masa en paciente VIH severamente inmunodepri- realce en anillo tras la administracin de contraste intra-
mida, inicindose tratamiento emprico. venoso, siendo muy especfica (pero poco sensible) la pre-
sencia de un realce nodular excntrico en dicha pared re-
Diagnstico diferencial alzante. En la RM las secuencias T2 y FLAIR muestran el
signo de la diana en las lesiones, con un centro hiperintenso
En nuestro caso nos encontramos ante una paciente con que se correlaciona con necrosis, un anillo hipointenso que
varias LOEs cerebrales, en un estado de importante in- se correlaciona con la zona inflamatoria, y una periferia hi-
munodepresin por la infeccin por VIH en estadio B3 perintensa en relacin a edema circundante. Las secuencias
con fracaso virolgico, junto a terapia corticoidea de uso T1 con gadolinio muestran un anillo realzante de la zona
prolongado y abuso de drogas. Tras las pruebas de ima- inflamatoria. Las lesiones suelen presentar una difusin au-
gen que evidencian lesiones que producen efecto masa, mentada en las secuencias de difusin, en la espectroscopia
quedan relegadas a un segundo plano alteraciones difusas suele existir un pico de lpidos y lactato elevado. El volumen
del sistema nervioso central (SNC) como seran la ence- sanguneo relativo a la lesin est disminuido. Comnmente
falopata por VIH (complejo demencia-SIDA, CDS) o la se asocian edema y hemorragia a estas lesiones.
leucoencefalopata multifocal progresiva (LMP), debiendo El diagnstico diferencial principal con lesiones mltiples
pensar en primer lugar en toxoplasmosis o linfoma prima- con realce en anillo es con el linfoma primario del SNC.
rio de SNC en su presentacin focal, considerando adems A favor del linfoma se encuentra la extensin subependi-
la posibilidad de tuberculoma u otros abscesos cerebrales maria, la restriccin en difusin (hipercelularidad), la afec-
infecciosos. tacin del cuerpo calloso y el marcado pico de colina. El
En un paciente VIH inmunodeprimido el diagnstico de volumen sanguneo relativo en la lesin linfomatosa est
toxoplasmosis es difcil, ya que frecuentemente no se ob- aumentado.
serva aumento de la inmunoglobulina M (IgM) ni grandes Adems de estas entidades ms frecuentes, en el paciente
variaciones en los ttulos de la inmunoglobulina G (IgG), VIH no puede dejarse atrs en el diagnstico diferencial la
por lo que no es til para el diagnstico, as que se estable- posibilidad de que las lesiones con realce puedan corres-
ce ms bien una sospecha que un diagnstico de certeza. ponder a abscesos cerebrales de etiologa bacteriana.
Por otro lado las caractersticas de las pruebas de imagen
son una excelente herramienta para encauzar el diagns- Evolucin
tico. La toxoplasmosis en la TC se suele manifestar como
lesiones mltiples, hipodensas, localizadas en los ganglios Tras el ingreso en planta, y ante las caractersticas de las
basales, tlamo y regin corticosubcortical, con el tpico lesiones, se inicia tratamiento emprico de forma obligada

134
CAPTULO 8
INFECCIN VIH
Y ENFERMEDADES ASOCIADAS

para toxoplasmosis cerebral con pirimetamina + sulfadia-


zina (posteriormente la serologa para toxoplasmosis fue
negativa para IgM y positiva para IgG), adems de cubrir
empricamente con antibioterapia de amplio espectro con
meropenem + metronidazol. Se solicita una resonancia
magntica (RM) cerebral con contraste (Imgenes 2 y 3).
Segn el informe de Radiologa se observan varias lesio-
nes ocupantes de espacio polilobuladas con componente
qustico en lbulo frontal izquierdo con importante efecto
de masa y edema perilesional, que ocasionan discreta her-
niacin falciana en la mitad anterior de la hoz cerebral con
desplazamiento de la arteria cerebral anterior izquierda y
deformacin ventricular con obliteracin del asta frontal
del ventrculo izquierdo. Las lesiones son de seal baja en
secuencia T1 y FLAIR y alta en T2, con restriccin de
difusin y marcada captacin de contraste de morfologa
anular. Las de mayor tamao miden 3 y 1,8 cm respec-
tivamente y hay otras menores de naturaleza similar en
contacto con el aspecto lateral y posterior. El resto de pa-
rnquima tiene una seal normal. Las lesiones descritas
plantean como diagnstico posible que se trate de abscesos
(tuberculosos?) y con menor probabilidad, por las caracte-
rsticas de la seal y la localizacin, linfoma o toxoplasma.
Tras dos semanas de tratamiento no hay respuesta clnica
ni radiolgica, persiste febril con cefalea, afasia y hemipa-
resia, adems de sufrir un episodio comicial, por lo que se
decide intervenir quirrgicamente.
Se realiza una craneotoma frontal izquierda encontrando
una duramadre a tensin. Se extirpa la cpsula del absceso
que llega a ventrculo pero no est abierto a l. Se reali-
za puncin y evacuacin del pus (30 ml), y exresis de un
fragmento laminar de 30x30x2 mm unido a un fragmento
irregular de superficie de aspecto cerebroide de 35x20x15
mm, al corte de aspecto qustico y contenido purulento. Se
mandan muestras para anlisis microbiolgico e histopa-
tolgico. Imgenes 2 y 3. Resonancia magntica de crneo. Se observan varias lesiones
ocupantes de espacio (LOEs) polilobuladas con captacin en anillo en lbulo
En la tincin de Gram de la muestra microbiolgica se vi- frontal izquierdo, asociando importante efecto de masa y edema perilesional.
sualizan bacilos gram positivos filamentosos ramificados
compatibles con Nocardia, creciendo dos semanas despus ictericia multifactorial e insuficiencia respiratoria global,
en cultivo estndar colonias de actinomiceto aerobio (se adems importantes secuelas motoras, evolucionando a
enva la cepa al Servicio de Bacteriologa del Centro Na- fallo multiorgnico y falleciendo un mes despus de la in-
cional de Microbiologa con resultado de Nocardia farcinica). tervencin.
El cultivo de hongos y reaccin en cadena de la polimerasa
(PCR) para genoma de Mycobacterium tuberculosis son nega- Diagnstico final
tivos. El informe anatomopatolgico describe un absceso
cerebral con proliferacin fibrovascular perifrica. Abscesos cerebrales por Nocardia farcinica en paciente gra-
Pese a aadir tratamiento antibitico especfico adecuado vemente inmunodeprimida con infeccin por VIH, trata-
(amikacina 500 mg cada 12 horas para sinergismo con car- miento corticoideo prolongado y abuso de drogas.
bapenem ya pautado) la paciente presenta evolucin trpi-
da, desarrollando hidrocefalia postquirrgica (Imagen 4),

135
CAPTULO 8
INFECCIN VIH
Y ENFERMEDADES ASOCIADAS

Discusin

La nocardiosis es una rara infeccin oportunista que afecta


principalmente a pacientes inmunodeprimidos, con gran
predileccin por la infeccin del SNC. Nocardia spp es un
bacilo ramificado aerbico gram positivo perteneciente a
la familia Actinomyces, catalasa positivo y cido-alcohol re-
sistente1. El principal factor de riesgo para desarrollar esta
infeccin es la inmunodepresin celular2 que presenta por
ejemplo un individuo con infeccin por VIH (sobre todo
con recuentos de CD4 menores a 100 clulas/mm3), as
como el tratamiento continuado con corticoides3 y el abu-
so de alcohol4 como sera nuestro caso.
Normalmente la infeccin primaria es adquirida por inha-
lacin (lo que ocurre en ms de dos tercios de los casos), o
va cutnea por inoculacin directa produciendo afeccin
local. Secundariamente la infeccin puede diseminarse va
hematgena, aunque es infrecuente encontrar hemocul- Imagen 4. Tomografa computarizada de crneo realizada tras la ciruga. Se
observan cambios postquirrgicos en regin frontal izquierda con importante
tivos positivos, quizs debido a las condiciones y el largo hidrocefalia y edema cerebral.
perodo necesarios para la incubacin. Se considera enfer-
medad sistmica cuando existen ms de dos lugares afec- co es insuficiente en la mayora de las ocasiones, requirien-
tados, encontrando especial tropismo del microorganismo do de tcnicas invasivas neuroquirrgicas con aspiracin o
por el SNC, siendo la causa del 2% del total de los abscesos extirpacin del absceso cuando el tamao es superior a 25
cerebrales5. La infeccin por N. farcinica es poco frecuente mm o ha fracasado el tratamiento mdico.
dentro de este grupo, y se ha asociado a un mayor riesgo El antibitico de eleccin es cotrimoxazol en dosis de
de diseminacin y de resistencia a antibiticos, y por tanto 15+75 mg por kg de peso cada 12 horas durante 6 meses
a mayor mortalidad; la afectacin cerebral aparece hasta hasta un ao, pudiendo asociar amikacina, imipenem o ce-
en el 30% de los casos6. falosporinas de tercera generacin mientras se esperan los
El diagnstico de esta entidad es difcil, debido a lo ines- resultados del antibiograma. Tambin se ha probado con
pecfico de la clnica con expresin paucisintomtica en xito el moxifloxacino en algunos casos de infeccin por N.
mltitud de ocasiones, adems de la escasa repercusin en farcinica9,10. Dentro de la peculiaridad de nuestra pacien-
los reactantes de fase aguda, la inexistencia de serologa te se encuentra el hecho de que estaba realizando trata-
especfica y el amplio abanico de diagnsticos diferenciales miento profilctico con cotrimoxazol, lo que tericamente
que se abre ante el hallazgo de LOEs nicas o mltiples7. protegera del desarrollo de esta infeccin, desconociendo
Todo ello, unido a la dificultad para el aislamiento e iden- realmente si la adherencia teraputica al tratamiento fue la
tificacin del microorganismo mediante cultivo (muestras adecuada o si la profilaxis fue inefectiva.
inadecuadas por mtodos no invasivos, contaminacin,
uso de productos txicos en el manejo de las muestras,
crecimiento lento) desembocan en un retraso diagnstico
importante en la mayora de las ocasiones, con una media
de 42 das hasta un ao desde la aparicin de la sintoma-
tologa inicial hasta la confirmacin del diagnstico. Ac-
tualmente disponemos de tcnicas de biologa molecular
como la PCR para su identificacin8, pero son recursos no
disponibles en todos los centros.
Las tasas de mortalidad en la nocardiosis cerebral son al-
tas (entre el 20-55%5, aunque ahora podran ser menores),
ms an si nos encontramos ante abscesos mltiples. El
abordaje teraputico no est bien establecido, aunque pa-
rece que el manejo conservador con tratamiento antibiti-

136
CAPTULO 8
INFECCIN VIH
Y ENFERMEDADES ASOCIADAS

Bibliografa

1. Corti ME, Villafane-Fioti MF. Nocardiosis: a review.


Int J Infect Dis 2003;7:243-50.
2. Kilincer C, Hamamcioglu MK, Simsek O, Hicdon-
mez T, Aydoslu B, Tansel O, et al. Nocardial brain abs-
cess: review of clinical management. J Clin Neurosci
2006;13:481-5.
3. Nocardiosis. Clin Infect Dis. 1996;22:891,903; quiz
904-5.
4. Lee GY, Daniel RT, Brophy BP, Reilly PL. Surgical
treatment of nocardial brain abscesses. Neurosurgery
2002;51: 668-71; discussion 671-2.
5. Mamelak AN, Obana WG, Flaherty JF, Rosen-
blum ML. Nocardial brain abscess: treatment strate-
gies and factors influencing outcome. Neurosurgery
1994;35:622-31.
6. Kumar VA, Augustine D, Panikar D, Nandakumar A,
Dinesh KR, Karim S, et al. Nocardia farcinica brain
abscess: epidemiology, pathophysiology, and literature
review. Surg Infect (Larchmt) 2014;15: 640-6.
7. Goodall L, Faggian F. Nocardia farcinica brain abs-
cesses in an HIV-positive patient. Int J STD AIDS
2012;23: 451-2.
8. Tatti KM, Shieh WJ, Phillips S, Augenbraun M, Rao
C, Zaki SR. Molecular diagnosis of Nocardia farcinica
from a cerebral abscess. Hum Pathol 2006;37: 1117-
21.
9. Fihman V, Bercot B, Mateo J, Losser MR, Raskine
L, Riahi J, et al. First successful treatment of Nocar-
dia farcinica brain abscess with moxifloxacin. J Infect
2006;52: e99-102.
10. Kandasamy J, Iqbal HJ, Cooke RP, Eldridge PR. Pri-
mary Nocardia farcinica brain abscess with secondary
meningitis and ventriculitis in an immunocompetent
patient, successfully treated with moxifloxacin. Acta
Neurochir (Wien) 2008;150: 505-6.

137
Paciente joven con exantema
y prdida de visin unilateral
Ruiz-Ruigmez, M; Fatoul del Pino, G;
Aomar Milln, I.
Hospital Universitario San Cecilio. Granada.

Caso clnico debe realizarse el diagnstico diferencial con los siguientes


grupos de patologas:
Se trata de un varn de 52 aos de edad con antecedentes
personales de infeccin VIH diagnosticado en 2004 cuan- Las causas infecciosas son la causa de retinitis ms fre-
do present un cuadro de retinitis por CMV. El paciente cuente en nuestro medio, siendo entidades a tener en cuen-
tena VIH en estadio C3 de la CDC, en tratamiento con ta dentro de este grupo las siguientes:
tenofovir-emtricitavina y efavirenz con buen cumplimien-
to y buen control viroinmunolgico. Como complicacio- Toxoplasmosis: suele tratarse bien de una infeccin
nes destaca un cuadro de meningitis tuberculosa tres aos congnita o bien de una reactivacin de una infeccin
antes a la consulta actual, sin destacar en la historia otros previa. Suele ser una de las causas ms frecuentes en
antecedentes de inters. Acude a nuestra consulta presen- pacientes inmunocompetentes. Las lesiones caracters-
tando un cuadro de un mes de evolucin consistente con ticas son lesiones pigmentadas y cicatrices coriorreti-
tos y ronquera, acompaado de sudoracin y exantema nianas con inflamacin en cmara anterior y vitritis,
difuso generalizado con afectacin palmo-plantar no pru- datos no compatibles con las lesiones que presentaba
riginosa, en las ltimas 48 horas refiere adems una pr- el paciente.
dida de la agudeza visual en el ojo derecho, motivo por el Tuberculosis: aunque es una causa poco frecuente en
que consulta. nuestro medio y suele ser de curso crnico y con afec-
La exploracin fsica general del paciente fue anodina, tacin tanto del segmento anterior como posterior es
destacando nicamente la presencia de un exantema en una entidad que no puede descartarse inicialmente.
tronco, abdomen y extremidades con afectacin de palmas Las manifestaciones oculares ms frecuentes incluyen
y plantas. En lo referente a las pruebas complementarias se iritis granulomatosa o no granulomatosa, y tubrculos
realizaron hemograma y bioqumica que fueron normales coroideos. El diagnstico se establece por lesiones tpi-
as como control viroinmunolgico con 566 CD4/l y car- cas oculares, as como investigacin de micobacterias,
ga viral del VIH indetectable. La valoracin oftalmolgica siendo necesaria en muchas ocasiones la realizacin de
mostr en ojo izquierdo lesiones antiguas de coriorretinitis biopsia ocular.
por fibrosis macular y atrofia coriorretiniana y en ojo de- Los virus herpes pueden presentar una forma aguda
recho, la presencia de una lesin blanquecina extensa y que afecta principalmente a pacientes inmunocompe-
profunda extendindose desde papila radialmente, siendo tentes, produciendo una necrosis retiniana y una for-
ms intensa en arcada temporal inferior, compatible con ma crnica que suele afectar a pacientes con menos de
retinitis. 50 CD4 con tpica afectacin retiniana y ausencia de
signos inflamatorios.
Diagnstico diferencial Sfilis es la causa bacteriana ms frecuente de infeccin
intraocular en paciente VIH independientemente de
Ante este cuadro se realiz un diagnstico diferencial con su estado viroinmunolgico. Debe sospecharse ante
las distintas causas de retinitis as como de enfermedades cualquier forma de uvetis, dando lugar a lesiones muy
exantemticas con afectacin ocular. El hecho de encon- diversas en el curso de una retinitis, pudiendo stas
trarnos ante un paciente VIH positivo nos orienta inicial- afectar a cualquier estructura ocular, siendo en mu-
mente hacia una posible etiologa infecciosa, sin embargo chas ocasiones indistinguibles con las producidas por
en este caso debe realizarse el diagnstico diferencial como herpes-virus. En nuestro caso, la aparicin de un exan-
si se tratase de un paciente inmunocompetente ya que en tema con afectacin palmo-plantar acompaando a la
el momento de la visita llevaba varios aos con carga viral afectacin ocular hace que debamos prestar especial
indetectable y cifra de linfocitos CD4 normales. inters a esta patologa.
Teniendo en cuenta la localizacin de las lesiones oculares Cndida spp generalmente produce lesiones mltiples

138
CAPTULO 8
INFECCIN VIH
Y ENFERMEDADES ASOCIADAS

retinianas focales bilaterales, blancas, infiltrativas y de


aspecto algodonoso que suelen aparecer en polo poste-
rior o alrededor de la papila, siendo ms frecuente en
consumidores de herona marrn.

Otras entidades infecciosas que pueden producir retinitis


son retinopata por VIH que se produce por depsito de
inmunocomplejos y accin directa sobre el endotelio, la
Bartonella henselae puede presentarse con diversas manifes-
taciones oftalmolgicas. Otras infecciones fngicas como
Cryptococcus spp, Pneumocystis jirovecii o Histoplasmosis o pa-
rasitarias como oncocercosis, leptospirosis o filariasis tam-
bin deben tenerse en cuenta sobre todo si hay anteceden-
tes epidemiolgicos.

El segundo grupo de enfermedades que debemos tener en


cuenta son los sndromes de enmascaramiento, de es-
pecial inters por la posibilidad de asociarse a tumores ma- Imagen 1. afectacin retiniana por lus.
lignos, siendo el linfoma de clulas B la neoplasia que ms
frecuentemente puede presentarse como uvetis, y debe El ltimo grupo a destacar es el de la uvetis secunda-
sospecharse ante uvetis crnica con inflamacin posterior rias, representado por procesos primariamente oculares
bilateral y mala respuesta al tratamiento esteroideo. Otros tratndose de cuadros lo suficientemente caractersticos
tumores malignos asociados a uvetis son el linfoma intrao- como para que la sola exploracin e historia oftalmolgi-
cular, el melanoma, las leucemias, el retinoblastoma y me- cas permitan acceder al diagnstico.
tstasis. Existen tambin sndromes de enmascaramiento
asociados a cuerpo extrao, desprendimiento de retina y Diagnstico final
distrofia retiniana entre otros.
Otro grupo importante dentro del diagnstico diferencial a Dados los antecedentes del paciente y ante la alta sospecha
incluir son las enfermedades sistmicas. La aparicin diagnstica se solicit VDRL con resultado 1/160. Revisa-
de un proceso uvetico en el contexto de una enfermedad mos los controles previos del paciente que eran negativos.
sistmica que lo justifique debe ser asumido en el contex- Por lo que estos datos junto a la valoracin oftalmolgica
to clnico de dicha enfermedad aunque debe excluirse permiti el diagnstico de retinitis lutica.
siempre la sfilis y tuberculosis. Dentro de este grupo los
procesos que no podemos olvidar son las patologas reu- Evolucin
mticas asociadas al complejo de histocompatibilidad B 27
(espondilitis anquilosante, artritis reactiva, artritis crnica Muy favorable con resolucin completa de las lesiones of-
juvenil y la artritis psorisica), la artritis reumatoide y la tlmicas y recuperacin completa de la agudeza visual tras
enfermedad de Still. Tambin hay que destacar la enfer- tratamiento con penicilina y corticoides.
medad de Behet, que presenta afectacin ocular entre el
25 y el 75% de los casos, siendo fundamentalmente uvetis Discusin
anterior, que aunque suele ser unilateral, acaba tpicamen-
te siendo bilateral y episdica, pudiendo cursar tambin La sfilis ocular es una entidad infrecuente que ha sido des-
como uvetis posterior, coriorretinitis o panuvetis. En la crita como manifestacin extracutnea en todos los esta-
sarcoidosis la afectacin ocular ocurre en el 20-40% de los dios de la sfilis, pudiendo afectar a cualquier estructura
casos, siendo casi en el 20% la primera manifestacin. Pue- ocular sin que existan signos patognomnicos en el exa-
de afectar la conjuntiva, la espiesclera y rara vez, la rbita men oftalmolgico que permitan un diagnstico clnico.
y la esclera, estando el segmento posterior de la vea afec- Del mismo modo que en las manifestaciones cutneas, a
tado en el 25% de los casos, generalmente asocindose a nivel ocular se presenta como la gran imitadora. En pa-
uvetis anterior por infiltracin granulomatosa de los vasos cientes con VIH la afectacin oftlmica por lues puede
de la coroides y de la retina. presentarse con una mayor agresividad (afectacin bila-

139
CAPTULO 8
INFECCIN VIH
Y ENFERMEDADES ASOCIADAS

teral, retinitis necrotizante) y con formas clnicas atpicas La realizacin de puncin lumbar para establecer el diag-
como la coriorretinitis placoide posterior. nstico ha sido ampliamente discutida sin demostrar ma-
Antes de la introduccin de la terapia antirretroviral de yor sensibilidad que la serologa.
gran actividad (TARGA), la retinitis por CMV era la in- El tratamiento de las lesiones luticas oftlmicas es el indi-
feccin ocular ms frecuente en pacientes con SIDA, afec- cado en casos de neurosfilis, con altas dosis de penicilina.
tando hasta casi un 40% de pacientes con CD4 inferiores En cuanto al pronstico, es favorable cuando se afectan a
a 50/mcl. A partir del ao 2000, el escenario de las uvetis capas de origen mesodrmico (coroides, iris, esclera y cr-
en estos pacientes cambia radicalmente con la aparicin nea) con muy buena respuesta al tratamiento. Sin embar-
de nuevas entidades como la uvetis por recuperacin In- go, los resultados observados tras tratamiento de lesiones
mune (URI) y la creciente importancia de otras causas de en estructuras ectodrmicas (retina, nervio ptico, crista-
uvetis infecciosas como la sfilis y la tuberculosis. lino), son ms desesperanzadoras debido al componente
El recuento de linfocitos CD4 en sangre perifrica es el degenerativo de las mismas.
principal indicador de riesgo de complicacin ocular, sin Debe tenerse en cuenta que el VIH puede modificar tanto
embargo en el caso de la sfilis la afectacin ocular no est la forma de presentacin como la evolucin y respuesta al
relacionada con el grado de inmunosupresin (recuento de tratamiento de la neuroles y de la sfilis ocular, presentan-
linfocitos CD4) sino con la elevada prevalencia de esta pa- do los pacientes coinfectados por VIH y T. pallidium, mayor
tologa. Actualmente la sfilis constituye la primera causa frecuencia de afectacin retininiana y del nervio ptico.
de infeccin bacteriana ocular, por delante del CMV en Este caso destaca la importancia de la sfilis que pese a su
pacientes con TARGA, ya que la inmunorecuperacin no baja frecuencia de compromiso ocular como manifestacin
presenta un efecto protector. extracutnea debe descartarse en pacientes con VIH por
Las manifestaciones oculares de la sfilis son frecuentes y sus formas de presentacin atpica y de curso ms agresivo.
pueden aparecer en cualquier estadio de la enfermedad, Pese a la escasez de casos publicados, quizs por la baja
siendo las manifestaciones oculares mas frecuentes la que- frecuencia o por el infradianstico de esta entidad, es im-
ratitis intersticial, uvetis, coriorretinitis, vasculitis retiniana portante sospechar y descartar una sfilis ante una afecta-
y neuropata ptica. En los pacientes VIH sin afectacin cin ocular en paciente VIH independientemente de su
inmunolgica como en la poblacin general, la forma de control viroinmunolgico para as instaurar tratamiento de
presentacin ms comn es la uvetis anterior, represen- forma precoz y consiguir un mejor pronstico, evitando as
tando el 80-70% de los casos, siendo muy infrecuente la recaidas y secuelas.
afectacin posterior, con menos del 20% de los mismos.
En pacientes inmunocomprometidos la coriorretinitis re-
presenta el 80% de los casos. Las lesiones que se pueden
encontrar en el fondo de ojo pueden ser muy diversas, la
presencia de hemorragias es mnima y generalmente estn
ausentes, lo que permite el diagnstico diferencial con el
CMV que presenta de forma constante hemorragias en
polo posterior.
El diagnstico de sfilis debe confirmarse tras sospecha cl-
nica mediante test treponmicos y no treponmicos, com-
portndose los pacientes con infeccin por VIH de la mis-
ma manera que los inmunocommpetentes; debe tenerse
en cuenta que las pruebas reagnicas pueden positivizarse
tardamente. En nuestro caso este dato es importante ya
que dos meses antes del diagnstico las serologas realiza-
das al paciente fueron negativas por lo que podramos es-
tar frente a un caso de seroconversin tarda. Actualmente
se recomienda la realizacin de pruebas serolgicas para
les en todos los casos de uvetis de origen desconocido,
no respondedoras al tratamiento, cuando exista retinitis o
vasculitis de retina, as como los casos en los que la uvetis
se acompae de exantema cutneo y/o cefalea.

140
CAPTULO 8
INFECCIN VIH
Y ENFERMEDADES ASOCIADAS

Bibliografa

1. Prez de Arcelus M, Salinas A, Garca Layana A. Ma-


nifestaciones retinianas de las enfermedades infeccio-
sas. An. Sist. Sanit. Navar. 2008 Vol. 31, Suplemento
3.
2. Hughes EH1, Guzowski M, Simunovic MP, Hunyor
AP, McCluskey P. Syphilitic retinitis and uveitis in
HIV-positive adults. Clinical and Experimental Oph-
thalmology 2010; 38: 8516 doi: 10.1111/j.1442-
9071.2010.02383.x.
3. Prez-Blzquez E, Redondo MI, Gracia T. Sida y of-
talmologa: una visin actual. An. Sist. Sanit. Navar.
2008 Vol. 31, Suplemento 3
4. Doris JP, Saha K, Jones NP, Sukthankar A. Ocular
syphilis: the new epidemic. Eye 2006; 20: 7035.
5. Villanueva AV, Sahouri MJ, Ormerod LD, Puklin JE,
Reyes MP. Posterior uveitis in patients with positive se-
rology for syphilis. Clin Infect Dis 2000;30:479-85.
6. Tran TH, Cassoux N, Bodaghi B, Fardeau C, Cau-
mes E, Lehoang P. Syphilitic uveitis in patients infected
with human immunodeficiency virus. Graefes Arch
Clin Exp Ophthalmol 2005;243:863-9. Epub 2005
Mar 15
7. Kiss S, Damico FM, Young LH. Ocular manifesta-
tions and treatment of syphilis. Semin Ophthalmol.
2005;20:161-7.
8. Hernndez-Bel P, Gmez-Maestra MJ, Torrijos-Agui-
lar A, Lpez J, Vilata JJ, Alegre V. Sfilis Ocular: Una
Presentacin Inusual de Sfilis Secundaria en un Pa-
ciente Inmunocompetente. Actas Dermosifiliogr 2010;
101 :184-6.
9. Foti C, Carnimeo L, Delvecchio S, Guerriero S, Cas-
sano N, Vena GA. Secondary syphilis with progressive
ocular involvement in an immunocompetent patient.
Eur J Dermatol 2009;19:288.

141
VIH, tuberculosis o toxicidad farmacolgica?
Garca Perea, L; Lluna Carrascosa, A;
Fuertes Rodrguez, A; Glvez Lpez, R.
Hospital Universitario San Cecilio. Granada.
Hospital El Carmelo de Chokw. Mozambique.

Caso clnico tratamiento TAT, con el siguiente rgimen: kanamicina,


pirazinamida, etionamida, etambutol y cicloserina y se
Mujer de 30 aos, natural de Mozambique, que estando suspende la quinolona. Adems, se inicia TAR con Teno-
trabajando en la Repblica de Sudfrica (RSA), comienza fovir (TDF), Lamivudina (3TC) y Efavirenz (EFV), junto
con un sndrome constitucional, por el que consulta en los con profilaxis con cotrimoxazol (CTZ).
servicios sanitarios de ese pas en febrero de 2014. Segn A los pocos das, la paciente presenta una disminucin del
el informe que aporta, la paciente presentaba un aceptable nivel de consciencia, con un Glasgow 10/15 y alteracio-
estado general, con una exploracin fsica sin alteraciones, nes del comportamiento, sin otra clnica acompaante ni
peso: 70 kg, talla 175 cm, IMC de 22,86. Se realizaron las alteraciones en la exploracin fsica. Se realiz una pun-
siguientes pruebas: cin lumbar que permiti estudiar las caractersticas del l-
quido cefalorraqudeo (LCR): presin de salida levemente
Test serolgico rpido de VIH: positivo. aumentada, aspecto trasparente como agua de roca, celu-
Baciloscopia de esputo: negativa. laridad normal, elevacin de protenas, concentracin de
PCR de M. tuberculosis: positiva. Se detecta resistencia glucosa baja y baciloscopia, tinta china, tincin de GRAM
a rifampicina. y RPR negativas.
Radiografa de trax: sin alteraciones.
Recuento de linfocitos T CD4, 215 cel/L (8%). Diagnstico diferencial

Es diagnosticada de infeccin VIH no conocida, estadio En primer lugar, la paciente ingresa asintomtica, salvo
3 de la OMS y de tuberculosis pulmonar multirresitente, por una sensacin de palpitaciones que pone de manifiesto
por lo que se inicia tratamiento antituberculoso (TAT) el un alargamiento del intervalo QT en el ECG. El sndrome
4/02/2014 con el siguiente rgimen: Moxifloxacino 400 de QT largo (SQTL), se define como un trastorno de la
mg/da, Isoniazida 300 mg/da, Pirazinamida 2400 mg/ repolarizacin del miocardio caracterizada por un inter-
da, Terizidona 1 g/da oral y Kanamicina 1 g/da i.m. valo QT prolongado en el ECG. Se asocia con un mayor
El da 06/03/2015, inicia tratamiento antirretroviral riesgo de presentar una taquicardia ventricular polimorfa
(TAR) con rgimen Abacavir (ABC), Lamivudina (3TC) del tipo de Torsades de Pointes (TdP), que generalmente
y Efavirenz (EFV), que es suspendido posteriormente por es de corta duracin y autolimitada. Los principales snto-
aparicin de lesiones cutneas, que no se especifican en mas que refieren los pacientes son: palpitaciones, sncope,
el informe que adjunta. La paciente solicita ser traslada- convulsiones y en algunas ocasiones, muerte sbita. En los
da a su pas de origen, Mozambique, continuando con el adultos, las mujeres tienen intervalos QT ligeramente ms
TAT pautado y sin TAR. Llega a nuestro hospital en abril largos que los hombres. Un intervalo QT corregido en las
del 2014, con el informe de trasferencia del hospital de la mujeres se considera normal hasta 0,47 segundos.
RSA, con los datos hasta ahora mencionados. El SQTL puede ser congnito o adquirido, estando rela-
En la consulta de valoracin de nuestro centro, la pacien- cionado con alteraciones electrolticas (hipopotasemia o
te refiere palpitaciones y dolor torcico. En la exploracin hipomagnesemia) y con el consumo de mltiples medica-
fsica, se encuentra con un buen estado general, estable he- mentos. Dentro de los grupos farmacolgicos que pueden
modinmicamente y nicamente llama la atencin en la producirlo se encuentran: frmacos antiarrtmicos (la qui-
auscultacin cardaca la presencia de unos tonos rtmicos, nidina es la causa ms frecuentemente identificada de TdP
con episodios de pausa frecuente. Ante esta alteracin, se inducida por frmacos), antihistamnicos no sedantes, los
decide realizar un electrocardiograma (ECG) en el que se macrlidos, y algunos procinticos y psicotropos como el
evidencia una prolongacin del intervalo QT. Se ingresa a haloperidol, siendo recomendable utilizarlos con precau-
la paciente para: ajustar el TAT, reiniciar el TAR y com- cin en pacientes con las alteraciones electrolticas mencio-
pletar el estudio de las alteraciones del ECG. Se ajusta el nadas. La mayora de los pacientes que presentan una TdP

142
CAPTULO 8
INFECCIN VIH
Y ENFERMEDADES ASOCIADAS

del sistema nerviosos central (SNC), dficits nutricionales


(vitamina B12), trastornos endocrinos como la insuficien-
cia suprarrenal o alteraciones tiroideas, sndromes de de-
mencia, como la enfermedad de Alzheimer, la demencia
tipo vascular o la demencia asociada al VIH, neoplasias
del SNC o toxicidad farmacolgica. En el caso de nues-
tra paciente, dada la instauracin aguda del cuadro, nos
pareci razonable investigar en primer lugar las causas in-
fecciosas y los posibles efectos secundarios de los frmacos
administrados.
En los pacientes con infeccin VIH, la etiologa de las
infecciones del sistema SNC est muy relacionada con
el grado de inmunosupresin, y en muchas ocasiones se
pueden identificar con el anlisis del lquido cefalorraqu-
deo (LCR) y las pruebas de neuroimagen. En cuanto a los
agentes infecciosos, nos centramos en los microorganismos
ECG que muestra prolongacin del intervalo QT. oportunistas que con ms frecuencia producen afectacin
neurolgica en pacientes con infeccin VIH:
inducida por frmacos tienen uno o ms factores de riesgo
predisponentes: dosis y va de administracin del medica- Toxoplasmosis: representa una reactivacin de la in-
mento, consumo de varios frmacos productores de SQTL feccin previa y principalmente la padecen pacientes
conjuntamente o asociado a inhibidores del citocromo con SIDA con un recuento de CD4 <100 clulas/L.
P450, alteraciones electrocardiogrficas y/o electrolticas La clnica consiste en fiebre, cefalea, alteracin del
previas, SQTL congnito silente, enfermedades cardiacas nivel de consciencia, focalidad neurolgica o convul-
o el sexo femenino. La hipotermia, enfermedades del te- siones. Es caracterstica la presencia de anticuerpos
jido conectivo que expresan anticuerpos anti Ro/SSA y anti-toxoplasma y de lesiones ocupantes de espacio
la infeccin por el VIH, tambin se consideran causas o con edema perilesional en las pruebas de imagen. Las
factores predisponentes del SQTL. caractersticas neurorradiolgicas no son patognom-
En el caso de nuestra paciente, en el momento de plantear nicas y pueden ser muy similares a las del linfoma.
el diagnstico diferencial de las causas ms frecuentes del La encefalitis por virus JC o Leucoencefalopata mul-
SQTL, nos encontramos con limitaciones logsticas, ya que tifocal progresiva (LMP): es una enfermedad desmie-
el hospital no dispone de material de laboratorio que de- linizante causada por el virus JC como consecuencia
termine alteraciones electrolticas y tampoco disponemos de una reactivacin en pacientes con inmunosupresin
de ECG previos en la historia clnica. Sin embargo, ajus- grave, generalmente con CD4 <200 clulas/L. Sin
tndonos a los recursos disponibles, la paciente no refiri embargo, la LMP tambin se puede producir en el
en la anamnesis alteraciones gastrointestinales, ni urinarias contexto de un sndrome inflamatorio de reconstitu-
o patologa cardiaca previa. Adems, en la exploracin fsi- cin inmune (SIRI). Hay muy pocos casos registrados
ca no se objetivaron signos de desnutricin, que nos hicie- de LMP en pacientes con VIH en India y en frica en
ran sospechar alteraciones electrolticas subyacentes. Por comparacin con Estado Unidos y Europa, y aun no
otro lado, s tenamos el antecedente de haber iniciado re- se sabe muy bien por qu (falta de pruebas diagnsti-
cientemente el consumo de varios frmacos. Segn la lite- cas, muertes prematuras debidas a otras infecciones,
ratura revisada, el moxifloxacino es una de las quinolonas diferencias genticas). Clnicamente, los pacientes pre-
incluida en la lista de frmacos con un riesgo conocido de sentan dficits neurolgicos focales rpidamente pro-
producir SQTL. Sin poder descartar otras causas, en este gresivos incluyendo un deterioro cognitivo.
caso parece razonable sospechar que el SQTL podra ser Criptococosis: es una infeccin mictica invasiva cau-
secundario al moxifloxacino, aadido a los factores predis- sada por Cryptococcus neoformans. La meningoencefalitis
ponentes de la infeccin por VIH y el sexo femenino. es la manifestacin ms frecuente entre las personas
En segundo lugar, el deterioro cognitivo en pacientes in- con inmunosupresin avanzada, y la gran mayora de
fectados por el VIH es un sntoma de numerosos procesos los casos, al igual que la toxoplasmosis, ocurren en-
patolgicos, entre los que se pueden encontrar: infecciones tre los pacientes con SIDA con un recuento de CD4

143
CAPTULO 8
INFECCIN VIH
Y ENFERMEDADES ASOCIADAS

<100 clulas/L. Aunque su incidencia ha disminuido vo del LCR son el mejor mtodo de diagnstico.
entre los pacientes con acceso al TAR, sigue siendo Sndrome inflamatorio de reconstitucin inmune
la principal causa de mortalidad en pases en vas de (SIRI): en pacientes que inician TAR, la mejora del
desarrollo, y es en la regin del frica sub-sahariana sistema inmunolgico puede poner de manifiesto in-
donde se registra el mayor nmero de casos. La clnica fecciones latentes, empeorar los sntomas de las ya
ms frecuente es la fiebre y la cefalea, y el diagnstico presentes o simplemente presentar un deterioro clni-
definitivo se obtiene mediante observacin directa del co sin causa infecciosa que responde a corticoides. La
hongo en el LCR (tincin de tinta china) o antgeno sintomatologa neurolgica es una manifestacin grave
en LCR. del SIRI, con un 30 % de mortalidad, que en muchos
Neurosfilis: es una infeccin del SNC por Treponema casos obliga a suspender el TAR hasta su resolucin.
pallidum que puede ocurrir en cualquier momento
despus de la infeccin inicial y ocurre con relativa Por ltimo, mencionar que el citomegalovirus (CMV) pue-
frecuencia en los pacientes con infeccin VIH. En de reactivarse en pacientes con CD4<50 clulas/L y dar
los pacientes infectados por el VIH, la presencia de lugar a una encefalitis que clnicamente cursa con delirio,
una prueba no treponmica, como la reagina rpida confusin y alteraciones neurolgicas focales, y que el pro-
en plasma (RPR) 1: 32 o un recuento de CD4 <350 pio VIH se puede manifestar con alteraciones neurolgi-
clulas/L, se asocian con un mayor riesgo de neu- cas, siendo clsica la trada de demencia, trastornos del
rosfilis. En el curso temprano de la sfilis, las formas movimiento y sntomas depresivos.
ms comunes de neurosfilis alteran a las meninges o Continuando con el diagnstico diferencial, la paciente
el sistema vascular, mientras que en las formas tardas se encontraba en tratamiento con mltiples frmacos, en-
es ms frecuente la afectacin del parnquima de la tre los que destaca el EFV y la cicloserina. El EFV puede
mdula espinal (tabes dorsal). Se recomienda realizar dar lugar a trastornos del sueo y del humor, incluyendo
un estudio del LCR a los pacientes infectados por VIH trastornos de la personalidad en ms del 25% de los pa-
con sfilis en los que aparecen sntomas neurolgicos cientes dentro de las primeras 4 semanas de tratamiento,
(disfuncin cognitiva, dficits motores o sensoriales, aunque es algo transitorio y menos del 3% de los pacientes
alteraciones visuales o auditivas, parlisis de los ner- requiere suspensin del tratamiento. A su vez, se ha des-
vios craneales o sntomas o signos de meningitis). Sin crito con el uso de la cicloserina la aparicin de trastornos
embargo, el diagnstico de neurosfilis en personas psicticos, cuadros confusionales y convulsiones hasta en
infectadas por el VIH puede ser ms complicado, ya un 10% de los pacientes, siendo estas manifestaciones dosis
que la pleocitosis del LCR y la elevacin de protenas dependiente. Aunque el consumo de otros frmacos como
pueden ser consecuencia del VIH en s. Es necesaria la etionamida y el etambutol pueden dar lugar a altera-
la combinacin de varios test serolgicos para llegar al ciones mentales, son reacciones adversas menos frecuentes
diagnstico. El test no treponmico VDRL en LCR que con los anteriores.
es altamente especfico para la neurosfilis y cuando es Teniendo en cuenta que nuestro centro no tiene acceso a
reactivo se considera diagnstico; sin embargo, es poco la realizacin de pruebas de imagen neurolgicas y que el
sensible y un resultado negativo no es excluyente; por laboratorio no cuenta con test serolgicos, el diagnstico
este motivo, se aconseja completar el estudio con test diferencial se basa principalmente en la epidemiologa, la
treponmicos como FTA-ABS que son muy sensibles. clnica y en los datos del estudio del LCR mencionados
Meningitis tuberculosa: la infeccin del SNC por M. anteriormente. Apoyndonos en el grado de inmunosu-
tuberculosis se debe sospechar fuertemente en pacien- presin de la paciente y en la aparicin aguda de la clnica,
tes con una tuberculosis activa. Por lo general, la en- sin manifestaciones neurolgicas focales, la toxoplasmosis,
fermedad tiene un curso subagudo que comienza con la criptococosis, el CMV, la LMP y la neurosfilis parecan
cefalea, fiebre y cambios de personalidad, seguida de poco probables. Continuando con el estudio del LCR, las
manifestaciones neurolgicas ms floridas como me- alteraciones mencionadas pueden estar presentes tanto en
ningismo, disminucin del nivel de consciencia, afec- la tuberculosis como en la criptococosis, la neurosfilis y la
tacin de pares craneales, pudiendo evolucionar a la encefalitis por herpes, sin embargo, a pesar de tener una
aparicin de paresias, convulsiones y coma. El estudio baciloscopia del LCR negativa, el antecedente de un diag-
del LCR es fundamental, con elevacin de protenas, nstico reciente de una tuberculosis pulmonar nos hizo
pleocitosis linfoctica y una concentracin baja de glu- sospechar altamente que la paciente tuviera una meningi-
cosa. No hay que olvidar, que la baciloscopia y el culti- tis tuberculosa. Por ltimo, no se puede descartar que las

144
CAPTULO 8
INFECCIN VIH
Y ENFERMEDADES ASOCIADAS

alteraciones del comportamiento no estuviesen condicio- Diagnstico final


nadas por algunos de los frmacos mencionados, princi-
palmente la cicloserina. Finalmente, la paciente fue dada de alta hospitalaria con
los siguientes diagnsticos:
Evolucin
1. VIH estadio 4 de la OMS en TAR
En primer lugar, tras la suspensin del tratamiento con 2. Tuberculosis diseminada multirresistente.
moxifloxacino, se normaliz el ECG de la paciente, por 3. Toxicidad al tratamiento tuberculosttico multirresis-
lo que a las dos semanas se decidi reforzar el TAT con tente:
levofloxacino, con buena tolerancia y sin presentar altera- Afectacin cardiolgica: sndrome del QT largo
ciones posteriores en los ECG de control realizados. secundaria a moxifloxacino
En segundo lugar, el cuadro neurolgico de la paciente me- Alteraciones del comportamiento agravadas por
jor en las semanas siguientes al instaurar tratamiento con cicloserina
corticoides a altas dosis ante la sospecha de una meningitis Neuropata perifrica secundaria a etionamida,
tuberculosa. A pesar de suspender el tratamiento con ci- etambutol, cicloserina y encamamiento prolongado
closerina persistieron las alteraciones del comportamiento, Ototoxicidad secundaria a kanamicina
mostrndose con una actitud desinhibida. Es posible que la
manifestacin neurolgica se agravara con la introduccin Discusin
del TAR. Puesto que era una paciente con el antecedente
de una induccin fallida, con varias semanas sin el mismo El tratamiento de la tuberculosis multirresistente en un pa-
y que ya realizaba TAT, se decidi no suspender el EFV ciente con infeccin por VIH implica el uso de numerosos
del TAR, ya que su uso no pareci que influyera ni en la frmacos con mltiples toxicidades. Distinguir si se trata
aparicin ni en la mejora de la sintomatologa neurolgi- de un efecto secundario farmacolgico o de una manifes-
ca. Finalmente, se reinici la cicloserina sin que existiera tacin propia de la tuberculosis o del VIH, en ocasiones no
variacin clnica en la paciente. resulta fcil si no se cuenta con los medios diagnsticos ms
En tercer lugar, tras 6 semanas de ingreso, la paciente pa- adecuados. Sin embargo, en reas de alta prevalencia de
dece una polineuropata del enfermo crtico de etiologa estas patologas, donde los recursos sanitarios son escasos,
multifactorial con movilidad muy limitada. Los frmacos una actuacin mdica basada en datos epidemiolgicos, la
utilizados en el tratamiento de la tuberculosis multirre- exploracin fsica y una buena historia farmacolgica, nos
sistente como la etionamida, el etambutol o la cicloseri- puede permitir realizar un diagnstico bastante certero.
na pueden producir neuropata perifrica, que adems, al
utilizarlos conjuntamente, pontencian su toxicidad sobre
el SNC. Algunos autores recomiendan aadir 50-100 mg
de piridoxina de manera profilctica, actuacin que no es
posible en los pases en vas de desarrollo por la escasa dis-
ponibilidad de ese frmaco, a pesar de su bajo coste. La
paciente precis aporte de piridoxina y rehabilitacin para
volver a caminar.
Posteriormente, se detect en la paciente una sordera pro-
gresiva que se relacion con ototoxicidad secundaria al
tratamiento con aminoglucsidos. Tras completar 6 meses
de kanamicina, segn el protocolo del pas, y la suspensin
de la misma, no ha recuperado audicin, y en el momento
actual presenta importantes problemas de comunicacin.
Por ltimo, mencionar que se realizaron baciloscopias de
esputo mensualmente, siendo todas ellas negativas, as
como cultivos que se enviaron al laboratorio nacional de
referencia, sin haber recibido resultados de los mismos en
la actualidad.

145
CAPTULO 8
INFECCIN VIH
Y ENFERMEDADES ASOCIADAS

Bibliografa

1. Roden DM. Drug-induced prolongation of the QT


interval. N Engl J Med 2004; 350:1013.
2. Charles I Berul, Stephen P Seslar, Peter J Zimetbaum,
Mark E Josephson. Acquired long QT sndrome. Up-
ToDate. Marzo 2015.
3. Berenguer J, Moreno S, Laguna F, Vicente T, Adrados
M, Ortega A, et al. Tuberculous meningitis in patients
infected with the human immunideficiency virus. N
Engl J Med 1992;326:668-72.
4. Cecchini D, Ambrosioni J, Brezzo C, Corti M, et al.
Tuberculous meningitis in HIV-infected patients:
drug susceptibility and clinical outcome. AIDS 2007;
21:373-4.
5. Lawn SD, Bekker LG, Miller RF. Inmune reconstitu-
tion disease associated with mycobacterial infections in
HIV.infected individuals receiving antiretrovirals. Lan-
cet Infect Dis 2005;5:361-73.
6. Igor J Koralnik. Approach to HIV-infected patients
with central nervous system lesions. UpToDate. Oc-
tubre 2014.
7. Neil W Schluger. Diagnosis, treatment, and preven-
tion of drug-resistant tuberculosis. UpToDate. Marzo
2015.

146
CAPTULO 9
INFECCIONES DE TRASMISIN SEXUAL
Mujer de 55 aos con sntomas psicticos agudos
Trigo Rodrguez, M; Martn Prez, AJ;
Vinuesa Garca, D.
Hospital Universitario San Cecilio. Granada.

Caso clnico

Mujer de 55 aos que es trada al Servicio de Urgencias


por su hermano porque en las ltimas 3 semanas haban
notado un comportamiento extrao en la paciente. Entre
las alteraciones conductuales, su hermano refera: episo-
dios de agresin verbal, descuido del aseo personal y de su
domicilio, lapsos temporales en los que se ausentaba de su
domicilio durante varias horas sin que pudiesen contactar
con ella, entre otros. La paciente era natural de Almera,
residente actualmente en Granada desde haca 15 aos.
Administrativa de profesin, estaba prejubilada desde
haca 3 aos. En su historial no se recogan antecedentes
personales o familiares de inters, que tampoco eran reco-
nocidos por su acompaante. Tomaba como tratamiento
diario fluoxetina (pautado por su mdico de atencin pri-
maria por nimo depresivo). En la anamnesis directa la
paciente manifestaba temor por la posibilidad de padecer,
tanto ella como sus padres, un tumor en la cabeza (mo-
tivo por el cual ya haba demandado atencin mdica en Figura 1. RMN crneo. Corte coronal donde se aprecia atrofia cortical sin otras
lesiones.
mltiples ocasiones, en distintos servicios de urgencias, en
las ltimas semanas). Como dato fsico de inters, sus fa-
miliares haban detectado prdida ponderal de 10 kg que normales. Coagulacin bsica con cifras de INR 1,48
relacionaban con disminucin de la ingesta, que la pacien- y actividad de protrombina del 55,7%.
te consideraba necesaria, puesto que estaba convencida de
que iba a someterse a un procedimiento quirrgico. Tras valoracin inicial, se decide ingreso a cargo de Psi-
En la exploracin fsica inicial estaba afebril, resto de cons- quiatra para completar estudio de cuadro psictico de de-
tantes bien, a excepcin de discreta tendencia a la taqui- but. A las 48h del ingreso avisan a Medicina Interna de
cardia. Llamaba la atencin una delgadez marcada pero guardia para valoracin por fiebre y deterioro del nivel de
no se encontraron otros datos a destacar en el resto de la conciencia de la paciente. La paciente est febril (38,5)
exploracin fsica. y con bajo nivel de conciencia, semicomatosa con escasa
respuesta a estmulos dolorosos y nula a estmulos verbales,
Pruebas complementarias iniciales por lo que se decide realizar TC craneal urgente y poste-
rior puncin lumbar. La TC craneal slo demostr cam-
ECG: taquicardia sinusal a 118 lpm, eje a 60, con- bios atrficos corticales difusos, descartando lesiones que
duccin normal, sin alteraciones agudas de la repola- produjesen efecto masa o sangrados, por lo que se proce-
rizacin. di a realizacin de puncin lumbar con la que se obtuvo
Analtica urgente: glucemia basal, funcin renal e hi- LCR de aspecto claro con presin de salida de 14 cm de
droelectrolitos sin alteraciones. Protena C reactiva H2O. El anlisis bioqumico mostraba cifras de glucosa 69
(PCR) 63,57 mg/l. LDH 558,6 U/ml. En el hemo- mg/dl (glucosa srica 114 mg/dl) y protenas 40,7 g/L. En
grama destaca anemia normoctica, normocrmica la citologa se informaba de la presencia de 0 leucocitos/
con cifras de hemoglobina en 8,2 g/l. 8300 leucoci- mm3 y 1 hemate/mm3 y de ausencia de microorganismos
tos/mm3 con linfopenia (Linfocitos 11,3%), plaquetas en la tincin de Gram.

148
CAPTULO 9
INFECCIONES DE TRASMISIN SEXUAL

cinaciones y perturbaciones de la percepcin, y por una grave altera-


cin del comportamiento habitual del paciente. Siendo preciso para el
diagnstico el haber descartado la existencia de una causa orgnica.
Por tanto, el propio diagnstico es excluyente y obliga a
hacer un despistaje de patologa orgnica subyacente. No
hay que olvidar que, en las presentaciones atpicas (como
aquellas que cursan con alteracin del nivel de conciencia,
dficit de memoria o inicio tardo), debe plantearse la po-
sibilidad de un trastorno psictico secundario a patologa
mdica general o inducido por sustancias como primera
causa, siendo importante la exclusin de enfermedades de
evolucin aguda y grave en el primer momento de la aten-
cin del paciente.
No cabe duda de que el caso que nos ocupa podra cata-
logarse de inicio tardo: se trata de una mujer de 55 aos
sin datos de patologa psiquitrica previa. Pero, adems,
existen otros datos que hacen imprecisa la aplicacin del
trmino psicosis al cuadro clnico, como son: el bajo ni-
vel de conciencia con el que encontramos a la paciente y
la existencia de fiebre. Es sobre todo este primer dato, la
disminucin del nivel de conciencia, el que resulta cardi-
nal para la ratificacin de la existencia de un delirium o
sndrome confusional agudo, expresin ms adecuada a la
terminologa mdica actual, si atendemos a la definicin
que la Clasificacin Internacional de Enfermedades rea-
Figura 2. RMN crneo. Sin lesiones destacables. liza en su ltima edicin y que marca la orientacin de
nuestro diagnstico diferencial.
Ante la gravedad del cuadro clnico, y a pesar de la nor- El sndrome confusional agudo con frecuencia se asocia
malidad del anlisis del LCR, se decide iniciar tratamiento a sntomas psicticos y su clnica puede fluctuar en inten-
con Aciclovir a dosis de 10 mg/kg/8h por la posibilidad de sidad, presentndose habitualmente de forma sbita. Es
una encefalitis herptica. Se solicit PCR de Herpesvirus preciso recordar que es un trastorno potencialmente rever-
en LCR, PCR de enterovirus en LCR, RMN (Figuras 1 y sible sin olvidar que existen ciertas etiologas que pueden
2) y analtica completa con bioqumica general, reactantes producir, en caso de no resolverse, daos permanentes del
de fase aguda, serologas, ferrocintica y autoinmunidad, sistema nervioso central. Existen multitud de causas de di-
llegndose al diagnstico. versa ndole capaces de producirlo: endocrinolgicas, me-
tablicas, secundarias a consumo de drogas y frmacos, in-
Diagnstico diferencial fecciosas, afectacin estructural del SNC. La mayor parte
de ellas se resumen en la tabla 1.
Todos aquellos trastornos mentales en los que el paciente
pierde en algn momento el correcto juicio de la realidad Evolucin
son los reconocidos de forma tradicional como trastornos
psicticos. Dentro de este grupo cabra incluir enferme- Al da siguiente de la primera valoracin se recibe resul-
dades tan dispares como las demencias, el delirium (psico- tado de PCR de herpesvirus y enterovirus en LCR que
sis orgnicas o sintomticas), los cuadros psicticos indu- resultan negativas, por lo que se suspende tratamiento con
cidos por sustancias (psicosis txicas), la esquizofrenia, la Aciclovir. Tras los resultados iniciales de la bioqumica de
paranoia y la psicosis manaco-depresiva. Con objeto de sangre, se descarta la existencia de alteraciones de tipo me-
realizar una mejor clasificacin diagnstica, la CIE-10 ha tablico (las funciones renal y heptica estaban preserva-
restringido el trmino, considerndose actualmente psico- das y no existan alteraciones hidroelectrolticas ni datos de
sis aguda al Grupo heterogneo de trastornos caracterizados por desnutricin). En el estudio hormonal, la funcin tiroidea
el comienzo agudo de sntomas psicticos, tales como delirios, alu- era normal, y los autoanticuerpos (ANA, ENA y ANCAs)

149
CAPTULO 9
INFECCIONES DE TRASMISIN SEXUAL

CAUSAS DE SNDROME CONFUSIONAL AGUDO


fueron negativos. En el hemograma persista anemia nor-
Frmacos y txicos:
moctica normocrmica, detectada al inicio, que perma-
Opiodes, sedantes, analgsicos, benzodiacepinas, antihistamnicos an-
neca estable, asocindose en esta ocasin a leucopenia tipsicticos, litio, etanol y otras drogas de abuso, efectos secundarios (c.
(2770 leucocitos/mm3) con linfopenia (430/mm3). Desta- valproico, quinolonas, sd. serotoninrgico, etc), envenenamientos (monxido
de carbono, cianuro, metanol, etc).
caba tambin una VSG>140 mm/h.
Infecciones:
La paciente era soltera, tena varias parejas sexuales oca-
Sepsis, infecciones sistmicas, delirium febril, encefalitis VHS, sfilis, VIH,
sionales. Se desconoca si utilizaba medidas de proteccin enfermedad de Lyme.
en las relaciones sexuales. Se reconoca consumidora de Trastornos metablicos:
alcohol de forma social; sus familiares negaban tener co- Diselectrolitemias, endocrinopatas (tiroides, paratiroides, pncreas, etc),
nocimiento sobre consumo de otras sustancias. La RMN hipoxemia, hipercapnia, hiper e hipoglucemia, sndrome hiperosmolar e
hipoosmolar, metabolopatas congnitas (porfirias, Enf. Wilson, etc), dficit
craneal con contraste no detect lesiones ocupantes de es- nutricional (encefalopata de Wernicke, deficiencia vitamina B12, etc).
pacio ni alteraciones vasculares o de la mielinizacin su- Trastornos cerebrales:
gestivas de patologa neurolgica estructural. Infecciones SNC (meningitis, encefalitis, etc), epilepsia, traumatismo, ence-
falopata hipertensiva.
Varios das ms tarde se reciben resultados de serologa que
Fracaso orgnico:
fue positiva para VIH-1. A continuacin se realiz estudio
Insuficiencia cardiaca, insuficiencia respiratoria, fracaso renal, heptico,
inmunovirolgico que evidenci nmero total de CD4 en discrasias sanguneas, etc.
30 cl/mm3 (7%) y carga viral srica 1.062.917 copias/ml. Trastornos fsicos:
Se realiz carga viral en LCR siendo de 11.260 copias/ml. Quemaduras, electrocucin, hipo e hipertermia, sd. respuesta inflamatoria
Estas pruebas complementarias pusieron de manifiesto la sistmica, embolia grasa, traumatismos.

existencia de infeccin VIH avanzada, con grado de inmu- Tabla 1. Causas de sndrome confusional agudo. Modificado de Thompson, Al-
exander W. Pieper, Andrew A. Treisman, Glenn J. HIV-associated neurocognitive
nodepresin severo, y elevada carga viral tanto en sangre disorders. Uptodate. 2015
como en SNC. Se inici terapia antirretroviral, una vez
descartada la existencia de infeccin tuberculosa con test literatura anglosajona de los ltimos 20 aos, no se reco-
de IGRA que result negativo, con tenofovir, emtricitabina gen ms de 15 casos similares al aqu descrito, siendo algo
y cobicistat/elvitegravir, con buena tolerancia. ms elevada esta forma de presentacin en los pases con
Durante su ingreso, la paciente continu con alteracin alta endemia.
conductual por lo que precis seguimiento por psiquiatra, Entre los trastornos psiquitricos de aparicin en los pa-
que inici tratamiento antipsictico con haloperidol. Pro- cientes con infeccin VIH se han descrito cuadros psicti-
gresivamente se produjo una mejora cognitivo-conduc- cos con clnica alucinatoria y/o contenidos delirantes (per-
tual, persistiendo al alta, un mes despus de su ingreso, secutorios o de grandeza). Estos sntomas pueden ocurrir
cierta desorientacin temporal y personal. en el contexto de deterioro cognitivo o pueden constituir
La paciente es revisada tras el alta en consultas externas, la nica manifestacin psicopatolgica. Diversos factores
2 meses despus del inicio del tratamiento antirretroviral, podran explicar la aparicin de un sndrome psictico en
encontrndose asintomtica desde el punto de vista neu- estos sujetos:
ropsiquitrico y con recuperacin casi total del peso y acti-
vidad diaria habitual. Efecto directo del VIH sobre el cerebro. En algunos
pacientes estos sntomas pueden suponer el inicio de
Diagnstico final una encefalopata por VIH que posteriormente pro-
grese a una demencia.
El diagnstico final fue de sndrome confusional agudo se- Infeccin del SNC por parte de otros virus como el
cundario a encefalopata VIH en mujer con infeccin VIH Herpes simplex, el virus de Creutzfeld-Jacob o el citome-
de diagnstico tardo, con criterios de SIDA (Estadio C3). galovirus.
Primer episodio de una enfermedad del espectro de
Discusin la esquizofrenia. Las posibilidades de coexistencia de
ambos cuadros aumentan si tenemos en cuenta que
La encefalopata por VIH, manifestada en forma de sn- comparten factores de riesgo: jvenes en la segunda
drome confusional agudo con sntomas psicticos, es una o tercera dcada de la vida, conducta sexual de alto
forma de presentacin poco frecuente y una forma rara de riesgo y uso de drogas por va parenteral.
trastorno neurocognitivo asociado a la enfermedad. Tras Presencia de circunstancias estresantes capaces de des-
realizar una revisin bibliogrfica de casos similares en la encadenar una psicosis reactiva breve.

150
CAPTULO 9
INFECCIONES DE TRASMISIN SEXUAL

Es necesario remarcar la importancia del despistaje de or- J Neurol Neurosurg Psychiatry. 1995; 58:399-402
ganicidad en los cuadros psicticos, sobre todo en aquellos 3. Thompson, Alexander W. Pieper, Andrew A. Treis-
de presentacin atpica. Dentro del estudio a realizar ser man, Glenn J. HIV-associated neurocognitive disor-
preciso siempre descartar la infeccin por VIH. ders. Uptodate. 2015
En Espaa, se estima que unas 120.000 personas viven ac- 4. Shapshak P, Kangueane P, Fujimura RK, et al. Edito-
tualmente infectadas por el VIH y que alrededor del 30% rial neuroAIDS review. AIDS 2011; 25:123.
de estas personas no conocen dicha condicin. Mientras 5. Crum-Cianflone NF, Moore DJ, Letendre S, et al.
que el nmero de nuevos infectados y el nmero de muer- Low prevalence of neurocognitive impairment in early
tes por VIH han disminuido globalmente en la ltima d- diagnosed and managed HIV-infected persons. Neu-
cada, no se ha objetivado reduccin alguna en el nmero rology 2013; 80:371-9.
de diagnsticos tardos en los ltimos 5 aos. Se estima que 6. Clasificacin internacional de enfermedades 10. revi-
casi la mitad de los nuevos diagnsticos en nuestro pas sin, modificacin clnica edicin espaola. Ministerio
presentan un recuento de CD4 inferior a 350 cel/l (defi- de Sanidad, Servicios Sociales e Igualdad. Direccin
nitorio de diagnstico tardo). De hecho, en 2012 en An- General de Salud Pblica, Calidad e Innovacin. Sub-
daluca se diagnostic de forma simultnea el VIH y SIDA direccin General de Informacin Sanitaria e Innova-
en el 25% de los casos, segn la encuesta de prevalencia cin. www.msssi.gob.es
hospitalaria VIH-SIDA.
Que persista un porcentaje elevado de casos de diagnstico
tardo es relevante tanto a nivel individual como colectivo.
Desde el punto de vista del individuo, a mayor nivel de in-
munosupresin ser esperable una peor respuesta al trata-
miento y por tanto, un mayor riesgo de desarrollar SIDA o
de muerte. Y colectivamente supone un problema de salud
pblica, ya que se favorece la transmisin del virus al no
adoptar la persona infectada conductas de menor riesgo.
Se estima que aproximadamente la mitad de las nuevas
infecciones VIH son causadas por individuos que desco-
nocen estar infectados.
En general, se realiza el screening de VIH a los principales
grupos de riesgo y a aquellas personas posiblemente ex-
puestas. Sin embargo, las principales sociedades mdicas
especializadas y grupos de trabajo recomiendan la siste-
matizacin del estudio de contactos, las estrategias de ex-
clusin voluntaria y el llamado test universal, es decir,
la realizacin de pruebas diagnsticas de VIH al menos
una vez al ao, en todas la personas que tienen contac-
to con el sistema sanitario, como una buena medida para
prevenir la transmisin. Adems, el personal sanitario de-
bera estar ms sensibilizado para detectar ms casos de
infeccin oculta por VIH. Otra medida recomendada es el
tratamiento a todos los pacientes una vez diagnosticados,
pero la economa sigue jugando un papel en las decisiones
mdicas.

Bibliografa

1. Helleberg M, Kirk O. Encephalitis in primary HIV


infection: challenges in diagnosis and treatment. Int J
STD AIDS 2013; 24:489-93
2. Everall IP. Neuropsychiatric aspects of HIV infection.

151
Varn con infeccin por VIH y proctitis:
dudas diagnsticas y terapeticas
Ruiz Estvez, B; Martn Aspas, A;
Sifuentes Daz, E; Ros Holgado, I.
Hospital Universitario Puerta del Mar. Cdiz.

BACTERIAS VIRUS PROTOZOOS


Caso clnico
Neissseria gonorrhoeae Virus Herpes Simple Entamoeba hystolitica
Chlamydia trachomatis Cytomegalovirus
Varn de 32 aos, sin alergias, fumador de 10 cigarrillos Treponema pallidum
Escherichia coli
al da y de cannabis, bebedor ocasional. Niega consumo Shigella spp.
de otros txicos. Antecedentes de trastorno afectivo y de Clostridium difficile

la personalidad, con episodios maniacos e ingresos previos Tabla 1. Patgenos habituales de la proctitis infecciosa. Adaptado de Wilcox (1).
en Unidad de agudos de Psiquiatra, en tratamiento con
clonazepam, sertralina y quetiapina. menor calibre que el habitual, y rectorragias que haban
Infeccin VIH de transmisin homosexual, diagnosticado ido en progresin. No asociaba fiebre ni sntomas genera-
en otro centro en 2007, a raz de una lcera genital compa- les, tampoco datos clnicos de gravedad. A la exploracin,
tible con chancro primario sifiltico, con serologa de sfilis presentaba tacto rectal doloroso, sin lesiones visibles; no
positiva. Realiz tratamiento con penicilina intramuscular, lceras genitales ni adenopatas, ningn otro hallazgo a
confirmndose la curacin serolgica. Al diagnstico: Lin- destacar, tanto a nivel local como en otras localizaciones.
focitos CD4+ basales 920 cel/mm. Desconocemos carga Deciden realizar una rectoscopia que mostraba datos de
viral basal, pero s que tenaresistencias basales: L100I y intensa proctitis ulcerativa, tomndose una biopsia con los
R41K. HLAB57 negativo. No inici tratamiento antirre- hallazgos anatomopatolgicos de: fragmentos de mucosa de
troviral al tener ms de 500 linfocitos CD4+, acudiendo intestino grueso con inflamacin crnica y tejido de granulacin que
regularmente a revisiones, y se trat posteriormente de un muestran en algunas clulas endoteliales cambios citopticos vricos
segundo episodio de sfilis latente precoz. con inmunotincin positiva para Citomegalovirus (CMV).
A mediados de 2014 se traslada a nuestra ciudad y acude Con los resultados de las pruebas diagnsticas, es derivado
a consulta para valoracin. En ese momento se encuen- para tratamiento.
tra asintomtico con exploracin clnica normal. Refiere
pareja estable que reside en Madrid. No ha presentado Diagnstico diferencial
infecciones oportunistas en este tiempo. Se reevala su si-
tuacin inmunolgica: carga viral 58.079 copias/ml y lin- El paciente consulta por sntomas rectales. La patologa
focitos CD4+ 502 cels/mm3 (18%). Se solicita: analtica ano-rectal es una entidad relativamente frecuente en los
completa con estudio de orina, sin alteraciones; serologa pacientes VIH, particularmente en los homosexuales varo-
de virus hepatotropos (VHC negativo, VHB y VHA inmu- nes que practican sexo anal de manera receptiva, general-
nizado); IgG para toxoplasma negativa e IgG para citome- mente por causa traumtica o infecciones de transmisin
galovirus positiva (compatible con infeccin pasada); les sexual (ITS), siendo los hallazgos ms comunes: abscesos
con RPR no reactivo. En ese momento, se aconseja inicio perirrectales, fistulas anales, verrugas anales, lceras ines-
de tratamiento antirretroviral y se solicita un nuevo estudio pecficas, proctitis infecciosa, neoplasia anal intraepitelial
de resistencias que no muestra nuevas mutaciones respecto (asociada al virus del papiloma humano) y cncer anal.
al previo. Teniendo en cuenta su situacin inmunolgica, Con mucha menos frecuencia, estos sntomas pueden no
resistencias basales y posibles interacciones farmacolgi- estar asociados a la prctica sexual y ser expresin de una
cas, fundamentalmente con la medicacin antipsictica, se enfermedad inflamatoria intestinal, como colitis ulcerosa o
opta por iniciar abacavir con lamivudina y dolutegravir, enfermedad de Crohn (1). En este caso, la rectoscopia evi-
con buena tolerancia y eficacia, consiguindose carga viral denciaba una inflamacin de la mucosa rectal o proctitis,
indetectable a las cuatro semanas. por lo que dado los antecedentes del paciente (infeccin
Posteriormente, es atendido en el Servicio de Aparato Di- por VIH y tratado de sfilis en dos ocasiones), orienta hacia
gestivo por molestias rectales que haban comenzado unas una ITS. Los grmenes causantes de una proctitis infeccio-
dos semanas antes y consistan en importante dolor en la sa son variados, pudiendo estar implicados en su etiologa
regin rectal y ano, sobre todo al defecar, con heces de tanto bacterias, como virus o protozoos (Tabla 1). De ellos,

152
CAPTULO 9
INFECCIONES DE TRASMISIN SEXUAL

los ms frecuentes son: Neisseria gonorrhoeae, Chlamydia tracho- te diagnosticado de: PROCTITIS INFECCIOSA POR
matis, virus Herpes simplex y Treponema pallidum. Si bien pue- CHLAMYDIA TRACHOMATIS.
den presentarse de manera asintomtica, en la mayora de
los casos originan una clnica similar a la que presenta el pa- Discusin
ciente (dolor anal intenso, fundamentalmente al defecar, que
se suele asociar a sangrado y tenesmo rectal, con ausencia de El problema que se nos plantea en la evaluacin inicial
sntomas generales), y sta no suele aportar datos especficos del caso es el papel que desempea el CMV en la clnica
que ayuden al diagnstico etiolgico (2). Sin embargo, se nos actual del paciente. Como sabemos, la infeccin por CMV
plantea en el diagnstico diferencial la infeccin por CMV es relativamente frecuente en nuestro medio (la proporcin
que es el germen hallado en la biopsia rectal y que tambin de sujetos con evidencia de infeccin previa por CMV va-
puede causar proctitis, si bien constituye una causa muy ra a lo largo del mundo, con tasas de seroprevalencia que
poco comn (3). Menos probable en nuestro caso, seran oscilan entre el 40 y 100% de la poblacin adulta) (4). La
otros grmenes como Escherichia coli, Shigella spp, Entamoeba transmisin se produce por contacto directo con secrecio-
histolytica (generalmente en infecciones gastrointestinales, no nes contaminadas, como sangre, saliva, secreciones vagi-
compatible con la clnica del paciente) y Clostridium difficile nales o semen, y el espectro clnico es variado, dependien-
(habitualmente en relacin con antibioterapia previa, que do en gran medida de la situacin inmunolgica. As, en
no haba tomado). El cultivo habitual, estudio de parsitos sujetos inmunocompetentes, la primoinfeccin por CMV
y deteccin de toxina de Clostridium, son generalmente sufi- produce un cuadro clnico inespecfico, generalmente en
cientes para el diagnstico de estas entidades. forma de sndrome mononuclesido que se resuelve espon-
Por tanto, la cuestin que se nos plantea es: realmente tneamente sin mayores complicaciones, quedando el virus
el CMV est implicado en el cuadro actual o ha sido un en forma latente durante toda la vida del individuo, siendo
hallazgo casual? posible presentar perodos de reactivacin de la infeccin
que pueden pasar desapercibidos o manifestarse con snto-
Evolucin mas leves. Sin embargo, en pacientes imnunocomprome-
tidos (principalmente infeccin por VIH y trasplantados),
Rehistoriando al paciente, refiere relacin anal receptiva tanto la infeccin primaria como la reactivacin puede dar
sin preservativo con su pareja que vive en Madrid. En ese lugar a complicaciones importantes en cualquier rgano,
momento se decidi no iniciar tratamiento para CMV y con elevada morbimortalidad. En pacientes con proctitis y
se ampli el estudio diagnstico en busca de otros grme- buena situacin inmunolgica, el papel del patognico del
nes ms frecuentemente implicados y que, a su vez, produ- CMV es discutido cuando se detecta en una biopsia rectal,
cen clnica compatible. Para ello se solicit: deteccin de habindose descrito la resolucin clnica incluso en ausen-
material gentico por Reaccin en Cadena de Polimerasa cia de tratamiento(5), mientras que en inmunocomprome-
(PCR) rectal de herpes, gonococo y clamidia, as como cul- tidos y ante la presencia de la triada clsica (relacin anal
tivo de gonococo y serologa de sfilis. Estas pruebas estn receptiva, clnica de proctitis y sndrome mononuclesido),
indicadas de rutina en la evaluacin de todos los pacientes sugestiva de primoinfeccin, s puede justificarse al CMV
con proctitis infecciosa y sospecha de ITS, pues est descri- como causante de la misma, si bien tambin se ha descrito
ta la coinfeccin por varios grmenes a la vez (1,2). la resolucin espontanea. En nuestro paciente, la serologa
Tras la toma muestras, se paut tratamiento emprico con es compatible con infeccin pasada por CMV y mantiene
una dosis de ceftriaxona 1 gramo intramuscular y doxici- buenas cifras de linfocitos CD4+ (mayores de 500 cel/
clina oral a dosis de 100 mg cada 12 horas durante 21 das, mm).
con desaparicin completa de los sntomas a los pocos das Respecto a la proctitis por C. trachomatis, la presentacin
de inicio del tratamiento. Los resultados de las pruebas so- clnica puede variar en funcin de las serovariantes impli-
licitadas mostraron una PCR de C. trachomatis positiva (rea- cadas (6). En la proctitis sintomtica, aunque estn descri-
lizado mediante sistema de PCR en tiempo real), con resto tas otras serovariantes, es tpica la tipo L (L1, L2, L3) que
de los estudios negativos. produce una enfermedad denominada Linfogranuloma
venreo (LGV) que con mayor frecuencia ocasiona lceras
Diagnstico final genitales indoloras asociadas a adenopatas inguinales do-
lorosas y es poco frecuente en nuestro pas (rara en los pa-
Ante el contexto clnico, hallazgos microbiolgicos y res- ses industrializados, siendo endmica en algunas partes de
puesta favorable al tratamiento, el paciente fue finalmen- frica, Asia, Sudamrica y pases del Caribe), aunque se

153
CAPTULO 9
INFECCIONES DE TRASMISIN SEXUAL

han descrito algunos brotes recientes en Espaa, primero proctitis: A rare sexually transmitted disease. J Sex
en Barcelona y posteriormente en Madrid, la mayora de Med. 2014; 11:1092-5
ellos en sujetos con infeccin VIH y homosexuales (7); las 4. Friel TJ. Epidemiology, clinical manifestations, and
otras serovariantes (D-K) estn implicadas habitualmente treatment of cytomegalovirus infection in immuno-
en la uretritis no gonoccica y en la proctitis asintomtica. competent adults. [Monografa en internet]. UpToDa-
El tratamiento que se utiliza en las formas ms frecuentes te; actualizado en mayo 2014. Disponible en http://
de infeccin por C. trachomatis es doxicilina 100 mg cada www.uptodate.com
12 horas durante 7 das azitromicina 1g en dosis nica, 5. Studemeister A. Cytomegalovirus proctitis: A rare and
mientras que en el LGV o infeccin por la variante L, se disregarded sexually transmitted disease. Sex Transm
recomienda una duracin de 21 das con doxiciclina azi- Dis. 2011; 38;876-8
tromicina semanalmente durante 3 semanas, fundamen- 6. Marrazzo J. Clinical manifestations and diagnosis of
talmente para evitar la cronicidad de las lesiones (8). Con Chlamydia trachomatis infections. [Monografa en in-
respecto a la eleccin de doxiciclina frente azitromicina, se ternet]. UpToDate; actualizado en marzo 2015. Dis-
han publicado recientemente meta-anlisis que evidencian ponible en http://www.uptodate.com
un ligero aumento de la eficacia del tratamiento cuando se 7. Martin-Iguacel R, Llibre JM, Nielsen H, Heras
emplea doxicliclina (hasta del 3-7% frente a azitromicina), E,Matas L,Lugo R, et al. Lymphogranulomavene-
tanto en infecciones genitales como rectales (9, 10). En este reumproctocolitis: asilentendemicdiseaseinmen
caso, decidimos tratar al paciente con doxicilina y, puesto whohavesexwithmenin industrialisedcountries. Eur
que la tcnica de laboratorio que disponemos para PCR de J Clin Microbiol Infect Dis.2010; 29:917-25
Chlamydia no permite distinguir entre serovariantes, man- 8. Palacios R, Polo R, Blanco JL, Blanco JR, Camino X,
tuvimos el tratamiento durante 21 das. Cervero M, et al. Documento de Consenso del Grupo
de Estudio del Sida (GESIDA)/Plan Nacional sobre
Puntos claves el SIDA (PNS) sobre las infecciones de transmisin se-
xual en pacientes con infeccin por el VIH. Enferm
Ante un cuadro de proctitis es fundamental para un Infecc Microbiol Clin. 2011; 29:286.e1-286.e19
tratamiento correcto, realizar una historia clnica ade- 9. Kong FYS, Tabrizi SN, Law M, Vodstrcil LA, Chen
cuada, especialmente dirigida al diagnstico de ITS. M, Fairley CK, et al. Azithromycin versus doxycycli-
Ante una proctitis como enfermedad de transmisin ne for the treatment of genital Chlamydia infection:
sexual, las tcnicas diagnsticas invasivas deben reali- A meta-analysis of randomized controlled trials. Clin
zarse si con el estudio microbiolgico y el tratamiento Infect Dis 2014;59:193-205
de la ITS no se solucionan los sntomas. 10. Kong FYS, Tabrizi SN, Fairley CK, Vodstrcil LA, Hus-
El CMV no suele ser el agente causal de proctitis en ton WM, Chen M, et al. The efficacy of azithromycin
pacientes con buena situacin inmunolgica por lo and doxycycline for the treatment of rectal chlamydia
que deben valorarse otras etiologas. infection: a systematic review and meta-analysis. J An-
Ante un resultado positivo por PCR para C. trachomatis timicrob Chemother. 2015; 70:1290-7.
de muestra rectal por una proctitis sintomtica, si no
se puede determinar la serovariante, aconsejamos tra-
tamiento prolongado.

Bibliografa

1. Wilcox CM. Evaluation of the HIV-infected patient


with anorectal symptoms. [Monografa en internet].
UpToDate; actualizado en enero 2014. Disponible en
http://www.uptodate.com
2. Bissessor M, Fairley C, Read T, Denham I, Bradshaw
C, Chen M. The etiology of infectious proctitis in men
who have sex with men differs according to HIV sta-
tus. Sex Transm Dis. 2013; 40:768-70
3. Maatouk I, Moutran R, Josiane H. Cytomegalovirus

154
Amaurosis de causa atpica
Rubio-Marn, P; Caro-Pea, I;
Girn-beda, J; Rodrguez-Flix, L.
Hospital del SAS de Jerez de la Frontera.

Caso clnico

Describimos el caso de una mujer de 66 aos con hiper-


tensin arterial, dislipemia y neuralgia del trigmino que
acudi a Urgencias por prdida de visin del ojo izquier-
do, no dolorosa, de instauracin sbita y de varios das de
evolucin. En la exploracin oftalmolgica se observ en el
fondo de ojo izquierdo una papila hipermica, ms eviden-
te en el rea nasal, con amputacin de los vasos superiores Figuras 1 y 2. Retinografas en las que se observa la asimetra papilar, con hipe-
remia y borramiento de bordes y desaparicin de la excavacin de la papila del
y edema peripapilar (figuras 1 y 2); pupilas isocricas; de- ojo izquierdo (Figura 1).
fecto pupilar aferente relativo (DPAR); y un defecto cam-
pimtrico central y altitudinal superior en la exploracin
del campo visual por confrontacin. Agudeza visual (AV)
en ojo derecho 1 y en ojo izquierdo menor 20/400. El ojo
derecho no presentaba alteraciones. La protena C reacti-
va (PCR) y el Tiempo de Tromboplastina activado (TTPa)
estaban levemente elevados y la TAC craneal era normal.
Ingresa para estudio sistmico con el diagnstico de sos-
pecha de neuritis ptica de ojo izquierdo probablemente
isqumica anterior no artertica. La paciente presentaba Figuras 3 y 4. Retinografas de polo posterior. Resolucin del edema y recuper-
acin de la excavacin papilar en ojo izquierdo (figura 3).
lesiones eritematosas de aspecto residual en abdomen de
origen infeccioso o txico. Se aquejaba adems de astenia,
claudicacin mandibular y dolor musculares generalizados vio ptico con la aparicin del exantema generalizado nos
y no haba presentado fiebre. En el interrogatorio se descu- oblig a pensar en una alteracin txico-infecciosa, en
bre una enfermedad venrea en el cnyuge hace 2 aos (le- concreto una sfilis secundaria dados los antecedentes de
sin ulcerada genital que se resolvi con tratamiento). La su pareja de posible chancro hace dos aos. Otra entidad
afectacin neurolgica se manifestaba como pupilas isoc- dentro del diagnstico diferencial fue la arteritis de clu-
ricas y DPAR izquierdo. La visin cromtica estaba con- las gigantes ya que presentaba cefaleas frecuentes y clau-
servada. No presentaba dolor ocular con los movimientos dicacin mandibular. El sndrome antifosfolpido tambin
del globo ni a la palpacin. deba estar en mente por la posibilidad de afectacin del
Los potenciales evocados visuales presentaban asimetra de nervio ptico por un fenmeno tromboemblico.
amplitud y latencia a expensas de ojo izquierdo compati-
ble con neuropata ptica izquierda. La RMN fue normal. Evolucin
En ecodoppler de troncos supraarticos existan mnimos
cambios arterioesclerticos bilaterales con pequeas pla- En cuanto a las pruebas de laboratorio, hubo resultados
cas en bulbo carotideo sin signos de estenosis significativa. positivos en la serologa para lues (Anticuerpos Treponema
pallidum IgG + IgM (QL) positivo, FTA-Abs-Ig positivo,
Diagnstico diferencial RPR 1/16), y en la autoinmunidad anticuerpos anticar-
diolipina. Resto, incluyendo hemograma, bioqumica,
La clnica y la exploracin que presentaba la paciente mos- VSG, estudio de autoinmunidad, niveles de vitamina B12,
traban una neuropata ptica anterior izquierda de posible funcin tiroidea, niveles de enzima convertidora de angio-
origen isqumico no artertica que haba que confirmar tensina, serologa (Brucella, VHB, VHC, VIH) dentro de la
con ms pruebas complementarias. La afectacin del ner- normalidad con Mantoux negativo. Se descart neuroles

155
CAPTULO 9
INFECCIONES DE TRASMISIN SEXUAL

mediante puncin lumbar y anlisis de LCR con serologa proteinorraquia) en ausencia de sntomas; en caso de pre-
lutica FTA-Abs Ig totales negativos. sentarlos, la afectacin de la memoria y la personalidad
Por todo lo anterior, se concluy que la paciente presen- son los habituales. Los anticuerpos anticardiolipina, pro-
taba una papilitis, compatible con un cuadro de sfilis se- ducidos tambin por el Treponema pallidum, son indicadores
cundaria. Por ello, recibi 4.000.000 UI de penicilina G de sndrome antifosfolpido (SAF). Estos anticuerpos estn
sdica intravenosa cada 4 horas durante 14 das con reso- presentes en aproximadamente un tercio de los accidentes
lucin parcial del cuadro visual y desaparicin completa cerebrovasculares que se presentan en personas de media-
del cuadro dermatolgico, desarrollando posteriormente na edad. Para el diagnstico de un SAF es necesario su po-
atrofia papilar y macular secundarias y micropsia (Figuras sitividad en dos ocasiones y la expresin clnica. Realizar
3 y 4). La AV residual en el ojo izquierdo fue de 20/60. El el diagnstico de SAF en esa paciente tena implicaciones
RPR ha disminuido, FTA-Abs Ig totales y los anticuerpos teraputicas no exentas de complicaciones como es el caso
anticardiolipina se negativizaron. de la anticoagulacin. Una vez tratada la infeccin los anti-
cuerpos anticardiolipina se negativizaron. La neuritis ptica
Diagnstico final anterior cursa con afectacin de la agudeza visual entre mo-
derada y severa, as como un defecto pupilar aferente. La
Neuritis ptica isqumica anterior no artertica causada alteracin campimtrica ms frecuente es la aparicin de un
por sfilis secundaria. escotoma central unilateral. En la poblacin adulta las neu-
ropatas pticas anteriores isqumicas son las ms frecuen-
Discusin tes. La VSG y la negatividad de la autoinmunidad descart
una arteritis de la temporal, ms habitual en hipertensos,
La sfilis es una enfermedad venrea causada por Treponema diabticos y fumadores. Dada la presencia de factores de
pallidum. Esta enfermedad es conocida como la gran simu- riesgo haba que considerar un origen isqumico, aunque
ladora por la variedad de sntomas que puede presentar y todos los datos orientaban hacia la sfilis como la causante
que nos obliga a tenerla en cuenta en el diagnstico dife- ms probable de esta entidad. Por consiguiente, concluimos
rencial de diversas entidades. Su incidencia ha disminuido que nuestra paciente present una neuritis ptica isqumica
desde la aparicin de la penicilina, y no es infrecuente, por anterior no artertica causada por sfilis secundaria desesti-
este hecho, que se subestimen las manifestaciones oculares mndose la neuroles, la arteritis de la temporal y el sndro-
de la sfilis secundaria sobre todo en pacientes no VIH. La me antifosfolpido como posibles causas.
infeccin por sfilis no pareca a priori la causa ms proba-
ble de la afectacin visual en nuestra paciente, pero una Bibliografa
historia clnica exhaustiva, no solo de la misma sino tam-
bin de su pareja, modific el diagnstico de sospecha. 1. Contreras Zuiga E, Zuluaga Martnez S.X, Ocampo
En la sfilis secundaria la afectacin ocular slo se presenta V. Sfilis: la gran simuladora. Infect. 2008;12:349-56.
en el 5-10% de los casos. Las manifestaciones visuales pue- 2. Belin MW, Baltch AL, Hay PB. Secondary syphilitic
den ser mltiples, ya sea por accin directa del microor- uveitis. Am J Ophthalmol. 1981; 92: 210-14.
ganismo o de los cambios hematolgicos e inflamatorios 3. Spoor TC, Wynn P, Hartel WC, Bryan CS. Ocular
inducidos a nivel sistmico. Puede afectar a cualquier es- syphilis: acute and chronic. J Clin Neuroophthalmol
tructura ocular entre las cuales se encuentra la afectacin 1983; 3:197-203.
del nervio ptico en forma de papilitis (inflamacin de 4. Allevato M, Julin P, Cabrera H. Sfilis secundaria con
la porcin prelaminar del nervio ptico) y que cursa con compromiso ocular Folia Dermatol. 2007; 18:37-41.
prdida brusca de agudeza visual, generalmente de forma 5. Boumpadre, M.C Neuropata ptica aguda: diag-
unilateral, por lo que obliga a descartar la existencia de nsticos diferenciales. Rev Neurol. 2013; 57 (Supl
neuropata ptica isqumica anterior artertica (NOIAA) 1):S139-S147.
o no artertica (NOIANA), valorando la existencia de una 6. Peate I. Syphilis: clinical presentation, diagnosis and
enfermedad sistmica asociada. treatment. Nurs Stand. 2007;22: 48-55.
Es posible que en este caso se uniese una especial predis- 7. Zeltser B. Syphilis. Clin Dermatol. 2004;22:461-8
posicin a desarrollar fenmenos isqumicos junto con los 8. Rothschlid F. History of syphilis. Clin Infect Dis.
cambios inducidos por la infeccin. La neurosfilis o sfilis 2005;40:1454-63.
terciaria suele aparecer desde los 5 a 10 aos del contac- 9. Duong M. The rash of secondary syphilis. CMAJ.
to sexual, con bacteriemia y LCR anormal (pleocitosis y 2007;176:33-5.

156
CAPTULO 10
INFECCIONES TROPICALES
Y DEL VIAJERO
Fiebre y diarrea crnica tras un viaje a la India.
Una sola causa puede no ser suficiente
Bosch Guerra, X; Roca Oporto, C;
Gonzlez Alvarez, DA; Luque Mrquez, R.
Hospital Universitario Virgen del Roco. Sevilla.

Caso clnico

Mujer de 21 aos natural de la India, residente en Espaa


desde los dos aos de edad, y sin antecedentes personales
de inters, que consulta en Urgencias un mes antes, tras re-
gresar de viaje a la India, por cuadro de diarrea lquida de
unas 6-8 deposiciones diarias sin productos patolgicos, y
en la ltima semana, fiebre de hasta 39,5C. No presenta-
ba clnica respiratoria ni urinaria asociada, aunque refera
dolor abdominal leve.
El viaje tuvo motivos religiosos/familiares, matrimonio
concertado, con estancia de 6 meses principalmente en el
estado de Rajasthan (Noroeste del subcontinente Indio), en
zona rural, residiendo en casa de amigos y familiares con Figura 1. Frecuencia de sntomas asociados en TBC Intestinal. Adaptado de Tu-
berculosis Intestinal. Estudio descriptivo retrospectivo de 14 casos. SAEI 2013.
hbitos alimentarios segn cocina y tradiciones locales.
Refiere relaciones sexuales no protegidas con su marido
tras el matrimonio. No realiz baos en ros ni lagos. ca posterior no fue favorable con persistencia de la fiebre,
A la exploracin fsica presentaba una tensin arterial (TA) diarrea y disminucin del nivel de conciencia, por lo que
de 100/65 mm Hg con una frecuencia cardaca (FC) a 100 se decide iniciar antibioterapia emprica con ceftriaxona y
latidos por minuto (lpm) y una T de 39 C. El abdomen metronidazol, con sospecha de cuadro sptico tras legrado.
no estaba distendido, con ruidos hidroareos normales, y No obstante ante la ausencia de respuesta se decide consul-
con mnima sensacin dolorosa en fosa ilaca derecha sin tar con Enfermedades Infecciosas.
signos de irritacin peritoneal. El resto de la exploracin
fsica fue normal. Diagnstico diferencial
Analticamente presentaba una bioqumica heptica alte-
rada con patrn de colestasis disociada: GOT 473 UI/l, El cuadro diarreico de ms de tres semanas de evolucin,-
GPT 153 UI/L, FA 272 UI/l, GGT 323 UI/l y bilirrubi- con fiebre de corta duracin, la historia reciente de viaje
na en rango normal. La Protena C reactiva (PCR) era de a rea tropical, el tipo de viaje realizado, la situacin de
67 mg/l. En hemograma destacaban: 12000 leucocitos/ inmunodepresin inducida por el embarazo y la ausencia
mm3 con neutrofilia (polimorfonucleares 78%) y Hb 6,5 de respuesta inicial a antibioterapia emprica de amplio es-
g/l (VCM 82 fL, HCHM 30 pG). El resto de parmetros pectro, son detalles que resultan clave a la hora del enfoque
incluyendo funcin renal, recuento plaquetario y estudio diagnstico en esta paciente. El cuadro diarreico tras viaje
de coagulacin fueron normales. al trpico es el motivo de consulta ms frecuente en centros
Se realiz una radiografa de abdomen en supino sin ha- de atencin a patologa del viajero. Los microorganismos
llazgos relevantes, as como una ecografa abdominal que ms frecuente de origen bacteriano son: Escherichia coli en-
mostraba una esplenomegalia homognea sin otras alte- terotoxignica, Escherichia coli enteroagregativa, Campylobacter jeju-
raciones. La gota gruesa y frotis de sangre perifrica no ni, Shigella spp y Salmonella spp. Otros agentes a considerar
evidenciaron parsitos. Se realiz test de gestacin urgente son: Norovirus, Rotavirus, Giardia lamblia, Entamoeba histolytica
con resultado positivo, por lo que fue trasladada al Hospi- y Cyclospora. La mayora de ellos dan lugar a cuadros agu-
tal Maternal para valoracin ginecolgica, confirmndose dos, que se manifiestan en las primeras dos semanas de su
gestacin con feto sin signos de viabilidad, por lo que, tras adquisicin, autolimitados y con buena respuesta a trata-
reposicin hidroelectroltica y transfusin de hemoderiva- miento etiolgico, en caso de precisarlo. En esta paciente,
dos, es sometida a un legrado uterino. La evolucin clni- la duracin del cuadro hace poco probable que se deba a

158
CAPTULO 10
INFECCIONES TROPICALES
Y DEL VIAJERO

los microorganismos bacterianos ms frecuentes. Asimismo


todos ellos deberan haber respondido al tratamiento anti-
bitico iniciado. La etiologa vrica queda descartada por
la duracin del cuadro. El tipo de diarrea no inflamatoria
hace poco probable que se trate de Entamoeba histolytica. Es
ms probable por todo ello el origen parasitario, que ha-
bra que descartar mediante estudio en heces. Sin embargo
la esplenomegalia observada, as como los valores de PCR,
no concuerdan inicialmente con la posibilidad de etiolo-
ga parasitaria. La anemia severa al ingreso, sin evidencia
de sangrado externo, junto con la cronicidad del cuadro,
podran indicar infeccin de larga evolucin, o prdidas
hemticas intestinales crnicas no evidenciables. Dada la
alta tasa de infeccin por Salmonella tiphy en la India, ha-
bra que considerar la posibilidad de fiebre fifoidea con mi-
crosangrado intestinal por hipertrofia de placas de Peyer,
diagnstico que justificara la esplenomegalia asociada. Sin
embargo la fiebre tifoidea suele cursar como un cuadro sis-
tmico agudo que no concuerda con la cronicidad de la
clnica en esta paciente. Asimismo, la esplenomegalia no
es un dato especfico y existen numerosas causas infeccio-
sas como no infecciosas, que la pueden justificar. Las rela-
ciones sexuales no protegidas, hacen necesario descartar
infeccin por VIH con infeccin oportunista asociada, con Figura 2. TBC Intestinal con afectacin de leon terminal y ciego, engrosamiento
mural y pseudomasa que oblitera la luz intestinal. Adaptado de Tuberculosis
grmenes como Cryptosporidium o Cyclospora. Por ltimo hay Intestinal. Estudio descriptivo retrospectivo de 14 casos. SAEI 2013.
que tener en cuenta que el embarazo induce inmunode-
presin celular que puede facilitar infecciones por grme-
nes poco frecuentes como Mycobacterium tuberculosis.

Evolucin

En base a todo ello se solicitaron como parte del estudio diag-


nstico microbiolgico: coprocultivo en el que se aisl Salmo-
nella non-typhi (S. enteritidis, R a cido nalidixco, CMI >16
mg/ml); baciloscopia que result negativa y cultivo de mico-
bacterias; estudio de parsitos en heces que fue negativo, y
hemocultivos que resultaron tambin negativos. Los estudios
serolgicos que incluyeron VIH, VHA, VHB, VHC, Coxiella
burnetti, Rickettsia connori y Brucella melitensis fueron negativos.
Se intensific el aporte hidroelectroltico, y se inici dieta
oral progresiva con evolucin favorable: autolimitacin del
cuadro diarreico quedando afebril a los 5 das del inicio de
antibioterapia, con descenso de los reactantes de fase agu-
da (RFA) en analticas de control (Leucocitos 9000/ microl Fgura 3. Afectacin de mucosa cecal en paciente con TBC Intestinal. Pseudo-
masa y engrsamiento mural irregular friable. Adaptado de Tuberculosis Intesti-
y PCR en 12 mg/l). nal. Estudio descriptivo retrospectivo de 14 casos. SAEI 2013.
En base al aislamiento en heces, mejora clnica y analti-
ca, junto con la ausencia de signos clnicos sugestivos de tas externas con diagnstico de, gastroenteritis aguda por
proceso sptico, se decidi interrumpir la antibioterapia Salmonella non-typhi sobre cuadro diarreico crnico tras
emprica a los 7 das de su inicio. Tras 48 horas de observa- viaje al trpico, sin aislamiento microbiolgico y legrado
cin clnica, fue dada de alta para seguimiento en consul- uterino por aborto espontneo.

159
CAPTULO 10
INFECCIONES TROPICALES
Y DEL VIAJERO

A los quince das vuelve a ingresar por persistencia de la como enfermedad inflamatoria intestinal como principales
clnica diarreica, astenia, escasa ingesta oral y fiebre de sospechas. En este sentido se plante la realizacin de una
hasta 40 C, con tendencia al sueo. colonoscopia para el estudio diagnstico.
A su ingreso presenta una TA 90/40 mm Hg, FC 110 lpm Reinterrogando y profundizando en la anamnesis, la pa-
y T 39,8C. ciente refera una prdida de aproximadamente 20 Kg de
A la exploracin se mostraba consciente y orientada, aun- peso en los ltimos 3 meses, con un IMC actual de 23,9
que tendente al sueo. El resto de la exploracin neurol- Kg/m2 (IMC previo de 31,2 Kg/m2 con una prdida de
gica y cardiorrespiratoria fue normal. El abdomen estaba peso corporal en ese perodo de 23,6 %). Asimismo previo
blando y no distendido, con abundantes ruidos peristlti- al inicio del cuadro diarreico, presentaba profusa sudora-
cos y doloroso a la palpacin selectiva en FID, sin signos cin nocturna, que interpretaba como fiebre que sin em-
de irritacin peritoneal ni hepatoesplenomegalia. El tacto bargo nunca objetiv.
rectal mostr una ampolla rectal vaca, sin restos de san- Finalmente se inform desde el laboratorio de Microbio-
grado. No presentaba adenopatas palpables ni alteracio- loga el crecimiento de en coporcultivo de Mycobacterium
nes cutneas. tuberculosis, en la muestra de heces obtenida en el primer
Se realiz radiografa de trax en proyecciones poste- ingreso hospitalario (22 das antes).
roanterior y lateral sin alteraciones. En anlisis de sangre Se inici tratamiento con pauta estndar de Isoniazida (H),
destacaban: creatinina 1.22 mg/dl, urea 46 mg/dl, K 2.5 Rifampicina (R) y Pirazinamida (Z) (Rifater ) , ajustada a
mEq/l, Na 125 mEq/L, PCR 116 mg/l, amilasa 200 UI/l peso. Una semana despues, la paciente presento evidente
, GOT 520 UI/l, GPT 180 UI/L, FA 302 UI/l, GGT 380 mejora del estado general y nutricional, qued afebril con
UI/l y bilirrubina en rango normal. La cifra de leucocitos resolucin del cuadro diarreico y del dolor abdominal, por
totales fue de 10.000/mm3 con 600 linfocitos/ mm3. El he- lo que se no se consider necesaria la realizacin de colo-
mograma mostraba una anemia leve 12,7 g/l con plaque- noscopia.
tas y coagulacin en rango normal. El estudio ecogrfico La paciente recibi un total de 2 meses con pauta de HRZ
no mostr alteraciones respecto al previo. Se obtuvieron y 7 meses de HR, con visitas peridicas a consultas exter-
hemocultivos en los que se aisl Salmonella typhi (R a cido nas, evolucin favorable, negativizacin de coprocultivo al
Nalidixco, CMI >16 mg/ml), por lo que se inici trata- mes de tratamiento, y sin evidencia de afectacin extrain-
miento antibitico con ceftriaxona durante 14 das. testinal.
El aislamiento de dos subtipos distintos de la misma es-
pecie, en dos ingresos consecutivos, en un corto intervalo Diagnstico final
de tiempo y sin mejora clnica, motiv la tipificacin del
coprocultivo del primer ingreso realizada con MALDI- Tuberculosis intestinal y fiebre tifoidea y desnutricin cal-
TOF que finalmente se identifica como Salmonella tiphy con rico proteica asociada.
el mismo antibiograma que el hemocultivo. A pesar del
tratamiento antibitico endovenoso, durante dos semanas Discusin
con negativizacin de hemocultivo a los 5 das, la paciente
no present mejora con fiebre diaria vespertina, diarrea y La Tuberculosis (TBC) intestinal es la sexta localizacin
dolor abdominal. extrapulmonar ms frecuente (1). La India y el sudeste
El diagnstico de gastroenteritis aguda en el primer ingre- asitico son consideradas zonas de alta endemia, con un
so no justificaba la cronologa clnica del cuadro. El hecho incremento de la incidencia en los ltimos aos (2), e in-
de aislar Salmonella tiphy en hemocultivos permiti corregir cluso en pases como Pakistn se trata de la localizacin
el diagnstico inicial que sin embargo segua sin explicar extrapulmonar ms frecuente(3). La pobreza, malnutri-
todo el cuadro presente, tanto por la duracin, como por la cin, hacinamiento, infeccin por el VIH y embarazo son
ausencia de respuesta al tratamiento con ceftriaxona. Las factores de riesgo para el desarrollo de TBC. La principal
resistencias adquiridas a cefalosporinas de tercera genera- especie es Mycobacterium tuberculosis y el mecanismo ms fre-
cin, aunque constituyen una posibilidad, son extremada- cuente es la reactivacin de un foco pulmonar latente con
mente raras. Se planteaba por lo tanto dos opciones: que ingestin de esputos que contengan bacilos. Tambin pue-
exista una coinfeccin por un germen no identificado o de ocurrir por la diseminacin hematgena y desde rga-
bien ampliar el estudio a causas no infecciosas, entre las nos adyacentes. En el caso de infeccin primaria, la especie
que habra que descartar procesos neoplsicos (neuroen- ms frecuente es Mycobacterium bovis por ingesta de leche no
docrinos, linfomas o adenocarcinoma), o autoinmunes pasteurizada. El genotipo de Mycobacterium tuberculosis tiene

160
CAPTULO 10
INFECCIONES TROPICALES
Y DEL VIAJERO

importancia clnica puesto que la subespecie predominan- ra semana de infeccin, aunque su sensibilidad es inferior
te en Asia se asocia a una mayor tendencia a la disemina- (8). Las tcnicas serolgicas, Test de Widal, carecen de fia-
cin y a resistencias a antituberculosos de primera lnea (4). bilidad en personas de rea endmica (9). El tratamiento
Se observa una mayor incidencia en pacientes jvenes por sistemtico con quinolonas no est actualmente aceptado
el mayor nmero de placas de Peyer. La vlvula ileocecal por la emergencia de cepas resistentes (10), de forma que
es la zona ms frecuentemente afectada, con el desarro- en reas donde exista una alta tasa de resistencias, es de
llo de granulomas intramurales, hasta la progresiva apari- eleccin otros antibiticos entre los que se incluyen la cef-
cin de sntomas, que son con mayor frecuencia: dolor en triaxona con una duracin, dependiendo de la severidad,
FID, fiebre, sudoracin nocturna, diarrea, estreimiento y de entre 7-14 das.
hemorragia gastrointestinal. El diagnstico de certeza re- En esta paciente, es probable que la situacin de inmu-
quiere la presencia de granulomas caseificantes con bacilos nodepresin celular inducida por el embarazo favoreciese
cido-alcohol resistentes (BAAR), mediante tincin de ZN una diseminacin de una infeccin tuberculosa latente o
en la histologa, y cultivo positivo en medios de micobacte- una infeccin diseminada de inicio, adquirida durante su
rias, o reaccin en cadena para la polimerasa (PCR) para viaje, dada la ausencia de afectacin clnica ni radiolgica
Mycobacterium tuberculosis complex positiva. El aislamiento de pulmonar, lo que explicara adems la alteracin de la bio-
micobacterias en una muestra de heces, con clnica com- qumica heptica al ingreso. Concomitantemente, y dada
patible, se considera como diagnstico probable. Las prin- la evolucin de la fiebre en la ltima semana previa al in-
cipales pruebas para el diagnstico son la TAC y la ileo-co- greso, es de suponer que se infectara por Salmonella tiphy en
lonoscopia para la toma de muestras histolgicas, siendo en el ltimo periodo de su estancia en la India. Es probable
ocasiones necesario recurrir a laparotoma. El diagnstico que ambas situaciones contribuyeran al aborto, ya que en
final supone un reto, por la escasa sospecha inicial frente ambas est descrito.
a otras patologas ms frecuentes en nuestro medio, con Es de importancia resaltar que la globalizacin junto con
los que es necesario realizar diagnstico diferencial, como los viajes intercontinentales han hecho posible que sea cada
las enfermedades inflamatorias intestinales y las neoplasias vez ms frecuente observar cuadros clnicos poco comunes
intestinales (5). La baciloscopia tiene una sensibilidad de en nuestro medio, incluso olvidados. No es una situacin
entre el 10-30%, mientras que el cultivo muestra una sen- infrecuente que en ocasiones estn producidos por varios
sibilidad superior aunque los resultados definitivos requie- microorganismos coexistentes, lo que hace que junto al
ren hasta las 3-4 semanas. Es por ello que se utilizan con desconocimiento de estas entidades se retrase su diagns-
cada vez ms frecuencia tcnicas de biologa molecular de tico, y con ello aumente la morbimortalidad. Es por ello
amplificacin de DNA de Mycobacterium tuberculosis complex que es de vital importancia mantener una sospecha clnica
mediante PCR, aunque presentan mayor coste y menor elevada, y comprobar la concordancia de las pruebas mi-
sensibilidad que el cultivo (6). El tratamiento de eleccin es crobiolgicas complementarias con la clnica, y cronologa
la combinacin de al menos tres frmacos antituberculosos del cuadro clnico presente.
de primera lnea, de eleccin HRZ durante 2 meses segui-
do de HR durante 4-7 meses ms. No obstante la duracin
total no esta bien establecida y puede variar entre los 6 y 12
meses. En ocasiones, es necesario el abordaje quirrgico
ante cuadros obstructivos o perforacin intestinal.
La fiebre tifoidea es causada por Salmonella tiphy y se ad-
quiere mediante la ingesta de agua o comida contamina-
da. India y el sudeste asitico son zonas endmicas, dnde
existe una creciente preocupacin por el aumento de es-
pecies resistentes a quinolonas (7). La resistencia al cido
nalidxico es un marcador de la sensibilidad de Salmonella
spp. a quinolonas. Su presentacin clnica es comn a otras
infecciones de rea tropical con fiebre, vmitos, diarrea y
hepatoesplenomegalia como datos ms caractersticos. El
diagnstico de eleccin es el aislamiento en hemocultivos,
con una sensibilidad entre el 40-60%, mientras que los
cultivos en heces y orina se obtienen despus de la prime-

161
CAPTULO 10
INFECCIONES TROPICALES
Y DEL VIAJERO

Bibliografa

1. World Health Organization. Improving the diagnosis


and treatment of smearnegative pulmonary and extra-
pulmonary tuberculosis among adults and adolescents.
Recommendations for HIV-prevalent and resour-
ce-constrained settings (WHO/HTM/TB/2007.379);
2007.
2. Das K, Ghoshat UC, Dhali GK, et al. Crohns disease
in India: a multicenter study from a country where tu-
berculosis is endemic. Dig Dis Sci 2009; 54:1099107.
3. Shaikh R, Khalid MA, Malik A, et al. Abdominal tu-
berculosis: profile of 26 cases. Pak J Surg 2008; 24:
2179.
4. Thwaites G, Caws M, Chau TTH, et al. Relationship
between Mycobacterium tuberculosis genotype and
the clinical phenotype of pulmonary and meningeal
tuberculosis. J Clin Microbiol 2008; 46:13638.
5. Almadi MA, Ghosh S, Aljebreen AM. Differentiating
intestinal tuberculosis from Crohns disease: a diagnos-
tic challenge. Am J Gastroenterol 2009; 104:100312
6. Donoghue HD, Holton J. Intestinal tuberculosis. Cu-
rrent Opinion in Infectious diseases 2009; 22:4906.
7. Renuka K, Sood S, Das BK, Kapil A.High-level ci-
profloxacin resistance in Salmonella enterica serotype
Typhi in India. J Med Microbiol 2005; 54:999-1000
8. World Health Organization Department of Vaccines
and Biologicals. Background document: the diagnosis,
prevention and treatment of typhoid fever. Geneva:
WHO, 2003:19-23.
9. Ochiai RL, Wang X, von Seidlein L, Yang J, Bhutta
ZA, Bhattacharya SK, et al. Salmonella paratyphi A
rates, Asia. Emerg Infect Dis 2005;11:1764-6
10. Thaver D, Zaidi AK, Critchley J, Madni SA, Bhutta
ZA. Fluoroquinolones for treating typhoid and para-
typhoid fever (enteric fever). Cochrane Database Syst
Rev 2006;(1):CD004530.

162
Varn natural de Mali con eosinofilia
e hipertransaminasemia
Cuenca Gmez, JA; Pousibet Puerto, J;
Rodrguez Rodrguez, JP; Salas Coronas, J.
Hospital de Poniente. El Ejido, Almera.

Caso clnico 125UI/L, FA 91 UI/L, resto normal.


Serologa: VIH, VHC y lues negativo. VHB: AgHBs -,
Antecedentes personales: Varn de 29 aos, natural de AcHBs -, AcHBc +.
Mali, que reside en Espaa desde hace 4,5 aos. No ha re- Parsitos en heces con tcnica de concentracin (Rit-
gresado a su pas de origen. Su ruta migratoria fue: Costa chie) 3 muestras (x3): Blastocystis hominis.
de Marfil, Libia, Mali, Argelia, Marruecos y Fuerteventu-
ra. Trabaja en los invernaderos. Convive con 8 personas en Diagnstico diferencial
una casa con buenas condiciones higinicas. Antecedentes
de malaria en repetidas ocasiones. No hbitos txicos ni El diagnstico diferencial lo planteamos por un lado en la
otros antecedentes de inters. No realiza tratamiento de eosinofilia y por otro en la hipertransaminasemia.
forma habitual. No alergias medicamentosas conocidas. La eosinofilia es uno de los sndromes ms frecuentes con
los que se enfrenta el clnico en pacientes con patologa
Enfermedad actual: es remitido desde Atencin Primaria tropical importada. Aunque hay que considerar como cau-
a la consulta de Medicina Tropical (UMT) para estudio de sas posibles los procesos alrgicos y de hipersensibilidad,
hipertransaminasemia. El paciente acudi a su mdico de las enfermedades inflamatorias sistmicas y las neopla-
cabecera por dolor abdominal que relacionaba con la in- sias (hematolgicas y de rgano slido), la etiologa ms
gesta de alimentos, con deposiciones normales, y que cedi frecuente en pacientes procedentes de regiones tropicales
con un tratamiento que no aporta. En esa visita, se le apli- (nuestro paciente procede de Mali) son las parasitosis, fun-
ca el protocolo de atencin inicial al inmigrante, que inclu- damentalmente las causadas por helmintos. La ausencia
ye la realizacin de un hemograma completo, bioqumica de sintomatologa especfica de las primeras causas hace
con perfil hepatorrenal, serologa de VHB, VHC, VIH y ms probable el origen parasitario. Ya en la anamnesis, el
sfilis, as como estudio de parsitos en heces y en orina (en paciente hace referencia a episodios de hematuria en la
sujetos procedentes de regiones endmicas de esquistoso- infancia, posiblemente debido a una esquistosomiasis uri-
miasis). Refiere tambin prurito de caractersticas inespec- naria, enfermedad endmica de los pases de frica Occi-
ficas, sin relacin con la ducha. Antecedente de hematuria dental. Entre las posibilidades a considerar estn:
en la infancia para la que recibi un tratamiento que no
recuerda, quedando desde entonces asintomtico. Nematodos, se incluyen:

Exploracin fsica: TA: 145/90, Peso: 60 kg. Buen estado 1. Geohelmintos (uncinarias, Trichuris trichuria, Ascaris lum-
general, bien nutrido. Cabeza y cuello: no adenopatas, ni bricoides, Strongyloides stercoralis).
muguet. ACR: tonos rtmicos sin soplos; murmullo vesi- 2. Filariosis (oncocercosis, filariasis linftica, gnero Man-
cular conservado sin ruidos patolgicos. Abdomen blando, sonella, Loa loa).
depresible, no doloroso, no se palpan megalias ni masas. 3. Y otros menos frecuentes como Toxocara o Anisakis.
MMII: no datos de filariasis; mnima micosis interdigital
entre 4-5 dedos de ambos pies. Cestodos: Tenia solium, Tenia saginata, Hymenolepis nana, Equi-
nococcus granulosus.
Pruebas complementarias iniciales:
Trematodos: Schistosoma haematobium, Schistosoma intercalatum
Hemograma: leucocitos 9120/microL (57,9% Neutr- y Schistosoma mansoni, Fasciola heptica.
filos, 33,6% linfocitos, 8,5% eosinfilos; 775 eosinofi-
los/mm3). Hb 15,7 mg/dl, Hto 46,5%, VCM 89,1; En relacin con la afectacin heptica, las posibles causas
HCM 30,1; Plaquetas 279.000/microL. a considerar en este paciente son:
Bioqumica: AST 93 UI/L, ALT 184 UI/L, GGT

163
CAPTULO 10
INFECCIONES TROPICALES
Y DEL VIAJERO

Hepatitis virales. La hepatitis crnica por VHB afec- Lesiones tumorales: el carcinoma hepatocelular es una
ta a ms de 350 millones de personas en el mundo, y entidad muy prevalente entre pacientes procedentes de
la prevalencia en la mayora de los pases de frica zonas endmicas por VHB, en los que la adquisicin
Subsahariana es superior al 8%. En nuestro caso, el de la enfermedad en la infancia permite la aparicin
AgHBs es negativo, pero habra que valorar la posi- de este tipo de tumores en pacientes de edad joven.
bilidad de una hepatitis B oculta (AcHBc+, AgHBs-, Otras posibilidades a considerar seran hepatitis de
carga viral positiva). Tambin habra que considerar origen autoinmune, medicamentoso, alcohol, de cau-
la hepatitis crnica por VHC, aunque la prevalencia sa metablica, granulomatosa o idioptica.
de la enfermedad es menor en pacientes procedentes
de esta zona. Las otras etiologas (VHA, VHE) se ma- Evolucin
nifiestan por hepatitis aguda, que parece razonable
descartar. Se prescribe tratamiento tpico con ciclopiroxolamina y se
Parasitosis. Entre las que pueden ocasionar afectacin he- solicita estudio segn el protocolo de estudio y tratamiento
ptica destacan la Fasciola heptica, Ascaris lumbricoides (ms de la eosinofilia de la UMT, as como ecografa abdominal
asociada a colangitis, colecistitis, colestasis), y la esquis- y vesical, y carga viral de VHB para descartar hepatitis B
tosomiasis. La esquistosomiasis, sobre todo producida oculta. Los resultados se muestran a continuacin:
por S. mansoni, puede dar lugar hipertensin portal presi-
nusoidal, caracterizada por intensa fibrosis periportal, hi- Hemograma: leucocitos 5.430/microL (4,2% eosin-
peresplenismo y varices esofgicas, conservando hasta es- filos; 230 eosinfilos/mm3). Hb 14,4 mg/dl. Plaquetas
tadios finales de la enfermedad la funcin hepatocelular. 189.000/microL.
Rasgo drepanoctico (heterocigoto para HbS; HbS
37%).
Bioqumica: AST 48 UI/L, ALT 74 UI/L, GGT
81UI/L, FA 119 UI/L, resto normal, incluyendo me-
tabolismo del hierro, perfil renal y lipdico. IgE: 243
UI/mL.
Orina: sedimento normal.
Coagulacin: TP 74%, INR: 1.2, TTPa 27.8 sg.
Serologa: Strongyloides (ELISA): NEGATIVA; Schistoso-
ma (ELISA): MUY POSITIVA. VHB: AgHBs -,
PCR VHB: POSITIVA; cuantificacin ADN VHB:
1.000 UI/mL.
Imagen 1. Hgado de ecoestructura heterognea, con intensa fibrosis de predo- Estudios parasitolgicos:
minio periportal
1. Parsitos en heces con tcnica de concentracin
(x3): NEGATIVOS.
2. Estudio de microfilarias en sangre (test de Knott):
NEGATIVO.
3. Parsitos en orina con tcnica de concentracin:
NEGATIVOS.
Alfa-fetoprotena: 1,4 ng/mL
Rx de trax y abdomen: sin alteraciones.
Mantoux: 10 mm.
Ecografa abdominal y vesical: hgado de tamao nor-
mal con alteracin de la ecoestructura. Hiperecogeni-
cidad del tejido periportal con intensa fibrosis (Imagen
1). Porta aumentada de calibre (15 mm) y bazo de 12
cm. Vejiga de paredes normales.

Imagen 2. Huevo de esquisistosoma (flecha) en biopsia de mucosa rectal.

164
CAPTULO 10
INFECCIONES TROPICALES
Y DEL VIAJERO

Ante el diagnstico de hepatitis B oculta y signos de hepa- de fiebre, cefalea, mialgias, diarrea, hepato-espleno-
topata crnica, con datos muy sugerentes de parasitacin megalia, adenopatas, tos con infiltrados pulmonares y
por Schistosoma, se solicita biopsia rectal que muestra mu- eosinofilia perifrica en el hemograma.
cosa con presencia de huevos de esquistosomas (Imagen 2).
Posteriormente, se decide realizacin de biopsia heptica La fase crnica va a depender fundamentalmente de
para valorar el grado de actividad de la enfermedad, con la carga parasitaria, de la duracin de la parasitacin
el siguiente resultado: Parnquima heptico reemplazado y de la especie de esquistosoma causante de la parasi-
por ndulos de pequeo tamao con tractos conectivos tacin. La podemos dividir bsicamente en dos enti-
fibrosos gruesos, con infiltrado inflamatorio linfohistioci- dades:
tario, necrosis parcelar erosiva y acmulos de eosinfilos. 1. Afectacin hepato-esplnica causada por las espe-
Cambios regenerativos en hepatocitos. Diagnstico final cies S. mansoni, S. japonicum, S. mekongi; que puede ocasio-
compatible con hepatopata crnica de origen viral (gra- nar hipertensin portal presinusoidal, hepato-espleno-
do 2 de actividad, grado 3 de fibrosis), y posiblemente se- megalia, formacin de varices esofgicas, y en estados
cundaria a esquistosomiasis aunque no se han visualizado avanzados, cirrosis heptica. El dao heptico causa-
parsitos. do por la infeccin por esquistosomiasis es debido a la
Se solicita serologa de VHDelta (AcVHD POSITIVA, ocupacin de los vasos pre-sinusoidales hepticos por
PCR VHD: NEGATIVA) y gastroscopia, a la que se niega huevos de esquistosomas y a la posterior proliferacin
el paciente. de clulas endoteliales en estos vasos. La proliferacin,
Ante los hallazgos descritos se inicia tratamiento con prazi- junto con una respuesta inmunolgica guiada por los
quantel (40 mg/Kg de peso, 1 da y con tenofovir 300mg/ linfocitos Th2, conduce a la formacin de granulomas
da, presentando actualmente buena adherencia al trata- alrededor del huevo. La formacin de mltiples gra-
miento, siendo la ltima carga viral de VHB negativa. nulomas produce fibrosis periportal, tambin conocida
como fibrosis 'pipestem'. La fibrosis, junto con el dao
Diagnstico final vascular y la obstruccin de los vasos, se traduce en
el desarrollo de la hipertensin portal pre-sinusoidal.
Hepatopata crnica secundaria a esquistosomiasis y he- sta es responsable de las diversas manifestaciones de
patitis B oculta la enfermedad, incluyendo la esplenomegalia y la for-
macin de circulacin colateral porto-sistmica, vari-
Discusin ces esofgicas en particular. La gravedad de todo este
proceso depender en ltima instancia del nmero de
El esquistosoma en un helminto trematodo del que se han huevos de esquistosomas que pasan al sistema venoso
descrito 11 especies, aunque solo 5 (S. mansoni, S. haemato- portal heptico. Esto significa que cuanto mayor es la
bium, S. intercalatum, S. japonicum y S. mekongi) afectan al ser carga parasitaria, mayor ser la cantidad de huevos en
humano. De ellos, los ms prevalentes en frica Subsaha- el sistema pre-sinusoidal y mayor probabilidad de en-
riana son el S. mansoni y el S. haematobium. fermedad heptica ms grave.
La parasitacin por este helminto puede producir manifes- 2. Afectacin de vas urinarias que se debe principal-
taciones agudas o crnicas: mente al S. haematobium. En este caso, el principal rga-
no afectado ser la vejiga con la formacin de plipos
La fase aguda se suele dar en viajeros o inmigrantes vesicales, placas calcificadas e incluso se ha asociado al
que visitan zonas endmicas de esquistosomiasis y se desarrollo de cncer vesical de tipo escamoso.
baan en lagos o ros de agua dulce. La primera ma-
nifestacin aparece alrededor de las 24 horas tras la En el caso de inmigrantes procedentes de zonas endmicas,
exposicin y es la denominada dermatitis del nadador. como sucede con nuestro paciente, nos vamos a encontrar
Se caracteriza por una dermatitis pruriginosa localiza- con la fase crnica de la enfermedad. Esta fase puede pa-
da de forma preferente en miembros inferiores. Puede sar inadvertida para el paciente y para los mdicos duran-
durar hasta una semana, pero se autolimita sin necesi- te aos porque la persona suele estar asintomtica y con
dad de medicacin. Posteriormente se puede producir analticas normales (sin eosinofilia ni datos de afectacin
la denominada fiebre de Katayama o esquistosomiasis heptica). Es por ello fundamental realizar un cribado en
aguda propiamente dicha. Aparece entre la 4 y la 8 este tipo de pacientes, por la posibilidad de que se pro-
semana post-exposicin. Se caracteriza por un cuadro duzcan complicaciones a largo plazo. Este cribado se rea-

165
CAPTULO 10
INFECCIONES TROPICALES
Y DEL VIAJERO

liza fundamentalmente mediante el examen microscpico Bibliografa


de los huevos del parsito en heces y orina. Otros medio
tambin utilizado es la serologa (ELISA, IFI, hemagluti- 1. World Health Association. [acceso el 05-04-2015].
nacin) para esquistosoma, pero esta prueba no distingue http://www.who.int/mediacentre/factsheets/fs204/
entre infeccin pasada o activa; y adems permanece po- en/
sitiva mucho tiempo despus de haber recibido el paciente 2. Global Atlas of Helminth Infections. [acceso el 05-04-
tratamiento, por lo que no es de utilidad para valorar la 2015]. Disponible en: http://www.thiswormyworld.
efectividad del tratamiento y por tanto, la curacin. Por l- org/maps/by-worm/schistosomiasis.
timo, ante alta sospecha de parasitacin por este helminto 3. Colley DG, Bustinduy AL, Secor WE, King CH. Hu-
y no haber encontrado huevos en heces u orina, se puede man schistosomiasis. Lancet2014;383: 2253-64.
realizar biopsias de mucosa rectal o vesical para estudio 4. El-Sayed HF, Abaza SM, Mchanna S, Winch PJ. The
anatomo-patolgico. En este caso veremos los huevos de prevalence of hepatitis B and C infections among im-
esquistosomas rodeados por un infiltrado eosinoflico. migrants to a newly reclaimed area endemic for Schis-
El tratamiento de esta helmintiasis se realiza fundamen- tosoma mansoni in Sinai, Egypt. Acta Trop. 1997;68:
talmente con prazicuantel, aunque en algunas regiones, 229-37
como por ejemplo en Brasil, se utiliza la oxamniquina. En 5. Silva JL, Souza VS, Vilella TA, Domingues AL, Col-
la fiebre de Katayama se aadirn una tanda corta de cor- ho MR. HBV and HCV serologial markers in patients
ticoides orales. with the hepatosplenic form of mansoni schistoso-
La hepatitis crnica por VHB es una enfermedad muy fre- miais. Arq Gastroenterol. 2011;48:124-30.
cuente en los pases de frica Subsahariana, siendo res- 6. Andrade JR, Silva LD, Guiamares CM, Basset-
ponsable de una gran morbimortalidad en esta poblacin. ti-Soares E, Cambraia RD, Teixeira R. Chronic he-
La hepatitis B oculta debe sospecharse en los pacientes con patitis B and liver schistosomiasis: a deleterious asso-
AcHBc como nico marcador positivo de la infeccin y da- ciation. Trans R Soc Trop Med Hyg 2014;108:159-64.
tos de hepatopata (elevacin de transaminasas o ecografa 7. Andrade ZA. Schistosomiasis and liver fibrosis. Parasi-
heptica patolgica). En el caso de que nos encontrsemos te Immunol.2009;31:656-63
un paciente con AcHBc como nico marcador de hepatitis 8. Vzquez Villegas J, Galindo Pelayo JP, Lujn Jim-
B sin datos de hepatopata asociados, habra que pensar nez R, Gmez Gmez E, Ramos Muoz JA, Loren-
como primera opcin que el paciente ha pasado una hepa- te-Serna J et al. Atencin inicial al paciente inmigran-
titis B en algn momento de su vida, habiendo perdido la te en Atencin Primaria. Medicina de familia (And).
inmunidad posteriormente. A estos pacientes sera preciso 2000;1:162-68.
vacunarlos para conseguir una inmunidad completa. 9. Raimondo G, Allain JP, Brunetto MR, Buendia MA,
En el caso que presentamos, existe eosinofilia se justificara Chen DS, Colombo M, et al. Statements from the
por la parasitacin por S. mansoni. Pero para explicar la hi- Taormina expert meeting on occult hepatitis B virus
pertransaminasemia tenemos una doble causa, por un lado infection. J Hepatol 2008;49:6527.
la presencia de hepatitis crnica por el VHB y por otro la 10. Said ZN. An overview of occult hepatitis B virus infec-
parasitacin por S. mansoni. Diversos estudios en zonas de tion. World J Gastroenterol 2011;17:1927-38.
alta endemicidad para la infeccin por VHB y/o infeccin
por el virus de la hepatitis C (VHC) y la parasitacin por
S. mansoni, muestran que es posible que la co-infeccin, ya
sea por VHB o VHC, junto con S. mansoni pueden empeo-
rar de forma significativa la evolucin de la enfermedad
hepato-esplnica.
Por todo ello, la importancia de este caso radica en que la
eosinofilia y el aumento de transaminasas son dos motivos
de consulta frecuentes en el estudio de pacientes inmigran-
tes y en concreto en pacientes subsaharianos. Ambos datos
analticos pueden ser debido a patologas potencialmente
graves para el paciente y con gran morbi-mortalidad. De
ah la relevancia de conocer su correcto manejo y orienta-
cin clnica.

166
Varn joven con afectacin pulmonar y eosinofilia
Hidalgo Prez, PV; Snchez Lpez, P;
Martnez Mateu, C; Lazo Torres, AM.
Complejo Hospitalario Torrecrdenas. Almera.

Caso clnico

Anamnesis

Hombre de 29 aos, natural de Senegal y residente en Al-


mera desde hace 7 aos, sin viajes posteriores de regreso a
su pas de origen. Jornalero agrcola. Negaba antecedentes
personales de inters, hbitos txicos y toma de tratamien-
to habitual.
Fue remitido a consulta de Medicina Interna por iniciar,
hace 2 meses, clnica de dificultad respiratoria y opresin
retroesternal a la inspiracin profunda. Refera adems tos
escasamente productiva, sibilantes y, en ocasiones, sensa-
cin distrmica no termometrada. Durante ese tiempo,
le pautaron de forma ambulatoria varios frmacos para Imagen 1. La Radiografa de trax PA muestra un discreto infiltrado bibasal (ms
medial en base derecha y ms perifrico en izquierda).
alivio sintomtico (analgsicos, mucolticos, antitusgenos
e inhaladores) y recibi tratamiento con levofloxacino oral
durante 1 semana, pero con ninguna de estas medidas pre- dl, triglicridos 51 mg/dl, GOT 34 UI/L, GPT 32 UI/L,
sent mejora clnica. De hecho, se quejaba de aumento de fosfatasa alcalina 80 UI/L, sodio 137 mEq/L, potasio 4,2
las sibilancias tras la inhalacin de fluticasona/salmeterol. mEq/L, PCR 0,10 mg/dl, CPK 389 UI/L. Marcadores
No asociaba sntomas a otros niveles, salvo flatulencia y cardiacos negativos. Hierro 120 ug/dl, ferritina 138 ng/
estreimiento de forma intermitente. ml, Inmunoglobulina E 1523 UI/ml, VSG 22 mm. Hemo-
grama: hemoglobina 16,6 g/dl, leucocitos 7490 mm3 (37%
Exploracin fsica neutrfilos, 21.90% eosinfilos), eosinfilos totales 1640/
mm3 (dicho valor se confirm con una segunda determina-
Estado general conservado, permaneciendo en todo mo- cin analtica). Coagulacin bsica con parmetros dentro
mento normotenso y afebril. Adecuada coloracin, hidra- de la normalidad. Sistemtico de orina sin hallazgos pato-
tacin y perfusin. No adenopatas en territorios accesibles lgicos. Serologas: VIH negativo, Hepatitis C: anti-HCV
ni evidencia de lesiones muco-cutneas. Eupneico en re- negativo, Hepatitis B: HBsAg, HBcAc y HBeAc positivos.
poso, con buena tolerancia al decbito y saturacin ba- Mantoux negativo. Gasometra arterial basal: pH 7,40,
sal de oxgeno del 97%. No ingurgitacin yugular. Tonos pO2 75 mmHg, pCO2 38 mmHg, satO2 96%, HCO3 25
cardiacos rtmicos a 80 lpm sin soplos ni roces audibles. mEq/l. Espirometra con datos de leve obstruccin area
Hipoventilacin pulmonar con algunos sibilantes aisla- de vas inferiores (FEV1: 70%, FEV1/FVC: 65%, FEF 25-
dos. Abdomen blando y depresible con leve molestia a la 75: 75% del valor terico) con cada brusca de la curva
palpacin profunda en epigastrio, sin palpar masas o vis- flujo-volumen con concavidad hacia arriba tras aparicin
ceromegalias. No edemas ni signos de patologa vascular de flujo espiratorio mximo. En la radiografa de trax se
en miembros inferiores. Resto de la exploracin fsica, sin objetivaban pequeos infiltrados focales y abigarrados en
hallazgos relevantes. ambas bases pulmonares, siendo de localizacin ms peri-
frica en el hemitrax izquierdo (imagen 1).
Pruebas complementarias

Bioqumica: glucosa 83 mg/dl, urea 26 mg/dl, creatini-


na 0,83 mg/dl, cido rico 5,3 mg/dl, colesterol 166 mg/

167
CAPTULO 10
INFECCIONES TROPICALES
Y DEL VIAJERO

Diagnstico diferencial Evolucin

Presentamos el caso clnico de un varn sin historia previa El paciente fue tratado con Ivermectina 200 mcg/kg du-
de hiperreactividad bronquial, que a los 29 aos debuta rante 2 das, presentando mejora clnica y posterior desa-
con dificultad respiratoria, sibilancias y eosinofilia. Las pa- paricin de la eosinofilia. No ha vuelto a presentar sinto-
tologas que cursan con eosinofilia e infiltrados pulmonares matologa respiratoria.
se agrupan en las denominadas eosinofilias pulmonares.
Las entidades que conforman este grupo, permitieron rea- Discusin
lizar el correcto diagnstico diferencial (Cuadro 1).
La sintomatologa se haba iniciado 2 meses atrs y no do- Las eosinofilias pulmonares son un grupo heterogneo de
cumentaba historia previa de alergias ni toma de frma- enfermedades caracterizadas por infiltrados pulmonares y
cos u otras drogas, por lo que inicialmente se excluy que aumento del nmero de eosinfilos en sangre perifrica.
pudiera ser secundario a una reaccin de hipersensibili- En ausencia de este ltimo requisito, la biopsia pulmonar
dad (primera causa de eosinofilia en la poblacin general). y/o lavado broncoalveolar pueden utilizarse para comple-
Al ser procedente de una regin tropical, la posibilidad tar el diagnstico (1). Dentro de las eosinofilias pulmona-
de una helmintiasis pareca ms razonable y se consolid res, encontramos un subconjunto denominado sndrome
de Lffler o Loeffler. Fue descrito por primera vez en 1956
1. Frmacos y toxinas por Wilhelm Lffler (3), quien identific pacientes con eo-
- AINES.
- Antibiticos: nitrofurantoina, minociclina, sulfamidas, ampicilina, daptomicina.
sinofilia y lesiones pulmonares transitorias causadas por
- Fenitona. Ascaris lumbricoides. Posteriormente tambin se describi
- L-triptofano.
secundarias a Strongyloides stercoralis, Ancylostoma duodenale y
2. Helmintos y hongos.
Necator americanum (2). A pesar de ser un trmino acuado
- Paso transpulmonar de larvas (sd. Lffler o Loeffler): Ascaris lumbricoides,
ancyilostoma, Strongyloides stercolaris. para describir una neumona eosinoflica en el contexto
- Invasin del parnquima pulmonar, en su mayora helmintos. de una infeccin por parsitos, actualmente algunos au-
- Siembra hematgena: triquinosis, estrongiloides diseminada, larva migrans,
esquistosomiasis. tores utilizan este trmino sin importar el factor desenca-
- Eosinofilia pulmonar tropical: Wuchereria bancrofti, Brugia malayi. denante. En nuestro caso, fue secundario a la infeccin de
- Aspergilosis broncopulmonar alrgica.
Strongyloides stercoralis, que es una parasitosis endmica en
3. Enfermedades inmunolgicas no alrgicas: sndrome de Churg-Strauss,
sarcoidosis, granulomatosis de Wegener. regiones tropicales y subtropicales, cuya importancia radi-
4. Idiopticas: Neumona eosinoflica aguda o crnica. Sndrome hipereo- ca en su capacidad exclusiva de multiplicarse dentro de su
sinoflico idioptico. Enfermedad pulmonar idioptica.
hospedador durante aos. Puede cursar de forma silente,
5. Otros: neoplasias (enfermedad de Hodgkin), infecciones por no helmintos
(histoplasmosis, coccidioidomicosis).
pero cuando provoca sintomatologa lo ms frecuente es
dolor abdominal y episodios de diarrea alternando con es-
Cuadro 1. Causas de eosinofilia pulmonar. treimiento (4). A nivel respiratorio, suelen presentar tos
irritante y molestia retroesternal que se agrava con la tos
como nuestra primera opcin. Adems, las causas de ori-
gen idioptico suelen constituir un diagnstico por exclu-
sin; y el resto de patologas (enfermedades autoinmunes,
neoplasias y otras infecciones no helmnticas), eran poco
probables dentro del contexto epidemiolgico, y sobre
todo clnico, del paciente.

Diagnstico final

Ante nuestra sospecha clnica inicial, se solicitaron 3 mues-


tras para examen de parsitos en heces, visualizndose al mi-
croscopio larvas de Strongyloides stercolaris en la tercera muestra
enviada (imagen 2). Tambin se confirm mediante ELISA
IgG Ac. Strongyloides con resultado muy positivo (ndice: 10,
positivo>1.1). El diagnstico final fue sndrome de Lffler se-
cundario a parasitacin por Strongyloides stercolaris. Imagen 2. Larva de Strongyloides Stercolaris.

168
CAPTULO 10
INFECCIONES TROPICALES
Y DEL VIAJERO

o la respiracin profunda. En algunos casos, la estrongiloi-


diasis puede desarrollar una clnica que se confunde con
asma pero que, paradjicamente, empeora con el uso de
corticosteroides (4,5). La radiografa de trax suele mos-
trar infiltrados migratorios que pueden variar de tamao,
siendo ms evidentes cuando la eosinofilia supera el 10 por
ciento. Para entender su patogenia es fundamental cono-
cer su ciclo de vida (5). Esta parasitacin comienza con
el contacto de la piel humana con larvas filariformes que
se encuentran en el suelo u otros materiales contaminados
con heces humanas. stas migran va hematgena a los
pulmones donde penetran en los sacos alveolares, ascien-
den por el rbol traqueo-bronquial y son deglutidas hasta
llegar al duodeno y yeyuno. A diferencia de otros helmin-
tos, pueden completar su ciclo dentro del husped humano
y volver a va hematgena atravesando la mucosa del co-
lon, aumentando progresivamente su carga parasitaria. De
este modo, si la respuesta inmune se mantiene estable, la
parasitacin puede persistir durante aos y surgir la clni-
ca tiempo despus de la infeccin inicial, como ocurri en
nuestro paciente. En cambio, si la respuesta inmune falla
(ej. infeccin por VIH) puede diseminarse a otros rganos
y desarrollar una entidad potencialmente grave, denomi-
nada sndrome de hiperinfeccin (5,6).

Bibliografa

1. Cottin V, Cordier JF. Eosinophilic pneumonias. Aller-


gy. 2005; 60: 841-57.
2. Ottesen EA, Nutman TB. Tropical pulmonary eosi-
nophilia. Annu Rev Med. 1992; 43: 417-24.
3. Lffler W. Transient lung infiltrations with blood eosi-
nophilia. Int Arch Allergy Appl Immunol. 1956.
4. Wilson ME, Weller PF. Eosinophilia. In: Guerrant RL,
Walker DH, Weller PF, eds. Tropical Infectious Disea-
ses: Principles, Pathogens and Practice. 3th ed. Phila-
delphia: Churchill-Livingstone Elsevier. 2011; p. 939.
5. Siddiqui, AA, Genta, RM, Berk, SL. Strongyloidiasis.
In: Guerrant RL, Walker DH, Weller PF, eds. Tropical
Infectious Diseases: Principles, Pathogens and Practi-
ce. 3th ed. Philadelphia: Churchill-Livingstone Else-
vier. 2011; p.805.
6. Lam CS, Tong MK, Chan KM, Siu YP. Disseminated
strongyloidiasis: a retrospective study of clinical course
and outcome. Eur J Clin Microbiol Infect Dis. 2006;
25: 14-8.

169
Varn subsahariano con lesin nasal
de evolucin trpida
Martnez Mateu, C; Hidalgo Prez, PV;
Glvez Contreras, MC; Ortega Ramrez, JR.
Complejo Hospitalario Torrecrdenas. Almera

Caso clnico

Varn de 34 aos natural de Gambia que acude por pri-


mera vez en noviembre de 2012 a consultas externas de
Enfermedades Infecciosas, derivado desde Otorrinolarin-
gologa (ORL) por lesin nasal y pilares amigdalinos com-
patible con Rinoescleroma.
Entre los antecedentes personales destaca la presencia,
desde los 20 aos, de una herida en la nariz, en la par-
te superior del tabique nasal, que supuraba contenido sin
olor ftido, tratada en su pas tard en cicatrizar unos dos
meses aos despus reaparece rinorrea y sensacin de obs-
truccin nasal. Presentaba, adems, temblores nocturnos
ocasionales desde haca 3-4 aos que interpretaba como
fiebre. Figura 1. Lesin proliferativasubepitelialcon refuerzo estroma fibroso ygran
densidad celular inflamatoria linfoplasmocitaria rica en histiocitosde citoplasma
El paciente era natural de Gambia, con ruta migratoria: claro vacuolado(HE 5x).
Gambia-Libia-Italia-Espaa (2010). Vive con 4 personas
en condiciones higinico-sociales aceptables. Sin alergias
medicamentosas conocidas.
Acude a consulta, remitido desde ORL, por lesin nasal
y en techo de pilares amigdalinos, de las que se tomaron
biopsias y donde se demostr, por estudio anatomopato-
lgico (Figuras 1 y 2), la presencia de clulas de Mikulicz
que albergaban estructuras bacterianas coco-bacilares, y
sospecha diagnstica de rinoescleroma sin diagnstico
microbiolgico. En el momento de esta primera visita, el
paciente presentaba rinorrea permanente y sensacin de
obstruccin nasal que le impeda respirar correctamente.
A la exploracin fsica, se evidenciaba cicatriz en zona na-
sal superior. Ausencia de vula. Amgdalas palatinas pe-
queas. Resto de exploracin sin hallazgos significativos.
Figura 2. A, Fase proliferativa/granulomatosa con denso infiltrado crnico
linfoplasmocitario rico en clulas de Miculicz (HE10x) B, Clulas histiocitarias de
Pruebas complementarias: citoplasmas claroscon estructuras bacilares intracelularesclulas de Mikulicz
(HE40x) C, Infiltrado rico en clulas plasmticas madurascon inmunoexpresin
positiva con CD138 (CD13810x) D, Bacilos intracitoplasmticos en clulas de
Hemograma: Hemoglobina 14g/dL, VCM 99, Leu- Mickuliz puestos con mayor realce con tcnica de plata (Warthin-Starry 40x).
cocitos 5.48x103/L (Neutrfilos 2.99x103 /L), Pla-
quetas 187x103 /L. VSG 4 mm/h. sAg negativo AntiHBc positivo, AntiHBs positivo (32).
Bioqumica: Perfil hepatorrenal normal. PCR 0.00 Cultivo de esputo: Desarrollo de flora orofarngea. No
mg/dL. Protenas totales 7.58 g/dL. Antiestreptolisina se observan BAAR. No desarrollo de micobacterias.
55 U/mL, Factor Reumatoide 9,7 U/mL. Radiografa de trax: ndice cardiotorcico normal.
Serologas: Negativa para Brucella, Citomegalovirus, No imagen de infiltrado ni condensaciones.
Les, VIH y Hepatitis C. IgG 3,6 para virus de Eps- TAC de senos paranasales (Figura 3): Ocupacin por
tein-Barr, IgG 354 e IgM 0,65 para Toxoplasma. HB- tejidos blandos de ambas fosas nasales, provocando ex-

170
CAPTULO 10
INFECCIONES TROPICALES
Y DEL VIAJERO

pansin de dichas fosas, as como desmineralizacin y


prdida de definicin de los cartlagos de los cornetes.
Tabique nasal desplazado hacia la izquierda. Lesiones
polipoideas.

Se remite de nuevo a ORL para limpieza quirrgica y


toma de muestras para estudio microbiolgico y el pacien-
te no vuelve a consulta hasta 2015.
Figura 3. Tomografa computerizada. Planos axiales de senos paranasales.
Masa de material isoatenuado con los tejidos blandos en ambas fosas nasales,
Diagnstico diferencial con extensin al orificio nasal externo y asimetra en la neumatizacin de ambos
agujeros.

El diagnstico diferencial inicial incluira infecciones por


hongos, enfermedades granulomatosas del tipo tuberculo- El agente etiolgico implicado es Klebsiella rhinoscleromatis,
sis, sarcoidosis, lepra, vasculitis, granulomatosis de Wege- un bacilo gram negativo encapsulado, que puede formar
ner o de carcinoma verrucoso as como carcinoma basoce- esporas en circunstancias anaerobias y excretar una exoto-
lular y otras infecciones como leishmaniasis mucocutnea, xina que hace progresar el escleroma en ausencia de ba-
rinoesporidiosis, esporotricosis, blastomicosis y paracocci- cilos viables. El periodo de incubacin y la contagiosidad
diodomicosis. no se conocen con precisin, aunque suele haber antece-
dentes de exposicin prolongada. La infeccin comienza
Evolucin en la mucosa nasal, subepitelial, desde donde se extiende
a estructuras vecinas. Si bien se crea que su afectacin se
El paciente no acudi a revisin, ya que tuvo que despla- limitaba a nariz y tracto respiratorio superior, se han des-
zarse por motivos laborales y se perdi el seguimiento. Tres crito casos donde hay afectacin de faringe, nasofaringe,
aos despus reaparece en consulta con el mismo proble- cavidad oral, senos paranasales (sobre todo el seno maxi-
ma. No est claro si ha seguido tratamiento. lar), rbita (que puede acarrear una invasin intracraneal),
Ha sido valorado de nuevo por ORL, tomando en esta laringe, trquea y bronquios. La afectacin nasal aparece
ocasin muestras para estudio microbiolgico, donde se en el 95-100% de los casos.
aslan Staphylococcus aureus y Klebsiella rhinoescleromatis. Culti- Los sntomas pueden aparecer hasta 10 aos antes de que
vo negativo para micobacterias. Ambos patgenos son sen- se realice un diagnstico firme y pueden ir, segn la locali-
sibles, segn antibiograma, a Ciprofloxacino, por lo que se zacin afecta, desde obstruccin nasal leve hasta disfagia o
decide tratamiento con este antibitico, a dosis de 500mg dificultad respiratoria franca, incluso asfixia.
cada 12 horas durante 6 meses, seguimiento estrecho con Se han descrito varios factores predisponentes como, por
A. Primaria para vigilar tolerancia y adherencia al mismo. ejemplo, unas condiciones higinicas deficientes, vivir en
zonas rurales superpobladas, hacinamiento o estados nu-
Diagnstico final tricionales carenciales, como anemia ferropnica, aunque
debe de coexistir una cierta susceptibilidad por parte del
Los datos tanto microbiolgicos como anatomopatolgicos husped.
caractersticos apuntan efectivamente hacia un Rinoescle- Podemos distinguir tres etapas progresivas, aunque solapa-
roma. bles, segn la clnica del rinoescleroma:
La primera etapa, exudativa o catarral, caracterizada por
Discusin la abundancia de PMN y restos celulares. Los sntomas
imitan a un resfriado comn, aunque con frecuencia, los
El rinoescleroma en una infeccin crnica granulomatosa pacientes presentan adems una secrecin nasal purulenta
rara que es un diagnstico a tener en cuenta en pases sub- y maloliente. Suele durar varios meses y el diagnstico es
desarrollados o pacientes inmigrantes de dichas regiones difcil, de hecho es en esta etapa donde ms frecuentemen-
que presenten inflamacin pseudoneoplsica con obstruc- te se demora. Tanto el examen como la biopsia pueden ser
cin crnica de vas respiratorias altas. inespecficos en esta etapa.
Es endmica en frica, Oriente medio, Amrica central La segunda etapa es de proliferacin (hipertrfica, granu-
y Sudamrica, Europa central y del este, China, India e lomatosa), donde aparecen ndulos granulomatosos, de-
Indonesia. formidades y dificultad respiratoria por la obstruccin de

171
CAPTULO 10
INFECCIONES TROPICALES
Y DEL VIAJERO

las cavidades nasales. Esta etapa dura de meses a aos y los 3. Abalkhail A,Satti MB,Uthman MA,Al Hilli F,Darwi-
sntomas estn relacionados con las estructuras afectadas, sh A,Satir A. Rhinoscleroma: a clinicopathological
normalmente asociadas a la obstruccin nasal. Se observa- study from the Gulf region. Singapore Med J. 2007;
rn linfocitos, abundantes clulas plasmticas con cuerpos 48: 148-51.
de Russell y las denominadas clulas de Mikulicz, en cuyo 4. Hart AC, Rao KS.Rhinoscleroma. J Med Microbiol.
citoplasma hay granulaciones gramnegativas, caractersti- 2000; 49: 3956.
cas del rinoscleroma. 5. Talwar A, Patel N, Chen L, Shah R, Margouleff D.
La tercera etapa sera la cicatricial, caracterizada por la Rhinoscleroma of the tracheobronchial tree: bron-
formacin de tejido fibrtico, con gran cantidad de tejido choscopic, PET, and CT correlation. Indian J Chest
conectivo y escasez de clulas de Mikulicz y clulas plas- Dis Allied Sci. 2008; 50: 2258.
mticas. 6. Kim RN, Han J, Kwon YT. Nasal Rhinoscleroma in
El diagnstico de rinoescleroma se basa en el aislamiento, a nonendemic area: a case report. J Korean Med Sci.
bien en una muestra de tejido nasal o en hemocultivo de 2003; 18: 4558.
Klebsiella rhinoscleromatis y en la demostracin de clulas de 7. Razek AAKA, Elasfour AA.MR Appearance of Rhi-
Mikulicz en el estudio histolgico, si bien slo el 50-60% noscleroma.AJNR Am J Neuroradiol. 2008; 20: 575
de los casos se confirman con estos mtodos. 8.
En el TAC aparecern masas de partes blandas homog- 8. Mukara BK, Munyarugamba P, Dazert S, Lhler
neas e irregulares, con deformacin nodular, que no real- J. Rhinoscleroma: a case series report and review of
zan con contraste, y de bordes definidos que no invaden the literatura. Eur Arch Otorhinolaryngol. 2014; 271:
planos adyacentes. 18516.
Las remisiones y las recadas suelen ser frecuentes, siendo 9. Navazo Egua AL, Garca Vicario F. Rinoscleroma.
la remisin espontnea rara. El tratamiento va a depender Acta Otorrinolaringol Esp. 2010; 61:1602.
del estadio en el que est la enfermedad y la ciruga queda-
r reservada para la toma de biopsias y la reconstruccin
de la va area en las lesiones obstructivas (a veces es nece-
saria hasta una traqueostoma).
Como la tasa de recidiva es grande, se aconseja tratamien-
to antibitico de larga duracin. El uso de estreptomicina,
doxiciclina/tetraciclina, cefalosporinas de segunda y ter-
cera generacin, fluoroquinolonas, rifampicina y sulfona-
midas han demostrado buenos resultados. Ciprofloxacino
es un antibiotico con buena penetracin tisular, amplio es-
pectro antibacteriano y escasos efectos secundarios. Pue-
den, adems, ser eficaces antibiticos de uso tpico, como
pomada de rifampicina al 3%. La duracin del tratamien-
to puede ir desde 6 semanas a 6 meses, hasta que los culti-
vos y el examen histolgico se negativicen.
La mortalidad relacionada con el rinoescleroma es muy
baja y suele ser consecuencia de la obstruccin al flujo a-
reo o por complicacin de alguna intervencin correctora.

Bibliografa

1. Bailhache A, Dehesdin D, Franois A, Marie J-P,


Choussy O. Rhinoscleroma of the sinuses. Rhinology.
2010; 46: 33841.
2. Maguina C, Escalante CJ, Plenge OF, Centeno
J, Guerra H,Montoya M,et al. Rhinoscleroma: eight
peruvian cases. Rev Inst Med Trop S Paulo. 2006; 48:
2959.

172
Masa tumoral colnica con final feliz
Lpez Montesinos, I; Lozano Salazar, AD;
Garca Escudero, A; Domnguez Castellano, A.
Hospitales Universitarios Virgen Macarena y Virgen del Roco. Sevilla.

Caso clnico

Se presenta el caso de un varn de 41 aos, sin alergias a


medicamentos conocidas. Como antecedentes personales
destaca hipertensin arterial y asma extrnseco. Realiza
tratamiento habitual con enalapril/hidroclorotiazida des-
de hace 3 aos.
El paciente acude al servicio de Urgencias de nuestro cen-
tro en Octubre de 2014 por presentar desde hace 4 das
dolor a nivel de fosa ilaca derecha. El dolor es de inten-
sidad moderada y mejora parcialmente con analgesia ha-
bitual. No se atena/exacerba con la ingesta de alimentos
y/o los movimientos posturales ni se acompaa de otros
sntomas gastrointestinales (nuseas, vmitos, diarrea y/o
estreimiento) ni sistmicos (fiebre, prdida de peso, aste- Figura 1. Imagen panormica que muestra lesin necrotizante submucosa,
geogrfica, que se extiende al tejido adiposo perivisceral. Se observa un ganglio
nia, etctera). linftico con hiperplasia folicular (HE, aumento original 0.3x).
A la exploracin fsica presenta un buen estado general,
est estable hemodinmicamente, afebril y eupnico respi- intestinal. No obstante, parece razonable establecer como
rando aire ambiente. No se objetivan signos de deshidrata- primera posibilidad diagnstica la sospecha de neoplasia
cin ni lesiones cutneo-mucosas. La exploracin neurol- de coln. Otra patologa a tener en cuenta podra ser la
gica y cardiopulmonar es anodina. enfermedad inflamatoria intestinal.
A nivel abdominal se palpa masa no dolorosa en flanco de- Desde el punto de vista de las enfermedades infecciosas,
recho de consistencia dura, adherida a planos profundos. no debemos olvidar la actinomicosis intestinal como causa
Las pruebas complementarias revelaron una leucocitosis de masa abdominal. Por otra parte, en el caso de nuestro
de 11930/L con eosinofilia de 1500/L y PCR de 37.41 paciente, la presencia de eosinofilia en sangre perifrica,
mg/l, como nicos valores destacables. La TAC abdomi- que si bien podra ser atribuida al antecedente de asma ex-
nal inform de un engrosamiento a nivel de coln ascen- trnseco, nos obliga a descartar una parasitosis. Por ltimo,
dente que produca estenosis de la luz, de probable ori- y aunque menos probable, se podra valorar tambin la
gen neoplsico, sin evidencia de adenopatas plvicas ni posibilidad de tuberculosis intestinal.
retroperitoneales. Se complet estudio con colonoscopia,
visualizndose a nivel de colon ascendente una lesin de Evolucin
crecimiento intraluminal, ulcerada y sangrante al tacto. Se
tomaron biopsias objetivando cambios inflamatorios ines- Ante la sospecha de un proceso neoformativo, se decidi
pecficos, sin signos histolgicos de malignidad. intervencin quirrgica. Se practic hemicolectoma de-
recha, sin incidencias. El diagnstico anatomopatolgico
Diagnstico diferencial fue de proceso inflamatorio granulomatoso, focal, necro-
tizante, en empalizada, con eosinofilia intensa, compatible
Nos encontramos ante un paciente joven con una masa con anisakiasis colnica invasiva (Figura 1-2). El paciente
abdominal de reciente aparicin. No tiene antecedentes fue derivado a consultas externas de enfermedades infec-
familiares relevantes ni factores predictores de enfermedad ciosas. Haciendo una anamnesis dirigida, refera consumir
neoplsica colnica. Por otra parte, el cuadro actual es de pescado poco cocinado casi a diario. Se solicit serologa
curso subagudo y no se asocian datos clnicos sugestivos de IgE frente Anisakis, resultando sta positiva (3.59 kUA/l) y
malignidad como prdida ponderal o alteracin del hbito se inici tratamiento con albendazol 400 mg/12h durante

173
CAPTULO 10
INFECCIONES TROPICALES
Y DEL VIAJERO

21 das. En la revisin realizada tras finalizar tratamiento


el paciente estaba asintomtico y sin eosinofilia.

Diagnstico final

Anikasiasis colnica invasiva

Discusin

El primer caso de anisakiasis fue descrito por primera vez


en 1876, pero no fue hasta 1960 cuando el parsito fue
identificado por Van Thiel. El Anisakis es un nematodo
de distribucin mundial que parasita en su forma adulta a
mamferos marinos. Los huevos son expulsados con las he-
ces del animal (primera fase larvaria), los cuales son inge- Figura 2. Detalle de la necrosis, empalizada histiocitaria e infiltrado eosinoflico
(HE aumento original 20x).
ridos por pequeos crustceos tipo plancton (segunda fase
larvaria) que a su vez son ingeridas por peces y cefalpo-
dos (tercera fase larvaria). El ser humano al consumir estos helmnticos. No obstante, este hecho no excluye el diag-
peces o cefalpodos es parasitado, comportndose como nstico, ya que segn se ha descrito en la literatura previa4,
hospedador accidental. fisiopatolgicamente, a partir de los 7 das de la ingesta, se
La mayora de las infecciones humanas son causadas por empiezan a formar granulomas y lesiones ulcerosas y tras
Anisakis simplex u Pseudoterranova decipiens. La infeccin ocu- unas 2 semanas, tiene lugar la prdida del parsito, mien-
rre comnmente entre consumidores de pescado crudo o tras que la inflamacin y ulceracin se perpeta.
poco cocinado. De los 20.000 casos reportados en todo el Por otra parte, los test serolgicos tambin pueden ser ti-
mundo, ms del 90% ocurren en Japn. La mayora del les para el diagnstico. Los pacientes con infeccin por
resto de casos se han descrito en reas costeras de Holan- Anisakis presentan tpicamente una elevacin inicial de an-
da, Alemania, Francia y Espaa. ticuerpos IgM anti-Anisakis simplex que tras el primer mes
Clnicamente se describen cuatro formas de anisakiasis: se transforman a otros isotipos (IgG, IgA e IgE)5.
gstrica, intestinal, extra intestinal o alrgica. La anisakia- En cuanto al tratamiento, la extraccin temprana de la
sis intestinal suele desarrollarse varios das despus de la larva a travs de endoscopia es la terapia de eleccin. La
ingesta del pescado infectado, en forma de dolor y disten- retirada quirrgica es otra opcin, principalmente, en
sin abdominal. Puede asociar una masa inflamatoria pal- aquellos casos en los que ocurren complicaciones como la
pable que puede causar obstruccin intestinal. La diarrea obstruccin intestinal. En el caso de que se opte por un
con productos patolgicos puede ser otra manifestacin. tratamiento conservador, algunas evidencias sugieren que
Mientras que en Japn la forma clnica predominante es el tratamiento con albendazol (400 800 mg/da durante
la gstrica, en los pases Europeos es la intestinal. El leo 6 21 das) puede ser eficaz.
distal es el tramo que se afecta con mayor frecuencia, aun-
que ocasionalmente, pueden darse formas colnicas. En
el caso de nuestro paciente, ste presenta una clnica tpica
intestinal, con la singularidad de la afectacin a nivel de
coln ascendente. Cabe plantear posibilidad de invagina-
cin intestinal secundaria a la lesin inflamatoria. Esto es
una eventualidad bastante rara, habindose descrito hasta
el ao 2010 slo 11 casos en la literatura inglesa y japonesa.
En cuanto al diagnstico, la historia de consumo reciente
de pescado crudo o poco cocinado junto con dolor abdo-
minal orienta el cuadro. La visualizacin de la larva por
endoscopia o en la pieza de anatoma patolgica otorga
el diagnstico definitivo. En nuestro caso, en el informe
anatomopatolgico se especifica que no se observan restos

174
CAPTULO 10
INFECCIONES TROPICALES
Y DEL VIAJERO

Bibliografa

1. Valls A, Pascual CY, Martn Esteban M. Anisakis y


anisakiosis. Allergologia et Immunopathologia 2003;
31: 348-55.
2. Natasha S. Hochberg, Davidson H. Hamer. Anisaki-
dosis: Perils of the Deep. Clinical Infectious Diseases
2010; 51: 80612.
3. Miura T, Iwaya A, Shimizu T, Tsuchiya J, Nakamura
J, Yamada S et al. Intestinal anisakiasis can cause in-
tussusception in adults: An extremely rare condition.
World Journal of Gastroenterology 2010; 16: 1804-7.
4. Audicana MT, Kennedy MW. Anisakis simplex: from
Obscure Infectious Worm to Inducer of Immune
Hypersensitivity. Clinical Microbiology Reviews 2008;
21: 36079.
5. Nieuwenhuizen, Lopata A. Anisakis A food-borne
parasite that triggers allergic host defences. Internatio-
nal Journal for Parasitology 2013; 43: 1047-57.

175
Paciente natural de Rumana con fiebre de 2 meses
de evolucin, tos y artromialgias
Rodrguez Rodrguez, JP; Pousibet Puerto, J;
Garca Jerez, MA; Salas Coronas, J.
Hospital de Poniente. El Ejido, Almera.

Caso clnico

Paciente varn de 42 aos de edad, natural de Rumana,


que reside en Espaa desde hace 10 aos. Trabaja en la
construccin, y por este motivo cambia su residencia a
Guinea Ecuatorial durante el ltimo ao, realizando viajes
a Espaa con periodicidad semestral. Entre sus anteceden-
tes destaca ser fumador de 20 cigarrillos/da y beber unos
20 gramos de etanol diarios. Adems fue vacunado de tu-
berculosis en la infancia. Imagen 1. Radiografa de trax al ingreso: Infiltrado en lbulo superior derecho
con atelectasia asociada.
Durante su estancia en frica permanece prcticamente
asintomtico salvo por un episodio de malaria diagnosti-
cado y tratado de forma ambulatoria (no realizaba qui- Las pruebas complementarias iniciales fueron:
mioprofilaxis frente a esta enfermedad). No haba tenido
contacto con animales, bebido agua no embotellada, con- Hemograma: Hemoglobina 6.9 mg/dL, Hemato-
sumido productos lcteos locales ni tampoco haba realiza- crito 21.3%, VCM 92.8 fl, MCH 30.2 pg, Plaquetas
do baos en agua dulce. 112.000/mm3, Leucocitos 4.770/mm3 (Neutrfilos
Despus de un ao regresa a Espaa por segunda vez, y 56.3%, Linfocitos 22.9%, Eosinfilos 0.9%).
tras permanecer en nuestro pas siete das, viaja a Rumana Bioqumica: Glucemia 58 mg/dL, Creatinina 0.45
para visitar a sus familiares. A la semana de llegar a su pas mg/dL, Sodio 135 meq/L, Potasio 4 meq/L, Urea
de origen comienza con un cuadro pseudogripal consisten- 28 mg/dL, ALT 10 UI/L, AST 20 UI/L, LDH 274
te en fiebre y artromialgias, tos escasamente productiva, UI/L, Protena C Reactiva 18.53 mg/dL.
sin expectoracin purulenta, dolor torcico pleurtico en Coagulacin: normal.
hemitrax derecho, sudoracin nocturna, astenia y prdi- Radiografa de trax: Infiltrado en lbulo superior de-
da de unos 5 kilogramos (kg) de peso. recho con atelectasia asociada (Imagen 1).
Tras permanecer un mes en Rumana regresa a Espaa. A Electrocardiograma: ritmo sinusal a 100 lpm, con eje
su llegada consulta en Atencin Primaria por persistencia normal y sin alteraciones de la conduccin ni de la
de la sintomatologa. Realiza tratamiento con amoxicili- repolarizacin ventricular.
na-clavulnico durante siete das, pero ante la ausencia de
mejora acude a nuestro centro hospitalario. Diagnstico diferencial
A la exploracin fsica se observaba un hbito astnico,
eupneico, con temperatura axilar de 39C, tensin arterial En resumen, se trata de un paciente que consulta por sn-
de 93/60 mmHg, frecuencia cardaca a 90 lpm y satura- drome febril, artromialgias y deterioro del estado general
cin de oxgeno basal del 97%. Peso: 55 kg. Presentaba de un mes de evolucin, con clnica infectiva respiratoria
muguet orofarngeo y no se palpaban adenopatas perif- asociada. En las pruebas complementarias analticas se
ricas. La auscultacin cardaca fue normal, pero en la res- detecta la presencia de anemia, trombopenia e hipoglu-
piratoria se apreciaba disminucin del murmullo vesicular cemia; as como un infiltrado en lbulo superior derecho
en ambas bases pulmonares con crepitantes hmedos en en la radiografa de trax. Destaca en la anamnesis el an-
la base derecha. El abdomen era blando a la palpacin, tecedente epidemiolgico de regreso a Espaa tras larga
no doloroso, con ligera hepatomegalia de borde liso, sin estancia en Guinea Ecuatorial 6 semanas antes y que no
masas, ni signos de irritacin peritoneal. Miembros inferio- haba realizado quimioprofilaxis antipaldica.
res sin edemas ni datos de trombosis venosa profunda. La El diagnstico diferencial inicial se debe centrar en torno
exploracin neurolgica fue normal. a sndrome febril en paciente procedente de rea tropical

176
CAPTULO 10
INFECCIONES TROPICALES
Y DEL VIAJERO

(Tabla 1), prestando especial atencin a aquellas etiologas DIAGNSTICO MacLean Doherty OBrien
et al. N=587 et al N=195 et al N=587
causantes de afectacin pulmonar. (1994) (1995) (2001)
La enfermedad ms importante a descartar en viajeros con Malaria 32% 42% 27%
fiebre al regreso del trpico es la malaria1, ms an si el Infeccin respiratoria 11% 2,6% 24%
paciente proviene de una zona endmica como es frica Hepatitis viral 6% 3% 3%
Subsahariana. En algunas series la malaria supone hasta Infeccin gastrointestinal 4.5% 6,5% 14%
el 30% de los cuadros febriles en viajeros procedentes de Infeccin urinaria 4% 2,6% 2%
esa regin del mundo. Con frecuencia, los pacientes pre- Dengue 2% 6% 8%

sentan adems sintomatologa inespecfica como dolor ab- Fiebre tifoidea 2% 2% 3%

dominal, diarrea, clnica miccional o respiratoria, en este Mononucleosis infecciosa 2% 0,5% 0.4%

ltimo caso, simulando infecciones del tracto respiratorio Tuberculosis 1% 2% 1%

inferior2. La malaria grave puede presentar infiltrados Rickettsiosis 1% 0,5% 2%

pulmonares tanto por la propia enfermedad como por so- Absceso heptico amebiano 1% 0% 0%

breinfeccin bacteriana (malaria complicada). Las mani- Faringitis 1% 2% 0%

festaciones hematolgicas son frecuentes, detectndose la Meningitis 1% 1% 1%

presencia de anemia en un 41% de los casos y trombope- Infeccin aguda por VIH 0.3% 1% 0.4%

nia en 62-83%. La hipoglucemia (definida como glucemia Miscelnea 6.3% 5% 5.2%

menor a 40 mg/dl) est recogida como uno de los criterios No diagnosticadas 25% 25% 9%
1
de gravedad de la malaria importada3. Tabla 1. Principales causas de fiebre a la vuelta del trpico .
Si estudiamos los casos de fiebre en viajeros al regreso
del trpico, el 11% de ellos presentan clnica de infeccin piratoria, pero su evolucin suele ser rpida a un distrs
respiratoria. En el estudio etiolgico de neumonas del via- severo.
jero, se deben considerar las infecciones virales, bacteria- Otra etiologa a considerar es el sndrome del viajero de
nas (siendo frecuentes las etiologas atpicas), tuberculosis, la clase turista, ya que el tromboembolismo puede cursar
sndrome de Leffler, paragonimiasis, fiebre Q , coccidio- como fiebre aislada (14% de los casos).
micosis, meliodiosis y malaria1. Respecto al estudio del sndrome febril a la vuelta del
En nuestro paciente decidimos realizar estudios para des- trpico, es esencial recoger en la historia clnica las ex-
cartar tuberculosis pulmonar, ya que presentaba un in- posiciones y actividades de riesgo que puedan asociarse a
filtrado en lbulo superior del pulmn derecho sugeren- infecciones potenciales.
te y un cuadro constitucional asociado que podran ser En nuestro caso, nuestro paciente no refera haberse ba-
compatibles con este diagnstico. Para viajeros con largas ado en zonas de agua dulce (ros, lagos) en Guinea
estancias como es este caso, la incidencia de tuberculosis Ecuatorial, por lo que la esquistosomiasis (fiebre de Kata-
puede ser tan alta como para los residentes de los pases yama) y la leptospirosis son menos probables.
endmicos4. Negaba haber tenido contactos sexuales de riesgo que
Se han descrito casos de melioidosis con afectacin pulmo- orientasen a infeccin aguda por VIH, o a la transmisin
nar con lesiones y cavidades en lbulos pulmonares supe- de otras ETS. Tampoco haba consumido lcteos no higie-
riores semejantes a los de la tuberculosis, pero esta patolo- nizados, que nos hiciesen sospechar fiebre por brucelosis.
ga es endmica de Sudeste Asitico y Oceana. No haba presentado exantemas (que orientasen a infec-
La histoplasmosis tambin produce afectacin pulmonar cin por Rickettsias ni Borrelia), rash cutneo o manifesta-
y es endmica en algunas zona de frica (adems de en ciones hemorrgicas (que hiciesen sospechar dengue o al-
Amrica y algunas regiones de Asia) pero nuestro paciente gn otro virus hemorrgico).
no refera actividades en grutas ni ruinas, que es la forma
ms habitual de inhalacin de esporas. Evolucin
La fiebre tifoidea tambin puede acompaarse de clnica
respiratoria, aunque el paciente negaba consumo de agua Con los anteriores datos clnicos, epidemiolgicos y analti-
no tratada, y tampoco presentaba clnica abdominal. cos orientativos se realizan de forma paralela:
No presentaba eosinofilia, que en un paciente con fiebre
y clnica respiratoria nos hubiese orientado inicialmente a Test de diagnstico rpido de malaria (Antgenos pan-
sospechar helmintiasis. malrico y de P. falciparum): positivos.
Los hantavirus producen tambin fiebre y afectacin res- Frotis (extensin fina): microcitosis, leve trombopenia,

177
CAPTULO 10
INFECCIONES TROPICALES
Y DEL VIAJERO

parsitos intraeritrocitarios compatibles con Plasmo- nancia ponderal de 8 kg. Tambin se apreci una nota-
dium falciparum y formas extraeritrocitarias (gametos) ble mejora analtica y radiolgica. Continu tratamiento
aislados. Grado de parasitemia: 8-10%. con rifampicina e isoniazida 4 meses ms al no detectarse
mutaciones de resistencias a dichos frmacos y recibirse el
Ingresa en UCI con diagnstico de Malaria por P. falcipa- antibiograma del cultivo de micobacterias que no mostr
rum con criterios de gravedad, realizndose tratamiento resistencia a ninguno de los frmacos de primera lnea tes-
intravenoso (iv) con artesunato (4 dosis de 180 mg a las 0, tados.
12, 24 y 48 horas). A las 24 horas el paciente se encontraba En el seguimiento tampoco se objetivaron efectos secunda-
afebril y la parasitemia era del 0.01% (indetectable a las rios debidos al tratamiento con derivados de la artemisina,
48 horas). como hepatitis y sobre todo, anemia hemoltica secundaria
Posteriormente se traslada a planta de Medicina Interna, a la administracin del artesunato.
estando resuelto ya el cuadro de fiebre y artromialgias,
donde completa tratamiento de forma secuencial con 3 Diagnstico final
comprimidos de Eurartesim oral durante 3 das (120
mg/da de piperaquina + 960 mg de dihidroartemisina). Paludismo por Plasmodium falciparum con criterios de
Adems de lo referido anteriormente, desde que el pacien- gravedad (parasitemia del 8-10%).
te ingres en Urgencias del hospital se tomaron medidas Tuberculosis pulmonar.
de aislamiento respiratorio, ya que se sospech la posibili-
dad de una infeccin tuberculosa pulmonar. Se inici tra- Discusin
tamiento con ceftriaxona 2 gr/24 horas intravenoso ante
la posibilidad de tratarse de una malaria complicada con La malaria es la primera enfermedad parasitaria a nivel
proceso neumnico, y posteriormente se modific a pipe- mundial. Est producida por 6 especies distintas del gne-
racilina-tazobactam 4 gr/8h iv ante los hallazgos en TAC ro Plasmodium (P. falciparum, P. vivax, P. ovale wallikeri, P. ovale
solicitado en la planta de Medicina Interna que mostr la curtisi, P. malariae y P. knowlesi), variando en funcin del rea
presencia de una neumona cavitada en lbulo superior geogrfica implicada. La especie ms frecuente a nivel glo-
derecho. bal es P. falciparum3.
El Mantoux fue negativo (0 mm) y a pesar de inducir el Los sntomas ms frecuentes son la fiebre, la cefalea y las
esputo, solo se obtuvo una muestra en la que la bacilosco- artromialgias. En el 16-33% de los casos existe espleno-
pia result negativa. Se solicit la realizacin de una fibro- megalia. El hallazgo analtico ms frecuente y precoz es la
broncoscopia, demostrndose la presencia de Mycobacterium trombopenia, que est presente hasta en el 80-85% de los
tuberculosis en el broncoaspirado. pacientes2,3.
Inici tratamiento antituberculoso con rifampicina, isonia- En nuestro pas, la mortalidad global en viajeros se sita en
zida, pirazinamida y etambutol (RHZE) ajustado al peso torno al 2-3 %, siendo menor en pacientes semi-inmunes.
(450/225/1.200/825 mg al da) con buena tolerancia, sus- El retraso diagnstico y teraputico son factores de mal
pendindose el tratamiento antibitico. Permaneci ingre- pronstico. En todo enfermo que regresa de zona endmi-
sado las 3 primeras semanas de tratamiento en aislamiento ca, presente fiebre o no, se debe descartar malaria de for-
respiratorio por imposibilidad de realizarlo en domicilio. ma precoz. Inicialmente se debe realizar un frotis (grueso
Durante el ingreso se recibieron otros resultados analticos: o fino) y en paralelo un test de diagnstico rpido. El es-
tndar oro sigue siendo la gota gruesa, que es la tcnica
Serologas de VIH, VHB, VHC, sfilis, Strongyloides y ms sensible, pero requiere de microscopistas expertos. La
Schistosoma: negativas. extensin fina es menos sensible, pero ms especfica para
Hemocultivos: negativos. identificar la especie e infecciones mixtas y cuantificar la
Estudio de microfilarias en sangre (test de saponina): parasitemia3.
negativo. El cuadro se considera malaria grave si cumple alguno de
Parsitos en heces (3 muestras) y orina (1 muestra) con los criterios de gravedad de la OMS (Tabla 2). La actitud
tcnicas de concentracin: negativas. teraputica es distinta en funcin de la especie y la grave-
dad o no. El paludismo grave por P. falciparum debe tratarse
Tras 2 meses de tratamiento con 4 frmacos, el paciente con artesunato iv con un mnimo de 3 dosis en las prime-
present una evolucin clnica favorable, estando asinto- ras 24 horas y hasta que se tolere la va oral, ya que es un
mtico en la revisin por consultas externas, con una ga- frmaco ms seguro y eficaz que la quinina. El tratamiento

178
CAPTULO 10
INFECCIONES TROPICALES
Y DEL VIAJERO

Tipo Criterios mantener al menos un ao el tratamiento. Se deben mo-


Clnicos Alteracin del nivel de conciencia (no explicable por otras
causas).
nitorizar las posibles toxicidades y efectos adversos de los
Postracin (que le impide andar o sentarse sin ayuda).
frmacos4.
Convulsiones (>2 crisis comiciales en 24 h).
Algunos autores han descrito que la malaria disminuye la
Insuficiencia respiratoria aguda (PaO2<60 mmHg con FiO2
respuesta inmune tanto celular como humoral del hospe-
21%). dador frente a M. tuberculosis, lo que explicara un manejo
Shock (PAS < 70 mmHg a pesar de reposicin de volu- ms complejo de la coinfeccin7.
men).
Ictericia asociada a lesin de rgano diana (bilirrubina>2.5
mg/dl). Como conclusiones importantes
Sangrado espontneo (sin otra causa atribuible).
Analticos y Hipoglucemia (40 mg/dl). En todo paciente con fiebre procedente de rea tropi-
radiolgicos cal ha de descartarse la malaria.
Acidosis metablica (pH<7.30 o bicarbonato <15 mmol/l).
Anemia normoctica grave (Hb < 15%, Hto < 15%). La malaria con criterios de gravedad debe ser tratada
Hemoglobinuria (Presencia de hemoglobina en orina). en una Unidad de Cuidados Intensivos.
Hiperlactacidemia (> 5 mmol/l o > 45 mg/dl). El tratamiento de primera eleccin son los derivados
Insuficiencia renal aguda (Creatinina srica > 3 mg/dl). de la artemisina. En el caso de la malaria grave, debe
Edema agudo de pulmn o sndrome de distrs respirato- emplearse artesunato intravenoso seguido de una pau-
rio agudo. ta estndar de tratamiento oral.
Parasitolgicos Hiperparasitemia (> 2.5% en personas no inmunes).
La tuberculosis puede ir asociada a un cuadro de ma-
3
Tabla 2. Criterios de gravedad de la malaria importada (OMS) . laria. El diagnstico y tratamiento es el mismo que en
pacientes no coinfectados, a pesar de que se han des-
se completa con un segundo frmaco por va oral, reco- crito mecanismos inmunolgicos que pueden ocasio-
mendndose dihidroartemisina-piperaquina o autuovaco- nar un manejo ms difcil de la coinfeccin.
na-proguanil3.
El tratamiento de la malaria grave debe realizarse en una
Unidad de Cuidados Intensivos para el manejo de posibles
complicaciones3. Como efecto adverso ms importante del
tratamiento con artesunato se han descrito casos de ane-
mia hemoltica alrededor de los 14 das tras el inicio del
tratamiento, por lo que se recomienda seguimiento clnico
y analtico durante la hospitalizacin, y a las 2 y 4 semanas
despus de su administracin3,5.
La tuberculosis es una enfermedad cosmopolita que afec-
ta a millones de personas en todo el mundo, ntimamen-
te ligada a la pobreza. La sintomatologa puede ser muy
inespecfica, sirviendo de gua los sntomas respiratorios6.
El diagnstico se basa en el aislamiento microbiolgico de
M. tuberculosis. La radiografa es muy sensible para las for-
mas pulmonares y puede dar cualquier patrn. Una radio-
grafa de trax normal descarta la enfermedad en ms del
95% de los adultos inmunocompetentes. La identificacin
se realiza con baciloscopias seriadas y cultivo. Se deben
investigar mutaciones de resistencia para frmacos de pri-
mera lnea4.
Las ltimas recomendaciones consideran iniciar trata-
miento antituberculoso con 4 frmacos (RHZE) por del
aumento de resistencias al mismo. En ausencia de las mis-
mas se podra iniciar con 3 (RHZ). La fase de induccin
dura 2 meses, completndose con RH durante 4 meses
ms. En los casos de afectacin sea o de SNC se aconseja

179
CAPTULO 10
INFECCIONES TROPICALES
Y DEL VIAJERO

Bibliografa

1. Aparicio PT, Torrus D, Trevio B, Zubero Z, Lo-


pez-Velez R. Guas de aproximacin al viajero con
fiebre al regreso del trpico. Guas Clnicas SEIMC
2006.
2. World Health Organization (WHO). Malaria rapid
diagnostic test performance: results of WHO product
testing of malaria RDTs: round 4 (2012). 140 pgs.
ISBN 978 92 4 150472 0
3. Muoz J, Rojo-Marcos G, Ramirez-Olivenza G, Sa-
las-Coronas J,Trevio B, Perez Arellano JL, et al.
Diagnstico y tratamiento de la malaria importada en
Espaa: recomendaciones del Grupo de Trabajo de
Malaria de la Sociedad Espaola de Medicina Tropi-
cal y Salud Internacional (SEMTSI). Enferm Infecc
Microbiol Clin 2015; 33: e1-e13
4. Gonzalez-Martn J, Garcia-Garcia JM, Anibarro L,
Vidal E, Esteban J, Blanquer R, et al. Documento de
consenso sobre diagnstico, tratamiento y prevencin
de la tuberculosis. Enferm Infecc Microbiol Clin 2010;
28: 297.e1297.e20.
5. Jaurguiberry S, Ndour PA, Roussel C,Ader F,Safeu-
kui I,Nguyen M, et al. Post-artesunate delayed he-
molysis is a predictable event related to the lifesaving
effect of artemisinins.Blood2014; 124: 16775.
6. WHO.Global tuberculosis report 2013.World Health
Organization,Geneva; 2013.
7. Wiwanitkit V.Co-infection between tuberculosis and
malaria: A consideration on interaction of molecules
and pathogenesis.J Vector Borne Dis2006;43:1957.

180
Paciente de 46 aos que consulta por fiebre
y eosinofilia al regreso de un viaje a Etiopa
Pousibet-Puerto, J; Cuenca-Gmez, JA;
Rodrguez-Rodrguez, JP; Salas-Coronas, J.
Hospital de Poniente. El Ejido. Almera.

Caso clnico

Se trata de una paciente espaola, mdico de profesin,


que consulta por fiebre al regreso de un viaje por Etiopa.
Entre los antecedentes personales slo destacan ser fuma-
dora de 10 cigarrillos/da, y dos episodios varios aos an-
tes de pielonefritis aguda. Previamente a este viaje haba
estado en dos ocasiones en Nepal (2 y 3 aos antes) que
haban cursado sin incidencias. Imagen 1. Regin de Gondar, norte de Etiopa, donde se encuentran las monta-
as Simiens.
Cuarenta das antes de la fecha de la primera consulta
haba regresado de un viaje programado por Etiopa con
gua, junto con otros 13 excursionistas de diversas naciona- En la primera visita se solicitaron las siguientes exploracio-
lidades. Visitaron fundamentalmente zonas rurales practi- nes complementarias:
cando senderismo por las Montaas Simiens en la zona de Hemograma: Leucocitos 7.810 (57% Neutrfilos,
Gondar, al norte del pas. Como actividades de riesgo rea- 23% linfocitos, 16% Eosinfilos -1.250 Eos/mm3-.);
lizadas durante el viaje destacaron el consumo de agua no Hb 13,8 mg/dL; Hto 41,9%; HCM 25,3; VCM 77;
embotellada y de productos lcteos locales, camin descalza Plaquetas 200.000/mm3.
por el terreno, tuvo contacto con animales (cabras, vacas y Bioqumica: ALT 56 UI/L, AST 32 Ul/L, GGT 96
perros) y se ba en ros y pozas de la zona (imagen 1). Ul/L, FA 116 UI/L, Protena C reactiva (PCR) 14,1
La paciente acudi un mes antes del viaje a la consulta de mg/dL, Ig E 67 mg/dL.
Vacunacin Internacional. Realiz quimioprofilaxis frente Coagulacin y sedimento de orina: normales.
al paludismo con atovacuona-proguanil y ya se encontra- Frotis sanguneo, test de diagnstico rpido y PCR de
ba correctamente vacunada frente a VHA, VHB, ttanos, paludismo: negativos.
fiebre tifoidea y fiebre amarilla por los viajes que haba Estudio de parsitos en orina y heces con tcnicas de
realizado anteriormente. concentracin: Negativos.
A la semana de regresar de dicho viaje comienza con tos Hemocultivos: negativos.
seca, fiebre de hasta 39,5C, escalofros y artromialgias Radiografa de trax y abdomen: sin alteraciones.
generalizadas, que trata con paracetamol, ibuprofeno y Ecografa abdominal: sin alteraciones.
amoxicilina-clavulnico durante 6 das, con desaparicin
transitoria de la sintomatologa. Posteriormente presen- Diagnstico diferencial
ta de nuevo fiebre elevada, cefalea y astenia intensa. Los
ltimos das comienza tambin con dolor abdominal tipo En resumen, se trata de una mujer de 46 aos, sin enfer-
retortijn y heces pastosas sin productos patolgicos. Entre medades previas relevantes, que consult por un cuadro
ambos periodos febriles present ppulas pruriginosas en de fiebre prolongada, tos seca, artromialgias generalizadas
ambos miembros inferiores que desaparecieron espont- y dolor abdominal una semana despus de haber regresa-
neamente en pocos das. do de un viaje a Etiopa donde realiz diversas actividades
En la exploracin fsica presentaba un buen estado general, de riesgo (ingesta de agua no embotellada, bao en ros de
con TA: 110/60mmHg, normocoloreada y eupneica. No se la zona). Haba realizado profilaxis antipaldica y estaba
palpaban adenopatas ni se objetivaron lesiones cutneas. correctamente vacunada. En la analtica general, los nicos
La auscultacin cardiorrespiratoria fue normal, y el abdo- datos llamativos eran la presencia de eosinofilia perifrica y
men presentaba discreta hipersensibilidad generalizada a la una discreta elevacin de las pruebas hepticas. El diagns-
palpacin profunda, sin llegar a palpar visceromegalias. El tico diferencial en esta paciente debe plantearse en base a la
resto de la exploracin fue rigurosamente normal. fiebre tras regresar una regin tropical y la eosinofilia.

181
CAPTULO 10
INFECCIONES TROPICALES
Y DEL VIAJERO

Los primeros estudios microbiolgicos resultaron negati- Mecanismo de transmisin Enfermedades


Artrpodos Malaria.
vos, as como las pruebas de imagen solicitadas. El cua- Fiebre amarilla.
dro clnico de esta paciente resulta bastante inespecfico, Dengue.
Fiebres hemorrgicas: fiebre del Valle del Rift,
aunque sugiere una etiologa infecciosa. En este sentido, fiebre hemorrgica de Crimea-Congo.
sus antecedentes epidemiolgicos y la eosinofilia orientan Tripanosomiasis africana.
Leishmaniasis.
a una infeccin parasitaria. Rickettsiosis.
La fiebre es uno de los principales motivos de consulta tras Fiebre de Chikungunya.

viajar a una zona tropical1. El origen de la fiebre vara no- Ingesta de alimentos o agua Diarrea bacteriana (incluido clera) y por
protozoos.
tablemente segn la zona geogrfica. En los pacientes que Fiebre tifoidea.
Hepatitis A y hepatitis E.
regresan de frica Subsahariana, como es nuestro caso, la Poliomielitis.
causa ms frecuente y la primera entidad que hay que des- Paragonimiasis.
cartar por su potencial gravedad es el paludismo o malaria. Contacto con agua dulce Leptospirosis.
Esquistosomiasis.
En nuestro caso, los estudios realizados (frotis sanguneos,
Contacto con animales Rabia.
test de diagnstico rpido y PCR de malaria) fueron ne- Otros Otras fiebres hemorrgicas. Meningitis menin-
gativos. En la tabla 1 se describen las enfermedades que goccica.

con mayor frecuencia se adquieren tras viajar a esta regin Tabla 1. Principales enfermedades que cursan con fiebre en pacientes que
regresan de frica Subsahariana.
segn el mecanismo de transmisin. No obstante, siempre
hay que tener presente que los viajeros pueden adquirir
otras infecciones que tienen una distribucin global, como Hel- Nematodos Cestodos Trematodos
mintosis
la gripe, mononucleosis infecciosa, infeccin por el VIH, Strongyloides stercoralis Echinococcus Schistosoma
hepatitis vricas, tuberculosis, brucelosis, rickettsiosis o fie- Ascaris lumbricoides granulosus spp.
Uncinarias Taenia solium Fasciola heptica
bre Q2. Trichuris trichuria Taenia saginata Clonorchis
Respecto a la eosinofilia, se define como la presencia en Toxocara spp. Hymenolepis sinensis
Filarias (Wuchereria ban- nana Opisthorchis
sangre perifrica de ms de 500 eosinfilos/mm3 3. Puede crofti, Onchocerca volvulus, viverinni
deberse a causas farmacolgicas, reacciones de hipersen- Loa loa, filarias del gnero Paragonimus
Mansonella) spp.
sibilidad, enfermedades inmunolgicas no alrgicas, neo- Anisakis
Triquinella spiralis.
plasias, enfermedades endocrino-metablicas, infecciones Gnathostoma spp
parasitarias y no parasitarias como la infeccin por el virus Otros Isospora belli
de la inmunodeficiencia humana (VIH), formas crnicas parsitos Dientamoeba fragilis
Sarcocystis spp
de tuberculosis o lepra, algunos protozoos como es el caso
de la Isospora belli y ciertas micosis como la aspergilosis4. Tabla 2. Principales enfermedades parasitarias que cursan con eosinofilia
en el humano.
En el caso de nuestra paciente que procede de una regin
tropical, la infeccin por helmintos constituye la causa ms
frecuente, y al igual que la fiebre importada, depende de Evolucin
las actividades de riesgo que el paciente realiza durante el
viaje5. Caminar descalzo sobre el terreno es un factor de Adems de las pruebas complementarias expuestas ante-
riesgo para la adquisicin estrongilodiasis y uncinarias; el riormente, se solicitaron serologas para Brucella, rickettsias,
haber estado en contacto con agua dulce para la adqui- Strongyloides, Schistosoma y Dengue que fueron negativas.
sicin de esquistosomiasis; la picadura de ciertas especies Ya en la primera visita, y tras recibir los resultados ana-
de mosquitos/moscas para filariasis; el comer berros para lticos iniciales en los que apareca eosinofilia, la clnica
fasciolasis, etc. compatible y el antecedente epidemiolgico de baos en
En las helmintosis, la eosinofilia est asociada especfica- agua dulce en una zona endmica de esquistosomiasis, se
mente a la invasin y migracin de los parsitos por los te- realiz el diagnstico de presuncin de fiebre de Katayama
jidos, siendo el grado de eosinofilia perifrica proporcional (esquistosomiasis aguda). La paciente recibi tratamiento
a la extensin de la invasin tisular. En la tabla 2 se descri- con praziquantel 40 mg/Kg durante 3 das y simultnea-
ben las principales infecciones parasitarias importadas que mente prednisona 30 mg/da durante 5 das.
pueden causar eosinofilia. La paciente contact con los compaeros del viaje. Tres de
ellos, ingleses, se encontraban con clnica similar (fiebre,
eosinofilia, tos seca y dolor abdominal), al igual que el gua
de nacionalidad rumana que se encontraba ingresado en

182
CAPTULO 10
INFECCIONES TROPICALES
Y DEL VIAJERO

un centro hospitalario de su pas. A los pocos das comuni- jas van a emigrar tambin va hematgena a un rgano
caron a nuestra paciente que en los tres pacientes ingleses diana, hacia los plexos vesicales (S. haematobium) o mesen-
se haban conseguido aislar huevos de Schistosoma mansoni tricos (S. mansoni, S. intercalatum, S. mekongi, S. japonicum).
en heces. Un compaero de viaje espaol contact con La hembra produce gran cantidad de huevos que emigran
nosotros y aunque se encontraba asintomtico fue citado por los capilares y atraviesan la mucosa vesical o intestinal
para valoracin, con estudio inicial (analtico, serologa, segn la localizacin, eliminndose por la orina o heces.
parsitos en heces y orina) negativo. La clnica va a depender de la fase de la infeccin. La pe-
La paciente evolucion favorablemente con desaparicin netracin cutnea de la cercaria puede provocar un exan-
de la fiebre, la clnica pulmonar y gastrointestinal, as como tema local pruriginoso temporal denominada dermatitis
analticamente, con descenso de la PCR y normalizacin del nadador7. El paso de las esquistosmulas por el pul-
de la eosinofilia y enzimas hepticas en controles posterio- mn produce un cuadro de tos seca, sibilancias y a veces
res durante su seguimiento. A las 4 semanas se le adminis- infiltrados pulmonares cambiantes. Una vez llegan a nivel
tr una segunda dosis de praziquantel (40 mg/kg 3 das). intestinal se produce dolor y diarrea a veces con sangre. En
Los controles de parsitos en heces y orina realizados en el caso del S. haematobium, el signo caracterstico es la hema-
visitas sucesivas fueron negativos, y la serologa de Schisto- turia terminal. El tiempo que transcurre desde la infeccin
soma repetida en 2 ocasiones ms fue tambin negativa. El inicial hasta que comienza la eliminacin de los huevos por
compaero de viaje espaol present una seroconversin heces u orina oscila entre 1 y 3 meses habitualmente. La
de la serologa de Schistosoma a los 4 meses, recibiendo tra- variabilidad clnica es importante, con individuos que se
tamiento con praziquantel en ese momento. encuentran asintomticos y otros con clnica muy florida.
La paciente, al igual que todos sus compaeros, se haba La esquistosomiasis crnica es la forma ms habitual de
baado en ros de la zona, lo que constituye el mecanismo presentacin en nuestro medio, siendo diagnosticada de
de transmisin de la esquistosomiasis, ya que las larvas o mayoritariamente en poblacin inmigrante. Puede ser
cercarias se mueven libremente por el agua y penetran a una enfermedad desde asintomtica hasta producir gra-
travs de la piel sana. El cuadro clnico que presentaba es ves complicaciones a nivel genitourinario (engrosamientos
similar al descrito en otros excursionistas en los que se aisl mucosos vesicales, plipos/ndulos, hidronefrosis, insufi-
el huevo del parsito en heces. Por todo ello, el diagnstico ciencia renal terminal, carcinoma vesical) o intestinal (fi-
de la paciente fue el de esquistosomiasis aguda o sndrome brosis heptica, hipertensin portal con hiperesplenismo,
o fiebre de Katayama. varices esofgicas). Se calcula que unas 300.000 personas
mueren cada ao por complicaciones derivadas de esta en-
Diagnstico final fermedad, sobre todo por insuficiencia renal o carcinoma
vesical y sangrado por varices esofgicas.
Fiebre de Katayama (esquistosomiasis aguda). La fiebre de Katayama es una reaccin de hipersensibili-
dad sistmica durante la migracin de la larva y la pues-
Discusin ta de huevos precoz de los gusanos adultos. Ocurre entre
14-84 das tras la infeccin primaria y se produce de for-
La esquistosomiasis est causada por trematodos del gne- ma habitual en personas no expuestas previamente (via-
ro Schistosoma, que infectan tanto al ser humanos como a jeros). Las manifestaciones de la enfermedad suelen tener
animales. Las cinco especies que producen la enfermedad un comienzo sbito, con fiebre elevada, mialgias, cefalea,
en el humano son S. haematobium, S. mansoni, S. intercalatum, tos seca, molestias abdominales e incluso urticaria8,9. La
S. japonicum y S. mekongi. Es la segunda enfermedad parasi- presencia de eosinofilia ha sido descrita como una cons-
taria tras la malaria que provoca mayor morbimortalidad a tante en estos casos, en especial eosinofilia intensa (>3.000
nivel mundial6. Tiene una distribucin especfica en zonas eosinfilos/mm3). Sin embargo, algunas series de pacientes
geogrficas de frica, Amrica y Asia. La transmisin se indican que hasta un 30% de los casos podra presentar ci-
produce a partir de las larvas (cercarias) que viven en agua fras normales de eosinfilos, dependiendo del momento en
dulce, penetran la piel del ser humano, invaden el sistema el que se realiza la analtica. Por tanto, la esquistosomiasis
linftico y venoso como formas inmaduras denominadas aguda puede ser diagnosticada en ausencia de eosinofilia8.
esquistosmulas, y llegan al pulmn va hematgena. Tras Las escasas descripciones de casos de esquistosomiasis agu-
el paso por el pulmn se dirigen al sistema porta heptico, da en pases endmicos, asociadas a la baja frecuencia de
donde se desarrollan en esquistosomas adultos (machos y casos en viajeros en pases no endmicos, han favorecido
hembras), formando parejas. Segn la especie, estas pare- que existan dificultades en el diagnstico y tratamiento de

183
CAPTULO 10
INFECCIONES TROPICALES
Y DEL VIAJERO

la fiebre de Katayama. Suele ayudar en el diagnstico el Bibliografa


que habitualmente los casos de la enfermedad se agrupen
en clusters, afectando a varios viajeros del grupo. 1. Freedman DO, Weld LH, Kozarsky PE, Fisk T, Robins
Mientras que en la esquistosomiasis crnica el diagnstico R, von Sonnenburg F, et al. Spectrum of disease and
se realiza principalmente a travs de tcnicas serolgicas y relation to place of exposure among ill returned trave-
la identificacin de huevos del parsito en heces y orina7, lers. N Engl J Med. 2006; 354:11930.
en la fase aguda la sensibilidad de la serologa no suele su- 2. Wilson ME, Weld LH, Boggild A, Keystone JS, Kain
perar el 50%, dependiendo de la tcnica utilizada y del KC, von Sonnenburg F, et al. Fever in returned trave-
momento en el que se tome la muestra. La sensibilidad de llers: Results from the GeoSentinel Surveillance Ne-
la parasitologa en heces y orina en fase aguda tampoco twork. Clin Infect Dis. 2007; 44:15608.
suele superar el 20- 30% con el examen de 3 muestras8. 3. Schulte C, Krebs B, Jelinek T, Nothdurft H.D, von
Por tanto, el diagnstico de la esquistosomiasis aguda se Sonnenburg F, Lscher T. Diagnostic significance of
realiza en muchas ocasiones en base al antecedente epide- blood eosinophilia in returning travellers. Clin Infect
miolgico de bao en agua dulce en regiones endmicas de Dis. 2002;34:407-11.
esquistosomiasis, a las manifestaciones clnicas y analtica 4. Garca Surez J, Burgaleta Alonso de Ozalla C, de
compatible. La negatividad de las pruebas microbiolgi- Miguel Llorente D, Lpez Rubio M. Protocolo diag-
cas no excluye el diagnstico en esta fase. En centros de nstico de la eosinofilia. Medicine 2001; 8(52): 2780-
referencia se est empleando en el diagnstico una PCR 2780-3.
de Schistosoma en sangre que parece que puede aumentar 5. Montes de Oca Arjona M, Saldarreaga Marn A, Brun
sensiblemente el rendimiento diagnstico. Romero F. Protocolo diagnstico de la eosinofilia en
Existe controversia en el tratamiento de la esquistosomiasis pacientes de pases con riesgo de parasitosis. Medicine
en la fase aguda. El praziquantel no es activo contra los hue- 2006; 9: 3791-4.
vos ni las formas inmaduras de Schistosoma (esquistosmula), 6. Gryseels B, Polman K, Clerinx J, Kestens L. Human
por lo que la administracin en la fase aguda no elimina por schistosomiasis. Lancet. 2006; 368: 110618.
completo el parsito, y por tanto, se cree que no previene el 7. Gray DJ, Ross AG, Li YS, McManus DP. Diagno-
desarrollo de esquistosomiasis crnica8,10. A pesar de esto, al- sis and management of schistosomiasis. BMJ. 2011;
gunos grupos administran praziquantel en la fase aguda con 342:d2651.
la intencin de disminuir la carga parasitaria inicial, tenien- 8. Jaurguiberry S, Paris L, Caumes E. Acute schistoso-
do en cuenta que se debe repetir a las 4-8 semanas despus miasis, a diagnostic and therapeutic challenge. Clin
de los sntomas, cuando todos los helmintos han madurado Microbiol Infect. 2010; 16:22531.
y, por tanto, son sensibles al tratamiento8. 9. Ross AG, Vickers D, Olds GR, Shah SM, MacManus
Dado que la fiebre de Katayama es una reaccin de hiper- DP. Katayama syndrome. Lancet Infect Dis. 2007;
sensibilidad a la migracin de los helmintos y la oviposi- 7:21824.
cin, la recomendacin general es administrar corticoides 10. Nguyen LQ , Estrella J, Jett EA, Grunvald EL, Nichol-
orales como base del tratamiento. La respuesta a la corti- son L, Levin DL. Acute schistosomiasis in nonimmune
coterapia en la fase aguda de la esquistosomiasis suele ser travelers: Chest CT findings in 10 patients. AJR Am J
excelente. Se han descrito casos de encefalitis, miocarditis Roentgenol. 2006; 186:13003.
y empeoramiento de los sntomas respiratorios tras la ad-
ministracin de praziquantel en fase aguda, posiblemente
causados por la destruccin de algunos helmintos situados
en el torrente vascular. Por tanto, tambin es recomenda-
ble asociar corticoides al tratamiento con praziquantel en
estos pacientes8,9,10.
En resumen, la fiebre de Katayama o esquistosomiasis agu-
da es una entidad que requiere de un elevado ndice de sos-
pecha por parte del clnico. Las tcnicas serolgicas y los
estudios parasitolgicos son con frecuencia negativos, y debe
sospecharse sobre todo en viajeros procedentes de regiones
endmicas de esquistosomiasis, que se han baado en ros o
lagos de agua dulce, y que presentan eosinofilia perifrica.

184
Varn subsahariano de 37 aos
con disnea, tos, diarrea y prurito
Salazar de Troya, C; Mercado, R;
Ruiz-Mesa, JD; Mediavilla Gradolph, C.
Hospital Universitario Regional de Mlaga

Caso clnico de clnica respiratoria consistente en golpe de tos, disnea


y expectoracin blanquecina, y se sustituye piperacilina/
Se presenta el caso de un varn de 37 aos de edad, natu- tazobactam por meropenem. El da +9 presenta nuevo
ral de Ghana, que reside en Espaa desde 1998, las ltimas pico febril, y se aade Anfotericina B liposomal a su tra-
visitas a su pas fueron en 2005 y 2008. Ex -fumador de tamiento por coprocultivo positivo a Candida glabrata. En
7 paquetes/ao y est diagnosticado de asma con brotes este momento el paciente presenta discreta mejora clnica
aislados, sin tratamiento en la actualidad. Sus padres falle- y analtica. El da +14 se suspende vancomicina y se inicia
cieron con 70 aos aproximadamente (no causa conocida) valganciclovir por PCR positiva para CMV.
y tiene 6 hermanos que viven en frica. El da +16 se observa importante elevacin de eosinfilos
En junio de 2011 ingresa en el Servicio de Digestivo por coincidente con la recuperacin de la pancitopenia tras
dolor en epigastrio y vmitos sin relacin con las comi- el trasplante, llegando hasta un pico mximo de 15,130
das de un mes de evolucin. En la ecografa abdominal se x109/L. Se solicita extensin de sangre perifrica, aspirado
objetiva un engrosamiento de la pared gstrica asimtrica de mdula sea y estudio de parsitos en heces.
con prdida de pliegues y diferenciacin de capas, junto
a adenopatas mesentricas en torno a la salida del tron- Diagnstico diferencial
co celaco. La gastroscopia es informada como neoplasia
extensa y ulcerada que compromete antro, ploro y bul- Se trata en resumen de un varn de origen africano que
bo duodenal, estenosis pilrica y esofagitis de la unin. El no viaja a su pas desde hace 5 aos, trasplantado de m-
TAC abdominal muestra datos compatibles con neoplasia dula sea por un linfoma no Hodgkin de clulas grandes B
gstrica con ulceraciones fundamentalmente a nivel del gstrico estadio IV que tras el trasplante presenta un cua-
antro, lquido en cuanta moderada subheptico y pelvis dro febril con sntomas respiratorios y digestivos asociados
menor. El resultado de la anatoma patolgica fue compa- a prurito y eosinofilia. El diagnstico diferencial inicial es
tible con linfoma no Hodgkin de clulas grandes B gstrico muy amplio puesto que se trata de un sndrome febril en
estadio IV de Lugano. Tras ser valorado por Hematologa un paciente inmunodeprimido. Se inici cobertura anti-
se decide tratamiento con R-CHOP (x 6), con PET-TAC bitica de amplio espectro sin que se consiguieran aislar
post 3 y post 6 ciclos negativos. bacterias en los hemocultivos. En el contexto de las dia-
En abril de 2012 presenta una hemorragia digestiva alta, rreas que el paciente presentaba se solicitaron varios co-
realizndose PET-TAC y gastroscopia con biopsia, confir- procultivos que fueron negativos en el inicio, en otro se
mndose la recada. Se inicia una segunda lnea de tra- detect Candida glabrata para la que recibi doble terapia
tamiento de rescate con R-ESCHAP (x 3) sin repuesta. antifngica a pesar de lo cual continuaba sintomtico. Con
Como tercera lnea de tratamiento recibe R-MINE (x 3), la recuperacin de la pancitopenia, aparece en analticas
con PET-TAC de control que muestra discreto aumento de control una hipereosinofilia que es la que finalmente
de captacin gstrica que podra considerarse fisiolgica. dirige y orienta al diagnstico final. Dentro de las causas
En junio de 2013 se realiza auto-trasplante (Auto-TPH), que producen eosinofilia se descartan causas farmacol-
habiendo recibido previamente radioterapia con irradia- gicas o txicas. Tampoco parece tratarse de una reaccin
cin corporal total como acondicionamiento. En el da -3 de hipersensibilidad mediadas por IgE, aunque el paciente
de acondicionamiento comienza con prurito generaliza- tena antecedentes de asma bronquial, ni de enfermedades
do, pautndose tratamiento sintomtico. El da +6 tras el autoinmunes sistmicas como granulomatosis de Churg-
trasplante (post TPH) presenta pico febril junto a diarreas Strauss, fascitis eosinoflica, granulomatosis de Wegener o
acuosas sin productos patolgicos por lo que al tratamien- artritis reumatoide, que cursan con clnica diferente.
to previo (con aciclovir, ciprofloxacino, fluconazol) se le As como tampoco cumple criterios de sndrome hipeosin-
aade vancomicina y piperacilina/ tazobactam. El da +8 oflico o eosinofilia familiar.
post TPH se asocian al prurito sntomas digestivos, cuadro En nuestro caso se realiza frotis de sangre perifrica y co-

185
CAPTULO 10
INFECCIONES TROPICALES
Y DEL VIAJERO

procultivo en busca de parsitos dados los antecedentes


epidemiolgicos del paciente. La presencia de eosinofilia
en el contexto de las infecciones es casi exclusiva de las
infecciones por helmintos. De forma excepcional, puede
aparecer esta alteracin analtica en algunas infecciones
bacterianas (ej. resolucin de una escarlatina o en formas
crnicas de tuberculosis o lepra), enfermedades vricas (re-
solucin de una infeccin vrica o infeccin VIH), micosis
(especialmente en la coccidioidomicosis) o algunas proto-
zoosis concretas (Isospora belli, Dientamoeba fragilis, Sarcocystis
sp y Blastocystis hominis). Aunque las helmintosis son mucho
ms frecuentes en el paciente procedente de regiones tropi-
cales o que ha viajado a las mismas, no debe olvidarse que
existen algunas parasitosis autctonas capaces de producir
eosinofilia. Entre los helmintos que con mayor frecuencia Figura 3. Cultivo en agar sangre.
son capaces de ocasionar eosinofilia se encuentran Echi-
nococcus granulosus, Anisakis spp. Fasciola heptica, Strongyloides disminucin franca del prurito.
stercoralis, Taenia spp. Toxocara canis, Trichinella spp. etc En revisiones posteriores en consulta externa de E. Infec-
Es muy importante considerar los aspectos geogrficos, ya ciosas presenta resolucin de eosinofilia y exmenes de he-
que determinadas parasitosis poseen una distribucin lo- ces repetidos para Strongyloides negativos.
calizada (Ej. Loa-Loa en pases centroafricanos, Clonorchis En la ltima revisin con Hematologa en marzo de 2015
sinensis en Sudeste asitico), mientras otras tienen distribu- la analtica de sangre es completamente normal y el PET-
cin mundial (Ej. Echinococcus granulosus, Strongyloides sterco- TAC no muestra hallazgos sugerentes de lesiones neoplsi-
ralis, etc.) cas macroscpicas, estando en situacin de remisin.

Evolucin Discusin

En el frotis de sangre perifrica no se visualizaron parsi- La estrongiloidiasis es una infeccin producida por Stron-
tos, sin embargo en el estudio de parsitos en heces (figura gyloides stercolaris. Se trata de un nematodo intestinal que se
1 y 2) se detectan larvas de Strongyloides stercolaris. Reinterro- encuentra ampliamente diseminado en el suelo en pases
gando al paciente, ya haba padecido prurito generalizado tropicales y subtropicales y que afecta a unos diez millones
en los ciclos previos de quimioterapia y en las analticas de personas en el mundo aunque la prevalencia exacta no
anteriores al trasplante ya se observaba eosinofilia. se conoce debido a que existen casos de portadores asin-
tomticos. El ciclo vital del parsito es complejo porque
Diagnstico final tiene dos fases, una libre y otra parasitaria y la hembra es
capaz de reproducirse por partognesis dentro del husped
Ante el diagnstico de hiperinfestacin por Strongyloides humano. En el ciclo de vida libre, las larvas rabditiformes
stercolaris se inicia tratamiento con ivermectina 15 mg/da son eliminadas con las heces y pueden convertirse en larvas
durante 3 das y se realizan controles de coprocultivo pos- filariformes infectantes (que son las que penetran en la piel
teriores que son siempre negativos. Tras el tratamiento, el del ser humano) o en machos y hembras adultos que viven
paciente no presenta diarrea, ni sntomas respiratorios, con libremente, que se aparean y producen huevos que eclosio-
narn como larvas rabditiformes de nuevo. Como acaba-
mos de mencionar, el inicio del ciclo parasitario se produce
cuando las larvas filariformes atraviesan la piel humana y
son transportadas va hematgena hasta los alvolos atra-
vesando los capilares pulmonares. De ah migran por la
trquea hasta la faringe, donde son deglutidas y llegan has-
ta el intestino delgado. Es en este tramo intestinal donde se
transforman en hembras que son capaces de poner huevos
Figura 1. Frotis directo de heces. Figura 2. Frotis directo de heces. que posteriormente eclosionaran larvas no infectivas que

186
CAPTULO 10
INFECCIONES TROPICALES
Y DEL VIAJERO

sern eliminadas por las heces. de heces para identificar las larvas. Las tcnicas para de-
La clnica de infeccin aguda aparece al poco de produ- tectarlas incluyen frotis directo con tincin de lugol yodo,
cirse la exposicin y puede durar semanas. Al inicio del acetato de etilformalina, el papel de filtro de Harada-Mori
cuadro suelen predominar los sntomas respiratorios (tra- y cultivos en placas de agar nutriente. El mtodo de culti-
queitis, tos, bronquitis) y a las dos semanas comienza la vo en placa de agar sangre, en el que las bacterias crecen
clnica gastrointestinal (anorexia, dolor abdominal, diarrea siguiendo los trayectos serpiginosos de las larvas mviles a
y vmitos). No es hasta la tercera o cuarta semana cuando las 24 48 horas de la incubacin a temperatura ambien-
se detectan las larvas en las heces. te, es el mtodo de eleccin debido a su alta sensibilidad
En algunos casos, las larvas en la luz intestinal pueden vol- y facilidad de implementacin en los laboratorios de mi-
verse infectivas y volver a atravesar la pared, proceso de- crobiologa. En la hiperinfestacin, las larvas filariformes
nominado autoinfeccin, e iniciar as de nuevo el ciclo y pueden aislarse en heces, esputo, sangre, lquido asctico e
dando lugar a una infeccin crnica. incluso en lquido cefalorraqudeo. Los test serolgicos son
La persistencia del parsito en el husped no suele dar sensibles pero muy poco especficos, dan falsos negativos
sntomas en el inmunocompetente pero si el paciente pa- en pacientes inmunodeprimidos y falsos positivos en infec-
dece algn trastorno que altere su inmunidad, el parsito ciones por helmintos.
se disemina dando lugar a dos sndromes potencialmente Todos los pacientes infectados por Strongyloides stercolaris de-
graves: la hiperinfestacin por Strongyloides y la estrongiloi- ben ser tratados independientemente de que tengan snto-
diasis diseminada. mas o no. De los frmacos antiparasitarios, la ivermectina
La hiperinfestacin es debida a un aumento masivo de las es la que ha demostrado ms eficacia frente a albendazol
larvas en la circulacin, que escapan de su circuito habi- y tiabendazol. Para los casos de infeccin localizada se
tual atravesando el duodeno e infectan a otros rganos, recomienda ivermectina 200 mcg/kg al da durante dos
especialmente al pulmn. En pacientes inmunodeprimi- das. No hay regimenes estndar para el tratamiento de la
dos puede producir enterocolitis grave, empeoramiento hiperinfeccin o enfermedad diseminada. Se debe admi-
de la funcin pulmonar con tos, sibilancias y ronquera, nistrar hasta que dejen de detectarse larvas en al menos
y en algunos pacientes puede progresar a una insuficien- dos muestras separadas unas dos semanas. En caso de que
cia respiratoria aguda y la muerte. Entre los factores de el paciente no responda, se debe asociar albendazol o tia-
riesgo principales para la hiperinfestacin se encuentran bendazol.
la toma crnica de corticoides, el trasplante de clulas he- El pronstico de la hiperinfestacin es malo con una tasa
matopoyticas, el alcoholismo y la infeccin por VIH y por de mortalidad que alcanza el 80% si el tratamiento no se
HLTV-1. instaura de manera precoz.
En la enfermedad diseminada por Strongyloides pueden Es necesario realizar screening a pacientes naturales de zo-
encontrarse afectados adems rin, hgado, corazn, nas tropicales o que han viajado a esas regiones en caso
glndulas endocrinas y el sistema nervioso central. Los sn- de que presenten alguna causa de inmunodepresin (VIH,
tomas principales suelen ser fiebre, dolor abdominal, tos neoplasias hematolgicas, trasplante, toma de corticoides,
persistente, prdida de peso, vmitos y anemia. En las for- etc) para poder detectar posibles casos de estrongiloidiasis
mas graves, existe riesgo de traslocacin bacteriana a nivel a tiempo.
intestinal con la consecuente bacteriemia.
Para el diagnstico es necesaria una alta sospecha clnica
dado que los sntomas pueden ser muy inespecficos. En
ocasiones, la eosinofilia es el dato gua pero ocurre que en
las infecciones crnicas la subida de eosinfilos puede ser
intermitente y en la forma de hiperinfestacin, dada la in-
munodepresin del paciente, tanto la serie blanca como
la roja suelen estar alteradas por lo que la subida de estas
clulas puede no ser evidente.
No existe una prueba gold estndard que confirme la
infeccin por Strongyloides. El test de inicio se basa en el exa-
men de heces en busca de larvas aunque un nico intento
puede ser fallido hasta en 70% de los casos. Pueden llegar
a ser necesarios hasta siete exmenes de distintas muestras

187
CAPTULO 10
INFECCIONES TROPICALES
Y DEL VIAJERO

Bibliografa

1. Paul B. Keiser and Thomas B. Nutran. Strongyloides


stercoralis in the Immunocompromised Population.
Clin Microbiol Rev. 2004; 17: 20817.
2. Llagunes J, Mateos E, Pea JJ, Carmona P, De Andrs
J. Hiperinfeccin por Strongyloides stercoralis. Med
Intensiva 2010;34:3536.
3. Domnguez-Castellano A, Molina G i l - B e r m e j o
J. Sociedad Andaluza de Enfermedades Infecciosas
(SAEI). Varn subsahariano de 34 aos con epigastral-
gia y vmitos de cinco meses de evolucin. Med Clin
(Barc). 2011;136: 44753.
4. Izquierdo I, Briones J, Lluch R, Arqueros C, Martino
R. Fatal Strongyloides Hyperinfection Complicating
a Gram-Negative Sepsis after Allogeneic Stem Cell
Transplantation: A Case Report and Review of the
Literature. Case Rep Hematol2013;(2013):860976.
5. Ramrez-Olivencia G, Cuesta Espinosa MA, Bravo
Martn A, Iglesias Nez N, Rodrguez de las Parras
E, Lago Nez M, Puente Puente S. Imported stron-
gyloidiasis in Spain. Int J Infect Dis 2014; 18: 327.
6. Prez-Arellano JL, Pardo J, Hrnandez Cabrera M,
Carranza C, ngel-Moreno A, Muro A. Manejo
prctico de una eosinofilia. An Med Interna (Madrid)
2004;21:244-52

188
CAPTULO 11
INFECCIONES EN PACIENTES
INMUNODEPRIMIDOS Y TRASPLANTADOS
Neumona en paciente con artritis reumatoide
en tratamiento con anti TNF- (etanercept)
Morales Jimnez, G; Gallo Padilla, L;
Anguita Santos, F.
Complejo Hospitalario Universitario de Granada
(Hospital Universitario San Cecilio). Granada.

Caso clnico

Mujer de 50 aos de edad diagnosticada de Artritis Reu-


matoide desde 2011, hipotiroidismo subclnico, hipovita-
minosis D y tabaquismo activo (25 cigarrillos/da desde
hace 34 aos). La paciente segua el siguiente tratamiento
desde diciembre de 2013: etanercept 50 mcg (1 inyeccin
subcutnea/semana), metotrexato (10 mg orales cada 12
horas los lunes), cido flico (5 mg los martes), predniso-
na (5 mg diarios), hidroferol (1 ampolla diaria) y analgesia
condicional a dolor. Estaba vacunada correctamente fren- Figura 1. Radiografa de trax al ingreso en UCI.
te a neumococo, virus de la hepatitis B y virus de la gripe y
presentaba un Mantoux negativo previo al inicio del trata- descartar, si bien, las infecciones fngicas producidas por
miento con antiTNF-. Aspergillus, Histoplasma, Criptococo y Pneumocystis jirovecii, entre
Acudi al servicio de urgencias por malestar general, mial- otras, y algunas bacterianas como Legionella y Neumococo, u
gias y fiebre de hasta 39C que se haba iniciado cuatro otras menos comunes, como Listeria o Nocardia tambin de-
das antes. Asociaba adems, tos con expectoracin puru- beran formar parte del diagnstico diferencial. De entre
lenta, disnea que haba ido progresando hasta hacerse de las infecciones vricas que se manifiestan con frecuencia en
mnimos esfuerzos y deposiciones diarreicas (2-3 veces/da) forma de neumona en este tipo de pacientes, habra que
sin productos patolgicos desde el comienzo del cuadro. considerar adenovirus, influenza (A y B) y parainfluenza, virus
Durante las primeras horas presenta hipotensin franca respiratorio sincitial (VRS) y citomegalovirus (CMV).
refractaria a una correcta reposicin hdrica, oligoanuria, La tuberculosis pulmonar es probablemente la infeccin
fiebre resistente a antitrmicos e hipoxemia (SpO2 90% respiratoria cuya asociacin ms se ha estudiado en pa-
con una FiO2 del 28%), por lo que se consulta con la Uni- cientes tratados con antiTNF-. Estudios recientes mos-
dad de Cuidados Intensivos (UCI) y se procede al traslado. traron que Espaa y Portugal tienen tasas sustancialmente
A su llegada destacaban los siguientes resultados analti- ms altas que otros pases de Europa en este tipo de pa-
cos: neutrofilia del 97% sin leucocitosis, funcin renal sin cientes. La tuberculosis habitualmente ocurre poco tiem-
alteracin, hiponatremia leve (Sodio 132 mEq/L) sin otras po despus del inicio del antiTNF-, generalmente como
alteraciones electrolticas, LDH 602 U/l y protena C re- reactivacin de una infeccin latente, mientras que, cuan-
activa 405 mg/l. No presentaba coagulopata y en gasome- do aparece tardamente, representara (en la mayora de
tra venosa destacaba un pH 7.51, pO2 21 mmHg, pCO2 casos) una nueva infeccin que progresa de forma directa
47 mmHg, bicarbonato 16.8 mmol/L y sat O2 88%. En a enfermedad activa. El amplio abanico de sntomas que
la radiografa de trax era evidente una condensacin en puede causar sta hace que siempre forme parte del diag-
el lbulo superior izquierdo (LSI), bien delimitada y con nstico diferencial en estos casos.
broncograma areo y atelectasia basal izquierda (Fig.1) La criptococosis (Cryptococcus neoformans y Cryptococcus gattii)
es una infeccin fngica invasiva cada vez ms prevalente
Diagnstico diferencial en pacientes inmunocomprometidos. El microorganismo
se encuentra normalmente en el suelo y en altas concen-
Estn bien recogidas en la literatura las potenciales in- traciones cerca de nidos de aves, pudiendo producirse la
fecciones oportunistas que con relativa frecuencia se pre- inhalacin. En modelos animales de infeccin por Crypto-
sentan en pacientes tratados con frmacos con actividad coccus, la inhibicin de TNF- est asociada con proble-
antiTNF-. La afectacin pulmonar de nuestra paciente mas de reclutamiento de clulas T activadas CD4 + y clu-
hara de la tuberculosis una de las principales entidades a las de Langerhans en la zona de la infeccin, conllevando

190
CAPTULO 11
INFECCIONES EN PACIENTES
INMUNODEPRIMIDOS Y TRASPLANTADOS

un mayor riesgo de difusin. Adems de afectar al SNC, alguna alteracin radiolgica en el tercer da. El patrn
puede tambin causar infecciones respiratorias, pudien- ms comn es un infiltrado unilobular irregular que pro-
do manifestarse desde una forma asintomtica hasta un gresa hacia la consolidacin, siendo los derrames pleurales
sndrome de distrs respiratorio o una neumona. Herra- tambin comunes. El diagnstico de la infeccin se basa en
mientas de diagnstico incluyen: investigacin de hongos la clnica, el cultivo (especificidad aproximada del 100%,
y determinacin del antgeno del criptococo en suero y, pero sensibilidad variable segn se trate de esputo (60%),
en un gran nmero de casos, es preciso el anlisis de una sangre (40%) o biopsia pulmonar (90%)), la antigenuria y
muestra histolgica. la visualizacin por inmunofluorescencia directa.
En la aspergilosis pulmonar en pacientes inmunocompro- En los ltimos aos se han detectado cada vez ms casos
metidos el principal responsable suele ser Aspergillus fumiga- de infeccin por Mycoplasma pneumoniae en adultos, especial-
tus, y puede presentar una gran variedad de sntomas. La mente ancianos. La transmisin se produce por va area
trada clsica incluye fiebre, dolor pleurtico y hemoptisis. y suele ocurrir de forma epidmica. La clnica puede ser
Sin embargo, la ausencia de esta trada no es infrecuente, y superponible a la de una neumona tpica, aunque ocasio-
debe considerarse esta enfermedad en pacientes con facto- nalmente existe un dato caracterstico, la miringitis bullo-
res de riesgo en los cuales la radiografa de trax a menudo sa, que apoya el diagnstico. Los mtodos de diagnstico
revela ndulos y/o infiltrados pulmonares. El diagnstico incluyen el cultivo, la reaccin en cadena de la polimerasa
de la aspergilosis invasiva se basa en el aislamiento del mi- (PCR), y la serologa con IgM e IgG (ms frecuente). Di-
croorganismo (o marcadores del mismo como el galacto- ferentes cuestiones pueden dificultar el manejo, como el
manano srico), pruebas de imagen y una alta sospecha tiempo que tarda en obtenerse el resultado de la serologa
clnica. y la falta de correlacin completa que se ha dado en dife-
Se ha descrito neumona por Pneumocystis jirovecii en pacien- rentes ensayos al comparar resultados de PCR, serologa
tes tratados con inhibidores de TNF-, especialmente en y cultivo
tratamiento con otros inmunosupresores concomitantes. Dentro de los influenza virus, destacamos por su poten-
La transmisin se produce por va area mediante contac- cial gravedad y la posibilidad de provocar pandemias el
to persona-persona. El diagnstico debe considerarse en tipo A. Pese a que la distribucin suele ser estacional, no
pacientes con factores de riesgo (neoplasias, especialmente es infrecuente encontrar esta infeccin en pacientes inmu-
hematolgicas, VIH, trasplantados, pacientes con trata- nodeprimidos fuera de las estaciones tpicas. En estos ca-
miento inmunosupresor, etc.) que presentan una imagen sos no es extrao encontrar una neumona gripal primaria
radiolgica compatible (compromiso intersticial de predo- (que generalmente tiene un pronstico muy grave, con una
minio perihiliar bilateral, con opacidades reticulares o no- mortalidad cercana al 80%), algo mucho ms infrecuente
dulillares y/o reas de consolidacin irregular), y se realiza en la poblacin general, donde la principal complicacin
por microscopa mediante la tincin de una muestra de es la neumona bacteriana secundaria a la infeccin gri-
esputo (o del lavado bronquioalveolar). pal. El diagnstico puede hacerse por cultivo, serologa, o
Listeria monocytogenes tiene como mecanismos de contagio a travs de pruebas de diagnstico rpido con la enzima
principales el consumo de alimentos y bebidas contamina- inmunoensayo (EIA), inmunofluorescencia, o PCR. Por
das o el contacto estrecho con animales. El principal factor otra parte, el Virus Respiratorio Sincitial (VRS), al igual
de riesgo es, sin duda, el estado de inmunodepresin. Aun- que la gripe, suele ser causante de infecciones respirato-
que son ms comunes las bacteriemias, infecciones gas- rias de manera estacional. La transmisin se produce de
trointestinales o del sistema nervioso central, se han dado manera tambin similar, provocando una afectacin inicial
casos de listeriosis pulmonar, que generalmente tienen un del bronquiolo que puede evolucionar a una neumona.
pronstico grave. El diagnstico definitivo se obtiene me- Existen mtodos de diagnstico rpido basados en la PCR
diante aislamiento en diferentes muestras de cultivo, de- viral que detectan mltiples virus respiratorios de mane-
pendiendo de la clnica. ra simultnea y rpida. No obstante, es necesario ser cau-
Legionella pneumophila se encuentra generalmente en am- telosos en la interpretacin, ya que hasta el 15% de las
bientes hmedos. La transmisin se produce por va res- personas sanas albergan alguno de estos virus en el tracto
piratoria y puede ser la causante de una neumona con respiratorio.
caractersticas atpicas y una sintomatologa que puede El citomegalovirus (CMV) es uno de los principales cau-
llegar a ser muy florida (tos, fiebre, diarrea, hiponatremia, santes de infecciones en pacientes inmunodeprimidos, es-
fracaso renal, confusin, ataxia, etc.) y que le confiere una pecialmente en trasplantados, ya sea de forma primaria
importante gravedad. Casi en el 100% de los casos existe o como reactivacin. Por ello, siempre deben solicitarse

191
CAPTULO 11
INFECCIONES EN PACIENTES
INMUNODEPRIMIDOS Y TRASPLANTADOS

serologas (presencia de IgM o aumento x4 de IgG) y/o


deteccin de la carga viral por PCR.

Evolucin

La paciente ingres en la Unidad de Cuidados Intensivos


donde se inici aporte de vasoactivos as como tratamiento
antibitico emprico con levofloxacino 500 mg y ceftriaxo-
na 2 gramos.
Se solicitaron hemocultivos seriados, urocultivo, cultivo de
esputo, baciloscopias (x3) y antgenos de Legionella y Neu- Figura 2. TAC con contraste intravenoso realizado en planta.
mococo en orina. En las primeras 12 horas se recibi una
antigenuria positiva para Legionella (con confirmacin pos- ligera prdida de volumen y atelectasias laminares bibasa-
terior mediante inmunofluorescencia y tras tratamiento les. (Fig. 2)
trmico de la orina), resultando tambin positivo para la Desde el primer momento del ingreso se retiraron tanto el
misma un cultivo de esputo en das posteriores. Se ajust antiTNF- como el metotrexato y se aument la dosis de
el tratamiento manteniendo levofloxacino 500 mg/12 ho- prednisona (30 mg/da), mantenindose la paciente sin da-
ras y sustituyndose la cefalosporina por azitromicina 500 tos de actividad de su proceso reumatoideo. Al alta conti-
mg/24 horas. El resto de pruebas microbiolgicas solicita- nu slo tratamiento con prednisona en pauta descendente
das resultaron negativas. (por indicacin de Reumatologa) y complet tratamiento
La evolucin fue trpida durante los primeros das con con Levofloxacino durante 6 das ms. Actualmente, se ha
desaturaciones repetidas que obligaron a iniciar Oxigeno- reiniciado tratamiento con metotrexato (5 mg orales/12
terapia Nasal de Alto Flujo (ONAF) sin llegar a precisar horas los lunes) y prednisona 5 mg/24 horas, sin necesidad
ventilacin mecnica. As mismo, se mantuvo dependiente de reiniciar antiTNF-.
de vasoactivos para matener una TAM>70 mmHg y pre-
sent un cuadro sincopal. Se realiz un ecocardiograma Diagnstico final
que mostr un derrame pericrdico leve y se descart la
presencia de un empiema mediante ecografa torcica. Shock sptico secundario a neumona multilobar por Legio-
Clnicamente se mantuvo alerta y consciente, con ortop- nella en paciente inmunodeprimida
nea y taquipnea. Persistieron accesos de tos con expectora-
cin purulenta y cedi el cuadro intestinal en las primeras Discusin
24 horas (coprocultivo y toxinas de Clostridium difficile ne-
gativas). Al 5 da comenz a experimentar mejora pro- Si bien existe cierta controversia respecto al papel de los
gresiva, pudiendo retirarse la ONAF y la noradrenalina y antiTNF- en el aumento de incidencia de diversas infec-
desapareciendo los picos febriles. Finalmente, se traslad a ciones, teniendo en cuenta su accin sobre el sistema in-
nuestro servicio de Enfermedades Infecciosas tras 10 das mune y los datos recogidos en los estudios ms recientes, s
de estancia en UCI (ya sin azitromicina, que se mantuvo parece existir una asociacin clara, al menos, con las infec-
durante 8 das), donde continu con una evolucin favora- ciones causadas por microorganismos intracelulares. En el
ble, pudiendo procederse a su alta hospitalaria. ao 2011, la FDA (Food and Drug Administration) public
Analticamente cabe destacar: evolucin favorable de la una alerta advirtiendo el riesgo de infeccin por Legionella
Protena C Reactiva (al alta 32.09 mg/l), determinacin y Listeria en pacientes en tratamiento con cualquier an-
de procalcitonina inicial de 5.2 ng/ml (al alta 0.79 ng/ml), ti-TNF-, resaltando la especial importancia que tena la
funcin renal conservada y discreta elevacin de GPT (57 valoracin riesgo-beneficio en pacientes en los que se iba
U/l) y GGT (67 U/l). a iniciar este tipo de tratamiento. Por tanto, el tratamiento
Otras pruebas complementarias solicitadas: determinacio- antibitico inicial de las mismas debera incluir siempre un
nes serolgicas para Coxiella burnetti, Chlamydia pneumoniae y frmaco activo frente a este tipo de microorganismos.
Mycoplasma, PCR para tipificacin de especie de Legionella Las infecciones en ese grupo de pacientes parecen darse
(realizada en Majadahonda, serotipo 1) y TAC torcico con mayor frecuencia durante los 6 primeros meses de tra-
con contraste (consolidaciones extensas en LSI y en lbulo tamiento, y existen otros factores como la edad o el uso
inferior homolateral, con abundante broncograma areo, de otros inmunomoduladores concomitantes que se han

192
CAPTULO 11
INFECCIONES EN PACIENTES
INMUNODEPRIMIDOS Y TRASPLANTADOS

propuesto en distintos trabajos como factores de riesgo Bibliografa


adicionales, aunque con diferentes resultados en cuanto
a su significacin estadstica. En nuestro caso, la paciente 1. Stone JH. Tumor necrosis factor-alpha inhibitors: Risk
desarroll la infeccin 10 meses despus de iniciar el trata- of bacterial, viral, and fungal infections. En: UpToDate,
miento con Etanercept y, en efecto, se encontraba tambin Post TW (Ed), UpToDate, Waltham, MA.
en tratamiento con corticoides y metotrexato, lo cual apo- 2. Wallis SW. Tumor necrosis factor-alpha inhibitors:
ya las dificultades de algunos autores para asegurar una Risk of bacterial, viral, and fungal infections. En: Up-
relacin directa con el tratamiento antiTNF-. Por otra ToDate, Post TW (Ed), UpToDate, Waltham, MA.
parte, la mayora de estudios coinciden en que el riesgo es 3. Lanternier F, Tubach F, Ravaud P et al; Research
menor con Etanercept que con Infliximab o Adalimumab Axed on Tolerance of Biotherapies Group. Incidence
por su diferente mecanismo de accin, no obstante, no son and risk factors of Legionella pneumophila pneumo-
pocos los casos comunicados con el primero, por lo que nia during anti-tumor necrosis factor therapy: a pros-
esta afirmacin no debe reducir el grado de sospecha de pective French study. Chest. 2013;144:990-8.
los clnicos. 4. Bodro M, Carratal J, Paterson DL. Legionellosis and
El diagnstico de Legionelosis en este caso se realiz de biologic therapies. Respir Med. 2014;108:1223-8.
manera adecuada y facilit en gran medida el tratamiento. 5. Ellerin T, Rubin RH, Weinblatt ME. Infections and
Resulta imprescindible solicitar tanto el antgeno en orina AntiTumor Necrosis Factor Therapy. Arthritis &
como un cultivo de esputo (incluso del lavado broncoal- Rheumatism. 2003; 48: 3013-22.
veolar si la sospecha es alta y la evolucin desfavorable). La
prueba de deteccin del antgeno urinario (rpida, sensi-
ble, especfica y barata), slo es til para el diagnstico de
infeccin de L. pneumophila tipo 1 (el serotipo, por otra par-
te, ms frecuente). La recomendacin general es realizar
tanto el cultivo en medios especficos (capaz de detectar in-
fecciones por cualquiera de las especies y serogrupo) como
la antigenuria. Es interesante destacar que el tratamiento
trmico de la orina no supone la desaparicin de la positi-
vidad (en infeccin cierta) y s la eliminacin de falsos po-
sitivos, dado que el antgeno detectado en la prueba es un
componente soluble del lipopolisacrido de la pared celu-
lar de Legionella y es termoestable. Respecto al tratamiento,
se emplearon Levofloxacino y Azitromicina. Si bien an
existe controversia sobre la idoneidad de la monoterapia
frente a la biterapia, lo que s se ha establecido en distintos
ensayos comparativos es que los macrlidos ms nuevos
y las quinolonas respiratorias son la terapia de eleccin,
mostrndose superiores frente a tratamientos previos. Para
los pacientes inmunodeprimidos con enfermedad grave, la
pauta de tratamiento ms usada consiste en levofloxacino
750 mg una vez al da durante 21 das.
En conclusin, la tasa de incidencia de legionelosis en los
pacientes que reciben tratamiento con antiTNF- es alta
en comparacin con respecto a la poblacin general. Por
tanto, siempre debera solicitarse la antigenuria en com-
binacin con el cultivo, y el tratamiento antibitico de
primera lnea para la neumona en pacientes que reciben
antiTNF- debera ser activo contra L. pneumophila.

193
Sndrome febril y lesin cavitada pulmonar
en paciente inmunodeprimido
Rivera Cabeo, I; Fernndez Fernndez, R;
Reguera Iglesias, JM.
Hospital Regional de Mlaga

Caso clnico

Varn de 50 aos, sin alergias medicamentosas conoci-


das, residente en Ceuta y sin datos epidemiolgicos de in-
ters. Antecedentes personales de hipertensin arterial y
diagnosticado de cirrosis heptica por virus de hepatitis C
(VHC) y alcohol (grado C de Child y MELD 25), con ml-
tiples complicaciones (encefalopata heptica, hemorragia
digestiva alta, ascitis refractaria y peritonitis bacteriana es-
pontnea).
Recibi trasplante heptico ortotpico de cadver en
mayo del 2010 y se llev a cabo tratamiento inmunosupre-
sor con cellcept (micofenolato mofetilo) 1.000 mg/12h,
sandimmun (ciclosporina) 75 mg/12h que mantiene has-
ta la actualidad, adems de omeprazol y captopril. Buena Figura 1.
evolucin postrasplante hasta seis meses antes del ingreso,
en que present cuadro de ictericia secundaria a estenosis U/I, FA 379 U/L, GGT 406 U/I, tiempo de protrombi-
de la anastomosis biliar, recibiendo tratamiento de dilata- na (TP) 60% y gasometra venosa con pH 7,2, pCO2 23
cin con baln hidrosttico mediante CPRE. Una biopsia mmHg, pO2 30.9 mmHg, HCO3 11 mmol/l. Una radio-
heptica en ese momento fue compatible con recada del grafa de trax mostr un infiltrado en lbulo superior de-
VHC sobre el injerto. Tras el alta no acudi a consulta de recho (LSD) as como otro infiltrado cavitado en lbulo in-
seguimiento. ferior derecho (LID) (Figura 1). En la ecografa abdominal
En marzo del 2014 el paciente acude al hospital por cua- se observ ascitis en cuanta moderada en ambas gotieras
dro de malestar general, tos con expectoracin verdosa, parietocolicas y pelvis menor y esplenomegalia de 16 cm.
sensacin distrmica no termometrada y disnea progre- Tras la extraccin de hemocultivo, urocultivo, cultivo de
siva de 10 das de evolucin. En la exploracin fsica se esputo, antigenuria a neumococo y Legionella, frotis na-
encontraba consciente y orientado, con mal estado gene- so-faringeo para descartar virus influenza y PCR a CMV,
ral, hipotenso (70/40 mm de Hg), taquicrdico (110 Lat/ se inici tratamiento antibitico con ceftriaxona y levo-
min), taquipneico con uso de musculatura accesoria y fe- floxacino, soporte hemodinmica con fluidos e inotropos
bril (39). Ictericia intensa de piel y mucosas. No presenta (noradrenalina) e ingreso en la unidad de cuidados inten-
alteraciones en cabeza y cuello. Los tonos cardiacos son sivos (UCI). Se estableci el diagnstico de shock sptico
rtmicos y sin soplos, presentando una hipoventilacin ge- secundario a neumona comunitaria cavitada en paciente
neralizada en la auscultacin respiratoria. El abdomen era inmunodeprimido.
blando y depresible con cicatriz de laparotoma previa, sin
dolor a la palpacin, masas ni organomegalias. En las ex- Diagnstico diferencial
tremidades inferiores los pulsos pedios estaban presentes y
no existan edemas. Estamos ante un varn de 50 aos receptor de un tras-
En las pruebas complementarias realizadas inicialmente plante heptico y en tratamiento con micofenolato y ci-
destaca: hemoglobina 10 g/dl, leucocitos 31.450/mm3 closporina que presenta un cuadro de neumona cavitada
con neutrofilia del 95%, plaquetas 102.000 mm3, PCR de la comunidad que evoluciona de manera rpida a shock
169 mg/l, procalcitonina 4.76 ng/ml, glucosa 95 mg/dl, sptico y fallecimiento del paciente. Posiblemente las claves
creatinina 4,46 mg/dl (previa 2,25 mg/dl), bilirrubina to- para el diagnostico nos la ofrezca la imagen radiolgica y
tal 21,6 mg/dl (directa 18 mg/dl), GOT 40 U/I, GPT 23 el curso clnico rpidamente progresivo, que prcticamen-

194
CAPTULO 11
INFECCIONES EN PACIENTES
INMUNODEPRIMIDOS Y TRASPLANTADOS

te descartan cualquier etiologa no infecciosa, como podra


ser la patologa inflamatoria y la tumoral.
Los procesos infecciosos siguen un patrn temporal deter-
minado en los pacientes trasplantados. Clsicamente se
establecen tres periodos postrasplante segn el riesgo para
cada tipo de infeccin. El periodo temprano comprende
el primer mes postrasplante y es cuando se producen in-
fecciones postoperatorias convencionales, similares a la de
pacientes imunocompetentes que presentan ciruga, y ge-
neralmente nosocomiales. El perodo intermedio, desde el
2 al 6 mes, se caracteriza por ser el de mxima depresin
funcional de los linfocitos T. Se producen infecciones por
patgenos intracelulares como CMV, virus del herpes hu-
mano 6 (HHV-6), P. jirovecii, C. neoformans, T. gondii y reacti- Figura 2.
vacin de M. tuberculosis. El tercer perodo comienza tras el
6 o mes postrasplante y en l la incidencia de infecciones y Las micobacterias seria otro grupo etiolgico a conside-
tumores depende fundamentalmente del estado de inmu- rar y sobre todo la tuberculosis. La tuberculosis es hasta
nodepresin farmacolgica, infecciones previas, episodios 70 veces ms frecuente en los receptores de un trasplante
de rechazo, necesidad de dilisis y disfuncin crnica del de rgano slido que en la poblacin general. La imagen
injerto. radiolgica que presentaba nuestro paciente se correspon-
Nuestro paciente se encontraba en el tercer periodo (4 dera ms con la afectacin pulmonar de la tuberculosis
ao postrasplante), pero presentaba una inmunodepresin posprimaria. Aunque esta se caracteriza por afectar pre-
farmacolgica importante y una mala funcin del injerto. ferentemente a los lbulos superiores (segmentos apical y
Estos dos factores obligan a reconsiderar su situacin como posterior), presentar cavitaciones pulmonares y en aproxi-
un periodo intermedio prolongado, caracterizado por la madamente la mitad de los casos la afectacin es bilateral.
depresin funcional de los linfocitos T. Clnicamente suele presentarse de forma ms subaguda.
Por tanto, diversas infecciones podran explicar el cuadro No podemos descartar una neumona adquirida en comu-
de nuestro paciente. El CMV es un agente infeccioso muy nidad (NAC) como posible causa del proceso de nuestro
frecuentemente aislado en pacientes sometidos a trasplan- paciente y de hecho el tratamiento inicial as lo hizo sospe-
te heptico. Aunque la enfermedad mas frecuente es la char. Las NAC en los pacientes inmunodeprimidos suelen
hepatitis, tambin puede causar neumonitis, caracterizada tener la misma etiologa que en inmunocompetentes y la
tpicamente por afectacin intersticial bilateral difusa, con mayor parte de los casos estn producidos por S. pneumo-
patrn en vidrio deslustrado e insuficiencia respiratoria. niae, H. influenzae. y Legionella spp. Los agentes productores
Son muy raros los infiltrados focales y menos aun cavita- de neumonas atpicas Mycoplasma pneumoniae o Chlamydia
dos. Otros patgenos virales como VHS, varicela-zoster, pneumoniae no suelen producir imgenes radiolgicas pare-
virus herpes-6, aunque pueden producir afectacin respi- cidas a las del caso y otros patgenos bacterianos serian
ratoria no suelen producir un cuadro clnico similar. muy infrecuentes, salvo S. aureus, aunque lo habitual es su
Ms del 80% de la infecciones fngicas que ocurren en crecimiento en hemocultivo. Para finalizar Nocardia ssp,
pacientes trasplantados hepticos estn producidas por pude causar neumona cavitada, pero no es usual un curso
Candida spp. y Aspergillus spp. Candida spp. es excepcional que fulminante.
se comporte como un patgeno pulmonar primario. Asper- En este momento nuestro paciente queda pendiente de las
gillus spp. es tambin un patgeno de las etapas temprana o pruebas complementarias y mientras tanto se llevo a cabo
intermedia postrasplante. Raramente ocurre en fases ms un tratamiento antimicrobiano emprico y dirigido a aque-
tardas, salvo que haya situacin de inmunodepresin in- llas entidades que ms probablemente podan ser causa de
ducida. La afectacin respiratoria es lo habitual y la TC este proceso.
de trax es la tcnica de eleccin para el diagnstico, ob-
servndose ndulos, usualmente mltiples, cavitados, de Evolucin
localizacin preferentemente perifrica y que a veces exhi-
ben el denominado signo del halo, imagen causada por la Tras ingresar en UCI se realiz un TAC de trax donde se
hemorragia que circunda la lesin necrtica central. evidenciaron infiltrados algunos cavitados de predominio

195
CAPTULO 11
INFECCIONES EN PACIENTES
INMUNODEPRIMIDOS Y TRASPLANTADOS

en segmentos posteriores de LSD, importante consolida- Discusin


cin con broncograma areo en LID y rea de cavitacin
con nivel en su interior, pequeos focos de vidrio deslus- Rhodococcus equi es un cocobacilo Gram positivo (en medio
trado de predominio en ambos LLSS, sin adenopatas slido es coco y en medio lquido es bacilo), acido alcohol
de tamao significativas (Figura 2). resistente, no fermentador. Se considera un microorganis-
A las 24 horas de su estancia en UCI y ante el deterioro de mo productor de zoonosis, comportndose como patge-
la situacin respiratoria se procedi a intubacin oro-tra- no intracelular y afectando fundamentalmente a humanos
queal, as mismo se inicio depuracin renal mediante con deterioro de la inmunidad celular. Se aisl por primera
hemofiltracin (CVVH) y se incrementaron las dosis de vez en 1923 con el nombre de Corynebacterium equi, pero en
noradrenalina para intentar mantener la estabilidad he- 1980 se modific el trmino a Rhodococcus ya que la com-
modinmica. Se extrajeron nuevos cultivos (broncoaspira- posicin de su pared lo hace ms parecido a nocardia y
do y hemocultivo) y se aadi caspofungina al tratamiento mycobacterias que a corinebacterias. La principal va de
antimicrobiano. En el esputo extrado en urgencias, la tin- adquisicin es la exposicin a tierra contaminada con es-
cin auramina fue positiva para coco-bacilos acido-alcohol tircol de caballo (entre el 32 y 50% de los casos descritos
resistentes los cuales adquirieron caractersticas tintoriales haba contacto con animales de granja o estircol) aunque
de Gram positivos en la tincin de Gram. La antigenuria tambin se puede adquirir por ingestin/inhalacin de
a neumococo y Legionella, la PCR a Influenza A y B, as material contaminado, inoculacin traumtica o sobrein-
como la PCR a CMV y la IFD en esputo a P. jiroveci fueron feccin de heridas1,2.
negativas. Afecta principalmente a personas con inmunidad celular
En una nueva analtica presentaba leucocitos 55.400 mm3 muy deteriorada, sobre todo pacientes VIH con CD4<200,
(97% neutrfilos), Hb 9,2 mg/dl, plaquetas 80.000 mm3, tumores linfoproliferativos, trasplante de rgano slido y
tiempo de protrombina 30%, factor V 20%, urea 231 mg/ tratamiento inmunosupresor. La presencia de una infec-
dl, creatinina 3,45 mg/dl (hemofiltracin), Bilirrubina cin por R. equi en un paciente inmunocompetente obliga
total 20,3 mg/dl y en la gasometra persista la acidosis a realizar un estudio de inmunidad3.
metablica. La evolucin clnica no mostr cambios, per- El primer caso de infeccin en humanos fue descrito en
sistiendo la situacin de shock. Se recibieron los resultados 1967 (anteriormente solo se haban descrito infecciones en
provisionales de los cultivos extrados previamente (hemo- animales, sobre todo potros y cerdos), y an siendo una en-
cultivo, cultivo de esputo y broncoaspirado) encontrndose tidad rara, su incidencia se vio incrementada a partir de los
en todos ellos coco-bacilos Gram positivos identificados aos 80 debido a la pandemia de VIH (era preHAART).
por MALDI-TOF como Rhodococcus equi. La PCR para S.Topino et al4 realizaron una revisin de la literatura, en
DNA de Mycobacterium tuberculosis complex fue negativa. la que recopilaron 272 casos de infeccin por R.equi en
Tras dicho resultado se sustituyo la ceftriaxona por mero- pacientes VIH entre 1986 y 2008. La mayora de los pa-
penem, continuando con levofloxacino y caspofungina. No cientes eran varones con una edad media de 33,6 aos,
obstante la evolucin clnica continu siendo desfavorable, el 61% cumplan criterios de SIDA y 154 pacientes pre-
presentando una situacin de shock refractario y finalmen- sentaban cifras de CD4 < 50 cel/mm2. Hubo afectacin
te falleciendo al cuarto da de ingreso. pulmonar en el 97,3% de los casos, de los cuales el 82,3%
Los resultados microbiolgicos definitivos confirmaron fueron neumonas. La tasa de mortalidad fue del 46,9%
el crecimiento de Rhodococcus equi (resistente a beta-lact- con una mortalidad atribuible del 17,3%. Aunque no hubo
micos, excepto a carbapenem) en cultivos respiratorios y diferencias estadsticamente significativas entre la era pre
hemocultivo. El cultivo de micobacterias y hongos fue ne- HAART y HAART en cuanto a edad, gnero, CD4, mor-
gativo. talidad y mortalidad atribuible, s que hay evidencia de que
el TARGA asociado a antibioterapia prolongada mejora el
Diagnstico final pronstico clnico de la enfermedad y que ante una lesin
cavitada en paciente con CD4 bajos hay que sospechar
Shock sptico secundario a Neumonia cavitada bacterie- esta infeccin. En cuanto al grupo de pacientes trasplanta-
mica por Rhodococcus equi en paciente con trasplante hep- dos, V. Menon et al5 describen el caso de un paciente con
tico. trasplante renal que sufre infeccin por R equi con mlti-
ples recidivas, y realizan una revisin de la literatura donde
recopilan 39 casos de infeccin por Rhodococcus en pacien-
tes trasplantados descritos hasta el ao 2012. La mayora

196
CAPTULO 11
INFECCIONES EN PACIENTES
INMUNODEPRIMIDOS Y TRASPLANTADOS

de los casos (58,5%) corresponden a trasplantados renales, lgico ya que el retraso diagnstico es frecuente y puede
mientras que el trasplante heptico ocupa la tercera posi- condicionar, en ocasiones, el mal pronstico del cuadro.
cin con un 7,3% de los casos. En su mayora son varones
con una edad media de 49,9 aos donde el 100% de los Bibliografa
pacientes reciban tratamiento esteroideo, asociado en mu-
chos casos a otros inmunosupresores. El lugar ms frecuen- 1. Yamshchikov AV, Schuetz A, Lyon GM. Rhodococcus
te de aparicin de la infeccin fue el pulmn (90,2%). Los equi infection. Lancet Infect Dis 2010; 10: 350-9
antibiticos ms empleados fueron glicopptidos, macr- 2. Guerrero R, Bhargava A, Nahleh Z. Rhodococcus
lidos, quinolonas, carbapenemes y rifampicina, usados en equi venous catter infection: a case report and review
37 de los 39 casos de forma combinada con una duracin of the literatura. Journal of Medical Case Reports
media de 6,5 meses y en un 33,1% de los casos fue necesa- 2011; 5:358
rio una reduccin o cese del tratamiento inmunosupresor. 3. Herath S, Lewis C, Nisbet M. Increasing awareness of
Desde el punto de vista clnico, la afectacin pulmonar es la Rhodococcus equi pulmonary infection in the immu-
ms frecuente, pudindose observar cavitaciones en la ra- nocompetent adult: a rare infection with poor progno-
diografa de trax hasta en un 50% de los casos. En cuanto sis. NZMA 2013; 126: 165-213
al tratamiento, se recomienda una terapia combinada, en 4. Topino S, Galati V, Grilli E, Petrosillo N. Rhodococcus
la que al menos uno de los agentes antimicrobianos ten- equi Infection in HIV-Infected Individuals: Case Re-
ga capacidad para penetrar en los macrfagos (como por ports and Review of the Literature. AIDS patient care
ejemplo la rifampicina) y con una duracin prolongada (al and STDs 2010; 24: 211-22.
menos dos meses), no obstante y dado que es una infec- 5. Menon V, Gottlieb T, Gallagher M, Cheong EL. Per-
cin poco frecuente, su manejo se basa en ocasiones en los sistent Rhodococcus equi infection in a renal trans-
casos descritos en la literatura, no habiendo un consenso plant patient: case report and review of the literature.
claro sobre la eleccin de antibioterapia, duracin de la Transpl Infect Dis 2012; 14: E126-E133.
misma, reduccin de la inmunosupresin o necesidad de 6. Hsueh PR, Hung CC, Teng LJ, Yu MC, Chen YC,
profilaxis secundaria1,5,6. Wang HK et al. Report of Invasive Rhodococcus equi
En nuestro caso, el paciente estaba claramente inmuno- Infections in Taiwan, with an Emphasis on the Emer-
deprimido al tratarse de un sujeto trasplantado heptico gence of Multidrug-Resistant Strains. Clinical Infec
con mala funcin del injerto y en tratamiento con dos Dis 1998; 27: 370-5
frmacos inmunosupresores (ciclosporina y micofenola- 7. Septic Shock Due to Rhodococcus equi in a Patient
to), no obstante, viva en un rea urbana y desconocemos with Chronic Myelomonocytic Leukemia. Arch Bron-
contacto con animales, granjas u otros antecedentes epi- coneumol 2013; 49: 497-9.
demiolgicos compatibles con este tipo de infeccin. Esta 8. Mikic D, Djordjevic Z, Sekulovic L, Kojic M, Toma-
inmunosupresin juega un papel muy importante, ya que novic B. Disseminated Rhodococcus equi infection in
puede condicionar no slo la gravedad de la infeccin pul- a patient with Hodgkin lymphoma. Vojnosanit Pregl
monar sino su diseminacin sistmica7,8,9. Como ya hemos 2014; 71: 317-24.
mencionado, la infeccin primaria ocurre en el pulmn en 9. Ferretti F, Boschini A, Labichino C, Gerevini S, Nardi
aproximadamente el 80% de los casos, siendo excepcional P, Guffsnti M et al. Disseminated Rhodococcus equi
el shock sptico de origen pulmonar debido a R. equi como infection in HIV infection despite highly active antire-
ocurre en el caso que presentamos. El deterioro de la in- troviral therapy. BMC Infect Dis 2011; 11:343
munidad celular es el factor ms importante en el desarro-
llo de la infeccin diseminada por Rhodococcus. De acuerdo
con los datos de la literatura, las recadas, bacteriemias e
infeccin diseminada por R. equi rara vez ocurre en pa-
cientes VIH negativos, al contrario de lo que ocurre en
pacientes con SIDA. Otro factor fundamental en la apari-
cin de bacteriemia e infeccin diseminada por Rhodococcus
es el retraso diagnostico y teraputico, no obstante, se han
registrado aproximadamente un 10% de casos de infeccin
diseminada a pesar de un tratamiento adecuado8. Por tan-
to es fundamental la sospecha clnica y el estudio microbio-

197
Mujer de 70 aos con lesiones cerebrales mltiples
Faro Mguez, N; Parra Rosado, P;
Vinuesa Garca, D.
Hospital Universitario San Cecilio. Granada

Caso clnico

Mujer de 70 aos, independiente para las actividades b-


sicas de la vida diaria. Fumadora habitual, ndice paque-
tes-ao 65. Antecedentes personales de EPOC avanzado
con hiperreactividad bronquial e infecciones respiratorias
de repeticin desde la infancia. Ha realizado tratamientos
frecuentes con esteroides motivo por el que presenta una
diabetes mellitus esteroidea y osteoporosis secundaria.
Cinco meses antes del cuadro actual precis ingreso en
Neumologa por reagudizacin de su EPOC. Se reali-
z aislamiento en esputo de Aspergillus fumigatus. Tras
aplicar Criterios de Bulpa fue considerada una probable
Aspergillosis pulmonar invasiva (API) y tratada con Vo-
riconazol. Durante el ingreso hospitalario se objetiv un
dficit de IgG 1 e IgG total. Al alta precis oxigenoterapia
domiciliaria y se mantuvo Prednisona 20 mg/24 h. A los
tres meses, nuevo ingreso hospitalario en Neurologa por
tetraparesia, desde donde se traslad a Infecciosas, sien- Imagen 1. TC con contraste iv de crneo. Se aprecian cuatro lesiones hipoden-
sas con halo hipodenso (edema perilesional).
do diagnosticada de polirradiculopata y colitis por CMV.
Tras recibir tratamiento con Ganciclovir e Inmnoglobu-
linas IV, fue trasladada para rehabilitacin a hospital de mmol/l. PCR 447 mg/l. Funcin renal e iones, transami-
crnicos evolucionando de forma favorable. nasas, enzimas de colestasis y coagulacin sin alteraciones.
Acude en esta ocasin derivada desde dicho hospital tras En la radiografa de trax (AP) se visualiza derrame pleu-
presentar de forma brusca disartria, bradipsiquia, hemipa- ral bilateral y refuerzo hiliar bilateral (ms evidente hiliar
resia izquierda y desviacin de la comisura bucal a la dere- derecho). Ante la focalidad neurolgica descrita, se realiza
cha. En los das previos al ingreso actual, haba presentado TC con contraste intravenoso de crneo aprecindose cua-
cefalea con caractersticas de organicidad y tos con expec- tro lesiones hipodensas con halo hipodenso (edema perile-
toracin verdosa. No fiebre termometrada. sional): ncleos de la base derecha (23x19mm), sustancia
A su llegada al Servicio de Urgencias: afebril, hipotensa paraventricular derecha (23x16mm) y dos en rea parie-
(TA 92/63 mmHg), taquicrdica (FC 110 lpm) y taquip- to-occipital (23x19 y 23x16mm) (Imagen 1). Se ingresa con
neica (FR 24 rpm). En la exploracin fsica destacaba di- el diagnstico de lesiones cerebrales a estudio pensando en
sartria, parlisis facial izquierda de origen central, hemipa- etiologa infecciosa, por la inmunosupresin que presen-
resia y hemihipoestesia izquierda. Reflejo cutneo-plantar taba la paciente y los dos ingresos previos descritos ante-
izquierdo extensor e indiferente derecho. Hipofonesis gene- riormente. Se inicia tratamiento con Pirimetamina + sul-
ralizada con roncus dispersos en la auscultacin pulmonar. fadiazina y se aade antibioterapia emprica con Linezolid
Presentaba tambin lesiones compatibles con candidiasis 600 mg/12h y Ceftriaxona 2 gr/12h para cubrir etiologa
orofarngea y lesiones cutneas ulceradas en miembros in- pigena ms frecuente.
feriores. Resto de la exploracin sin hallazgos de inters.
Se solicita analtica urgente en la que destaca leucocito- Diagnstico diferencial
sis (14.600/mm3) con neutrofilia (855% PMN), resto de
hemograma dentro de la normalidad. Gasometra arterial La lesin intracraneal ms frecuente en adultos es la me-
con insuficiencia respiratoria crnica global. Lactato 35 tstasis (hasta el 50%), se presenta habitualmente como

198
CAPTULO 11
INFECCIONES EN PACIENTES
INMUNODEPRIMIDOS Y TRASPLANTADOS

lesin nica, siendo su origen ms habitual el carcinoma


pulmonar no microctico (20% de las ocasiones precede
al hallazgo del tumor primario). Los tumores primarios
del sistema nervioso central (SNC), la patologa vascular,
y las enfermedades inflamatorias y granulomatosas son
otras causas potenciales de masas cerebrales. El absceso
cerebral es una causa poco frecuente en la poblacin ge-
neral (incidencia 03 a 13 casos/100.000 habitantes), pero
su incidencia aumenta de forma considerable en pacien-
tes inmunodeprimidos. Hasta en el 86 % de los casos es
posible establecer una condicin predisponente, siendo lo
ms frecuente patologa infecciosa del rea otolgica y na-
so-sinusal, seguida de cardiopata congnita o endocardi-
tis (13%), proceso hematolgico (13%), inmunosupresin
(9%), patologa pulmonar previa (8%) y meningitis (6%).
En su etiopatogenia podemos distinguir tres mecanismos:
A) por contigidad desde rea otorrinolaringolgica (50-
60%), se presentan como lesiones nicas; B) como siembra Imagen 2. RMN cerebral (Secuencia T1). Se aprecia el realce en anillo de la
captacin de contraste de las lesiones cerebrales.
hematgena desde un foco sptico a distancia (20-30%),
suelen ser lesiones mltiples con distribucin en territorio
vascular; y C) secundario a traumatismo craneal o neuro- de producirse, suele producir un patrn retculo-nodular.
ciruga previa (10%), por inoculacin directa. En la po- Dentro de las causa bacterianas debemos considerar My-
blacin general, los microorganismos aislados con ms fre- cobacterium tuberculosis y Nocardia spp. La tuberculosis delsis-
cuencia son Streptococcus spp. (34%) y Enterococcus spp. (14%) tema nervioso central incluye tres formas clnicas: la me-
en el contexto de infecciones nasosinusales y otgenas; y ningitis, el tuberculoma intracraneal y la aracnoiditis. En
Staphylococcus spp. (18%), por inoculacin directa tras neu- pases de baja prevalencia la forma clnica ms frecuente es
rociruga o traumatismos, embolismos spticos de endo- la meningitis en contexto de reactivacin de infeccin la-
carditis o comunicaciones cardacas congnitas. Hongos, tente tuberculosa. La presencia de masas (tuberculomas),
parsitos y micobacterias constituyen menos del 2 % de presentan realce en anillo en estadios ms evolucionados,
los casos. Los resultados difieren cuando nos referimos a resultando casi exclusivos de reas endmicas. En caso de
poblacin inmunodeprimida. En estos casos, aumenta la afectacin del SNC, encontramos afectacin pulmonar en
presencia de infecciones oportunistas y la reactivacin de el 50 % casos, siendo lo ms habitual las lesiones apicales y
infecciones latentes cobran mayor importancia. La etiolo- presencia de cavernas como consecuencia de reactivacin
ga depender de las caractersticas epidemiolgicas, as de infeccin latente.
como del tipo y gravedad de la inmunosupresin. Nocardia spp. puede producir afectacin del SNC, bien de
Ante una paciente inmunodeprimida con lesiones mlti- forma aislada (9 % de los casos) o bien en el contexto de
ples cerebrales en el territorio de arteria cerebral media Nocardiosis diseminada (32 % de los casos). En estos casos
debemos plantearnos la posibilidad de abscesos mltiples la afectacin primaria y concomitante ms frecuente la pul-
cerebrales en el contexto de diseminacin hematgena o monar. Slo en el 2% de casos se acompaa de afectacin
reactivacin de microorganismos oportunistas. En nues- cutnea. Esta diseminacin se produce fundamentalmen-
tro caso se acompaaba de clnica infectiva respiratoria y te en presencia de determinados factores predisponentes:
un derrame pleural bilateral, probablemente relacionados trasplante de rgano slido y de progenitores hematopo-
con los hallazgos del SNC. yticos, tratamiento crnico con glucocorticoides, VIH
En presencia de abscesos cerebrales mltiples con realce en CD4<100/ul, neoplasia activa y diabetes mellitus.
anillo en una TC con contraste es fundamental sospechar Dentro de las causas bacterianas, Listeria monocytogenes, en el
una toxoplasmosis. Se trata de una zoonosis causada por contexto de meningoencefalitis, puede producir formacin
Toxoplasma gondii. Su reactivacin genera infecciones graves de abscesos cerebrales en determinados grupos de riesgo
en determinadas situaciones de inmunosupresin celular (ancianos, neonatos e inmunodeprimidos); sin embargo es
(habitualmente VIH con recuento CD4<100/mm3). No excepcional la afectacin pulmonar.
es habitual que produzca afectacin pulmonar, y en caso En cuanto a las etiologas fngicas destacar la aspergilosis

199
CAPTULO 11
INFECCIONES EN PACIENTES
INMUNODEPRIMIDOS Y TRASPLANTADOS

Imagen 3. RMN cerebral (Secuencia Difusin). Masas con captacin en anillo y Imagen 4. TC con contraste iv de trax . Se observan derrames pleurales bilater-
restriccin de la difusin, rodeadas de edema vasognico. ales mltiples loculados.

diseminada. Produce afectacin del SNC en el 10-20 %. exclusiva de zonas endmicas, solicitamos determinacin
En ausencia de un adecuado control inmunitario local pul- de interferon gamma release assays(IGRA) que resulta ne-
monar (donde presentan un papel importante de los neu- gativo.
trfilos) es posible el desarrollo de angioinvasin por hifas En hemograma destaca la presencia de un recuento de
maduras, diseminacin hematgena y siembra a distancia. neutrfilos dentro de la normalidad (>1500/mm3). Esto,
Afecta casi exclusivamente a pacientes con neutropenia junto a la ausencia de otros factores predisponentes, hace
severa (<500/mm3 neutrfilos) de causa hematolgica o improbable el desarrollo de una aspergilosis invasiva.
tras tratamiento quimioterpico. Existen otras infecciones Se solicita ecocardiografa transtorcica que informan
fngicas capaces de producir afectacin concomitante de como normal y TC de trax con contraste intravenoso
pulmonar y del SNC. Tanto por su baja incidencia, por el (Imagen 4) que confirma la presencia de derrames pleura-
contexto epidemiolgico y el tipo de inmunosupresin son les bilaterales mltiples loculados. Se decide la realizacin
poco probables en el contexto clnico actual. de toracocentesis diagnstica, por su mayor accesibilidad
y menor riesgo que el SNC, para estudio bioqumico, re-
Evolucin cuento celular y microbiolgico. El lquido pleural obteni-
do presenta un moderado consumo de glucosa, predomi-
Tras su ingreso se realiza RMN Craneal con contraste in- nio de clulas mononucleares y cumple criterios de Light
travenoso con gadolinio (secuencias pT1 (Imagen 2), pT2, para exudado. La muestra es enviada para realizar tincin
FLAIR, DWI (Imagen 3), Espectroscopia). Se aprecian de Gram, cultivo en medios comunes y micobacterias, as
masas con captacin en anillo y restriccin de la difusin, como PCR para deteccin de Mycobacterium tuberculosis que
rodeados de edema vasognico, dos en lbulo occipital informan como negativa.
derecho (24x23x26mm y 21x17x20mm), una temporo-pa- En el Gram de lquido pleural se observa un gram positivo
rietal frontal derecho (27x24x21mm) muy sugestivas de filamentoso ramificado sugerente de nocardia. Posterior-
abscesos pigenos. Captacin de contraste en corteza y mente se produce crecimiento en cultivo de N. asteroides.
zona profunda de hemisferio cerebral sugestivo de infartos Ante la presencia de nocardiosis con afectacin pulmonar,
subagudos o microabscesos. sistema nervioso central y probablemente cutnea se inicia
Con el objetivo de descartar toxoplasmosis cerebral se so- antibioterapia intravenosa con trimetoprim-sulfametoxa-
licita de forma urgente serologas para Toxoplasma (IgG e zol (TMP-SMX), linezolid 600 mg/12h e imipenem 0.5
IgM negativa), serologa VIH, tambin negativa; as como gr/6 h. Posteriormente tras confirmar con antibiograma
subpoblaciones linfocitarias que se encuentran dentro de que se trata de N. asteroides multisensible, se sustituye imipe-
los lmites normales. Es excepcional la presentacin de to- nem por ceftriaxona a dosis altas.
xoplasmosis cerebral con CD4>100. Esto, junto a serolo- Aunque inicialmente mejora, posteriormente evolucio-
gas negativas, nos permite descartar esta posibilidad etio- na de forma trpida desde el punto de vista neurolgico,
lgica por lo que se suspende pirimetamina + sulfadiazina. presentando adems hiponatremia grave en contexto de
Pese a que los tuberculomas se presentan de forma casi SIADH secundaria a afectacin del SNC. Permanece con

200
CAPTULO 11
INFECCIONES EN PACIENTES
INMUNODEPRIMIDOS Y TRASPLANTADOS

bajo nivel de consciencia hasta que se produce el falleci- mos resultados del antibiograma se debe plantear reajuste
miento en el contexto de fracaso multiorgnico. de antibioterapia. A las 2 semanas de inicio se debe valo-
rar respuesta clnica. En caso de ausencia de respuesta se
Diagnstico final debe reevaluar el tratamiento (resistencias primarias, es-
casa penetracin a nivel de SNC o necesidad de drenaje
Nocardiosis diseminada con afectacin pulmonar, del sis- quirrgico). En caso de evolucin clnica favorable se pue-
tema nervioso central, y probablemente cutnea. de plantear antibioterapia oral a las 3-6 semanas. Se debe
completar posteriormente tratamiento durante al menos
Discusin un ao.
La gravedad de la nocardiosis no slo depende de la difi-
Nocardia spp. es una bacteria gram positiva ramificada fila- cultad diagnstica y de su capacidad de invadir tejidos con
mentosa aerbica. Se encuentra ubicua en el medio am- afectacin del SNC, sino de la capacidad de evasin del
biente. Siempre acta como patgeno en el ser humano sistema inmune y de formar formas de L-Nocardia (varian-
y nunca debe ser considerado colonizacin. Es posible la tes Nocardia spp. que modifican su pared bacteriana). Estas
inoculacin cutnea, pero la va de entrada ms comn formas persisten en el tiempo pese a tratamiento adecuado
es la inhalatoria. Por este motivo el pulmn es el rgano y motivan recadas de la infeccin tras suspensin del mis-
ms frecuentemente afectado (de forma aislada en el 39 mo. Por dicho motivo se recomiendan pautas prolongadas
% de los casos y en ms del 70 % considerando afectacin de antibioterapia.
concomitante de otros rganos). La mayor importancia Resulta fundamental considerar el uso crnico de gluco-
radica en su capacidad para invadir tejidos, produciendo corticoides como una de las formas ms habituales de in-
diseminacin a distancia en presencia de determinados munosupresin. Producen alteracin a nivel de inmunidad
factores predisponentes: trasplante de rgano slido y de innata (deterioro funcin fagoctica y dficit produccin
progenitores hematopoyticos, tratamiento crnico con mediadores proinflamatorios), como inmunidad celular.
glucocorticoides, VIH CD4<100/ul, neoplasia activa y Distintos estudios han documentado aumento del riesgo
diabetes mellitus. En caso de afectacin de dos o ms r- de infeccin dependiente de la dosis (> 20 mg/da o do-
ganos hablamos de nocardiosis diseminada (56 % de los sis acumulada > 700 mg. prednisona), especialmente para
casos). Lo ms habitual es afectacin pulmonar y del SNC infecciones bacterianas habituales, virus, y hongos patge-
(32%), con afectacin cutnea en el 2% de casos. Podemos nos. Es necesario una inmunosupresin muy significativa
encontrar afectacin aislada del SNC (9%), o cutnea por para desarrollar una infeccin oportunista (otros frmacos
inoculacin directa (8%). Son lesiones indistinguibles de inmunosupresores o condiciones inmunosupresoras subya-
infecciones pigenas habituales. centes). Nos encontramos ante una paciente con un dficit
No poseemos de mtodos indirectos que nos permitan su inmunidad humoral previo, al cual se le aaden los efectos
diagnstico, por lo que es necesario su aislamiento en cul- del uso crnico de esteroides a dosis bajas, lo cual favorece
tivo. Se trata de una bacteria de crecimiento lento (hasta 3 la aparicin de infecciones oportunistas y reactivacin de
semanas) que precisa medios especiales de cultivo, por ello infecciones previas.
es fundamental orientar al microbilogo ante la sospecha En cuanto a la forma de presentacin clnica de los abs-
diagnstica. Esto permitir realizar un Gram y Ziehl-Ne- cesos cerebrales, merece destacar que el sntoma ms fre-
elsen (presenta cido-alcohol resistencia parcial) que nos cuente es la cefalea, encontrndose la trada clsica (fie-
permita un diagnstico precoz. bre, cefalea, y dficit neurolgicos focales) nicamente en
En caso de afectacin pulmonar aislada, y debido a la alta el 20% de los pacientes. La ausencia de fiebre se produce
incidencia de afectacin del SNC asociada ocasionalmente hasta en la mitad de los casos (47%). En pruebas comple-
asintomtica, se recomienda realizar prueba de imagen del mentarias destaca la elevacin habitual de parmetros in-
SNC en todos los casos, incluso en ausencia de sntomas. flamatorios, no siendo raro encontrar los mismos dentro
La importancia de detectar afectacin del SNC, radica en de la normalidad (PCR normal 40%, velocidad de sedi-
que modifica la actitud con respecto al tratamiento. En mentacin globular normal 28%, o recuento de glbulos
caso de Nocardiosis diseminada con afectacin del SNC se blancos normal 40%)
recomienda iniciar antibioterapia emprica IV con al me- Actualmente la orientacin diagnstica se realiza median-
nos dos frmacos, incluyendo TMP-SMX e imipenem. Se te RMN de difusin (DWI), la cual ha demostrado ser su-
puede plantear aadir un tercer frmaco en caso de fallo perior a la TC y RM convencional en la diferenciacin
multiorgnico (amikacina o linezolid). Una vez obtenga- de los abscesos cerebrales de otras lesiones qusticas, sobre

201
CAPTULO 11
INFECCIONES EN PACIENTES
INMUNODEPRIMIDOS Y TRASPLANTADOS

todo tumores cerebrales primarios. Sin embargo, para de-


terminar la etiologa de un absceso cerebral el gold stan-
dard contina siendo la biopsia del mismo.

Bibliografa

1. Southwick FS, Calderwood SB, Thorner AR. Patho-


genesis, clinical manifestations, and diagnosis of brain
abscess. UpToDate, Post TW (Ed), UpToDate, Wal-
tham, MA. (Consultado el 10 de Abril de 2015).
2. Spelman D, Sexton DJ, Thorner AR. Clinical manifes-
tations and diagnosis of nocardiosis. UpToDate, Post
TW (Ed), UpToDate, Waltham, MA. ( Consultado el
12 de Abril de 2015).
3. Tunkel AR. Absceso cerebral. En: Mandell GL, Ben-
nett JE, Dolin R, eds. Enfermedades infecciosas. Prin-
cipios y prctica. Sexta ediccin. Madrid: Elselvier,
2006; 1150-1163.
4. Sethi P.K., Khandelwal D., Sethi N.K. , Torgovnick
J., Arsura E. Neuroimage: Disseminated nocardiosis.
Clinical Neurology and Neurosurgery 2008; 110: 98
100.
5. Zaatreh M, Alabulkarim. Images in clinical medicine.
Disseminated Central Nervous System Nocardiosis. N
Engl J Med 2006; 354:2802.

202
Diarrea crnica y lesiones cutneas vesiculosas
en un paciente inmunodeprimido
Ramrez Bueno, A; Antequera Martn-Portugus, I;
Ojeda Burgos, G; Mrquez Solero, M.
Hospital Universitario Virgen de la Victoria. Mlaga.

Caso clnico se completa con TC abdominal, cultivo de heces, determi-


nacin de toxina Clostridium difficile, estudios serolgicos de
Presentamos el caso clnico de un hombre de 68 aos con infecciones vricas y colonoscopia.
diarrea de ms de catorce das de duracin y prdida de En la TC de abdomen se aprecia un engrosamiento sim-
ocho kg de peso en tres meses. Entre sus antecedentesper- trico de las paredes de la tercera porcin duodenal con tra-
sonales destacan trasplante cardiaco por miocardiopata beculacin de la grasa adyacente (Figura 1). Los sntomas
dilatada de origen isqumico (2005), portador de marca- digestivos mejoraron con tratamiento sintomtico, fluido-
pasos endocavitario bicameral,insuficiencia renal crnica terapia intravenosa y reposo diettico.
grado IV ycarcinoma urotelialdiagnosticadounos meses Al quinto da de ingreso,el paciente comienza confiebre
antes.En curso de terapia inmunosupresoracon micofeno- elevada y lesiones vesiculosas diseminadas con afectacin
lato, everolimus y deflazacort. de extremidades, tronco, cara y cuero cabelludo muy pru-
El paciente acude a urgencias en Junio de 2014 refiriendo riginosas. En una analtica realizada en ese momento des-
un cuadrode cuatro-cinco deposiciones diarias de consis- tacan los siguientes hallazgos: 11000 leucocitos/mm3 con
tencia lquida y sin productos patolgicosjunto con dolor 9830 neutrfilos/ mm3, creatinina 2,4 mg/dL y protena C
abdominal tipo clico acentuado en el hipocondrio dere- reactiva 244 mg/L. Se realiza unaradiografa de traxen
cho y de aproximadamente dos-tres semanas de evolucin. la que seapreciaun infiltrado intersticial en el hemitrax
As mismo refera prdida de unos ocho kilos de peso en derecho con mayor afectacin de la base pulmonar. Ante
tres meses y que haba coincidido con el diagnstico re- estos hallazgos se inicia tratamiento emprico con aciclovir
ciente de un carcinoma urotelial en vejiga el cual an no y levofloxacino intravenosos y se solicitan cultivos de san-
haba sido tratado. En la exploracin destaca un abdomen gre y esputo, estudios serolgicos y se toma una muestra
distendido, timpnico a la percusin y doloroso a la pal- del exudado de una de las vesculas para deteccin de vari-
pacin con hipersensibilidad en fosa iliaca derecha. Sin cela zoster (VVZ) mediante PCR.
hallazgos relevantes en las pruebas complementarias rea-
lizadas en urgencias, se ingresa para estudio digestivo que Diagnstico diferencial

El listado de posibles causas de diarrea es bastante extenso


(ver tabla 1). La duracin de los sntomas nos permite aco-
tar las opciones posibles mediante la diferenciacin entre
las diarreas agudas, de menos de dos semanas de dura-
cin, y las diarreas crnicas, de duracin superior a las dos
semanas. Por otro lado la presencia de fiebre, escalofros,
sangre o restos de pus en las heces son datos clnicos que
nos orientan hacia un proceso inflamatorio habitualmente
de naturaleza infecciosa.
Las causas infecciosas deben ser especialmente considera-
das en pacientes inmunodeprimidos o trasplantados como
en nuestro caso. La probabilidad de las distintas etiologas
variar segn el tiempo transcurrido desde el trasplante
(ver tabla 2). En pacientes trasplantados de largo tiempo
de evolucin las infecciones ms frecuentes son las adqui-
ridas en la comunidad.1 En estos pacientes es especialmen-
Imagen 1. TC de abdomen realizado en urgencias en el que se aprecian signos te necesaria la realizacin de una colonoscopia y toma de
inflamatorios a nivel duodenal.
biopsia, que ofrecer datos acerca de la extensin del pro-

203
CAPTULO 11
INFECCIONES EN PACIENTES
INMUNODEPRIMIDOS Y TRASPLANTADOS

Duracin de Con componente Sin componente inflamatorio Primer mes postrasplante Infecciones derivadas del donante:
los sntomas inflamatorio VHB, VHC, toxoplasmosis, CMV.
Infecciones relacionadas con la
Diarrea Aguda Causas infecciosas: Salmo- Causas infecciosas: Noro-
ciruga: colitis isqumica, C. difficile.
(Duracin < 2 nella, E. coli productora de virus, Rotavirus, Toxinas
sem) toxina shiga. Campylobac- bacterianas (S. aureus, Baci- Desde el primer mes al sexto Infecciones por grmenes oportunis-
ter jejuni, Shigella, Yersinia llus cereus, Vibrio cholerae, postrasplante tas: Cryptosporidium, Microsporidi-
enterocoltica, Clostridium Clostridium perfringens, E. um, CMV, rotavirus.
difficile, Vibrio parahaemo- coli enterotoxingena, E.coli
A partir del sexto mes postrasplante Infecciones comunitarias: C. difficile,
lyticus, Entamoeba histolyt- enteroagregativa), parsitos
CMV.
ica, Aeromonas hidrophila, (Giardia lamblia, Cryptos-
Otros: farmacolgico, neoplsico,
Helmintos. poridium, Strongyloides
isqumico.
Causas no infecciosas: stercoralis).
colitis isqumica Causas no infecciosas: frma- Tabla 2. Causas de gastroenteritis en pacientes trasplantados en funcin del
cos, alergia alimentaria tiempo transcurrido desde el trasplante.

Duracin de Con componente Sin componente inflamatorio


los sntomas inflamatorio
Diarrea Causas Causas infecciosas: Plesiomonas, Ae-
este ltimo caso como un sntoma paraneoplsico.
Crnica infecciosas: romonas, Cryptosporidium, T. whipplei, Nuestro paciente present adems al quinto da fiebre y le-
(Duracin 2 Campylobacter Cyclospora
sem) jejuni, Clostridium Causas no infecciosas: vasculitis,
siones vesiculosas diseminadas, que fueron evolucionando
difficile, Aeromo- farmacolgica, alteraciones de la moti- hacia la formacin de costras, presentndose en distintos
nas, Entamoeba lidad (neuropata autnoma diabtica,
histolytica. diarrea postvagotoma), causa endo- estadios evolutivos a los pocos das y permitiendo el diag-
Causas no crinolgica (enfermedad de Addison, nstico clnico de un herpes zoster diseminado. Sin em-
infecciosas: hipertiroidismo, gastrinoma, vipoma,
enfermedad infla- somatostatinoma, sndrome carcinoide, bargo en una fase inicial, el diagnstico clnico no era tan
matoria intestinal, mastocitosis), evidente y se podan plantear otras hiptesis diagnsticas
neoplasia. intestino irritable, colitis microscpica,
sndrome malabsortivo (pancreatitis como un pefingoide ampolloso, un eritema multiforme o
crnica, intolerancia a la lactosa,
celiaqua, sndrome de intestino corto),
un exantema medicamentoso.
postcolecistectoma, neoplasia.

Tabla 1. Causas de diarrea segn el tiempo de evolucin y su presentacin clnica. Evolucin

ceso y adems podr sugerir otros diagnsticos como una El resultado de la PCR para VVZ en el exudado de una de
enfermedad inflamatoria intestinal, una colitis isqumica o las vesculas fue positivo. La serologa mostr IgG positivas
una neoplasia. El micofenolato es un inmunosupresor fre- para VVZ e IgM negativas. Otros estudios serolgicos so-
cuentemente empleado en pacientes trasplantados entre licitados (hepatitis A, hepatitis B, hepatitis C, virus de Eps-
cuyos efectos secundarios se encuentra la gastroenteritis, tein Barr, herpes simple I y II (VHS), Yersinia, Coxiella bur-
lo cual en ocasiones condiciona modificaciones en la dosis netti y VIH) aportaron resultados negativos o indicativos de
a prescribir. Finalmente la colitis por Clostridium difficile es infecciones pasadas. El cultivo de heces y una determina-
una opcin que siempre debe ser considerada en este gru- cin de toxina de Clostridium difficile en heces fueron as mis-
po de enfermos. mo negativos. La fiebre desapareci as como descendie-
En el paciente que presentamos, la duracin de los snto- ron los reactantes de fase aguda y descendieron el nmero
mas superior a dos semanas y la ausencia de productos pa- de deposiciones al da. Se realiz una endoscopia digestiva
tolgicos en heces nos llevan a considerar como poco pro- alta y baja hallndose signos inflamatorios en coln con
bables las causas ms habituales de cuadros agudos y las alguna ulceracin. Se tomaron muestras de mucosa col-
infecciones bacterianas invasivas. An as, dada su condi- nica en la que se apreciaron cambios citopticos y cuerpos
cin de paciente trasplantado e inmunodeprimido, podra de inclusin compatibles con infeccin viral. Se realiz una
esperarse una evolucin clnica atpica de causas comunes PCR con resultado positivo para CMV (112 copias) en la
de diarrea aguda. Hay que considerar infecciones poco muestra histlogica y en plasma (330 copias). La serologa
frecuentes en sujetos inmunocompetentes como la ente- para CMV mostr una IgG positiva y una IgM negativa.
ritis por citomegalovirus (CMV) as como las toxicidades Se inici tratamiento intravenoso con ganciclovir con dosis
por frmacos como el micofenolato. Los factores de riesgo ajustada a la funcin renal as como nutricin parenteral
cardiovascular propios de un paciente con patologa coro- total dados los datos de desnutricin calrico-proteica y la
naria nos obligan a considerar la colitis isqumica y dado persistencia de la diarrea. Se solicitaron niveles de mico-
su antecedente de neoplasia urotelial y mayor incidencia fenolato con resultados por debajo del rango terapetico.
de tumores comunicada en pacientes con tratamientos Se mantuvo tratamiento con ganciclovir intravenoso du-
crnicos inmunopresores, la posibilidad de una neoplasia rante 14 das tras lo cual se comenz con valganciclovir
digestiva o a otro nivel, pudiendo explicarse la diarrea en obtenindose la negativizacin en la carga viral srica en

204
CAPTULO 11
INFECCIONES EN PACIENTES
INMUNODEPRIMIDOS Y TRASPLANTADOS

la primera semana y una mejora clnica con descenso del para el diagnstico de la enfermedad con invasin de teji-
nmero de deposiciones, aumento de la consistencia de las do, particularmente en las muestras gastrointestinales.4 Los
mismas y progresiva tolerancia digestiva oral. Se mantuvo valores de antigenemia o DNAemia pueden ser negativos
el tratamiento con valganciclovir hasta cumplir 30 das. A o bajos, sobre todo en pacientes con afectacin gastrointes-
los pocos meses del alta el paciente sufri un nuevo de- tinal o retinitis. Por lo tanto, el diagnstico debe realizarse
terioro clnico con prurito intenso, ictericia, elevacin de sobre muestras de tejido con la visualizacin de cambios
los enzimas de colestasis, hiperbilirrubinemia y datos com- histopatolgicos compatibles con infeccin por citomega-
patibles con tumoracin de la va biliar con progresin lovirus, identificacin de cuerpos de inclusin o deteccin
intraheptica. Por decisin del paciente no se realizaron de antgenos virales por inmunohistoqumica.5,6 En el caso
ms pruebas diagnsticas ni teraputicas, transfirindose que presentamos la clnica, los hallazgos endoscpicos con
a cuidados paliativos. El paciente finalmente falleci como inflamacin colnica y ulceracin de la mucosa, los cam-
consecuencia de la progresin del tumor. bios celulares en las muestras histopatlogicas, los resulta-
dos positivos de la PCR para CMV en sangre y mucosa
Diagnstico final intestinal as como la exclusin razonable de otras causas
posibles, permitieron establecer el diagnstico de infeccin
Enteritis por citomegalovirus. por CMV con afectacin intestinal.
Herpes zoster diseminado. En la enfermedad por CMV el tratamiento de primera l-
nea es valganciclovir 900 mg/12h oral o ganciclovir a do-
Discusin sis de 5mg/kg intravenoso cada 12h.5 El ganciclovir debe
utilizarse en pacientes con enfermedad por CMV grave
La reactivacin de virus herpes es una complicacin que y cuando valganciclovir sea poco tolerado o inadecuada-
contina generando importante morbimortalidad en pa- mente absorbido. Tras la mejora puede hacerse terapia
cientes trasplantados. Aunque el CMV es el virus que ms secuencial a valganciclovir oral lo cual permite adems fi-
frecuentemente se investiga, el VHS y el VVZ tambin nalizar el tratamiento en rgimen ambulatorio tal y como
pueden presentar formas clnicas graves en este subgrupo se hizo en el caso que presentamos.5 Esta estrategia propor-
de pacientes, como ocurre en el caso que presentamos. ciona un tratamiento eficaz con una adecuada exposicin
CMV es el microorganismo que ms a menudo causa in- a las drogas, la reduccin de los costos de tratamiento y
feccin en pacientes con trasplante de rganos slidos. La evitar la hospitalizacin prolongada.7 Es preciso contro-
infeccin por CMV se produce entre el 30 y el 80% de lar la funcin renal durante el tratamiento as como, en
estos pacientes y la presencia de enfermedad sintomtica casos de viremia, realizar cargas virales semanales hasta
vara dependiendo del tipo de trasplante, la presencia de la negativizacin de la misma. Se recomienda mantener el
factores de riesgo (dependientes del estado inmunolgico tratamiento hasta la resolucin de los sntomas y la nega-
de receptor y donante), del tratamiento inmunosupresor tivizacin de la carga viral en al menos dos muestras, con
utilizado y las maniobras de prevencin.2 una duracin no inferior a cuatro semanas.5
En los pacientes trasplantados cardacos, la forma de pre- El virus de la varicela zoster, puede reactivarse hasta en el
sentacin ms frecuente de la infeccin por CMV es la neu- 20% de los pacientes trasplantados, siendo ms frecuente
mona y la afeccin gastrointestinal, que comporta especial en los pacientes trasplantados de corazn o pulmn.8 El
gravedad. En una cohorte americana de 1444 pacientes uso de micofenolato, el trasplante cardiaco y una edad su-
trasplantados seguidos prospectivamente, la enteritis por perior a los 50 aos son algunos de los factores de riesgo
citomegalovirus, represent el 6,3% de las causas de dia- para la reactivacin viral descritos en la literatura y que
rrea comunitarias y el 2,7% de las diarreas nosocomiales.2 presentaba nuestro paciente. Las formas clnicas en pa-
Sus manifestaciones clnicas consisten en dolor abdominal, cientes trasplantados pueden cursar con afectacin cut-
diarrea, hemorragia digestiva, perforacin y reactivacin nea diseminada y dao visceral pudiendo presentarse con
ulcerosa, aunque puede ser asintomtica. Ello da lugar a hepatitis, neumonitis o afectacin del sistema nervioso cen-
que su diagnstico sea en ocasiones difcil de establecer.1 tral. El tratamiento siempre est recomendado en pacien-
La serologa no es til en el diagnstico de enfermedad tes inmunodeprimidos.9
activa por CMV durante el postrasplante, los cultivos vira- Nuestro paciente present dos lesiones tumorales a lo lar-
les en sangre tienen un uso muy limitado por su baja sen- go de su evolucin; un cncer de vejiga y un tumor de va
sibilidad. En cambio, la deteccin del antgeno viral p65 biliar que no pudo aclararse si era consecuencia de la pro-
o ADN viral en muestras histopatolgicas es una opcin gresin del primero a nivel heptico o un segundo tumor.

205
CAPTULO 11
INFECCIONES EN PACIENTES
INMUNODEPRIMIDOS Y TRASPLANTADOS

En pacientes trasplantados hay descrito un aumento de la cipients: HSV, VZV, human herpes viruses, and EBV.
incidencia de tumores, respecto a la poblacin general lo Infec Dis Clin N Am. 2010; 24: 373-9
cual se ha atribuido a la inmunodepresin, a la estimula- 9. Carratal J, Montejo M, Prez-Romero P. Infections
cin crnica del sistema inmune y a la activacin de vi- caused by herpes viruses other than cytomegalovirus
rus oncognicos como los herpes virus 8, el virus de Eps- in solid organ transplant recipients. Enferm Infecc Mi-
tein-Barr o el papiloma virus humano. La incidencia de un crobiol Clin. 2012;30(Supl 2):63-9.
tumor solido a los 10 aos de inmunosupresin es de un 10. Ajithkumar T, Parkinson C, Butler A et al. Manage-
20%. Aunque los tumores cutneos y los del sistema reti- ment of solid tumours in organ-transplant recipients.
culoendotelial son los ms frecuentemente descritos, se ha The Lancet. Oncology 2007;8: 921-32
visto un aumento de la incidencia de otras neoplasias; los
relacionados con la va biliar son cinco veces ms frecuente
que en poblacin general mientras que la incidencia del
cncer de vejiga es tres veces mayor.10
Finalmente el paciente que presentamos expone la ten-
dencia de los pacientes sometidos a trasplantes de rgano
slido para desarrollar complicaciones relacionadas con su
disfuncin inmunolgica, tales como las reactivaciones por
virus herpes con presentaciones clnicas graves y el desa-
rrollo de neoplasias. El clnico debe de tener un grado de
sospecha elevado ante estas posibilidades en este subgrupo
de pacientes.

Bibliografa

1. Gurgu M, Muoz P. Infecciones en el trasplante car-


daco. Enferm Infecc Microbiol Clin 2007;25: 587-98.
2. Echenique IA, Penugonda S, Stosor V et al. Diagnos-
tic yields in solid organ transplant recipients admitted
with diarrhea. Clin Infect Dis. 2015; 60: 729-37.
3. Aguado JM, Navarro D, San Juan R, Castn JJ. Cy-
tomegalovirus infection insolid organ transplantation.
Enferm Infecc Microbiol Clin. 2012;30(Supl 2):57-62.
4. Kotton CN et al. Update international consensus gui-
delines on the management of cytomegalovirus in so-
lid-organ transplantation. Transplantation. 2013 Aug
27;96: 333-60
5. De la Torre Cisneros J. GESITRA-SEIMC/REIPI re-
commendations for the management of cytomegalovi-
rus infection in solid-organ transplant patients Enferm
Infecc Microbiol Clin. 2011;29: 73558.
6. Ljungman P, Griffiths P, Paya C. Definitions of cyto-
megalovirus infection and disease in transplant reci-
pients. Clin Infect Dis 2002; 34: 1094-7.
7. Caldes A, Gil-Vernet S, Armendariz Y, Colom H, Pou
L, Niubo J, et al. Sequential treatment of cytomega-
lovirus infection or disease with a short course of in-
travenous ganciclovir followed by oral valganciclovir:
efficacy, safety, and pharmacokinetics. Transpl Infect
Dis. 2010;12 :20412.
8. Shiley K, Blumberg E. Herpes viruses in transplant re-

206
Lesiones cutneas diseminadas
en paciente inmunodeprimido
Lpez Montesinos, I; Martnez Prez-Crespo, PM;
Suarez Barrenechea, AI; Valiente Mndez, A.
Hospitales Universitarios Virgen Macarena y Virgen del Roco. Sevilla.

Caso clnico respiratoria a 20 rpm y saturacin basal de oxgeno al 90%


por pulsioximetra. La exploracin neurolgica revel un
Se presenta el caso de un varn de 78 aos sin alergias a deterioro de la memoria a corto plazo, disartria y debilidad
medicamentos conocidas. Como antecedentes personales muscular generalizada. A la auscultacin cardiopulmonar
destaca Diabetes Mellitus tipo II, hiperlipemia y exfuma- presentaba disminucin del murmullo vesicular con cre-
dor de 20 paquetes/ao. Un ao antes del ingreso actual, pitantes gruesos bibasales. Las pruebas complementarias
haba sido diagnosticado de Miastenia Gravis bulbar re- revelaron leucocitosis de 15590/L (rango de laboratorio
quiriendo hospitalizacin en la Unidad de Cuidados In- 4.000-11.000/L) con neutrofilia de 15000/L (rango de
tensivos (UCI). Recibi tratamiento con inmunoglobulinas laboratorio 1.700-7.500/L) y protena C reactiva (PCR)
y corticoesteroides. Seis meses ms tarde, present nuevo de 187.2 mg/l (N<5mg/l), como nicos valores destaca-
episodio de crisis miastnica, aadindose azatioprina a su bles. Se realiz TC crneo (sin contraste) y radiografa de
tratamiento habitual. En el momento del ingreso estaba en trax sin hallazgos significativos. A las 24 horas de su in-
tratamiento con metformina 2550 mg/da, insulina lenta greso, se objetiva la aparicin de lesiones cutneas vesi-
14 UI/da, piridostigmina 300mg/da, prednisona 70 mg/ culo-pustulosas diseminadas (Imagen 1), que es motivo de
da y azatioprina 150mg/da. De forma basal, era parcial- ingreso a cargo de Enfermedades Infecciosas.
mente dependiente para las actividades bsicas de la vida
diaria, necesitando de un bastn para la deambulacin. Diagnstico diferencial
Tres meses antes de su ingreso, presenta cuadro de deterio-
ro cognitivo consistente en dificultad para recordar hechos Nos encontramos ante un paciente inmunodeprimido que
recientes, olvidos de las tareas cotidianas, cambios en el ingresa con un cuadro subagudo consistente en fiebre, sos-
carcter y descuido de la higiene personal. pecha de infeccin respiratoria y lesiones cutneas vesicu-
Ingresa inicialmente en planta de Neurologa por presen- lo-pustulosas diseminadas, en el contexto de un deterioro
tar deterioro de su estado general, fiebre de hasta 38.5C e cognitivo de meses de evolucin. En el diagnstico diferen-
infeccin respiratoria de vas altas de una semana de evo- cial deberamos incluir la infeccin por virus herpes sim-
lucin. En la exploracin inicial, el paciente estaba alerta, ple (VHS) o varicela zster (VVZ) diseminado, virus de la
hemodinmicamente estable, taquipnico con frecuencia inmunodeficiencia humana (VIH), bacteriana (Pseudomonas
aeruginosa, Nocardia spp...), Mycobacterium tuberculosis y mico-
bacterias no tuberculosas; as como infecciones fngicas
(criptococosis, histoplasmosis, aspergilosis y mucormicosis).

Evolucin

Se inici tratamiento emprico con piperacilina-tazo-


bactam 4/0.5 g/8h en perfusin extendida y aciclovir
750mg/8h. Se extrajeron hemocultivos y urocultivos
siendo negativos. Se realizaron aspirados de las lesiones
cutneas enviando muestras para reaccin en cadena de
polimerasa de VHS y VVZ, tincin Tzank (resultados ne-
gativos) y cultivo bacteriano. La serologa frente al VIH y
el antgeno galactomanano fueron negativos.
Clnicamente, el paciente presenta mala evolucin, con de-
Imagen 1. Lesiones vesiculopustulas umbilicadas monomorfas distribuidas en terioro del nivel de vigilancia (Glasgow Score de 4 puntos)
nuca, tronco y miembros.
y aumento del trabajo respiratorio. Se considera su ingreso

207
CAPTULO 11
INFECCIONES EN PACIENTES
INMUNODEPRIMIDOS Y TRASPLANTADOS

en UCI, sin embargo, el paciente haba notificado previa-


mente su objecin a medidas invasivas. Dado, que adicio-
nalmente tambin presentaba alargamiento de los tiempos
de coagulacin, tampoco se realiza puncin lumbar. Se
realiz nueva radiografa de trax, objetivando mltiples
ndulos bilaterales que se confirman en TC de trax (Ima-
gen 2). Analticamente, destaca elevacin de PCR hasta
669 mg/l.
Cinco das despus de la toma del aspirado de las lesio-
nes cutneas, se asla Nocardia spp. y se inicia tratamiento
emprico con trimetoprim-sulfametoxazol (TMP-SMX)
800mg/8h y amikacina 750mg/24h. Veinticuatro ho- Imagen 2. Imgenes nodulares de bordes mal definidos y discreto halo perifrico
en ambos lbulos superiores.
ras despus, la identificacin a nivel de especie median-
te mtodos fenotpicos nos revela que se trata de Nocardia
otitidiscaviarum, y dado que se describe en la literatura con cin con otras especies. Beaman et al. reportaron slo 10
un elevado porcentaje de resistencias, se sustituye TMP- (2.9%) de 347 casos en Estados Unidos hasta 1994.
SMX por carbapenmico, manteniendo amikacina. Dos Normalmente la infeccin diseminada ocurre en pacientes
das despus del aislamiento de Nocardia spp. se realiza inmunodeprimidos, incluyendo, como en el caso presenta-
estudio de susceptibilidad, realizado mediante E-Test si- do, pacientes en tratamiento prolongado con corticoides
guiendo metodologa de CLSI (Clinical and Laboratory y/o diabticos. Se considera infeccin diseminada cuando
Standard Institute): sensible a imipenem (Concentracin hay afectacin de dos o ms rganos no contiguos. La di-
mnima inhabitoria (CMI) 2 mcg/ml) y a TMP-SMX seminacin presumiblemente ocurre por va hematgena
(CMI 0.03 mcg/ml), se aade TMP-SMX y se retira ami- desde un foco pulmonar o cutneo. No obstante, es muy
kacina por deterioro severo de la funcin renal atribuido a infrecuente la identificacin del microorganismo en hemo-
amikacina (Filtrado glomerular segn MDRD 32.63 ml/ cultivos dadas las caractersticas del mismo1,3.
min/1.73m). No se dispona de otras alternativas terapu- La nocardiosis pulmonar es la presentacin clnica ms
ticas. A pesar de la antibioticoterapia dirigida, el paciente frecuente debido a que la inhalacin es la principal fuente
falleci 12 das despus de su ingreso. de infeccin, aunque su sintomatologa es inespecfica. No-
En relacin con enfermedad basal, el paciente fue tratado cardia spp. no coloniza de forma habitual el tracto respira-
con inmunoglobulinas a dosis de 0.4 g/Kg/da durante torio, por lo que su aislamiento es indicativo de infeccin.
5 das. Se retir azatioprina por hiperbilirrubinemia y se Radiolgicamente, se presenta en diferentes patrones, en-
mantuvo corticoterapia a dosis de 1mg/Kg/dia. tre ellos, la aparicin de ndulos mltiples.
El sistema nervioso central (SNC) es la localizacin extra-
Diagnstico final pulmonar ms frecuente (hasta el 44% en algunas series).
Suele presentarse en forma de abscesos parenquimatosos
Nocardiosis diseminada por Nocardia otitidiscaviarum con o, ms infrecuentemente, como meningitis. En nuestro
afectacin pulmonar, cutnea y probablemente cerebral paciente, no se objetivaron lesiones ocupantes de espacio
en un paciente inmunocomprometido. cerebrales; si bien el TC de crneo se realiz sin contras-
te yodado y presentaba artefactos por los movimientos del
Discusin paciente. Por otra parte, la no realizacin de puncin lum-
bar impidi el estudio del lquido cefaloraqudeo. No obs-
Edmund Nocard describi por primera vez el gnero No- tante, consideramos plausible la afectacin del SNC por
cardia en 1888 como patgeno bovino. Nocardia spp. per- Nocardia spp. ante la clnica neurolgica descrita.
tenece al orden Actinomycetales y a la familia Nocardiaceae. Se La piel es el siguiente rgano afectado en frecuencia (10%
trata de bacilos ramificados gram-positivos y parcialmente de los casos)4. Suele aparecer en forma de pstulas, absce-
cido-alcohol resistentes, con subramificaciones en ngulo sos y/o ndulos.
recto. Dentro de este gnero se han descrito 11 especies pa- El diagnstico definitivo de nocardiosis requiere el aisla-
tgenas humanas, siendo la ms frecuente Nocardia asteroides miento e identificacin del microorganismo. Dado que las
complex, que incluye N. asteroides, N. farcinica y N. nova. La colonias de Nocardia spp. pueden tardar hasta dos semanas
infeccin por N. otitidiscaviarum es infrecuente en compara- en evidenciarse, es muy importante notificar al laboratorio

208
CAPTULO 11
INFECCIONES EN PACIENTES
INMUNODEPRIMIDOS Y TRASPLANTADOS

cuando se sospeche esta infeccin. Las tinciones de Gram carla al laboratorio, para el empleo de medios selectivos
y de Kinyoun modificada son de especial relevancia para que optimicen los resultados microbiolgicos, dado que
obtener un diagnstico rpido de presuncin, mientras se Nocardia spp. es un microorganismo de lento crecimiento.
obtienen los cultivos definitivos. Pueden obtenerse mues- Siempre se debe intentar realizar la identificacin a nivel
tras de esputo, lavado bronquio-alveolar, lquido cefalo- de especie y estudio de susceptibilidad antimicrobiana, da-
rraqudeo, aspirado y/o biopsia de lesiones cutneas, en dos los diferentes patrones de sensibilidad de las distintas
funcin de la clnica del paciente. Si bien, aunque Nocardia especies.
spp. crece habitualmente en medios de cultivo convencio-
nales, en las muestras que pueden presentar contamina- Bibliografa
cin de flora mixta, puede ser necesario el uso de medios
selectivos como BCYE (buffered charcoal-yeast extract) o 1. Lederman ER, Crum NF. A case series and focused re-
Thayer-Martin4,6. Otras herramientas como secuenciacin view of nocardiosis: clinical and microbiologic aspects.
de DNA, reaccin en cadena de la polimerasa o ribotipado Medicine (Baltimore) 2004; 83: 300-13.
pueden ser tiles segn disponibilidad del laboratorio. En 2. Beaman BL, Beaman L. Nocardia: host-parasite rela-
nuestro caso, el diagnstico se realiz a travs del cultivo tionships. Clin Microbiol Rev 1994; 7: 21364
del aspirado de las lesiones cutneas, no se realiz tincin 3. Wilson JW. Nocardiosis: updates and clinical overview.
de Gram de la muestra, hecho que pudiera haber aportado Mayo Clinic Proceedings 2012; 87: 403-7.
un diagnstico ms temprano. 4. Dodiuk-Gad R, Cohen E, Ziv M, Goldstein LH, Cha-
En cuanto al tratamiento, es caracterstico del gnero No- zan B, Shafer J et al. Cutaneous nocardiosis: report of
cardia la marcada diferencia en los patrones de sensibilidad two cases and review of the literature. Intern J Derma-
entre especies. N. otitidiscaviarum es normalmente resistente tol 2010; 49: 1380-5.
a beta-lactmicos, incluyendo cefalosporinas de amplio 5. Brown-Elliott BA, Brown MJ, Conville P, Wallace RJ.
espectro, ampicilina, amoxicilina-clavulnico e imipinem; Clinical and Laboratory Features of the Nocardia spp.
susceptible a amikacina, fluorquinolonas y linezolid y, por Based on Current Molecular Taxonomy. Clin Micro-
ltimo, es frecuentemente resistente a TMP-SMX, aun- biol Rev 2006; 19: 25982.
que puede ser sensible3,5. De forma emprica, en el caso 6. Ambrosioni J, Lew D, Garbino J. Nocardiosis: updated
de nocardiosis diseminada en paciente inmunodeprimido clinical review and experience at a tertiary center. In-
con sospecha de afectacin del SNC se recomienda triple fection 2010; 38: 89-97.
terapia con TMP-SMX (muy buena penetracin en SNC), 7. Cercenado E, Marn M, Snchez-Martnez M, Cue-
imipenem y amikacina. Cercenado et al7 sugirieron que vas O, Martnez-Alarcn O, Bouza E. In Vitro Acti-
meropenem era 4 veces menos activo que imipenem fren- vities of Tigecycline and Eight Other Antimicrobials
te a Nocardia spp. No obstante, otros datos evidencian que against Different Nocardia Species Identified by Mole-
meropenem tiene mejor actividad que imipenem, excepto cular Methods. Antimicrob Agents Chemother 2007;
para N. farcinica y N. nova5. Linezolid puede ser una alterna- 51: 11024.
tiva a TMP-SMX, amikacina o imipenem de forma emp- 8. Thomas C, Mayer S, Gungor Y, Swarup R, Webster
rica3,6. En el caso presentado, el tratamiento antibitico se E, Chang Y et al. Myasthenic crisis: Clinical features,
modific en varias ocasiones en funcin de las observacio- mortality, complications and risk factors for prolonged
nes descritas en la literatura, antibiograma y las caracters- intubation. Neurology 1997; 48: 1253-60.
ticas clnicas del paciente.
Por otra parte, resaltar que cualquier infeccin puede ser
causa de crisis miastnica. Adems, se ha descrito que el
uso de antibiticos puede actuar como desencadenante,
especialmente los aminoglucsidos8. As, la posibilidad de
crisis miastnica aadido a la propia infeccin diseminada
por Nocardia spp. conlleva un pronstico ms sombro y di-
ficulta el manejo teraputico.
Como conclusin, queremos resaltar que ante un paciente
inmunocomprometido con clnica respiratoria y lesiones
cutneas es necesario incluir en el diagnstico diferencial
a Nocardia spp. La sospecha de nocardiosis hay que notifi-

209
CAPTULO 12
INFECCIONES POR HONGOS
Cuando la cara duele a un diabtico
Navarrete Lorite, MN; Ballestero Tllez, M;
Valiente Mndez, A; Domnguez Castellano, A.
Complejo Hospitalario Regional Virgen Macarena y Virgen del Roco. Sevilla.

Caso clnico

Presentamos el caso de una mujer de 66 aos de edad en-


tre cuyos antecedentes personales mdico-quirrgicos ms
relevantes se encuentran: la ausencia de reacciones alrgi-
cas medicamentosas conocidas; Diabetes Mellitus tipo 2
(DM tipo 2) de larga evolucin en tratamiento con insuli-
noterapia con mal control glucmico; cirrosis heptica de
probable origen autoinmune con actividad y datos clnicos
de hipertensin portal con mltiples descompensaciones
hidrpicas en tratamiento con deflazacort y 6-mercapto-
purina e insuficiencia renal crnica con hiperparatiroidis-
mo secundario en seguimiento por el Servicio de Nefrolo-
ga. La paciente fue sometida a colecistectoma en 2002
e intervenida de carcinoma lingual de clulas escamosas Imagen 1. Afectacin del seno maxilar izquierdo y cavidad nasal.
bien diferenciado e infiltrante en 2012, con vaciamiento
ganglionar en seguimiento por el servicio de Ciruga mxi- titucionales y el inicio agudo haran de ste un diagnstico
lofacial. poco probable. La sinusitis bacteriana principalmente por
La paciente acude en Febrero de 2015 al servicio de Ur- Pseudomonas spp. o la sinusitis fngica por Aspergillus spp. o
gencias por presentar desde tres semanas antes dolor ocular Mucorales sera un diagnstico muy probable teniendo en
izquierdo con prdida de visin progresiva, acompaada cuenta el cuadro clnico y los antecedentes personales de
de dolor en regin nasal, periorbitaria y frontal izquierda. la paciente.
Refera fiebre de 38C en la ltima semana.
En la exploracin realizada se aprecia, sobre regin perior- Evolucin
bitaria y maxilar izquierda, un rea enrojecida y dolorosa
a la palpacin. El ojo izquierdo muestra cierto grado de Ante la sospecha clnica de mucormicosis rinocerebral se
proptosis y la pupila ipsilateral en midriasis media arreac- realiza TAC urgente de senos paranasales, donde se ob-
tiva. El resto de exploracin resulta anodina con normali- serva ocupacin de los senos maxilar, frontal y esfenoidal
dad de constantes vitales. izquierdos, as como fosa nasal izquierda y celdillas etmoi-
dales de este lado. Se objetiva adelgazamiento de los tabi-
Diagnstico diferencial ques seos de las celdillas etmoidales, suelo de la rbita y
de la pared interna del seno maxilar izquierdo, con erosin
Nos encontramos ante una paciente con episodio agudo y de esta ltima. Existe aumento de atenuacin de la gra-
progresivo de dolor facial y fiebre, entre cuyos anteceden- sa y partes blandas del espacio masticador izquierdo, con
tes existe: inmunosupresin por corticoides, DM tipo 2 e aumento de densidad en fosa pterigopalatina izquierda.
intervencin quirrgica por carcinoma lingual. Estos hallazgos estaran en relacin con un proceso infec-
Entre las entidades clnicas que cursan con dolor facial, la cioso-inflamatorio. Sin evidencia de alteraciones intracere-
neuralgia del trigmino sera un diagnstico posible, pero brales (Imagen 1).
poco probable por la existencia de fiebre. El antecedente Al mismo tiempo se realiza estudio analtico urgente don-
neoplsico de la paciente habra que tenerlo presente, dado de en el hemograma se aprecia una anemia de perfil mi-
que el dolor facial podra ser consecuencia de una recidiva croctica e hipocrmica ya conocida. Leucocitos 11980
tumoral con invasin de los senos paranasales, aunque la clulas/microlitro (normal: 4000-11000 clulas/microli-
existencia de fiebre, la ausencia de signos o sntomas cons- tro) con neutrofilos absolutos 10850 clulas/microlitro (n:

212
CAPTULO 12
INFECCIONES POR HONGOS

En el mismo acto exploratorio y ante los hallazgos descri-


tos que confirmaban el diagnstico de sospecha inicial, se
realiza intervencin quirrgica urgente efectuando turbi-
nectoma media, exresis de mucosa del cornete inferior,
as como de septum y suelo de la fosa. Se practica mea-
totoma media muy amplia resecando toda la mucosa del
seno, etmoidectoma anterior y posterior, esfenoidotoma y
reseccin de mucosa enferma. Durante el acto quirrgico
se recogen muestras que son enviadas al servicio de micro-
biologa y de anatoma patolgica.
Una vez realizada la intervencin quirrgica urgente se
inicia la correccin de los factores de riesgo predisponen-
tes subyacentes (la acidosis metablica de la paciente y la
hiperglucemia). Igualmente y de forma simultnea a lo an-
terior se inicia tratamiento antifngico emprico intraveno-
so combinado con anfotericina B liposomal (5mg/kg/24
Imagen 2. Estructura microscpica de Rhizopus spp (examen directo con KOH). horas) y caspofungina (dosis de carga de 70 mg el primer
da seguido de 50 mg/24 horas ajustado al peso de la pa-
ciente). Se solicita como prueba de imagen de control una
RMN craneal.
Desde el laboratorio de Microbiologa se informa urgente-
mente de la existencia de un hongo patgeno pertenecien-
te al orden de los Mucorales, Rhizopus spp. Posteriormente
y tras su identificacin resulta ser Rhizopus oryzae (Imagen
2). Asimismo desde el laboratorio de Anatoma Patolgica
se informa de la existencia de angioinvasin por parte del
hongo y por tanto hallazgos compatibles con mucormico-
sis (Imagen 3).
La evolucin de la paciente es trpida por la existencia de
dolor facial de difcil control, que obliga a una nueva revi-
sin por parte del Servicio de Otorrinolaringologa, que
Imagen 3. Epitelio mucoso y estructura correspondiente al hongo patgeno (H&E). solicitan nuevo TAC craneal. La nueva prueba realizada
muestra cambios inflamatorios secundarios a la interven-
2000-7500 clulas/microlitro). Plaquetas 306000 clulas/ cin quirrgica previa, sin evidenciarse afectacin del sis-
microlitro (n: 150000-400000 clulas/microlitro). Coagu- tema nervioso central. En los siguientes das, la paciente
lacin normal. Bioqumica donde se objetivan valores de: presenta dolor y distensin abdominal acompaado de
glucosa 300 mg/dl (n: 70-100 mg/dl), urea 86.9 mg/dl (N: diarrea, se solicita deteccin de la toxina de Clostridium di-
7-18 mg/dl), creatinina 1.52 mg/dl (n: 0.6-1.4 mg/dl). So- fficile en heces que resulta positiva, inicindose tratamiento
dio 125 meq/L (n: 136-145 meq/L) y Potasio 5 meq/L dirigido. En las 48 horas siguientes, la paciente presenta
(N: 3.5-4.5 meq/L). PCR 103 mg/L (n: 0-5 mg/L). En empeoramiento del cuadro clnico a pesar del tratamiento,
gasometra venosa se aprecia acidemia metablica con pH y fallece.
7.20 (N: 7.35-7.45), PCO2 34 mmHg (n: 35-45 mmHg) y
HCO3 15 mmol/L (n: 18-23 mmol/L). En orina se ad- Diagnstico final
vierte la presencia de cuerpos cetnicos.
Ante esta situacin clnica, se realiza exploracin endosc- Mucormicosis rinocerebral por Rhizopus oryzae.
pica otorrinolaringolgica en quirfano objetivndose una Diarrea nosocomial asociada a la infeccin por Clostridium
mucosa inflamada con parches necrticos de todo el sep- difficile.
tum y suelo de fosa nasal izquierda. En los cornetes inferior Acidosis metablica secundaria a insuficiencia renal crni-
y medio existe necrosis con parches blanquecinos visibles. ca y probable cetoacidosis diabtica.
Resto de exploracin normal.

213
CAPTULO 12
INFECCIONES POR HONGOS

Discusin cin de cuatro pilares fundamentales: el diagnstico pre-


coz, la reversin de los factores de riesgo predisponentes, el
La mucormicosis rinocerebrales una infeccin invasiva desbridamiento quirrgico y la terapia antifngica.
por hongos saprfitos de la cavidad nasal y senos parana- El diagnstico precoz se basa en las pruebas complementa-
sales. Los microorganismos que principalmente estn im- rias anteriormente citadas.
plicados pertenecen al orden Mucoral y los gneros ms La identificacin y correccin de los factores predisponen-
frecuentes son Rhizopus spp., Mucor spp., Rhizomucor spp. y tes es fundamental para lograr una mejor respuesta al tra-
Absidia spp (1). tamiento quirrgico y antifngico.
Existen numerosos factores de riesgo implicados en el de- El desbridamiento quirrgico precoz agresivo de los tejidos
sarrollo de esta enfermedad: la DM tipo 2 con mal control afectados deber efectuarse tan pronto como se sospeche el
metablico, especialmente la situacin de cetoacidosis, el diagnstico de mucormicosis rinocerebral. Este tratamien-
tratamiento inmunosupresor o inmunomodulador, las neo- to permite eliminar aquellos tejidos afectados, limitando
plasias hematolgicas y la sobrecarga de hierro entre otros. la diseminacin de la infeccin y facilita el acceso de los
Sera la situacin de cetoacidosis el factor clnico subyacen- antifngicos a las reas afectadas. En ocasiones esta inter-
te que con ms frecuencia se ha encontrado en las distintas vencin quirrgica puede ser deformante, ya que a menu-
series de casos a lo largo de los aos (2). La cetoacidosis de- do requiere la extirpacin del paladar, cartlagos nasales,
terminara por un lado que los macrfagos se encuentren y la rbita.
con incapacidad para poner en marcha los mecanismos de La terapia antifngica de eleccin es anfotericina B liposo-
formacin de sustancias de estrs oxidativo, y llevar a cabo mal. La dosis inicial habitual es de 5 mg/kg al da pudin-
una fagocitosis eficaz; de otro lado la cetoacidosis determi- dose aumentar la dosis hasta 10 mg/kg al da en un intento
nara la existencia de una mayor cantidad de hierro libre por controlar esta infeccin. Si la infeccin no se controla
en suero, factor de estmulo necesario para el crecimiento se han ensayado en modelos animales y en algunas series
del hongo (3). de casos, con poca experiencia, las terapias combinadas.
La interaccin hongo-husped, tras la inhalacin de las Estas terapias han utilizado derivados azlicos, deferosirox
esporas y el fracaso de los mecanismos inmunolgicos, se y equinocandinas.
caracteriza por la angioinvasin. El hongo es capaz de in- Los derivados azlicos empleados in vitro han sido posaco-
vadir el endotelio desencadenando la trombosis de los va- nazol (POS), itraconazol (ITC), voriconazol (VRC) y flu-
sos sanguneos y provocando la necrosis del tejido afecto. conazol (FLC). De ellos, POS es el que mejor actividad in
Existen distintas formas clnicas de presentacin de la mu- vitro ha presentado frente a agentes de la mucormicosis.
cormicosis en funcin del rea afectada. As podemos en- Se ha empleado en estudios preclnicos donde parece til
contrar mucormicosis rinocerebral (la ms frecuente), y en terapia combinada, pero los datos clnicos son escasos.
tambin otras localizaciones como pulmonar, gastrointes- Un inconveniente aadido es que no existe formulacin
tinal, cutnea, renal, cerebral y diseminada. intravenosa (5,6)
El cuadro clnico de la forma rinocerebral se caracteriza Deferosirox (un quelante del hierro sin actividad siderfo-
por sntomas de comienzo insidioso siendo los ms fre- ra) aadido a anfotericina B liposomal, en el estudio DE-
cuentes la fiebre: (44%), ulceracin nasal o necrosis (38%), FEAT Mucor, demostr un exceso de mortalidad a los 90
inflamacin de la cara o periorbitaria (34%), disminucin das en el brazo de terapia combinada frente a anfotericina
de la visin (30%), oftalmoplejia (29%), sinusitis (26%) y B liposomal y placebo (7).
dolor de cabeza el (25%) (4). Las equinocandinas son frmacos muy seguros y con for-
Para el diagnstico es fundamental la sospecha clnica. Una mulacin intravenosa que han demostrado in vitro ser ti-
vez se tiene la sospecha, el TAC o RMN son las tcnicas les en terapia combinada. En un modelo con ratones se
de imagen de eleccin que permiten valorar la extensin aprecia una mayor supervivencia de aquel grupo tratado
de la enfermedad. Posteriormente, es de vital importancia con terapia combinada (8). En un estudio clnico retros-
que durante la intervencin quirrgica se realice una ade- pectivo con 41 pacientes se vi como la supervivencia fue
cuada toma de muestras de tejido para su posterior anlisis mayor en el grupo de pacientes que recibi terapia com-
microbiolgico y anatomopatolgico. Dado que la mayor binada (9).
parte de los hongos implicados en el cuadro clnico son Con lo anteriormente expuesto, la experiencia clnica en
saprofitos, es necesario demostrar la invasin del hongo en terapias combinadas es muy escasa debido por un lado a
el tejido. la baja frecuencia de presentacin de esta patologa y por
El tratamiento de la mucormicosis se basa en la combina- otro lado a la pobre existencia de datos clnicos, que se

214
CAPTULO 12
INFECCIONES POR HONGOS

limitan a estudios retrospectivos con escaso nmero de pa-


cientes. Existen datos preclnicos que abren el camino a
estas terapias combinadas principalmente con equinocan-
dinas, habindose demostrado tiles in vitro para inhibir
el crecimiento de hongos del orden de los mucorales. Sera
necesario realizar estudios prospectivos que valoraran la
eficacia de la terapia combinada anfotericina B liposomal
- equinocandinas y su impacto en la supervivencia de los
pacientes afectados por Mucormicosis.

Bibliografa

1. Petrikkos G, Skiada A, Lortholary O, Roilides E,


Walsh T, Kontoyiannis D. Epidemiology and Clini-
cal Manifestations of Mucormycosis. Clin Infect Dis
2012;54(S1):S23-34.
2. Roden M, Zaoutis T, Buchanan W, Knudsen T, Sarki-
sova T, Schaufele R, et al. Epidemiology and Outco-
me of Zygomycosis: A Review of 929 Reported Cases.
Clin Infect Dis 2005; 41:634-53.
3. Ibrahim A, Spelberg B, Walsh T, Kontoyiannis D.
Pathogenesis of Mucormycosis. Clin Infect Dis 2012;
54(S1):16-22.
4. Yohai R, Bullock J, Aziz A, Markert R. Survival factors
in rhino-orbital-cerebral mucormycosis. Surv Oph-
thalmol 1994; 39:3.
5. Sun Q , Fothergill A, McCarthy D, Rinaldi M, Gray-
bill. In Vitro Activities of Posaconazole, Itraconazo-
le, Voriconazole, Amphotericin B, and Fluconazole
against 37 Clinical Isolates of Zygomycetes. Antimi-
crob Agents Chemother 2002; 46:1581-2.
6. Greenberg R, Mullane K, Van Burik J, Raad I, Abzug
M, Anstead G, et al. Posaconazole as Salvage Thera-
py for Zygomycosis. Antimicrob Agents Chemother
2006; 50:126-33.
7. Spellberg B, Ibrahim AS, Chin-Hong PV, Konto-
yiannis DP, Morris MI, Perfect JR, et al. The Defe-
rasirox-AmBisome Therapy for Mucormycosis (DE-
FEAT Mucor) study: a randomized, double-blinded,
placebo-controlled trial. J Antimicrob Chemother.
2012;67:715-22.
8. Spellberg B, Fu Y, Edwards J, Ibrahim A. Combina-
tio Therapy with Amphotericin B Lipid Complex and
Caspofungin acetate of Disseminated Zygomycosis in
Diabetic Ketoacidotic Mice. Antimicrob Agents Che-
mother 2005; 49:830-32.
9. Reed C, Bryant R, Ibrahim AS, Edwards J, Filler SG,
Goldberg R, et al. Combination polyene-caspofungin
treatment of rhino-orbital-cerebral mucormycosis.
Clin Infect Dis. 2008;47: 364-71.

215
Varn de 60 aos fumador con lesin pulmonar
y masa ocupante de espacio a nivel cerebral:
no todo es lo que parece
Roca Oporto, C; Navarro Amuedo, MD;
Praena Segovia, J; Ruiz Ruiz, F.
Hospital Universitario Virgen del Roco.

Caso clnico

Varn de 60 aos con antecedente de tabaquismo activo


de 20 cigarrillos/da, hipertensin arterial en tratamiento
con ramiprilo y carcinoma basocelular nasal intervenido
mediante reseccin completa. El paciente no presentaba
otras patologas asociadas. El cuadro actual comienza de
forma abrupta cuando, estando previamente asintomti-
co, sufre una crisis comicial autolimitada con movimientos
tnico-clnicos de miembros derechos y con estado post-
crtico posterior. Es trasladado a urgencias de su hospital
de referencia (Hospital de Ro Tinto) donde se realiz to-
mografa computarizada (TC) craneal descubriendo lesio-
nes sugestivas de metstasis cerebral, por lo que se decidi
ingreso en Medicina Interna.
En planta, el paciente presentaba funciones superiores
normales, orientado en tiempo y espacio y con lenguaje Figura 1. RM cerebral corte coronal (potenciada en T2). Lesin qustica frontal izquierda.
coherente. No presentaba dficit campimtrico ni altera-
ciones de pares craneales. Balance motor y sensibilidad
superficial y profunda normales. Reflejo cutneo plantar
flexor bilateral. Afebril, tensin arterial 120/60. Eupneico
en reposo. A la auscultacin se encuentra rtmico sin so-
plos, murmullo vesicular conservado. El abdomen es blan-
do, con ruidos conservados sin masas ni megalias, cicatri-
ces residuales ni adenopatas. En extremidades inferiores
pulsos bilaterales simtricos y conservados sin edemas, sin
signos de TVP, con presencia de lesiones maculares ocre
no sobreelevadas de forma dispersa.
En la bioqumica los datos mas relevantes fueron: gluce-
mia de 125 mg/dl, creatinina 1.15 mg/dl, urea 66 mg/dl,
protenas de 5 gr/dl, y en el hemograma: 6.670 leucocitos/ Figura 2. 2-A. TAC trax con lesin pulmonar descrita. 2-B: Misma imagen duran-
te la biopsia torcica guiada por TC.
l con frmula normal, Hb 76 g/l, normoctica normocr-
mica, hematocrito 0.22 l/l, 234.000 plaquetas/l y coagu- de un engrosamiento nodular, irregular, captante a nivel
lacin normal. La TC craneal mostraba lesiones parietales de la pleura visceral lateral-basal izquierda, con pequeo
izquierdas bien delimitadas la mayor de ellas de 14 mm y derrame pleural loculado adyacente (figura 2A). La coexis-
una lesin de aspecto qustica en lbulo frontal izquierdo tencia de LOE cerebral, masa pulmonar y antecedente de
sugestiva de origen neoplsico. Se realiz una RM cerebral tabaquismo sugera el diagnstico de carcinoma pulmonar
donde se apreciaba una lesin de aspecto qustico en lbu- con metstasis cerebral por lo que se decidi el abordaje
lo frontal izquierdo alto con extensas zona de edema peri- de la lesin pulmonar. Se realiz biopsia de la lesin pul-
lesional, sugestiva de lesin metstasica sin poder descartar monar guiada por TC (figura 2B) y posterior estudio por
otras posibilidades diagnosticas (figura 1). Ante la sospecha anatoma patolgica sin demostrar la presencia de un tu-
del origen metastsico se solicit un estudio de extensin mor primario. Por ello se deriv al servicio de neurociruga
mediante TC toraco-abdominal demostrado la presencia de nuestro hospital (H.U. Virgen del Roco) para obtencin

216
CAPTULO 12
INFECCIONES POR HONGOS

de biopsia y estudio anatomopatolgico ya que la biopsia


pulmonar haba sido negativa. Finalmente el paciente fue
intervenido con el objetivo de la reseccin completa de la
lesin por la alta sospecha de neoplasia. Se realiz una cra-
neotoma frontal izquierda observando una tumoracin de
aspecto mucoide amarillenta, con numerosas formaciones Figura 3. Imgenes al microscopio del tejido cerebral con distintas tinciones. 3A:
Hematoxilina-eosina. 3B: Tincin Grocott. 3C: Tincin PAS positiva.
qusticas que se resec completamente. Se enviaron mues-
tras tanto a anatoma patolgica como a microbiologa.
plsico de la lesin. En las muestras de anatoma patol-
Diagnstico diferencial gica se observaron numerosas estructuras de morfologa
redondeada de entre 4 y 15 micras de dimetro cuya pared
La RM y TC craneal constituye el primer escaln diag- presentaba una tincin positiva con PAS, Grocott y azul
nstico para la deteccin y caracterizacin de las lesiones alcian (figura 3), compatibles con Cryptococcus neoformans. El
expansivas intracraneales. Las metstasis cerebrales son los cultivo en medio aerobio y en medio de cultivo de hon-
tumores intracraneales ms frecuentes. Su diagnstico se gos se inform como Cryptococcus neoformans. El diagnstico
ve facilitado normalmente gracias a la presencia de un tu- final fue el de absceso cerebral por criptococo (criptoco-
mor primario aunque hasta en un tercio de las metstasis coma) en un paciente inmunocompetente con un ndulo
cerebrales ste es desconocido. En nuestro caso, el diag- pulmonar presumiblemente de la misma etiologa.
nstico de cncer de pulmn con metstasis cerebral era
la primera opcin tanto por el antecedente de tabaquismo Evolucin
como por la lesin descubierta en el TC de trax simult-
nea al diagnstico de la LOE cerebral, siendo adems este Tras la intervencin quirrgica el paciente present una
tipo de tumor el que ms frecuentemente metastatiza en buena evolucin clnica inicial pero posteriormente sufri
cerebro. Sin embargo, la biopsia guiada por TAC de la un deterioro neurolgico progresivo con disminucin del
lesin pulmonar no permiti el diagnstico de confirma- nivel de consciencia y afasia sensitiva. Se realiz puncin
cin histolgico. Otros tumores metastsicos a considerar lumbar con obtencin de lquido normal siendo los cul-
eran el melanoma (en nuestro caso el paciente presentaba tivos, tinta china y ltex a criptococo negativos. El TC
un carcinoma basocelular), tumores digestivos y del apara- mostr una lesin focal compatible con un absceso cere-
to genitourinario, aunque el paciente no presentaba nin- bral, siendo reintervenido con extirpacin del absceso. El
gn dato que orientase a estos diagnsticos y el estudio de estudio microbiolgico demostr la presencia de Klebsiella
extensin haca poco probable estos dos ltimos. Tras el pneumoniae. Tras la intervencin y antibioterapia el paciente
estudio de extensin negativo haba que considerar otras present excelente evolucin clnica y resolucin comple-
posibilidades diagnsticas como los tumores primarios del ta sin complicaciones posteriores. Finalmente fue dado de
SNC (el grupo de los gliomas es el ms frecuente a nivel alta tras completar 6 semanas de tratamiento con anfoteri-
intraparenquimatoso), patologa infecciosa y procesos in- cina B liposomal y flucitosina, y posteriormente continu
flamatorios sistmicos. Dentro de las causas infecciosas des- tratamiento ambulatorio con fluconazol 800 mg al da du-
tacan por su frecuencia los abscesos de origen bacteriano, rante 6 meses. Tras finalizar el tratamiento el paciente se
infecciones por parsitos tipo hidatidosis cerebral y neuro- mantiene asintomtico con una calidad de vida excelente
cisticercosis, as como infecciones por hongos, aunque, en sin dficit neurolgico asociado.
este paciente, no existan datos de inmunosupresin que hi-
cieran considerar esta etiologa. En todas ellas el diagnstico Discusin
final requiere del estudio histolgico de la lesin que puede
obtenerse mediante puncin esterotxica (con menos mor- La infeccin por Cryptococcus neoformans en pacientes sanos
bilidad) o por reseccin completa de la lesin, como ocurri e inmunocompetentes es extremadamente rara con una
en nuestro caso ante la sospecha de malignidad. incidencia de slo 0,2/milln de habitantes al ao. De he-
cho, algunos autores defienden que detrs de un paciente
Diagnstico final considerado inmunocompetente con criptococosis subya-
ce algn tipo de inmunodepresin y que sta debera ser
El estudio anatomopatolgico mostraba ausencia de me- investigada. Los rganos ms frecuentemente afectados
tstasis y signos de malignidad descartando el origen neo- en sujetos no-VIH son el pulmn (36%) y el SNC (51%).

217
CAPTULO 12
INFECCIONES POR HONGOS

La va area constituye la puerta de entrada ms comn sos. El diagnstico requiere de su aislamiento en cultivo o
produciendo, a nivel pulmonar, un amplio espectro clnico en su defecto de su deteccin mediante estudio histolgico.
que abarca desde una simple colonizacin hasta una neu- El diagnstico se basa en la tincin de tinta china, que per-
mona grave con distress respiratorio. En sujetos inmuno- mite visualizar la capsula de la levadura, y en el cultivo. En
competentes es frecuente que la afectacin pulmonar no muestras de tejido su identificacin se realiza mediante tin-
tenga expresin clnica y normalmente se diagnostica por ciones para mucinas, como el mucicarmn y el azul alcin,
anormalidades en la radiografa de trax donde se aprecia PAS y las tinciones de plata (Grocott y metenamina), que
uno o varios ndulos bien delimitados no calcificados que tien la cpsula del hongo. La identificacin de especie re-
pueden ser confundidos con neoplasias pulmonares hasta quiere mtodos moleculares no disponibles en la mayora
el estudio anatomopatolgico. Otras anormalidades radio- de los laboratorios de microbiologa asistencial.
lgicas son la presencia de linfadenopatas hiliares, derra- La terapia quirrgica es el tratamiento de eleccin de las
me pleural, infiltrados lobares o cavitaciones. El pulmn masas criptocccicas del SNC mayores de 3 cm y aunque
constituye la puerta de entrada y posteriormente, por va su necesidad est siendo cuestionada debido a la disponibi-
hematgena, alcanza otros rganos como el SNC, ojos, lidad de mejores frmacos antimicticos, las series de casos
aparato genitourinario, etc. Ante la presencia de criptoco- publicadas muestran buenos resultados cuando la masa es
cosis pulmonar se recomienda el despistaje de afectacin accesible, ya que de esta forma se disminuye el inculo. Las
del SNC mediante puncin lumbar incluso cuando no recomendaciones del tratamiento antifngico recogidas en
existe clnica neurolgica en pacientes inmunocomprome- las actuales guas de la Sociedad Americana de Enferme-
tidos con alto riesgo de diseminacin. dades Infecciosas (con un nivel de envidencia BIII, basado
En el paciente inmunocompetente, a pesar de su presenta- en series de casos y recomendaciones de expertos) incluyen
cin excepcional, la afectacin del SNC es frecuente, sien- una terapia de induccin con anfotericina B desoxicolato
do la forma de presentacin hasta en el 51%, segn series 0,7 mg/Kg/d o anfotericina B liposomal 3-5 mg/Kg/d iv
de casos publicados. Se manifiesta predominantemente junto a flucitosina 100 mg/kg vo repartida en cuatro dosis
como masas granulomatosas denominadas criptococomas durante 6 semanas, seguida de un tratamiento de consoli-
y no como meningoencefalitis. En una revisin de la lite- dacin con fluconazol 400-800 mg/d entre 6 y 18 meses.
ratura, Li et al recogen slo 17 casos publicados hasta 2009 Se desconoce si el tratamiento exclusivo con fluconazol,
de criptococomas en inmunocompetentes. La clnica de con menos efectos adversos, podra constituir una alterna-
presentacin ms frecuente es la cefalea aislada (85.2%). tiva en pacientes inmunocompetentes
Otros sntomas se deben al efecto masa que produce, como Las infecciones por criptococo son muy infrecuentes en
dficits motores focales o aparicin de crisis comiciales. En sujetos inmunocompetentes y constituye una causa poco
el diagnstico de este tipo de infeccin la puncin lumbar comn de ndulo pulmonar y cerebral concomitante. En
tiene poca rentabilidad ya que, a diferencia de lo que ocu- el abordaje quirrgico de las lesiones ocupantes de espa-
rre con las meningoencefalitis, el cultivo del LCR suele ser cio cerebrales es importante ser sistemtico en el envo de
normal. En inmunocompetentes no suele sospecharse por muestras a microbiologa y a anatoma patolgica ante la
su baja incidencia y habitualmente el primer diagnstico posibilidad de encontrarnos con etiologas infrecuentes.
de sospecha lo constituyen los tumores primarios y mets-
tasis del SNC: de hecho, las pruebas de neuroimagen no Agradecimientos
permiten distinguir un tumor de un criptopcocoma. La
TC craneal queda restringida a la valoracin inicial o de Agradecimiento por la cesin de las imgenes a Elena A.
urgencia, siendo la RM la prueba de imagen de eleccin Durn Izquierdo, residente de anatoma patolgica del
debido a su mayor sensiblidad para el estudio de las le- Hospital Universitario Virgen del Roco.
siones intraparenquimatosas, pues permite conocer de for-
ma muy precisa la localizacin de la lesin y su relacin
anatmica con las estructuras vecinas. Las caractersticas
del criptococoma en la RM son indistinguibles de lesiones
neoplsicas e inflamatorias (como por ejemplo la neuro-
sarcoidosis) del SNC presentando las mismas caractersti-
cas: baja intensidad en T1 y alta intensidad en T2, aunque
la presencia de formaciones qusticas se da en la mayora
de los casos (65%) lo que puede distinguirlo de otros proce-

218
CAPTULO 12
INFECCIONES POR HONGOS

Bibliografa

1. Li Q , You C, Liu Q , Liu Y. Central nervous system


cryptococcoma in immunocompetent patients: a short
review illustrated by a new case. Acta Neurochir. 2010;
152:12936.
2. Daz-Prez JA, Garca-Vera JA, Mantilla-Hernndez
JC, Pradilla-Ardila G. Criptococoma en el sistema
nervioso central de un paciente no inmunoafectado.
Rev Neurol. 2008; 46:97-101.
3. Jung A, Korsukewitz C, Kuhlmann T, Richters M,
Fischer B, Niederstadt T et al. Intracerebral mass
lesion diagnosed as cryptococcoma in a patient with
sarcoidosis, a rare opportunistic manifestation induced
by immunosuppression with corticosteroids. J Neurol.
2012; 259 (10): 2147-50.
4. Kanaly CW, Selznick LA, Cummings TJ, Adamson
DC. Cerebellar cryptococcoma in a patient with un-
diagnosed sarcoidosis: case report. Neurosurgery.
2007; 60:E571.
5. Arbizu J, Domnguez PD, Diez-Valle D, Vigil C, Gar-
ca-Eulate R, Zubieta JL. Neuroimagen de los tumo-
res cerebrales. Rev Esp Med Nucl. 2011; 30:4765.
6. Perfect JR, Dismukes WE, Dromer F, Goldman DL,
Graybill JR, Hamill RJ et al. Clinical Practice Gui-
delines for the Management of Cryptococcal Disea-
se: 2010 Update by the Infectious Diseases Society of
America. Clin Infect Dis. 2010; Feb 1; 50:291-322.

219
Fiebre y cefalea en paciente
con sarcoidosis sistmica
Gonzlez-Estrada, A; Praena-Segovia, J;
Garca Ocaa, PP; Garca-Morillo, S.
Hospital Universitario Virgen del Roco. Sevilla.

Caso clnico

Mujer 45 aos que acude a urgencias por cefalea de una


semana de evolucin asociada a fiebre en las ltimas 48
horas.
Destacar entre sus antecedentes personales el diagnstico,
seis aos anteriores al episodio actual, de tuberculosis pul-
monar y meningitis tuberculosa diagnosticada y tratada
correctamente en otro centro. En el ltimo ao comienza
con cuadro de tos seca no productiva para la que recibe
tratamiento intermitente con corticoides. En los ltimos Figura 2: Resonancia Magntica Cerebral. Se objetiva lesin nodular ovalada
hipercaptante en la periferia del hemisferio cerebeloso derecho sin restriccin en
cuatro meses adems asocia fiebre por lo que es derivada a difusin.
consultas de medicina interna con el diagnstico sindrmi-
co de fiebre de origen desconocido y sospecha de neumo- estructuras micticas con tinciones de Ziel-Neelsen (ZN),
pata intersticial para estudio. Se realiza estudio de imagen PAS y Grocott. Adems se practic una puncin aspira-
mediante tomografia axial computarizada de alta resolu- cin con aguda fina (PAAF) de adenopata mediastnica
cin (TACAR) de pulmn donde se describen adenopatas que descart patologa neoplsica e infecciosa asociada.
mediastnicas y broncopulmonares bilaterales hipervascu- Los estudios microbiolgicos que incluyeron cultivo de
lares asociadas a un patrn intersticial bilateral tipo sep- bacterias, ZN y cultivo de micobacterias y cultivo de hon-
tal interlobulillar con escasa micronodularidad. Se realiza gos en lavado broncoalveolar (LBA) fueron negativos. La
adems una tomografa computerizada (TC) abdominal serologa de VIH, Brucella melitensis, Coxiella burnetii, Rickett-
donde se observa ligera hepatomegalia y adenopatas re- sia conorii y Rickettsia typhi, Toxoplasma spp., Leishmania spp.,
troperitoneales pre y latero articas de hasta 6 mm, siendo sfilis y virus hepatotropos result negativa.
los resultados de ambos TC compatibles con sarcoidosis. Ante los hallazgos histolgicos y exclusin de otras causas
El diagnstico de sarcoidosis se establece mediante realiza- de enfermedad granulomatosa se establece el diagnstico
cin de biopsia heptica y biopsia de adenopatas mediast- de sarcoidosis sistmica (afectacin pulmonar y heptica)
nicas. La histologa revela granulomas lobulillar y portal de recibiendo tratamiento con prednisona 30 mg al da, aza-
tipo sarcoideo sin necrosis, sin identificar micobacterias ni tioprina 150 mg al da e hidroxicloroquina 100 mg cada 12
horas con cese del cuadro febril a la semana.
Tras dos meses desde el inicio del tratamiento, y coinci-
diendo con corticorreduccin, la paciente acude a Urgen-
cias por cefalea intensa holocraneal de una semana de evo-
lucin que no cede con analgesia habitual y aparicin de
fiebre en las ltimas 48 horas. En el momento de su ingreso
se encuentra afectada por la cefalea intensa, pero con acep-
table estado general, mantiene buen nivel de consciencia y
esta orientada en las tres esferas. Fiebre de 38.7C y con
tensin arterial de 110/60 con FC 90 lpm. Auscultacin
cardiopulmonar, abdomen y miembros inferiores sin al-
teraciones. Exploracin neurolgica: pupilas isocricas y
Figura 1. Radiografa de Trax. Patrn micronodular bilateral de predominio normorreactivas, campimetra normal, pares craneales sin
perihiliar compatible con sarcoidosis pulmonar ya conocida, sin otros datos de
afectacin aguda.
alteraciones, no dficits sensitivos ni motores. Reflejos os-
teotendinosos conservados. No dismetra. Romberg nega-

220
CAPTULO 12
INFECCIONES POR HONGOS

tivo. No presentaba rigidez de nuca ni signos menngeos.


Ante la clnica se solicit radiografa (Rx) trax que mostr
un infiltrado micronodular bilateral y simtrico ya conoci-
do en relacin con su enfermedad de base (Figura 1), TC
crneo urgente en el cual no se identifican lesiones ocu-
pantes de espacio ni signos de hemorragia ni otras altera-
ciones intracraneales.
Se procedi a realizar una puncin lumbar (PL), donde se
obtuvo un lquido cefalorraqudeo (LCR) trasparente con
bioqumica que mostraba 2 clulas, glucosa de 62 mg/dl
(glucemia 98 mg/dl), protenas de 0,54 g/L y ADA 7,9
U/L. Se enviaron muestras para tincin de Gram y cultivo
que resultaron negativas. Ante la normalidad del LCR se Figura 3. Radiografa trax. Ndulos pulmonares paratraqueales derechos com-
patibles con criptococosis pulmonar tras el inicio de tratamiento inmunosupresor
solicit una resonancia magntica (RM) cerebral que ad- para neurosarcoidosis.
virti una lesin nodular ovalada hipercaptante en la pe-
riferia de hemisferio cerebeloso derecho, que no restringa sin de salida de LCR y envo de nuevas muestras a bioqu-
difusin, junto con la presencia de patrn de captacin de mica y microbiologa, en esta ocasin incluyendo estudio
aspecto micronodular difuso que afectaba a folias del cere- de hongos y micobacterias.
belo de forma bilateral (Figura 2). El diagnstico radiolgi- La presin de salida del LCR fue de 50 cm H20, por lo que
co fue de posible neurosarcoidosis. se decidi contactar con el servicio de Neurociruga que
realiz drenaje lumbar con mejora evidente del nivel de
Evolucin y Diagnstico diferencial consciencia quedando paucisintomtica. Se reciben los re-
sultados de estudio bioqumico del LCR que muestra nue-
Basndose en la aparicin de la clnica coincidiendo con la vamente escasa celularidad (6 cel/mm3 con 70% PMN)
reduccin de corticoides, los hallazgos de la RM cerebral, pero con aumento de protenas 0,95 g/l y consumo de glu-
las caractersticas bioqumicas del LCR (que slo mostraba cosa 30 mg/dl (glucemia capilar 100 mg/dl). El estudio
una discreta hiperproteinorraquia) y negatividad de hemo- microbiolgico solicitado aport el diagnstico tras revelar
cultivos y cultivo de LCR se estableci el diagnstico de la presencia de Cryptococcus neoformans (tincin con tinta chi-
por neurosarcoidosis. na, cultivo de hongos y cultivo aerobio). Los hemocultivos
Se aument la dosis de esteroides a 60 mg/da y se admi- extrados en este segundo pico febril tambin aislaron el
nistr un primer bolo de ciclofosfamida de 750 mg intra- mismo microorganismo. Se realiz estudio de inmunodefi-
venoso. En contra de lo esperado la paciente present una ciencias en el que present linfopenia de 87 linfocitos: CD4
evolucin trpida, con empeoramiento inicial de la cefa- 35 cel/uI, CD3 83 cel/uI, CD8 48 cel/uI, NK 0 cel/uI.
lea, pico febril y aparicin de visin borrosa, aadindose
a las 48 horas diplopia por parlisis del sexto par craneal Diagnstico final
y fluctuaciones en el nivel de consciencia. Ante estos ha-
llazgos se realiza nueva Rx trax (Figura 3) en la que se Meningitis criptoccica en paciente con sarcodosis pulmo-
muestran ndulos pulmonares derechos y una TC crneo nar y heptica en tratamiento inmunosupresor.
urgente donde se aprecia la misma imagen hipodensa ova- Masa cerebelosa compatible con posible neurosarcoidosis
lada en hemisferio cerebeloso derecho estable con respecto sin poder descartar criptococoma.
a la observada en la RM previa sin datos de hidrocefa- Criptococosis pulmonar.
lia ni efecto de masa. Se realiz fondo de ojo objetivando Debido a la localizacin de la lesin en fosa posterior y
papiledema bilateral. Se establece el juicio clnico sindr- dado la profundidad de la misma se desestim abordaje
mico de meningoencefalitis con hipertensin intracraneal, quirrgico de la lesin por los riesgos de la intervencin.
afectacin de pares craneales y masa cerebelosa realizan- Se inici tratamiento con Anfotericina B liposomal a dosis
do el diagnstico diferencial entre tuberculosis del sistema de 5 mg/kg/da ms flucitosina 100 mg/kg/da durante
nervioso central (SNC) con afectacin menngea y posible 6 semanas con mejora clnica inmediata y estabilizacin,
tuberculoma, criptococosis del SNC y menos probable in- que permiten la retirada del drenaje lumbar a la semana.
feccin por Nocardia spp. As mismo, se mantiene el tratamiento inmunosupresor
Se repite una nueva puncin lumbar con medicin de pre- con esteroides y azatioprina por su enfermedad sistmica

221
CAPTULO 12
INFECCIONES POR HONGOS

de base. Se realiz una nueva RM cerebral de control al TNF. La dosis depender de los sntomas y de la respuesta,
mes que mostr disminucin de la lesin. Sin embargo la debiendo individualizar el tratamiento en cada caso. La
evolucin de la paciente fue trpida con recurrencias de pauta de corticoides recomendada es de 0,5 mg/kg/da
los episodios de meningitis criptoccica precisando nuevas para las formas pulmonares y 1 mg/kg/da para las formas
derivaciones de LCR. extrapulmonares. Se recomienda entre un ao y dos aos
debido a las recadas (formas pulmonares y extrapulmona-
Discusin res excepto cutnea, respectivamente), la reduccin de los
corticoides se inicia cuando presenta estabilidad durante
La neurosarcoidosis es una manifestacin relativamente 6 meses2.
comn de la sarcoidosis sistmica, afectando del 5 al 15% La criptococosis del SNC es una infeccin fngica oportu-
de estos pacientes, con series que describen hasta el 27% nista (germen encapsulado), que sucede en pacientes con
de los casos. En estudios realizados post mortem, el 50% de afectacin de la inmunidad de tipo celular. Por este motivo
las sospechas clnicas de neurosarcoidosis fueron confirma- guarda estrecha relacin con los enfermos con sarcoidosis,
das. Esta enfermedad puede aparecer tras los 20 aos del tanto por las alteraciones de la inmunidad que ocurren en
diagnstico de sarcoidosis, normalmente sucede con la re- esta enfermedad como por el tratamiento inmunosupresor
duccin del tratamiento, no obstante, un 50% de los casos utilizado, tal y como ocurre en nuestra paciente, que pre-
de neurosarcoidosis se describe en los primeros dos aos de senta menos de 100 clulas CD4 en el estudio de subpobla-
evolucin de la enfermedad. En aproximadamente el 90% ciones. El tratamiento prolongado con corticoides favorece
de los casos existe enfermedad sistmica. Las lesiones pue- la diseminacin de la infeccin. La meningitis o menin-
den ocurrir a cualquier nivel del SNC, sin embargo tiene goencefalitis por criptococo deben ser consideradas siem-
predileccin por las meninges de la base del crneo, siendo pre en un paciente con sarcoidosis que comience con clni-
los nervios craneales los ms frecuentemente afectados. El ca neurolgica. Las manifestaciones clnicas, alteraciones
parnquima cerebral se afecta en el 50% de los casos, me- del LCR y hallazgos de la RM cerebral en la criptococosis
ninges 20-40%, siendo menos frecuente la afectacin me- del SNC son superponibles con la neurosarcoidosis, lo que
dular (10%). La sintomatologa es inespecfica e incluye un dificulta su diagnstico, especialmente en los pacientes que
amplio espectro clnico, desde neuropatas craneales, sn- presentan masa cerebral. Aunque los hallazgos de la RM
drome de irritacin menngea, hipertensin craneal hasta son superponibles, las masas por criptococoma comn-
alteraciones de la personalidad y cambios endocrinolgi- mente se localizan en cerebelo, ganglios basales y regiones
cos. As mismo, las pruebas de diagnstico, como la RM temporoparietales. Es muy importante mantener una alta
y LCR son poco especficas, caracterizndose por pleoci- sospecha clnica de esta entidad en pacientes con sarcoido-
tosis linfoctica, consumo de glucosa, protenas altas y pre- sis y sintomatologa neurolgica, ya que habitualmente, y
sin de salida elevada. La RM es la prueba diagnstica de sobre todo en los casos de criptococoma, el estudio micro-
eleccin, superior al TC debido a su elevada sensibilidad, biolgico del LCR puede ser inicialmente negativo para
siendo caracterstico un realce leptomenngeo, lesiones pa- Criptococcus neoformans y nicamente presentar una elevada
renquimatosas hipointensas en T1 e hiperintensas en T2, presin de salida del lquido e hiperproteinorraquia.3,4 La
engrosamiento de nervios craneales e hidrocefalia. Los ha- aparicin de hipertensin intracraneal (HIC) sin hidroce-
llazgos ms frecuentes, si bien no son especficos, son las le- falia en el TC es caracterstica en las fases iniciales, incluso
siones hiperintensas periventriculares en sustancia blanca con estudios microbiolgicos an negativos. La realizacin
y subcorticales. El diagnstico, por lo tanto es un reto, con- del fondo de ojo permite en estos casos advertir signos indi-
siderndose diagnstico probable cuando existe sarcoidosis rectos de hipertensin intracraneal aunque el diagnstico
sistmica y clnica sugestiva de neurosarcoidosis con exclu- requiere de la medicin de la presin de salida. El diag-
sin de otras causas de disfuncin neurolgica, y diagnsti- nstico microbiolgico de Cryptococcus neoformans se basa en
co definitivo tras respuesta favorable al tratamiento despus el aislamiento del hongo en cultivo, as como su identifi-
de un ao. En caso de no lograr confirmacin histopato- cacin con tinta china. Las tcnicas anatomopatolgicas
lgica de sarcoidosis sistmica y no descartar otras causas (AP) de deteccin de Cryptococcus neoformans se basan en la
de enfermedad neurolgica se considera neurosarcoidosis deteccin de su cpsula mediante tcnicas de inmunohis-
posible1. El tratamiento requiere terapia inmunosupresora, toqumica. A pesar del diagnstico del LCR, siempre que
siendo los esteroides la primera lnea. De segunda lnea, sea posible cuando se detecta una masa cerebral por RM,
ahorradores de esteroides, como azatioprina y micofeno- el diagnstico precisa de una biopsia de la lesin con estu-
lato de mofetilo, metotrexato, ciclosporina y agentes anti dio de AP y pruebas microbiolgicas, ya que las lesiones

222
CAPTULO 12
INFECCIONES POR HONGOS

macroscpicamente son indistinguibles (criptococoma vs necesario en las infecciones por Cryptococcus spp. corregir
neurosarcoidosis).5 Se recomienda como tratamiento para la situacin de inmunosupresin siempre que sea posible.
la criptococosis del SNC en pacientes no VIH anfoterici-
na B liposomal asociado a flucitosina durante al menos 4 Bibliografa
semanas seguido de fase de consolidacin con fluconazol
a dosis de 400-800 mg al da durante 8 semanas, y pos- 1. Hamzeh N. Sarcoidosis. Med Clin North Am. 2011;
terior mantenimiento con fluconazol 200 mg da de 6 a 95: 12234.
12 meses.6 En los casos de criptococoma no hay estudios 2. Gascn-Bayarri J, Ma J, Martnez-Ylamos S, Mu-
prospectivos en pacientes no VIH y las recomendaciones rillo O, Re R, Rubio S. Neurosarcoidosis. Report
se basan en estudios realizados en poblacin VIH. Se reco- of 30 cases and a literature survey. Eur J Intern Med.
mienda prologar la duracin de la fase de induccin y con- 2011; 22:12532.
solidacin segn respuesta microbiolgica, radiolgica y 3. Jung A, Korsukewitz C, Kuhlmann T, Richters M,
clnica. Se recomiendan al menos 6 semanas de induccin Fischer B, Niederstadt T, et al. Intracerebral mass
y de 6 a 18 meses de consolidacin. La ciruga est indi- lesion diagnosed as cryptococcoma in a patient with
cada para lesiones accesibles con ms de 3 cm si producen sarcoidosis, a rare opportunistic manifestation induced
efecto masa/hidrocefalia o para alcanzar el diagnstico. Se by immunosuppression with corticosteroids. J Neurol.
recomienda derivacin de LCR si la presin intracraneal 2012; 259: 2147-50.
es mayor a 25 cmH2O7. 4. Kanaly CW, Selznick LA, Cummings TJ, Adamson
En nuestro caso no se diferenciaron mediante estudio histo- DC. Cerebellar cryptococcoma in a patient with un-
patolgico ambas entidades ya que no se realiz la biopsia diagnosed sarcoidosis: case report. Neurosurgery.
cerebelosa debido a la localizacin de la lesin, dificultades 2007; 60: E571.
tcnicas del abordaje quirrgico y la elevada morbi-morta- 5. Lin YJ, Yang TM, Lin JW, Song MZ, Lee TC. Pos-
lidad asociada. As mismo, el diagnstico de criptococosis terior fossa intracranial inflammatory pseudotumor: a
del SNC estaba demostrado, por lo que se deba continuar case report and literature review. Surg Neurol. 2009;
el tratamiento con anfotericina B y flucitosina. Tambin 72: 712-6.
se justificaba el mantenimiento del tratamiento inmuno- 6. Johns CS, Michelle TM. The clinical management of
supresor para el diagnstico confirmado mediante AP de sarcoidosis. A 50-year experience at the Johns Hopkins
sarcoidosis sistmica. No obstante, la inmunosupresin re- Hospital. Medicine (Baltimore). 1999; 78: 65-111.
querida para el tratamiento de la sarcoidosis con afecta- 7. Perfect JR, Dismukes WE, Dromer F, Goldman DL,
cin cerebral es mayor que para otras localizaciones. Graybill JR, Hamill RJ, et al. Clinical Practice Gui-
Este caso implica un gran reto diagnstico, no solo por el delines for the Management of Cryptococcal Disea-
alto grado de solapamiento que existe entre ambas enfer- se: 2010 Update by the Infectious Diseases Society of
medades, sino tambin por la dificultad del acceso de la America. Clin Infect Dis 2010; 50: 291-322.
lesin. Por una parte, el diagnstico de neurosarcoidosis
requiere un alto ndice de sospecha, la exclusin de otros
procesos neurolgicos y es fundamental descartar infeccio-
nes oportunistas que puedan asociarse a la alteracin de
la inmunidad celular que acompaa a la sarcoidosis. As
mismo, es de suma importancia no olvidar que en pacien-
tes inmunodeprimidos las infecciones oportunistas, espec-
ficamente en sarcoidosis con afectacin sistmica y clnica
neurolgica y ante la presencia de masa cerebral, se deber
de incluir dentro del diagnstico diferencial el criptococo-
ma y/o la meningitis criptococcica, ya que la normalidad
del LCR no excluye el diagnstico del mismo. De la misma
manera, la continuacin del tratamiento inmunosupresor
a elevadas dosis por la sospecha de neurosarcoidosis pudo
contribuir a la aparicin de posteriores recurrencias y mala
evolucin clnica, por lo que en la medida de lo posible, se
recomienda retirar la causa de la inmunosupresin, siendo

223
Ndulo pulmonar solitario
en paciente inmunocompetente
Gonzlez Surez, M; Jimnez Rodrguez, E;
Merchante Gutirrez, N; Melguizo Moya, I.
Hospital Universitario Nuestra Seora de Valme. Sevilla.

Caso clnico tpicamente endobronquiales. Sin embargo, un 20%


se presenta como NPS perifrico con bordes bien de-
Se presenta el caso de una mujer de 25 aos que acude a finidos.
Urgencias por un cuadro de 72 horas de evolucin carac- 2. Metstasis pulmonares. Aunque se presentan comn-
terizado por tos no productiva y ruidos respiratorios, sin mente como ndulos pulmonares mltiples, tambin
fiebre u otra sintomatologa acompaante. pueden presentarse como NPS como en el caso de me-
Como antecedentes a destacar, reside en mbito urbano y lanoma, sarcoma y carcinomas de colon, mama, rin
tiene contacto con animales domsticos (canario y perros). y testculo. Corresponde a menos del 10% del total de
No presenta alergias medicamentosas ni antecedentes fa- los NPS.
miliares ni personales de inters, salvo tabaquismo activo 3. Otros: linfoma primario extraganglionar, plasmocito-
de 7 paquetes/ao. ma, schwannoma.
En radiografa de trax realizada en el Servicio de Ur-
gencias del centro hospitalario al que consulta, se aprecia Causas benignas (50-60%)
lesin redondeada en lbulo superior derecho y ante la cl-
nica respiratoria presentada, con sospecha de neumona 1. Infecciosas. Los granulomas infecciosos causan apro-
adquirida en la comunidad, inicia tratamiento con levo- ximadamente el 80% de los NPS benignos. La tuber-
floxacino 500 mg cada 24 horas. Tras 7 das de tratamien- culosis y las infecciones fngicas (histoplasmosis, coc-
to desaparece la sintomatologa. cidioidomicosis, criptococosis y blastomicosis) pueden
La paciente fue revisada en consultas externas de Neumo- ser el origen de stos. Otras causas infecciosas menos
loga de otro centro. En radiografa de trax de control que comunes son los abscesos bacterianos, el aspergiloma
se realiz un mes posterior al inicio del cuadro se observa y la infeccin por Pneumocystis jirovecii.
la persistencia de la imagen redondeada, bien delimitada, 2. Tumores benignos. Los hamartomas causan el 10% de
de unos 3 cm de dimetro, situada en lbulo superior dere- los ndulos benignos pulmonares. Otras causas menos
cho, por lo que se inicia estudio de ndulo pulmonar soli- comunes son fibromas, lipomas, leiomiomas, heman-
tario (NPS) (Imgenes 1 y 2). giomas, amiloidoma y neumocitoma.
3. Vasculares. Entre ellas, malformaciones arterioveno-
Diagnstico diferencial sas, contusiones e infartos pulmonares.
4. Inflamatorias. A destacar como causa de NPS las si-
El NPS se define radiolgicamente como una nica lesin guientes enfermedades sistmicas:
pulmonar intraparenquimatosa, redondeada u ovalada, de Granulomatosis de Wegener
bordes bien definidos y con un dimetro no mayor a 3 cm. Artritis reumatoide
Puede ser la forma de presentacin tanto de procesos be- Sarcoidosis
nignos como malignos, constituyendo un desafo diagns- 5. Malformaciones congnitas como el quiste broncog-
tico. A continuacin se discute el diagnstico diferencial nico.
del NPS:
Centrndonos en nuestro caso, nos encontramos ante una
Causas malignas (40-50%) paciente joven, en principio inmunocompetente, sin ante-
cedentes familiares ni personales de enfermedad neopl-
1. Neoplasia primaria de pulmn. El adenocarcinoma es sica. De tratarse de un NPS de etiologa tumoral podra
el subtipo histolgico que se presenta de forma ms corresponder a una neoplasia primaria de pulmn de pre-
frecuente como NPS, representando el 30-50% de las sentacin precoz en mujer fumadora, o bien de un hamar-
lesiones malignas, seguido del carcinoma de clulas es- toma, por ser el tumor benigno ms frecuente.
camosas con el 10-37%. Los tumores carcinoides son Dentro de las causas infecciosas, la mayora son infeccio-

224
CAPTULO 12
INFECCIONES POR HONGOS

nes poco frecuentes en pacientes no inmunodeprimidos, a dad de descartar una forma diseminada de criptococosis,
excepcin de la tuberculosis. Por lo que en principio, salvo as como de realizar tratamiento antifngico. El antgeno
que la paciente no tuviera inmunodeficiencia no conocida de criptococo fue negativo, el TAC trax y abdominal no
hasta el momento, sera una etiologa poco probable. mostr datos dignos de mencin y la RMN de crneo fue
Aunque podra ser la forma de presentacin de una enfer- normal.
medad sistmica, la ausencia de otros datos clnicos hacen Dada la ausencia de sntomas neurolgicos y la normali-
en principio este diagnstico menos probable. dad de las pruebas complementarias no se realiz puncin
Por ltimo, dentro de las causas menos frecuentes, una lumbar para descartar criptococosis menngea.
malformacin vascular o congnita podran ser la causa de Por otro lado, ante la ausencia de datos de infeccin sist-
la lesin radiolgica. mica (estudio analtico, TAC de trax y abdomen y RM de
crneo sin hallazgos) en una paciente inmunocompetente
Evolucin se consider resuelta la infeccin tras tratamiento quirrgi-
co y se decidi no realizar tratamiento antifngico.
Dada la persistencia de la lesin en radiografa de trax se
solicit tomografa axial computarizada (TAC) de trax en Diagnstico final
la que se objetiv una consolidacin pulmonar de 5 cm,
redondeada, de bordes polilobulados y con broncograma Neumona abscesificada por Criptococcus neoformans mani-
areo sin signos de cavitacin en segmento posterior de festada como ndulo pulmonar solitario en paciente in-
lbulo superior derecho. Ante estos hallazgos, se realiz munocompetente.
fibrobroncopia donde se apreci una estenosis de aspec-
to inflamatorio en segmento apical de bronquio principal Discusin
derecho sin otros hallazgos, que se biopsi. Las muestras
de biopsias obtenidas no mostraron hallazgos de clulas La infeccin por Criptococcus neoformans se presenta con ms
neoplsicas. La baciloscopia y el cultivo de lavado bron- frecuencia en pacientes inmunodeprimidos y su tendencia
coalveolar en medio convencional y de Lowestein resulta- es hacia la diseminacin. Sin embargo, puede afectar a in-
ron negativos. No se procesaron muestras para cultivo de dividuos inmunocompetentes, causando infecciones locali-
hongos. zadas, clnicas o subclnicas. Se ha estimado que alrededor
En aquel momento la analtica general no mostr ningn del 20% de los pacientes que presentan criptococosis sin
dato significativo y la serologa result negativa para virus infeccin por VIH no tiene una enfermedad adyacente ni
de hepatitis B, C y virus de la inmunodeficiencia humana factores de riesgo evidentes.
(VIH) en dos ocasiones. Existen diversas tcnicas para el diagnstico de criptoco-
Ante la incertidumbre de un origen neoplsico de la lesin cosis pulmonar, entre ellas estudio histolgico, cultivo de
y falta de resultados concluyentes, se decidi la realizacin hongos, antgeno criptoccico srico y pruebas de imagen.
de videotoracoscopia exploradora. Se realiz bisegmentec- El diagnstico microbiolgico se puede establecer median-
toma derecha y linfadenectoma. La anatoma patolgica te el cultivo de hongos en muestra de lavado broncoalveo-
de la pieza quirrgica no mostr hallazgos de enfermedad lar, esputo y el tejido pulmonar.
neoplsica. Se apreciaba una histologa compatible con En formas localizadas dichas pruebas pueden ser nega-
granuloma histioctico puro, revelando las tcnicas Grocott tivas, siendo necesaria la biopsia, como ocurri en nues-
y PAS la presencia de Criptococcus neoformans. tro caso, en el que el diagnstico solo fue evidente con los
En este momento se nos remite a la paciente. En la primera hallazgos histolgicos. Por tanto, hay que tener en cuenta
visita a nuestras consultas se encuentra asintomtica y la este aspecto a la hora del diagnstico diferencial del NPS
exploracin general fue rigurosamente normal. El primer y valorar la necesidad de realizacin de tcnicas invasivas
paso fue descartar algn tipo de inmunodeficiencia ante para la obtencin de muestras cuando el diagnstico no
el hallazgo inesperado de criptococosis pulmonar. Reali- est aclarado.
zamos nueva serologa VIH, as como un estudio de au- Un aspecto controvertido es la necesidad de realizacin de
toinmunidad con resultados negativos, y la cuantificacin puncin lumbar de rutina para descartar una meningoen-
del complemento e inmunoglobulinas result normal. Para cefalitis criptoccica en pacientes inmunocompetentes,
poder descartar linfopenia idioptica de linfocitos T CD4 como ocurra en nuestro caso. En general, la mayora de
se solicitaron niveles de CD4 con resultado normal (1408). expertos la recomiendan en pacientes con sntomas neu-
Otro segundo aspecto que nos planteamos fue la necesi- rolgicos, con factores predisponentes de diseminacin o

225
CAPTULO 12
INFECCIONES POR HONGOS

con ttulos elevados (>1:512) de antgeno criptoccico s-


rico. Estos pacientes tienen mayor riesgo de diseminacin
extrapulmonar y siembra en sistema nervioso central. No
est claro si fuera de estos supuestos es seguro no realizar
la puncin lumbar. En nuestro caso, dada la ausencia de
clnica neurolgica con RM de crneo normal, optamos
por no realizarla.
Otro aspecto a destacar de nuestro caso es la necesidad
o no de realizar tratamiento antifngico adicional. Se re-
comienda que en presencia de inmunodepresin se valore
tras hallazgos de infeccin localizada, el inicio del trata-
miento antifngico. Puesto que nuestra paciente era in-
munocompetente, no presentaba sintomatologa alguna ni
datos de enfermedad sistmica, se decidi no llevar a cabo
tratamiento y vigiliar la evolucin.
Se llevaron a cabo revisiones peridicas durante 15 meses,
permaneciendo la paciente asintomtica en todo momen-
to, por lo que fue dada de alta.

Bibliografa

1. Midthun DE, Swensen SJ, Jett JR. Approach to the


solitary pulmonary nodule. Mayo Clin Proc 1993;
68:378-85.
2. Campbell GD. Primary pulmonary cryptococcosis.
Am Rev Respir Dis 1966; 94:236-43.
3. Ost D, Fein A. Management strategies for the soli-
tary pulmonary nodule. Curr Opin Pulm Med 2004;
10:272-8.
4. Chu HQ , Li HP, He GJ. Analysis of 23 cases of pul-
monary cryptococcosis. Chin Med J (Engl) 2004;
117:1425-7.
5. Lindell RM, Hartman TE, Nadrous HF, Ryu JH. Pul-
monary cryptococcosis: CT findings in immunocom-
petent patients. Radiology 2005; 236:326-31.
6. Baddley JW, Perfect JR, Oster RA, et al. Pulmonary
cryptococcosis in patients without HIV infection: fac-
tors associated with disseminated disease. Eur J Clin
Microbiol Infect Dis 2008; 27:937-43.
7. McWilliams A, Tammemagi MC, Mayo JR, et al. Pro-
bability of cancer in pulmonary nodules detected on
first screening CT. N Engl J Med 2013; 369:910-9.
8. Mandell GL, Bennet JE, Dolin R. Enfermedades in-
fecciosas. Principios y prctica. 6 ed. Vol 3. Madrid:
Elsevier; 2006.

226
CAPTULO 13
INFECCIONES POR MICOBACTERIAS
Orquiectoma y nefrectoma tras una prolongada
infeccin genitourinaria: un largo y tortuoso camino
Prez Romero, T.
Hospital Costa del Sol. Marbella.
Roca Oporto, C; Praena Segovia, J; Luque Mrquez, R.
Hospital Universitario Virgen del Roco. Sevilla.

Caso clnico

Se trata de un varn de 46 aos fumador sin otros fac-


tores de riesgo cardiovascular y apendicectomizado que
fue sometido en el ao 2000 a una orquiectoma izquier-
da por orquiepididimitis recurrente. Doce aos ms tarde
acude en repetidas ocasiones a Urgencias por un cuadro
de disuria, tenesmo y hematuria, siendo diagnosticado de
infecciones del tracto urinario realizando varios ciclos de
antibioterapia emprica. Todos los urocultivos practicados
resultaron negativos.
Al ao siguiente es valorado en consulta de Urologa por
piuria estril persistente de un ao de evolucin. Se so-
licita una tomografa computarizada (TC) que muestra
litiasis de pelvis renal izquierda que se extiende a grupo Figura 1. TC de abdomen. Rin izquierdo con mltiples lesiones qusticas y
calcificaciones intrarenales.
calicial inferior con hidronefrosis junto con litiasis ureteral
izquierda y mltiples imgenes de litiasis en grupos medios
y superiores. La cortical renal izquierda est adelgazada y ria: latidos cardacos rtmicos, crepitantes basales y dismi-
lobulada, lo que induce a pensar que la obstruccin de la nucin bibasal del murmullo vesicular. Abdomen algo dis-
va es crnica. Adems presenta quistes renales izquierdos, tendido, blando, no doloroso a la palpacin, con matidez
uno de ellos con calcificaciones groseras y contenido denso en flancos sin peritonismo. Herida quirrgica con buen
de 33 mm y otro de 30 mm que muestra un engrosamiento aspecto. No lesiones cutneas. Catter venoso sin datos de
de pared con una imagen nodular densa (quiste Bosniak flebitis. Sonda urinaria con orina de apariencia normal.
3) (Fig 1; a,b,c,d). Se remiten al hospital de referencia con En las pruebas complementarias destaca de la analtica
la sospecha de ureterohidronefrosis complicada de origen hemograma con leucocitosis de 18.390, hemoglobina 10.8
litisico. Se solicita ecografa doppler de abdomen con po- g/L, hematocrito 31%; resto normal. Bioqumica: AAT 53
tenciador de seal que confirma los datos de la TC. mU/ml, amilasa 27 mU/ml, bilirrubina directa 0.73 mg/
Dos meses despus ingresa de forma programada para dl, creatinina 1.15 mg/dl, protena C reactiva 352 mg/l
realizacin de una ureteroscopia izquierda con extraccin (0-5), Protenas totales 5.6 g/dl; resto normal. Coagula-
de varios restos litisicos a vejiga y finalmente se realiza cin INR 1.51. Serologa: VIH negativo. Hemocultivos y
nefroureterectoma izquierda por laparoscopia. urocultivo: negativos. La radiografa de trax no mostraba
En el postoperatorio inmediato presenta fiebre persistente alteraciones. En planta se realiz paracentesis diagnstica
con hemocultivos negativos que no responde a antibiote- con obtencin de lquido peritoneal (LP) con las siguien-
rapia emprica. Se realiza TC de abdomen donde se des- tes caractersticas: Adenosinadesaminasa (ADA) 91.5mU/
cartan complicaciones en lecho quirrgico pero se objetiva mL, glucosa 0.17g/L, protenas 35.5g/L, recuento de
una moderada cantidad de lquido peritoneal y adenopa- clulas 768 cel/mm3, polimorfonucleares 45%, mono-
tas mesentricas y retroperitoneales. El paciente se tras- nucleares 55%. Gram: no se observan grmenes. Cultivo
lada a planta de Enfermedades Infecciosas para estudio y negativo. Tincin de Ziehl-Neelsen: no se observan bacilos
seguimiento. cido alcohol resistentes (B.A.A.R). PCR TBC complex
En la exploracin fsica presenta buen estado general. Bien negativo. Cultivo de Lwenstein: pendiente.
hidratado y perfundido. Coloracin normal de piel y mu-
cosas. Constantes vitales: TA 135/ 80mmHg, Peso 72 Kg.
No adenopatas cervicales. Auscultacin cardiorrespirato-

228
CAPTULO 13
INFECCIONES POR MICOBACTERIAS

Diagnstico final

El diagnstico definitivo fue TUBERCULOSIS GENI-


TOURINARIA que se manifest por ORQUIEPIDIDI-
MITIS y PIELONEFRITIS TUBERCULOSA trece aos
ms tarde con resultado de nefrectoma. Adems present
PERITONITIS TUBERCULOSA en el postoperatorio
de la nefrectoma.

Discusin

La incidencia de tuberculosis extrapulmonar (TBE) en Es-


paa es de 3,71/100.000 hab segn datos del registro de
Figura 2. Demostracin de B. A. A. R. mediante tcnica de Ziehl- Neelsen sobre enfermedades de declaracin obligatoria correspondien-
pieza de nefrectoma.
tes a 2010. La tuberculosis genitourinaria (TBGU) es la
segunda causa ms frecuente de TBE despus de la TB
Diagnstico diferencial ganglionar y su frecuencia en pases industrializados oscila
entre 3,3 y 20% habiendo descendido en los ltimos aos,
Una de las causas ms frecuentes de nefrectoma es la pio- lo que ha conducido a que no se considere habitualmente
hidronefrosis de origen litisico, y efectivamente, nuestro esta etiologa en el diagnstico diferencial del paciente con
paciente presentaba datos clnicos y de imagen que orien- patologa renal.
taban a este diagnstico. Sin embargo la TC postquirr- La presentacin clnica es insidiosa, habitualmente como
gica solicitada para evaluar el sndrome febril mostraba la disuria y piuria con urocultivos negativos. La presencia de
presencia de adenopatas mesentricas y lquido peritoneal calcificaciones en parnquima renal y vas urinarias, las ca-
tabicado, que no encajaban con el diagnstico de sospe- vitaciones y la hidronefrosis secundaria a estenosis cicatri-
cha. cial del sistema pielo-ureteral condiciona que pueda con-
La clave nos la dio la anatoma patolgica que fue infor- fundirse con pionefrosis litisica, como ocurri en nuestro
mada como pielonefritis granulomatosa caseificante. Se paciente. La baja rentabilidad de la baciloscopia en orina
revis la pieza macroscpicamente y se realiz tcnica de nos obliga a tener una alta sospecha diagnstica y a incluir
Ziehl Neelsen en la que se informaba de la presencia de esta etiologa en el diagnstico diferencial de varios sndro-
litiasis, calcificaciones y abundantes BAAR (figura 2). Al no mes urinarios, especialmente en pacientes con radiografa
sospecharse sta etiologa no se haba remitido muestra a de trax normal. El escaso nmero de bacilos presentes en
microbiologa. la orina retrasa el crecimiento en medios de micobacterias,
A su traslado a planta de Enfermedades Infecciosas, se rea- a pesar del envo de varias muestras de la primera orina
liza nueva paracentesis evacundose 2700 cc de lquido as- de la maana ("concentrada" durante la noche). Ello con-
ctico serofibrinoso con caractersticas bioqumicas descri- diciona un retraso en el diagnstico y nos obliga a realizar
tas. Finalmente en el cultivo de Lwenstein del LP creci un seguimiento prolongado del enfermo antes de descartar
tras 25 das de incubacin Mycobacterium tuberculosis sensible una TBGU.
a todos los antituberculosos de primera lnea. Aunque se describen patrones radiolgicos caractersticos
de la tuberculosis renal ("rin mastic"), a veces resultan
Evolucin indistinguibles de las imgenes de riones atrficos oca-
sionadas por pionefrosis recurrente de origen litisico. Por
Se inicia tratamiento antituberculoso, con Isoniazida, Ri- ello es necesario elevar nuestra sospecha en pacientes con
fampicina y Pirazinamida con buena tolerancia. Ante la calcificaciones renales para prevenir autonefrectomas,
persistencia de la fiebre, se aade tratamiento con esteroi- especialmente en los pases con endemia de tuberculosis
des en dosis decrecientes quedando afebril en el momento como ocurre en Espaa.
del alta. La evolucin fue favorable con desaparicin de la La peritonitis tuberculosa que present el paciente pudo
ascitis. Cur tras cumplir 6 meses de tratamiento antitu- deberse a extensin por contigidad desde el espacio retro-
berculoso. peritoneal al intraperitoneal; o a una rotura de un "quis-
te" con vertido de su contenido en el peritoneo durante la

229
CAPTULO 13
INFECCIONES POR MICOBACTERIAS

nefrectoma laparoscpica, lo que justificara su aparicin 6. Huang LH, Wen MC, Hung SW, et al. Renal tuber-
aguda en el postoperatorio inmediato. culosis presenting as a complicated renal cyst. Urology
El diagnstico del TBGU debera confirmarse mediante 2012; 80: e69.
el cultivo del microorganismo o su identificacin mediante 7. Zumla A, Raviglione M, Hafner R. et al, Tuberculosis.
mtodos moleculares, sin embargo esto no siempre es posi- Review. N Engl J Med 2013; 368: 745-55
ble. El hallazgo de granulomas necrotizantes en el estudio
anatomopatolgico es altamente sugestivo de esta etiologa
y justifica el inicio del tratamiento antituberculoso, ms
an en esta forma clnica donde pueden coexistir varias
localizaciones simultneas que si no se tratan pueden pro-
gresar hacia la lesin irreversible del parnquima renal. En
nuestro caso el cultivo del lquido peritoneal permiti la
confirmacin microbiolgica.
A pesar de que su incidencia est disminuyendo, es nece-
sario considerar la TBGU en el diagnstico diferencial de
la piohidronefrosis de origen litisico por las graves conse-
cuencias del diagnstico tardo, motivado por la presenta-
cin clnica inespecfica, por la baja rentabilidad del diag-
nstico microbiolgico, por la posibilidad de la coinfeccin
del tracto urinario con otros uropatgenos y porque las
pruebas de imagen tambin pueden ser no concluyentes o
interpretadas como pionefrosis.
Y por ltimo, y no menos importante, es necesaria la bs-
queda en la historia clnica del paciente de otra forma cl-
nica de tuberculosis que haya pasado inadvertida o que
hay sido mal interpretada e iniciar tratamiento antituber-
culoso ante la presencia de granulomas necrotizantes en el
informe anatomopatolgico, aunque no tengamos confir-
macin microbiolgica.
As que el largo y tortuoso camino que sigui el bacilo en
nuestro paciente tuvo como consecuencia una prdida que
pudo haber sido evitada.

Agradecimiento: Dra R Terrones del Servicio de Microbiolo-


ga y Dra. F. Snchez del Servicio de Anatoma Patolgica.
Hospital Virgen de Rocio.

Bibliografa

1. Abbara A, Davidson R.N. Etiology and management


of genitourinary tuberculosis. Nat Rev Urol 2011; 8:
678-688.
2. Bhatt, C. Lodha, S. Paraspinal sinuses? Do remember
renal tuberculosis. BMJ Case Reports 2012.
3. Figuereido, A.A. Lucon A. M. Urogenital Tuberculo-
sis: Update and Review of 8961 Cases from the World
Literature. Rev Urol 2008; 3: 207-17.
4. Teo EY, Wee TC. Images in clinical medicine. Renal
tuberculosis. N Engl J Med 2011, 365: e26.
5. Lawn SD, Zumla AI. Tuberculosis. Lancet 2011; 378: 5772

230
Epigastralgia con vmitos postprandiales
en paciente con espondilitis anquilosante
y sospecha de enfermedad de crohn
Serrano Martnez, JL; Redondo Orts, M;
Roa Chamorro, R; Pasquau Liao, J.
Hospital Universitario Virgen de las Nieves. Granada.

Caso clnico Diagnstico diferencial

Varn de 53 aos sin alergias y con antecedentes persona- Con el diagnstico sindrmico de estmago retencionista
les de hipertensin arterial, hipotiroidismo y espondilitis por estenosis intestinal secundaria a masa transmural infil-
anquilosante en seguimiento por el Servicio de Reumato- trante de duodeno y colon contiguo se plante el diagns-
loga desde haca 2 aos. Estaba en tratamiento con anti- tico diferencial entre:
cuerpo monoclonal anti factor de necrosis tumoral (inflixi-
mab) a dosis de 5 mg/kg cada 8 semanas, encontrndose Neoplasias:
en su 50 dosis, adems de con enalapril 10 mg y levotiro-
xina 50 g al da. No haba realizado viajes fuera de Espa- Tumores periampulares: adenocarcinoma duodenal y
a, no haba tenido contacto con animales ni consumido ampuloma.
aguas o productos lcteos no higienizados. Es derivado a Cncer de colon.
consultas de Digestivo por cuadro de epigastralgia pos- Linfoma duodenal.
tprandial acompaada de vmitos alimenticios y prdida Tumor del estroma gastrointestinal (GIST).
de 6 kg de peso de 3 meses de evolucin. La exploracin
fsica result normal salvo por la presencia de un abdomen Enfermedad de Crohn.
timpanizado y doloroso a la palpacin del epigastrio, pero
sin rebote ni otros signos de irritacin peritoneal. En el he- Enfermedades infecciosas:
mograma presentaba anemia microctica de 10,3 g/dl de
hemoglobina y 76 fl de volumen corpuscular, con frmula Tuberculosis intestinal e infecciones por otras mico-
y bioqumica srica normales. Respecto a los marcadores bacterias.
tumorales se solicitaron el antgeno carcinoembrionario, Histoplasmosis intestinal.
CA-125 y CA-19.9 que estaban en rango de normalidad. Actinomicosis.
Se realiz una tomografa computarizada (TC) abdominal Ameboma.
en la que se observaba un estmago dilatado, as como el Abscesos pigenos.
bulbo y primera porcin duodenal de 47 mm (figura 1A),
secundario todo ello a una masa de 40 mm que estenosa- Evolucin
ba el segmento duodenal inmediatamente distal con infil-
tracin de la grasa mesentrica y colon ascendente. Estos Tanto en la histologa de la biopsia duodenal como ileal
hallazgos se confirmaron con la realizacin de una endos- no se visualizaron clulas sugerentes de malignidad con
copia digestiva alta (EDA) y se complet el estudio con una test inmunohistoqumicos negativos. Sin embargo en la
colonoscopia total que revel una ileitis. Se tomaron en muestra de leon se identificaron granulomas no caseifi-
ambas exploraciones biopsias que se remitieron a Anato- cantes compatibles con enfermedad inflamatoria intesti-
ma Patolgica y Microbiologa solicitndose cultivos bac- nal, siendo el paciente diagnosticado de enfermedad de
terianos convencionales, hibridacin de DNA por reaccin Crohn ileocecal y duodenal. Se comenz tratamiento con
en cadena de la polimerasa (PCR) de micobacterias, baci- prednisona 60 mg diarios en pauta descendente y mesa-
loscopias y cultivos de larga incubacin para hongos y en lazina 1 g cada 8 horas con lo que hubo ligera mejora
medio de Lowenstein-Jensen. Cabe destacar que previa a de los vmitos, pero con persistencia de la epigastralgia.
la instauracin del tratamiento con infliximab se realiz En cuanto a las pruebas microbiolgicas, las baciloscopias,
test de Mantoux con booster, radiologa simple de trax y cultivos bacterianos y fngicos y la PCR para micobacte-
serologas de virus de hepatitis B y C por parte de Reuma- rias fueron negativas, pero finalmente el cultivo de mico-
tologa, siendo todos ellos negativos. bacterias en medio de Lowenstein-Jensen result positivo
para Mycobacterium tuberculosis complex sensible a todos los tu-

231
CAPTULO 13
INFECCIONES POR MICOBACTERIAS

berculostticos de primera lnea. Se descart infeccin por incidencia de enfermedad inflamatoria en pacientes con la
virus de la inmunodeficiencia humana (VIH) y se instaur enfermedad reumatolgica del 22,96%. Esto se debe a un
una pauta de tratamiento consistente en 4 frmacos (iso- trasfondo gentico y ambiental similar entre ambas enti-
niazida 300mg, rifampicina 600 mg, pirazinamida 25 mg/ dades, destacando de stos ltimos los factores microbia-
kg y etambutol 15 mg/kg al da) durante 2 meses seguido nos como las infecciones subclnicas por Klebsiella spp.2. Por
de 4 meses ms con isoniazida y rifampicina, aunque se ello ante un paciente con enfermedad reumtica inmu-
tuvo que reintroducir de manera precoz el tratamiento es- nomediada la aparicin de sntomas digestivos sugerentes
teroideo con prednisona 40 mg al da ante la progresin de debe de hacernos sospechar una enfermedad inflamatoria,
la obstruccin intestinal, como se pudo ver en el trnsito como ocurri en nuestro caso en el que la clnica y hallaz-
baritado. La respuesta a corticoides fue parcial planten- gos iniciales apuntaban a una enfermedad de Crohn.
dose dilatacin endoscpica de la estenosis pero se desesti- Con el advenimiento de las terapias biolgicas el pro-
m por el alto riesgo de rotura de pared intestinal pues la nstico vital y funcional de muchas enfermedades auto-
masa tuberculosa presentaba una alta actividad metabli- inflamatorias ha mejorado en la ltima dcada, pero su
ca en la tomografa por emisin de positrones (PET) que uso no est exento de riesgos, siendo el fundamental las
sugera que se encontraba en fase inflamatoria. Por ello infecciones. stas se presentan en forma de reactivacin
se decidi tratamiento quirrgico destacando como hallaz- de procesos crnicos como la tuberculosis, hepatitis vricas
gos intraoperatorios una gran dilatacin de duodeno con y herpes zster o bien son de carcter oportunista como
inflamacin contigua de la cabeza pancretica y coldoco las producidas por Pneumocystis jiroveci3. La ms importante
junto a engrosamiento de la pared ileal y ciego. Se toma- y frecuente es la reactivacin de la infeccin tuberculosa
ron muestras de intestino, leon y lquido peritoneal y se latente (ITL), normalmente en su forma miliar. Entre los
procedi a realizar gastroyeyunostoma transmesoclica en distintos tratamientos los anticuerpos monoclonales, y de
cara gstrica posterior. Las muestras intraoperatorias con- stos infliximab, han demostrado tener un riesgo mayor
firmaron el diagnstico creciendo en todas ellas Mycobacte- de reactivacin que las protenas anlogas del receptor
rium tuberculosis y visualizndose en la Anatoma Patolgica de TNF, como etanercept4. El estudio de Wallis et al5 es-
tanto bacilos como granulomas, algunos de los cuales pre- tima una incidencia de 54 casos de tuberculosis por cada
sentaban necrosis caseosa. La evolucin del paciente fue 100.000 pacientes tratados con infliximab frente a 28 casos
muy favorable, desapareciendo la epigastralgia y vmitos por cada 100.000 pacientes tratados con etanercept. A esto
as como ganando peso, si bien tuvo que reintervenirse por hay que sumar el riesgo aumentado de tuberculosis que la
estenosis iatrgena de asa eferente. Se complet el esque- propia enfermedad autoinmune subyacente confiere, y que
ma teraputico de 6 meses con tuberculostticos resultan- va de 2 a 8,9 veces ms6. Por ello el despistaje de ITL se
do los cultivos y PCR de control obtenidas mediante en- hace imprescindible previo al comienzo con estas terapias,
doscopias negativas. Respecto a su proceso reumatolgico consistente en una historia clnica del paciente detallada,
el paciente se encuentra actualmente en tratamiento con la realizacin de un test de Mantoux y de una radiografa
etanercept, otro frmaco anti-TNF, sin recurrencia de la simple de trax.
enfermedad tuberculosa. La tuberculosis intestinal por lo general surge de la inges-
ta directa de productos contaminados o por diseminacin
Diagnstico final hematgena de una tuberculosis miliar, siendo la forma in-
testinal aislada infrecuente7. Existen diferentes variantes de
El diagnstico final fue de tuberculosis intestinal hiper- la enfermedad entre ellas la forma ulcerosa, inflamatoria/
plsica o pseudotumoral por Mycobacterium tuberculosis con hipertrfica y estenosante. La localizacin ms frecuente
afectacin duodenal e ileocecal secundaria a inmunosu- es la ileocecal en ms del 90% de los casos. La ausencia
presin humoral por infliximab. de sntomas o signos patognomnicos hace el diagnstico
clnico difcil, especialmente el diferenciarla de la enferme-
Discusin dad inflamatoria intestinal, mimetizndose ambas entida-
des patolgicas8. As pueden establecerse ciertas diferencias
Desde que en la dcada de los 50 y 60 se describiera la aso- entre ellas9. El diagnstico se basa en el uso combinado de
ciacin entre la espondilitis anquilosante y la enfermedad pruebas radiolgicas y endoscpicas, junto a la histologa
inflamatoria intestinal la aparicin de ambas enfermeda- y microbiologa de la lesin. Aunque la validacin de la
des en un mismo paciente ha sido constatada con amplia PCR en la tuberculosis intestinal es limitada se ha estima-
experiencia. Ya en 1963 McBride et al1 establecieron una do su sensibilidad entre 25-96% y su especificidad entre

232
CAPTULO 13
INFECCIONES POR MICOBACTERIAS

el 89-100%10. El tratamiento de primera lnea con 4 fr-


macos (isoniazida, rifampicina, pirazinamida y etambutol)
durante 6 meses presenta una tasa de xito del 90-95%.
La ciruga no suele ser necesaria y se reserva para compli-
caciones como la obstruccin, perforacin, fistulizacin o
masas que no responden al tratamiento mdico. En la ma-
yor parte de los casos se recomienda ensayar el tratamiento
mdico antes de llevar a cabo la intervencin quirrgica.

Bibliografa

1. Mcbride JA, King MJ, Baikie AG, Crean GP, Sircus


W. Ankylosing spondylitis and chronic inflammatory
diseases of the intestines. Br Med J. 1963; 2:483-6.
2. Rashid T, Wilson C, Ebringer A. The link between
ankylosing spondylitis, Crohn's disease, Klebsiella,
and starch consumption. Clin Dev Immunol. 2013;
2013:872632.
3. Murdaca G, Span F, Contatore M, Guastalla A, Pen-
za E, Magnani O, et al. Infection risk associated with
anti-TNF- agents: a review. Expert Opin Drug Saf.
2015; 14:571-82.
4. Fallahi-Sichani M, Flynn JL, Linderman JJ, Kirschner
DE. Differential risk of tuberculosis reactivation among
anti-TNF therapies is due to drug binding kinetics and
permeability. J Immunol. 2012; 188:3169-78.
5. Wallis RS, Broder M, Wong J, Beenhouwer D. Granu-
lomatous infections due to tumor necrosis factor bloc-
kade. Correction. Clin Infect Dis 2004; 39: 1254-5.
6. Carmona L, Hernndez-Garca C, Vadillo C, Pato
E, Balsa A, Gonzlez-Alvaro I, et al. Increased risk of
tuberculosis in patients with rheumatoid arthritis. J
Rheumatol. 2003; 30:1736-9.
7. Zumla A, Raviglione M, Hafner R, von Reyn CF. Tu-
berculosis. N Engl J Med. 2013; 368:745-55.
8. Koschny R, Junghanss T, Mischnik A, Karner M, Kre-
uter M, Roth W, et al. Development of miliary tuber-
culosis under infliximab in a patient with spondyloar-
thritis and suspected Crohn's disease. Z Gastroenterol.
2013; 51:1177-83.
9. Garca-Snchez V, Barreiro de Acosta M. Enfermedad
de Whipple, tuberculosis intestinal y otras enferme-
dades infecciosas crnicas. En: Ponce-Garca J, Cas-
tells-Garangou A, Gomolln-Garca F, editores. Tra-
tamiento de las enfermedades gastrointestinales 3 ed.
Barcelona: Elsevier Doyma S.A.; 2011; p. 238.
10. Chang K, Lu W, Wang J, Zhang K, Jia S, Li F, et al.
Rapid and effective diagnosis of tuberculosis and ri-
fampicin resistance with Xpert MTB/RIF assay: a
meta-analysis. J Infect. 2012; 64:580-8.

233
Mujer de 84 aos con bronquiectasias crnicas,
disnea, tos y hemoptisis
Garca Ocaa, P; Praena Segovia, J;
Gonzlez Estrada, A; Garca Morillo, JS.
Hospital Universitario Virgen del Roco. Sevilla.

Caso clnico

Mujer de 84 aos que ingresa en el servicio de urgencias


por cuadro de tos de una semana de evolucin, asociado
a disnea y expectoracin hemoptoica desde el da previo,
motivo por el que consulta. Entre sus antecedentes perso-
nales destacaba: hipertensin arterial, hipertiroidismo con
bocio endotorcico y fibrilacin auricular no anticoagula-
da. Tuberculosis en la infancia con bronquiectasias crni-
cas y lobectoma inferior izquierda en 2001. No dispone-
mos de estudios de funcin respiratoria. En Junio de 2008
presenta derrame pericrdico recidivante requiriendo pe-
ricardiocentesis por taponamiento cardaco, con diagnsti-
co de probable pericarditis tuberculosa en base a la clnica,
la bioqumica del lquido pericrdico con pleocitosis linfoci-
taria, ADA y LDH elevados y los hallazgos de la resonan- Figura 1. Radiografa PA de Trax. Cardiomegalia. Ocupacin de hemitrax
izquierdo por derrame pleural junto a componente atelectsico ipsilateral.
cia magntica. Los cultivos para bacterias fueron negativos
pero no se realiz estudio para micobacterias. Se complet
tratamiento correcto con cuatro frmacos rifampicina, iso- izquierdo por probable derrame pleural ms componente
niazida, pirazinamida y etambutol durante 2 meses, seguido atelectsico ipsilateral sin desplazamiento de la trquea y
de rifampicina e isoniazida durante seis meses, sin recidiva el mediastino [figura 1]. En la analtica de urgencias desta-
clnica, aunque con persistencia de derrame residual lateral. caba un deterioro de la funcin renal con creatinina de 2,9
En tratamiento en la actualidad con: levotiroxina 175 mcg, mg/dl y urea de 138 mg/dl. Los reactantes de fase aguda
sertralina 50 mg, furosemida 40 mg, omeprazol 20 mg, lor- estaban elevados con una protena C reactiva (PCR) de
metazepan 2 mg, metamizol 575 mg, AAS 100mg. 200 mg/l y 31.000 leucocitos (96% PMN). El estudio de
La paciente acude por tos de una semana de evolucin coagulacin mostr un INR de 1,3. El resto de la analtica
junto con expectoracin hemoptoica (de hasta 14 cuchara- presentaba valores dentro del rango de normalidad.
das/da) en las ltimas 24 horas, acompaada de disnea sin Se realiz una toracocentesis diagnstica con obtencin
fiebre o sensacin distrmica. No semiologa de insuficien- de lquido de aspecto achocolatado envindose muestras
cia cardiaca. No historia de infecciones respiratorias en el a microbiologa y bioqumica. La bioqumica del lquido
ltimo ao, no prdida de peso ni otra clnica asociada. pleural (LP) presentaba 120.000 clulas, 90% PMN, LDH
La exploracin a su llegada a urgencias revelaba: ausencia 60.000 mU/mL, glucosa de 2,03 g/l, protenas 31,4 g/l,
de fiebre, taquipnea a 30 respiraciones/minuto, con una y ADA 15 mU/mL, no se determin el pH. La tincin de
saturacin basal de oxgeno del 85%. Presentaba taqui- Gram y Ziehl-Neelsen del lquido pleural (LP) resultaron
cardia con fibrilacin auricular a 140 sstoles por minuto negativas. Se repitieron cultivos seriados de bacterias y mi-
y tensin arterial de 120/80 mmHg. A la auscultacin se cobacterias en esputo y se inicia antibioterapia emprica
apreciaba hipoventilacin generalizada en hemitrax iz- con piperacilina-tazobactam.
quierdo, roncus y subcrepitantes dispersos en ambos cam- Ante el diagnstico de sospecha de empiema, en base a las
pos pulmonares con tonos cardiacos arrtmicos sin soplos o caractersticas macroscpicas del LP, se decide colocacin
extratonos. No presentaba ingurgitacin yugular, hepato- de drenaje endotorcico que se intent en dos ocasiones sin
megalia ni edemas en miembros inferiores. xito, motivo por el cual se solicita tomografa axial com-
Entre las pruebas complementarias que se realizaron, la putarizada (TAC) de trax. En este no se objetiv derrame
radiografa de trax mostraba una opacidad en hemitrax pleural, y por lo tanto empiema, en su lugar se advierte

234
CAPTULO 13
INFECCIONES POR MICOBACTERIAS

cia de un patrn micronodulillar asociado, debemos con-


templar la posibilidad de micobacterias tuberculosas y no
tuberculosas. Adems de bacterias, es importante incluir
en el diagnstico diferencial infeccin por hongos como
Aspergillus, Coccidioides, Histoplasma, Blastomyces, Cryptococcus,
Mucor y Pneumocystis spp. Ante la ausencia de aislamiento
microbiolgico se decide realizar broncoscopia con lava-
do broncoalveolar (LBA) para toma de muestras, enviando
cultivo de bacterias y micobacterias del LBA.

Evolucin

Ante la sospecha de una sobreinfeccin de bronquiectasias


crnicas y formacin de absceso pulmonar secundario, se
Figura 2. TAC trax con contraste. Absceso en lbulo inferior izquierdo de pare- consider como causa ms probable la infeccin por bac-
des engrosadas. Atelectasia de la lngula.
terias anaerobias. Tras dos semanas de tratamiento emp-
rico con piperacilina-tazobactam con adecuada respuesta
un absceso en lbulo inferior izquierdo, de paredes engro- clnica se continu al alta con amoxicilina-clavulnico a
sadas, hipercaptantes y anfractuosas de 67x50x70mm de dosis de 875/125 mg cada 8 horas. A los 10 das del alta
contenido licuefacto con burbujas en su interior, corres- la paciente reingresa por nuevo cuadro de disnea, tos y
pondiendo al lquido drenado mediante toracocentesis. expectoracin hemoptoica. En la radiografa de trax no
Adems presentaba atelectasia completa de la lngula y l- se objetiva progresin radiolgica del proceso. Ingresa y,
bulo inferior izquierdo, observndose en las zonas de pul- durante la hospitalizacin en planta, se reciben los resulta-
mn aireado patrn en vidrio deslustrado acompaado de dos de los cultivos de micobacterias obtenido de muestras
nodulillos centrilobulillares en pex izquierdo [figura 2]. de esputo y LBA.
Tras la TAC de trax se establece el diagnstico de infec-
cin de bronquiectasias crnicas con patrn micronoduli- Diagnstico final
llar complicado con absceso en lbulo inferior izquierdo.
En el cultivo de micobacterias obtenido del esputo creci
Diagnstico diferencial M. intracellulare en dos de dos cultivos obtenidos en das
diferentes, con tiempo de crecimiento inferior a 14 das.
Nos encontramos ante una paciente con diagnstico de Asimismo, se recibi el cultivo del LBA en el que tambin
bronquiectasias crnicas secundarias a TBC pulmonar creci M. intracellulare, sin otros aislamientos en el cultivo
previa, que ingresa con signos de sobreinfeccin objeti- de aerobios y anaerobios. El diagnstico final fue infeccin
vndose en la TAC de trax un absceso pulmonar. Ante de bronquiectasias con infiltrado micronodulillar por M.
este cuadro, se debe realizar el diagnstico diferencial con intracellulare y absceso pulmonar probablemente secundario
los principales agentes etiolgicos para la eleccin de un a M. intracellulare. Se complet tratamiento con claritromi-
tratamiento antibitico emprico adecuado. La causa ms cina, rifampicina y etambutol durante un ao tras la nega-
frecuente de absceso pulmonar son las bacterias anaero- tivizacin de esputo, sin presentar recidiva clnica ni nuevo
bias: Peptostreptococcus, Prevotella, Bacteroides y Fusobacterium aislamiento en esputo.
spp. que proceden de la boca del paciente. Otros germe-
nes causantes de absceso pulmonar son Staphylococcus au- Discusin
reus, Pseudomona aeruginosa, enterobacterias (Klebsiella sp. ms
frecuente), Legionella y Haemophilus influenzae spp., estas fre- Las infecciones por micobacterias no tuberculosas (MNT)
cuentemente ocasionan sobreinfeccin de bronquiectasias, presentan una incidencia creciente debido, en parte, a la
siendo las dos primeras comunes en pacientes con sobrein- disponibilidad de tcnicas moleculares que permiten la
fecciones frecuentes y que han recibido ciclos de antibio- identificacin de hasta 142 especies diferentes. Las MNT
terapia previamente. Otros microorganismos implicados, se encuentran ampliamente distribuidas en el medio am-
aunque menos habituales, son las bacterias del orden Acti- biente encontrndose en el agua, sus sistemas de conduc-
nomycetales como Nocardia, Actinomyces spp. Y, dada la presen- cin y abastecimiento, el agua del mar, la tierra, animales,

235
CAPTULO 13
INFECCIONES POR MICOBACTERIAS

la leche y otros alimentos. No se ha demostrado transmi- lidos previo al tratamiento, recomendando la asociacin
sin de persona a persona. Predominantemente producen de estos (claritromicina o azitromicina) a rifampicina y
infecciones pulmonares con clnica indistinguible de la tu- etambutol. Esta combinacin debe mantenerse durante al
berculosis pulmonar u otras sobreinfecciones bacterianas. menos un ao tras negativizacin del esputo. La respuesta
El patrn radiolgico se caracteriza por la presencia de al tratamiento es variable y es frecuente el fracaso terapu-
cavitaciones fibrocicatriciales o aparicin de infiltrados mi- tico. La reseccin quirrgica se contempla en casos indivi-
cronodulares mltiples. Otras formas de presentacin son duales cuando existe resistencia al tratamiento y/o falta de
infecciones extrapulmonares como linfadenitis, infeccin respuesta al mismo.
diseminada, infeccin de piel, partes blandas e infecciones
de catter. La prevalencia de estas formas es baja y depen- Qu aporta el caso?
de de la especie de MNT y del husped.
La presencia de MNT en muestras respiratorias no siempre En pacientes con patologa pulmonar predisponente como
corresponde a una infeccin y es frecuente su aislamiento EPOC o bronquiectasias FQ y no FQ es importante in-
como colonizante, siendo comn esta forma de aparicin cluir dentro del diagnstico diferencial las infecciones por
en determinadas especies como M. gordonae y M. fortui- MNT. Es necesario un alto ndice de sospecha para solici-
tum. Las tcnicas moleculares identifican la especie pero tar cultivos especficos y su aislamiento requiere confirma-
no permiten diferenciar los casos de infeccin de los de cin en al menos dos cultivos de esputo, para diferenciarlo
colonizacin. En 2007 la Sociedad Americana de Trax de la colonizacin.
American Thoracic Society (ATS) public los criterios Presentamos el caso de un absceso pulmonar por M. intra-
diagnsticos de infeccin pulmonar por MNT consideran- cellulare: una forma de presentacin inusual en la afectacin
do infeccin los casos que cumplan los siguientes criterios: pulmonar por MNT, siendo el nico caso descrito en la lite-
a) Criterio microbiolgico: aislamiento de MNT en ms de ratura por este agente y el segundo caso descrito por MNT.
2 esputos o un nico aislamiento en LBA (segn la especie) Para su confirmacin sera necesario el aislamiento de
o mediante biopsia pulmonar con histologa compatible; MNT en el contenido del absceso obtenido por puncin,
b) Criterio clnico: presencia de sntomas respiratorio o pa- aunque la negatividad obtenida en el cultivo para otras
trn radiolgico compatible y c) exclusin de otras causas. bacterias apoyan esta etiologa.
Las principales especies causantes de infeccin pulmonar
son M. avium y M. intracellulare, pertenecientes al complejo Bibliografa
Mycobacterium avium complex (MAC). Suelen producir infec-
cin en pacientes con patologa pulmonar predisponente 1. Johnson M M, Odell J A. Nontuberculous myco-
como EPOC, fibrosis qustica (FQ), trasplantados de pul- bacterial pulmonary infections. J Thorac Dis 2014;
mn y bronquiectasias no FQ. En un anlisis retrospecti- 6(3):210-220
vo de prevalencia de infeccin por MNT en pacientes con 2. Martnez-Cern E et al. Infeccin por micobacterias
bronquiectasias no FQ realizado durante 10 aos se ob- no tuberculosas en pacientes con bronquiectasias no
serv el aislamiento de MNT hasta en el 10% de los casos causadas por fibrosis qustica. Rev Clin Esp 2012;
de sobreinfeccin. Otros huspedes predispuestos son pa- 212:127-30
cientes VIH con CD4+ por debajo de 50/uL y aquellos en 3. Asai K, Urabe N. Acute empyema with intractable
tratamiento con anti-TNF (artritis reumatoide y enferme- pneumothorax associated with ruptured lung abscess
dad inflamatoria intestinal, entre otros). A pesar de afectar caused by Mycobacterium avium. Gen Thorac Cardiovasc
ms frecuentemente a pacientes con factores predisponen- Surg. 2011; 59(6):443-6.
tes tambin causan infeccin en pacientes sin enfermedad 4. Camarena-Miana JJ. Micobacterias atpicas y su im-
pulmonar subyacente. plicacin en patologa infecciosa pulmonar. Enferm
Se recomienda en el estudio de infeccin pulmonar por Infecc Microbiol Clin 2011; 29(Supl 5):66-75
MNT la obtencin de al menos tres cultivos de micobacte- 5. Fabbian F et al. Pleural effusion in an immunocompe-
rias y una prueba de imagen. Si la radiografa de trax es tent woman caused by Mycobacterium fortuitum. J Med
normal debe realizarse una TAC de alta resolucin. Microbiol. 2011;60(Pt 9):1375-8
Previo al inicio del tratamiento debemos valorar el tipo de 6. Griffith DE et al. An official ATS/IDSA statement:
infeccin, a veces indolente, y las comorbilidades del pa- diagnosis, treatment, and prevention of nontubercu-
ciente, debido a la toxicidad y la duracin del mismo. En lous mycobacterial diseases. Am J Respir Crit Care
MAC se recomienda determinar la sensibilidad a macr- Med 2007; 175: 367-416.

236
Varon de 33 aos con uma meningitis post-quirurgica
por uma micobacteria no identificada.
Un reto diagnstico y teraputico
Monsalvo Hernando, M; Gmez Durn, M;
Martn Aspas, A.
Hospital Universitario Puerta del Mar, Cdiz.

Caso clnico

Paciente varn de 33 aos de edad, con antecedentes de


Prpura de Schnlein-Henoch a los 16 aos y pleja del
brazo izquierdo secundaria a lesin traumtica del plexo
braquial a los 19 aos, que es sometido a exresis de cordo-
ma en la unin crneo-cervical, manifestado previamente
por cervicalgia intensa y parestesias en miembro superior
izquierdo (MSI). Presenta una buena evolucin inmediata
tras la ciruga, y es dado de alta con pauta descendente de
dexametasona.
Un mes ms tarde, reingresa en el Servicio de Neurociru-
ga por presentar fiebre de una semana de evolucin de
hasta 38.5, dolor en zona quirrgica y coleccin fluctuan-
te en la zona de cicatriz, sin presentar signos menngeos ni
de infeccin a nivel de la herida quirrgica. En la explora-
cin fsica llama nicamente la atencin la limitacin para
la abduccin de MSI.
A su ingreso se suspende tratamiento previo con corticoi-
des, se realiza analtica general completa que result nor-
mal, con 7.500 leucocitos (60% neutrfilos), Protena C
Reactiva 5,6 mg/l y procalcitonina 0,2 ng/ml, as como
puncin lumbar, cuyo anlisis de lquido cefalorraqudeo
(LCR) mostr: 220 clulas (80% linfocitos), protenas 105
mg/dl, glucosa 21.8 mg/dl. Se inici tratamiento con me- Imagen 1. Coleccin isointensa a la seal del LCR rodeando zona de crane-
otoma. Informado como seroma postquirrgico comunicado con coleccin
ropenem (2 g. iv/8 h) y linezolid (600 mg. iv/12 h). Asi- lquida que se dirige a tejidos blandos de la regin latero cervical izquierda.
mismo se realiza Resonancia Magntica Nuclear (RMN)
craneal, que muestra coleccin isointensa alrededor de Diagnstico diferencial
zona de craneotoma y en charnela crneo cervical, que
asocia realce leptomenngeo y que se extiende hasta el ni- Nos encontramos ante un caso de meningitis relacionada
vel correspondiente al platillo vertebral inferior de C3, sin con la asistencia sanitaria, siendo el principal dato gua
describir si el origen ms probable es resto tumoral o infec- del que disponamos, la hipoglucorraquia del paciente. Las
cioso (Imagen 1). Se realiz puncin de la coleccin, que principales causas de hipoglucorraquia son:
result ser LCR. El cultivo del LCR result negativo.
Realiza 14 das de tratamiento antibitico, durante los 1. Infecciosa: bacterias (en principio, una glucosa en
cuales el paciente permanece afebril y asintomtico. Se LCR menor de 18mg/dl es altamente sugestiva de
realiza puncin lumbar de control al final del tratamiento meningitis bacteriana)[1] principalmente Streptococcus
con los siguientes resultados: 40 leucocitos, protenas 93 pneumoniae, Neisseria meningitidis o Listeria monocytogenes;
mg/dl y glucosa 1,3 mg/dl. Es en este momento cuando micobacterias, hongos o virus (parotiditis, enterovirus,
nos consultan el caso. coriomeningitis linfocitaria, Virus Herpes Simple, Vi-
rus Varicela Zoster)[2].
2. Carcinomatosis Menngea.
3. Hemorragia subaracnoidea.

237
CAPTULO 13
INFECCIONES POR MICOBACTERIAS

4. Sarcoidosis con afectacin del sistema nervioso cen-


tral: ocurre en un 5-25% de pacientes con afectacin
sistmica, y los hallazgos ms frecuentes en RMN son
el realce leptomenngeo por leptomeningitis granulo-
matosa con afectacin fundamental de las cisternas
basales, observndose dilatacin de las mismas por
obstruccin del acueducto de Silvio [3]. Resulta menos
frecuente la presencia de masas intraaxiales nicas o
mltiples secundarias a la diseminacin de la enferme-
dad granulomatosa [4].
5. Falso resultado por interferencia analtica

La citologa del LCR fue negativa para clulas neopl-


sicas. No existan datos de sangrado en LCR ni RMN,
ni tampoco lesiones focales a nivel del parnquima ni de
meninges, por lo que parecen improbables o descartables
la carcinomatosis menngea, la hemorragia subaracnoidea
y la sarcoidosis como causas de la hipoglucorraquia. Por Imagen 2. Baciloscopia de LCR que muestra bacilos cido-alcohol resistentes.
otra parte, hasta el momento no se han descrito casos de
hipoglucorraquia secundaria al tratamiento con linezolid el buen estado general del paciente durante todo el ingre-
o meropenem. Se analiz el LCR en distintos equipos, y se so: se encuentra fumando un cigarrillo en la calle cuando
realiz la tcnica de las diluciones, confirmando los datos vamos a visitarlo. Tras reevaluar clnicamente al pacien-
de hipoglucorraquia y descartando la interferencia anal- te, y ante la discordancia entre la clnica y los resultados
tica. Por todo ello, la principal etiologa a descartar fue la analticos, decidimos realizar nueva puncin lumbar, que
infecciosa. confirm la hipoglucorraquia, la elevacin de protenas y
La meningitis postquirrgica es una complicacin infre- lactato (tabla 1). La tincin de Gram no observ microor-
cuente (0,3-1,5%), aunque de indudable trascendencia ganismos, pero la baciloscopia detect bacilos cido-alco-
clnica. El diagnstico diferencial nos hizo descartar la hol resistentes (imagen 2), siendo negativa la deteccin de
meningitis asptica postquirrgica, dadas la hipogluco- Mycobacterium tuberculosis mediante reaccin en cadena de
rraquia e hiperproteinorraquia. Epidemiolgicamente, polimerasa (PCR)(GeneXpert MTB/RIF).
los principales agentes etiolgicos de meningitis bacteria- Decidimos iniciar tratamiento con sospecha de infeccin
na postquirrgica o tras traumatismo cerebral penetrante por micobacteria no tuberculosa, con amikacina 1g/da
son estafilococos y bacilos gram-negativos aerobios facul- i.m., levofloxacino 500 mg/da v.o. y claritromicina 500
tativos (incluyendo Pseudomonas aeruginosa). En pacientes mg/12 h. v.o.
con colocacin de un dispositivo permanente (por ejem- El paciente permanece afebril y con mejora de los snto-
plo una derivacin ventrculo-peritoneal), las infecciones mas iniciales. El tratamiento con amikacina i.m. se suspen-
son causadas principalmente por grmenes presentes en di al mes por intolerancia, levofloxacino se suspendi tras
la piel, como Propionibacterium acnes. Nuestro paciente es- 46 das de tratamiento por clnica de tendinitis rotuliana
tara incluido en el primer grupo de riesgo. Sin embargo, bilateral (no confirmada por ecografa, pero resuelta tras
estos grmenes suelen producir la infeccin en el postope- la suspensin del frmaco) y la claritromicina se mantuvo
ratorio inmediato y se acompaan de gravedad clnica del hasta completar 56 das, suspendindose tras la normali-
enfermo y marcada elevacin de reactantes de fase aguda, zacin del LCR, que sigui sin alteraciones bioqumicas y
a diferencia de nuestro paciente, por lo que pensamos que con estudios microbiolgicos negativos en un control reali-
tena que tratarse de otro germen menos habitual [5]. zado al mes y medio de finalizar el tratamiento antibitico
(Tabla 1).
Evolucin
Diagnstico final
Desde el ingreso, el paciente ha permanecido afebril, pre-
sentando nicamente cervicalgia leve, y sin elevacin de Meningitis postquirrgica por micobacteria atpica, pro-
reactantes de fase aguda en ningn momento. Sorprenda bablemente de crecimiento rpido.

238
CAPTULO 13
INFECCIONES POR MICOBACTERIAS

LCR Da 0 Da Da +16 Da Da Da Da M. TUBERCULOSIS COMPLEX MICOBACTERIAS


+14 +29 +42 +91 +119 NO-TUBERCULOSAS
M. tuberculosis DE CRECIMIENTO LENTO
Clulas 220 40 30 40 50 20 0
M. bovis
(PMN/linfocitos) (20/80) (5/95) (5/95) (5/95) (5/95)
M. africanum Fotocromgenas:
Protenas 105 93 87 85 84 36.4 47 M. microti M. kanasasii
(mg/dl) M. canetti M. marinum
M. LEPRAE
Glucosa LCR/ 21.8/ 1.3/ 4/89 37.3/ 48.9/ 60.4/ 58.4/ Escotocromgenas:
plasma (mg/dl) ND ND 92 126 87 81 MICOBACTERIAS M. gordonae
NO-TUBERCULOSAS M. scrofulaceum
Lactato (mg/dl) ND ND 58 33.4 22.3 15.8 13.8
DE CRECIMIENTO RPIDO
Cultivo Neg Neg Neg Neg Neg Neg Neg No cromgenas:
M. fortuitum complex M. avium complex
BK/LW ND ND ++/neg Neg/ Neg/ Neg/ Neg/ M. fortuitum M. avium
neg neg neg neg M. peregrinum M. intracellulare
Otros ADA 6.0 M. porcinum M. terrae complex
PCR TBC M. chelonane M. ulcerans
neg M. abscessus M. xenopi
Protena M. abscessus M. simiae
C M. bolleti (antes M. M. sulgai
Reactiva masiliense) M. asiaticum
en M. smegmatis M. haemophilum
plasma M. mucogenicum
0.64 mg/
dl
Tabla 2. Especies de micobacterias que causan patologa en humanos.

previo en nuestro paciente pudo ser responsable de la ne-


gatividad de los cultivos para micobacterias.
Las tcnicas moleculares permiten reducir el tiempo de
diagnstico y detectar resistencias farmacolgicas, entre
otras ventajas, aunque hay que remarcar que an no tie-
nen unas indicaciones claras para su implementacin en
muestras distintas al esputo.
PMN: leucocitos polimorfonucleares; Neg: negativo; ND: no disponible; ADA: En el caso de nuestro paciente, la PCR GeneXpert fue em-
adenosin deaminasa; PCR TBC: deteccin de genoma de Mycobacterium tuber-
culosis mediante reaccin en cadena de polimerasa.
pleada sobre una muestra de LCR con baciloscopia previa
positiva (S: 81% y E: 99%) [8]. El resultado fue negativo. A
Tabla 1. Evolucin analtica y microbiolgica del LCR durante el tratamiento. pesar de no tener un diagnstico de certeza, este resultado,
junto con las caractersticas clnicas y epidemiolgicas ya
Discusin explicadas, nos hizo descartar razonablemente la infeccin
por M. tuberculosis y asumimos como lo ms probable que
Aunque las alteraciones del LCR son compatibles con la nos encontrramos ante una micobacteria no tuberculosa
etiologa tuberculosa, la ausencia total de sntomas, los ha- (Tabla 2).
llazgos en la RMN (habitualmente se aprecia hidrocefalia Las infecciones causadas por micobacterias no tubercu-
en el 75% de los pacientes, realce menngeo basilar en el losas son poco frecuentes, aunque estn tomando mayor
38%, infartos cerebrales en regin supratentorial en 15- relevancia con el paso del tiempo, principalmente en los
30% y tuberculomas en 5-10% [6], de los cuales nuestro pases con mayor desarrollo econmico y con la aparicin
paciente slo presenta realce menngeo en la zona que ro- de la infeccin por el VIH, llegando a suponer en algu-
dea la coleccin), la negatividad de la PCR y el anteceden- nos laboratorios de microbiologa entre el 15 y el 30% de
te quirrgico, hacen poco probable a M. tuberculosis como los aislamientos micobacterianos. Se han descrito casos de
la causa de la meningitis. meningitis por algunos de estos microorganismos en pa-
El diagnstico de tuberculosis puede realizarse mediante cientes inmunosuprimidos, con antecedentes de neuroci-
tcnicas convencionales (baciloscopia, con sensibilidad (S) ruga (como es el caso de nuestro paciente, que adems
45-80%, especificidad (E) 50-80% y valor predictivo posi- se considerara inmunosuprimido al estar recibiendo trata-
tivo (VPP) 50-80%; cultivos: S 71%; E 82% [7]) y tcnicas miento con corticoesteroides), traumatismos craneoence-
moleculares como la determinacin de PCR Gene-Xpert flicos, implantes intracraneales, otomastoiditis o infeccio-
MTB/RIF. Generalmente ambas determinaciones se nes sistmicas [9].
complementan dadas las limitaciones que presentan. Hay Un aspecto importante es diferenciar la colonizacin y
que tener en cuenta adems que el tratamiento antibitico contaminacin de la verdadera infeccin. Puesto que al-

239
CAPTULO 13
INFECCIONES POR MICOBACTERIAS

gunas micobacterias atpicas se encuentran ampliamente principalmente por las caractersticas de cada paciente.
distribuidas en agua y suelo, es importante descartar la po- En nuestro paciente, con los resultados microbiolgicos
sibilidad de la colonizacin y contaminacin. La presencia descritos, decidimos realizar tratamiento antibitico para
de estos grmenes en lquido estril no puede considerarse micobacteria no tuberculosa en base a la clnica y contex-
colonizacin, pero s puede tratarse de una contaminacin to epidemiolgico. Aunque no se ha podido confirmar el
de la muestra durante su manipulacin [10]. En nuestro agente etiolgico del cuadro, la resolucin de la meningitis
caso, las alteraciones bioqumicas del LCR apoyan el papel con el tratamiento prescrito apoya la etiologa sospechada.
patgeno de la micobacteria. La ausencia de lesiones abscesificadas y de tratamiento con
Micobacterium leprae no produce clnica compatible con el corticoides justificara que aunque la duracin del trata-
cuadro de nuestro paciente, ya que afecta principalmente miento fue breve, haya sido suficiente.
a la piel y el sistema nervioso perifrico. Las micobacterias
no tuberculosas de crecimiento lento (ms de 7 das tras Puntos clave
cultivo) producen principalmente patologa pulmonar, en
tracto gastrointestinal o ganglios linfticos sobre pacientes Las decisiones tomadas en cuanto al manejo de este pa-
predispuestos (patologa pulmonar de base o inmunodepri- ciente, tipo de antibioterapia y duracin de la misma, susci-
midos), ya que son escasamente patgenas [11]. El grupo taran controversia en cualquier grupo de infectlogos ante
que principalmente produce un cuadro clnico compatible quienes se exponga, pero es precisamente este el atractivo
son las micobacterias no tuberculosas de rpido crecimien- del caso. En nuestra prctica clnica no siempre es posible
to (menos de 7 das en cultivo) [10]. tener todos los datos necesarios para hacer un completo
Estas bacterias son ambientales y resistentes a frmacos diagnstico que nos permita ajustarnos a las guas de prc-
antituberculosos de primera lnea. Las tres especies ms tica clnica. La toma de decisiones en pacientes complejos
frecuentes son M. abscessus, M. fortuitum y M. chelonae. M como el que presentamos es siempre un reto diagnstico
abscessus es la ms abundante de las tres en la naturaleza, y teraputico que afrontar, aunando conocimientos epide-
pudiendo contaminar depsitos de agua o soluciones de miolgicos, clnicos, microbiolgicos y de radiologa.
lavado en los hospitales, y tambin la ms patognica, cur-
sando sobre todo con afectacin pulmonar (80% de los ca-
sos) y de herida quirrgica. M. chelonae suele dar afectacin
cutnea y en infecciones postraumticas, siendo rara la
afectacin pulmonar [12, 13]. M fortuitum causa fundamen-
talmente infecciones de heridas quirrgicas y traumticas
y relacionadas con catteres intravasculares.
Al pensar en una infeccin causada por micobacterias no
tuberculosas de crecimiento rpido, tuvimos que tener en
cuenta que su tratamiento es distinto que el del resto de
micobacterias. En particular, es muy distinto al de la tu-
berculosis. Esto se debe a la diferente sensibilidad in vitro
de estos organismos, que resultan ser resistentes a los tu-
berculostticos convencionales, mientras que pueden ser
sensibles a otros antibiticos de uso ms amplio. La du-
racin recomendada habitualmente es prolongada, espe-
cialmente en pacientes inmunodeprimidos (siempre que se
mantenga dicha situacin), y dependiendo de la localiza-
cin de la infeccin [14]. La experiencia en el tratamiento
de la meningitis por estos grmenes es escasa y se basa en
estudios observacionales y de revisin. Las tres principales
series que describen esta patologa muestran la gran va-
riabilidad en la presentacin clnica, los factores predispo-
nentes y la evolucin con el tratamiento de estos pacientes
[12, 13, 15]. La duracin de dicho tratamiento fue muy
variable (de algunos meses a varios aos) y condicionada

240
CAPTULO 13
INFECCIONES POR MICOBACTERIAS

Bibliografa 14. Griffith D, Aksamit T, Brown-Elliott B, et al. An Offi-


cial ATS/IDSA Statement: Diagnosis, Treatment, and
1. Spanos A, Harrell FE Jr, Durack DT. Differential diag- Prevention of Nontuberculous Mycobacterial Disea-
nosis of acute meningitis. An analysis of the predictive ses. Am J Respir Crit Care Med 2007;175:367-416
value of initial observations. JAMA. 1989; 262:2700-7. 15. Talati NJ, Rouphael N, Kuppalli K, Franco-Paredes C.
2. Chaudhuri A, Martin PM, Kennedy PGE, Andrew Spectrum of CNS disease caused by rapidly growing
Seaton R, Portegies P, Bojar M, et al. EFNS guideline mycobacteria. Lancet Infect Dis 2008; 8: 390-8.
on the management of communityacquired bacterial
meningitis: report of an EFNS Task Force on acute
bacterial meningitis in older children and adults. Eur J
Neurol 2008;15:649-59.
3. Smith JK, Matheus MG, Castillo M. Imaging mani-
festations of neurosarcoidosis. AJR A J Roentgenol
2004;182:289-95.
4. Hollander MD, Friedman DP. Neuroradiology
case of the day. Neurosarcoidosis. Radiographics
1998;18:1608-11.
5. van de Beek D1, Drake JM, Tunkel AR. Nosocomial
bacterial meningitis. N Engl J Med 2010;362:146-54
6. Bernaerts A, Vanhoenacker FM, Parizel PM, Van
Goethem JWM, van Altena R, Laridon A, et al. Tu-
berculosis of the central nervous system: overview of
neuroradiological findings. Eur Radiol 2003;13:1876-
90.
7. Thwaites GE, Tran Thi Hong Chau, Farrar JJ. Im-
proving the Bacteriological Diagnosis of Tuberculous
Meningitis. J Clin Microbiol 2004;42:378-9.
8. Lawn SD, Mwaba P, Bates M, Piatek A, Alexander H,
Marais BJ, et al. Advances in tuberculosis diagnostics:
the Xpert MTB/RIF assay and future prospects for a
point-of-care test. Lancet Infect Dis 2013;13:349-61.
9. Flor A, Capdevila JA, Martin N, Gavald J, Pahissa A.
Nontuberculous mycobacterial meningitis: report of
two cases and review. Clin Infect Dis 1996; 23: 1226-
73.
10. Garca-Martos P, Garca-Agudo L. Infecciones por
micobacterias de crecimiento rpido. Enferm Infecc
Microbiol Clin 2012; 30:192-200
11. Philley JV, Griffith D. Treatment of Slowly Growing
Mycobacteria. Clin Chest Med 2015; 36: 79-90.
12. Maniu CV, Hellinger WC, Chu SY, Palmer R, Alva-
rez-Elcoro S. Faliure of treatment for Chronic Myco-
bacterium abscessus meningitis despite adequate clari-
thromycin levels in cerebrospinal fluid. Clin Infect Dis
2001; 33: 745-8.
13. Lee M-R, Cheng A, Lee Y-C, Yang C-Y, Lai C-C,
Huang Y-T, et al. CNS infections caused by Myco-
bacterium abscessus complex: clinical features and
antimicrobial susceptibilities of isolates. J Antimicrob
Chemother. 2012;67: 222-5.

241
Varn de 43 aos procedente de Senegal
con cuadro constitucional y fiebre
Ramos Sesma, V; Velasco Fuentes, S;
Anguita Santos, F.
Hospital Universitario Clnico San Cecilio. Granada.

Caso clnico

Se trata de un varn de 43 aos, procedente de Senegal;


que acude al servicio de urgencias externas del hospital,
por fiebre termometrada de hasta 39C y dolor abdominal
de una semana de evolucin.
Se recogen los antecedentes con dificultad por la barrera
lingstica. Sin antecedentes personales de inters, lleva 13
aos en Europa, entre Espaa e Italia. No haba regresa-
do recientemente a su pas de origen. Casado, sin hbitos
sexuales de riesgo. Trabaja en el campo, actualmente pa-
rado. Bebedor ocasional de una unidad de bebida estndar
(UBE). Fumador activo de 1-2 cigarrillos de cannabis.
Acude por presentar fiebre de caractersticas bacterimicas
de hasta 39 C en las ltimas 48 horas, asociado a dolor Imagen 1. Radiografa AP de trax al ingreso en el servivio de urgencias. Muestra
velamiento del hemitrax derecho y caverna en lbulo superior izquierdo.
abdominal, localizado en epigastrio. Desde hace dos se-
manas, presenta tos escasamente productiva, con disnea
de moderados mnimos esfuerzos. Adems refiere cuadro de las asas intestinales, y en el electrocadiograma, una ta-
constitucional de 2 meses de evolucin, con astenia y pr- quicardia sinusal a 100 lpm con eje a 60.
dida ponderal cuantificada de 10 kg. No otra sintomatolo- El paciente se ingresa con shock sptico de origen respira-
ga por rganos y aparatos. torio, pancitopenia y hepatomegalia.
En la exploracin fsica destaca, sensacin de enfermedad, Ante las imgenes de la radiografa y el antecedente epide-
con caquexia importante y sequedad cutnea. Se encuen- miolgico, se solicita un BAAR en esputo, que resulta ser
tra hipotenso, pero con diuresis conservada, satura 97- positivo para micobacterias.
98 % a FiO2 0.21. Taquipnea a 24 rpm, sin tiraje. A la Las serologas solicitadas para VIH, virus hepatotropos,
auscultacin presenta hipofonesis generalizada, con tonos VEB, parvovirus, el resto de los estudios microbiolgicos y
rtmicos a 100 lpm. La exploracin abdominal destaca, de autoinmunidad son negativos.
hepatomegalia no dolorosa de 5 cm. Buen relleno capilar, Se solicita una bioqumica ms completa, donde destaca
pulsos presentes y simtricos. ferritina de 15150 ng/mL, hierro 86 ug/dL, transferrina
En las pruebas bioqumicas realizadas de urgencia tiene 86 mg/mL, ndice de saturacin de transferrina 79.7%,
una funcin renal conservada, alteraciones hidroelectro- vitamina B12 > 2000 pg/dL, triglicridos 133 mg/dL. Se
lticas: hiponatremia de 116 mg/dL, hipopotasemia 2.5 detecta una alteracin en la coagulacin que no presenta-
mg/dL, hipocalcemia (calcio corregido con protenas tota- ba al ingreso; con actividad de protrombina 68.45%, INR
les de 4.3 mg/dl) de 9.4 mg/dL, LDH 1330 U/L, BT 1.43 1.33 y fibringeno derivado 145.4 mg/dL.
mg/dl, GPT 27 mg/dl. Elevacin de los reactantes de fase
aguda (RFA) PCR 160 mg/l. Pancitopenia con 3040 leu- Diagnstico diferencial
cocitos/mm3 con 1830 PMN, hemoglobina 8.1 mg/dL,
57000 plaquetas/mm3. Coagulacin preservada. En resumen, se trata de un paciente con tuberculosis acti-
En la radiografa solicitada (imagen 1) se evidencia una va, con pancitopenia, hepatomegalia e hiperferritinemia
condensacin el LSD con broncograma areo, cavitacin de 15150 mg/dL.
en LI, donde se aprecia una imagen nodular y patrn al- La pancitopenia obliga a descartar una enfermedad en
veolo intersticial en lngula. la mdula sea o bien un proceso perifrico que explique
En la radiografa de abdomen se observa desplazamiento el consumo o la destruccin celular. Dentro de las causas

242
CAPTULO 13
INFECCIONES POR MICOBACTERIAS

con afectacin perifrica se encuentra el hiperesplenismo,


hepatopatas, determinados virus, procesos autoinmunes
o el consumo que se produce durante la sepsis. Entre los
mecanismos centrales, se incluiran procesos virales, leuce-
mias o sndromes mielodisplsicos o mieloproliferativos. Se
debe descartar consumo de frmacos o alcohol, historia de
hepatopata, mala alimentacin, as como enfermedades
hematolgicas o infecciones por VEB, CMV, parvovirus
B19 o VIH.
La hepatomegalia, se puede producir como consecuencia
de diversas patologas, como hepatitis secundaria a un pro-
ceso infeccioso, isqumico, txicos como frmacos o alco-
hol.
Puede producirse por acmulo de sustancias, como en la
hemocromatosis o por infiltracin de tumores, tanto benig-
nos como malignos. Puede ser consecuencia de alteracio-
nes en el retorno venoso, como en la trombosis de las venas
suprahepticas o en la insuficiencia cardiaca congestiva.
Alteraciones en la va biliar, como ocurre en la cirrosis bi-
liar primaria, pueden provocar tambin, un aumento de
tamao del hgado.
La hiperferritinemia se puede dar en una gran variedad
de procesos como la inflamacin, sepsis, patologa hep-
tica, hemocromatosis o transfusiones de sangre frecuen-
tes. Aunque los niveles de ferritina bajos se correlacionan
estrechamente con dficit frrico (1), no parece ocurrir lo
mismo, cuando los valores estn elevados. Esto guarda re-
lacin con el papel de la ferritina como mediador pro-in-
flamatorio y reactante de fase aguda (RFA) (1, 2).
Niveles marcadamente elevados son raros en la prctica
clnica. Son muy sugerentes de sndrome hemofagoctico
(SHF), enfermedad de Still o sndrome antifosfolpido ma-
ligno entre otros. En estos sndromes, no es slo un media-
dor de inflamacin, sino que participa en la fisiopatogenia
de la enfermedad como citoquina inflamatoria (2).
En resumen, y atendiendo a las pruebas complementarias,
se descartan las causas autoinmunes y los procesos virales,
como etiologa posible. Se debera ampliar el estudio con
test de Coombs, reticulocitos o aspirado de mdula sea
para descartar una causa central de la pancitopenia, aun-
que podra estar explicada en parte por el cuadro sptico. Imgenes 2 y 3. TAC de trax con contraste realizado en UCI. Muestra mltiples
cavernas, afectacin intersticial reticular y reas de panalizacin, con neumo-
Se necesitan pruebas de imagen para valorar el estado de trax y tubo de trax bien colocado.
la hepatomegalia y descartar/confirmar una esplenome-
galia o un tumor heptico que tambin justifica la clnica Evolucin
de desnutricin y caquexia del paciente. Con todo lo an-
terior, la hiperferritinemia podra ser secundaria a una he- Se inici tratamiento con rifampicina, isoniacina, etam-
patopata como la hemocromatosis o estar en el contexto butol y pirazinamida, cuyas dosis se ajustaron al peso del
del cuadro sptico como reactante de fase aguda, aunque paciente. A pesar de ello, el paciente desarroll un fallo
los niveles estn excesivamente elevados, por lo que no se respiratorio, hemodinmico y hematolgico, y requiri in-
debera olvidar el sndrome hiperferritinmico. greso en Unidad de Cuidados Intensivos (UCI).

243
CAPTULO 13
INFECCIONES POR MICOBACTERIAS

Se ampli el estudio con pruebas hematolgicas como fue- Diagnstico molecular relacionado con sndrome hemofagoctico

ron test de Coombs y haptoglobina que resultaron negati- Mutacin PRF


Mutacin SAP
vos, descartndose proceso hematolgico subyacente. No Mutacin MUNC13-4
se realiz aspirado de mdula sea. Cinco de los ocho criterios siguientes
La evolucin analtica fue la siguiente: desarroll una coa- Fiebre
gulopata con descenso de AP al 52%, APTT 25 s, DD 31, Esplenomegalia
Citopenias (que afecten a dos de las tres lneas)
empeoraron de las citopenias, requiriendo transfusiones Hipertrigliceridemia (> 265mg/dL) y/o hipofibrinogenemia (<150 mg/dL)
Hemofagocitsis demostrada en MO, bazo, hgado o ganglio linfticos
de plaquetas y hemates, aunque posteriormente se logr Hiperferritinemia > 500
un control de los niveles sanguneos. Las cifras de ferriti- Aumento de CD 25

na fueron en descenso, llegando a 2578 mg/dL. Present Tabla 1. Criterios de diagnstico de SHF (3,8)
negatividad en todos los cultivos en medios comunes de
sangre y orina extrados. Las baciloscopias seriadas que se 10000 mg/dL, altamente especficas. Esta correlacin en
realizaron se mantuvieron persistentemente positivas, con adultos no parece tan clara, sino ms bien basada en opi-
un antibiograma para M. tuberculosis sensible a rifampicina. niones de expertos, no tanto en datos publicados. Algunos
No se aisl el bacilo en ninguna otra muestra. La PCR se autores indican que la hiperferritinemia es ms frecuentes
elev a su ingreso en UCI descendiendo posteriormente. en pacientes con fallo renal, dao heptico, infecciones o
Fue necesario intubar al paciente por agotamiento mus- enfermedades hematolgicas malignas; no siendo tan es-
cular, presentando mucha desadaptacin a la ventilacin pecficos del SHF(6). Esta discrepancia parece estar en re-
invasiva. Desarroll un neumotrax parcelar, por lo que se lacin con la cohorte de pacientes estudiada en cada grupo
le coloc un tubo de trax dificultando an ms la ventila- de trabajo(7).
cin. Se solicit un TAC de trax para evaluar la extensin Atendiendo a las causas que pueden producir elevacin de
de la lesin (imagen 2 y 3): Hidroneumotrax moderado ferritina, parece poco probable que el paciente tenga una
derecho anterior con tubo de drenaje bien ubicado. Posi- enfermedad del Still o un sndrome antifosfolpido malig-
ble fstula pleurobronquial en LSD. Consolidacin en LSD no; porque el cuadro clnico no parece compatible con
multicavitada con dilatacin bronquial. Gran cavidad de dichas enfermedades. Por otro lado, dentro de todas las
85 mm en LSI, sin niveles hidroareos ni masas. Parn- patologas que podran explicar la hepatomegalia y la hi-
quima de lbulo medio, lngula y lbulos inferiores con perferritinemia, quedaran descartadas por la normalidad
afectacin intersticial reticular y reas de panalizacin. de las pruebas complementarias; excepto por el SHF, que
Infiltrados alveolares en LSI y parcheado en LII. No ade- explicara adems, el resto de los sntomas del paciente,
nopatas mediastnicas de tamao significativo. En TC ab- como la pancitopenia, que en este caso pude ser mixta,
dominoplvico solo destaca hepatomegalia homognea consecuencia de dicha enfermedad y del consumo secun-
A pesar de las medidas de soporte tomadas, y del trata- dario al proceso sptico.
miento tuberculosttico, no presenta mejora clnica. Fi- El SHF es un desorden histiocitario caracterizado por la
nalmente el paciente fallece. activacin y proliferacin de los macrfagos que producen
una fagocitosis descontrolada de las clulas sanguneas as
Diagnstico final como de los precursores hematopoyticos.
Para su diagnstico, Histiocyte Society desarroll unas guas,
Sepsis respiratoria por tuberculosis pulmonar y SHF se- basadas en criterios clnicos y de laboratorio. (Tabla 1)
cundario. (3,8). Se necesitan la confirmacin gentica de alguna de
las mutaciones asociadas al sndrome, o la presencia de
Discusin cinco de los ocho criterios menores.
Creemos que nuestro paciente presenta un SHF probable,
Los niveles de ferritina elevados se pueden dar en multi- ya que cumple cinco de los ocho criterios necesarios, como
tud de procesos, como ocurre en procesos spticos como son: fiebre, hepatomegalia, citopenias (anemia y plaqueto-
mediador proinflamatorio. Sin embargo, niveles muy ele- penia), hipofibrinogenemia e hiperferritinemia.
vados, sin ser especficos, pueden orientar a determinadas Aunque la esplenomegalia es el criterio diagnstico, en la
patologas, como sndrome hemofagoctico, Enfermedad literatura revisada se encuentra hepatomegalia aislada en-
Still del adulto o sndrome antifosfolpido maligno(2). tre el 61-88 %(2,4). No se pudo confirmar el diagnstico
En poblacin peditrica, cifras de ferritina por encima con el aspirado de mdula sea, ya que no se lleg a reali-
3000 mg/dL, son muy sugestivas de SHF; y valores > zar al paciente.

244
CAPTULO 13
INFECCIONES POR MICOBACTERIAS

Se suele clasificar en dos grupos. El primero agrupara No se plante en ningn momento, iniciar tratamiento es-
todos los casos de origen gentico o familiar, como con- pecfico frente al SHF. Al ser una entidad poco frecuente,
secuencia de mutaciones de los genes perforinas y otros no existen estudios controlados y aleatorizados, que indi-
implicados en la exocitosis de grnulos citotxicos, son quen cual es el mejor esquema teraputico. El tratamiento
considerados primarios. Ms comn entre los nios de difiere de si se trata de un nio o un adulto, o si existe algn
edades infantiles. Y un segundo grupo donde la enferme- desencadenante. Las terapias estn destinadas a contro-
dad se desarrolla como consecuencia de diversos procesos, lar la cascada inflamatoria que se desencadena a travs de
como infecciones, enfermedades autoinmunes y afecta corticoides, ciclosporina, eptopsido, inmunoglobulinas
con ms frecuencia a adultos. Sin embargo, esta clasifica- intravenosas o en los casos extremos trasplante de mdula
cin, no se ajusta a la realidad con exactitud. El SHF pri- sea. En los SHF secundarios se debe iniciar terapia espe-
mario puede aparecer a cualquier edad y slo se encuentra cfica frente al proceso desencadenante, junto con frmacos
mutaciones genticas en el 40% de los casos. Por otro lado, imunosupresores si la evolucin es trpida. A los de causa
en ambos grupos, el cuadro puede estar desencadenado gentica, los muy agresivos o refractarios, se les puede rea-
por una infeccin (3,8). lizar trasplante de progenitores hematopoyticos (3,4,5,8).
Si nos centramos en los sndromes hemofagociticos reac- En este caso nos planteamos que ante la mala evolucin
tivos, las infecciones suelen ser un desencadenante muy clnica se podra haber optado por la administracin con-
comn. Se ha asociado a gran variedad de procesos vi- junta de corticoides o algn otro inmunomodulador, junto
rales, bacterianos o fngicos. El VEB es la causa que se con el tratamiento antituberculoso, aunque no existen cla-
ha relacionado con ms frecuencia (3,5). Tambin se han ras evidencias cientficas, sin embargo, en la bibliografa
descrito casos con CMV, VIH, Salmonella typhi, Leishmania, revisada, con esta combinacin consigue una mejora de
etc, aunque en menor nmero. la supervivencia.
Otras causas que pueden desencadenar el sndrome son La evolucin que sufrieron los niveles de ferritina sricos,
desrdenes inmunes (artritis reumatoide, LES, enferme- con un marcado descenso tras la instauracin del trata-
dad de Still), alteraciones hematolgicas malignas (leuce- miento, persistiendo no obstante en niveles muy elevados
mia, linfoma,) (3,5). (>2500 ng/ml). Esto nos habla a favor de un SHF secun-
El SHF asociado a Mycobacterium tuberculosis es inusual, de dario, ya que si dichas cifras hubieran sido debidas exclusi-
hecho en la literatura slo se recogen 36 casos (4). Las ca- vamente a la infeccin no seran tan elevadas ni se habra
ractersticas clnicas y analticas de nuestro paciente, son evidenciado tal descenso.
similares a las descritas en las publicaciones, como son la Podemos concluir, que nuestro paciente presenta una una
edad media que es de 46 aos, la fiebre, la hepatomega- tuberculosis activa que desencadena un sndrome hemofa-
lia, que se encontraba presente en un 49% de los casos goctico. Aunque no podemos confirmar de manera histo-
publicados y la pancitopenia, en un 88%. Es importante lgica el sndrome, cumple criterios suficientes. Al ser un
resaltar que en el 83% de los pacientes haba evidencia proceso grave, con importante repercusin en el pronsti-
de tuberculosis diseminada y que la mayora presentaban co y elevada mortalidad, creemos que debe ser tenido en-
algn tipo de inmunosupresin o comorbilidad asociada, cuenta en el diagnstico diferencial e iniciar tratamiento
como insuficiencia renal en estado avanzado, neoplasia he- precoz ante su sospecha.
matolgica, infeccin por VIH o pacientes trasplantados
(4). Nuestro paciente presentaba una tuberculosis pulmo-
nar localizada y no se encontr ningn dato de inmunosu-
presin ni otra comorbilidad.
La asociacin de tuberculosis y SHF tiene un mal prons-
tico, casi con un 50% de mortalidad. Por las importantes
implicaciones que tiene para el paciente, debe ser conside-
rado de manera precoz dentro del diagnstico diferencial
del sndrome(4).
En nuestro caso, el paciente presentaba un proceso un pro-
ceso sptico de origen respiratorio, por lo que fue necesario
iniciar tratamiento antituberculoso con cuatro frmacos
de manera precoz, adems de medidas de soporte transfu-
sional y hemodinmico.

245
CAPTULO 13
INFECCIONES POR MICOBACTERIAS

Bibliografa

1. Alts A, Prez Lucena MJ, Bruguera M. Systematic


approach to the diagnosis of hyperferritinemia. Med
Clin (Barc) 2014; 142:412-7.
2. Rosrio C, Zandman-Goddard G, Meyron-Holtz EG,
DCruz DP, Shoenfeld Y. The hyperferritinemic sy-
ndrome: macrophage activation syndrome, Stills di-
sease, septic shock and catastrophic antiphospholipid
syndrome. BMC Med [Internet]. 2013 Jan [cited 2015
Apr 20];11:185. Available from: http://www.pubme-
dcentral.nih.gov/articlerender.fcgi?artid=3751883&-
tool=pmcentrez&rendertype=abstract
3. Rouphael NG, Talati NJ, Vaughan C, Cunningham
K, Moreira R, Gould C. Infections associated with
haemophagocytic syndrome. Lancet Infect Dis.
2007;7:814-22.
4. Brastianos PK, Swanson JW, Torbenson M, Sperati J,
Karakousis PC. Tuberculosis-associated haemophago-
cytic syndrome. Lancet Infect Dis. 2006; 6:44754.
5. Li F, Yang Y, Jin F, Dehoedt C, Rao J, Zhou Y, et al.
Clinical characteristics and prognostic factors of adult
hemophagocytic syndrome patients: a retrospective
study of increasing awareness of a disease from a sin-
gle-center in China. Orphanet J Rare Dis [Internet].
2015 Feb 15 [cited 2015 Apr 6];10(1):20. Available
from: http://www.pubmedcentral.nih.gov/articleren-
der.fcgi?artid=4355377&tool=pmcentrez&render-
type=abstract
6. Schram AM, Campigotto F, Mullally A, Fogerty A,
Massarotti E, Neuberg D. Marked hyperferritinemia
does not predict for HLH in the adult population.
Blood 2015;125:154852.
7. Ramrez C, Rubio C, de la Puebla RF, Aguilera
C, Espejo I, Fuentes F. Significado clnico de los va-
lores elevados de ferritina srica. Med Clin (Barc)
2004;122:5324. Available from: http://db.doyma.
es/cgi-bin/wdbcgi.exe/doyma/mrevista.fulltext?pi-
dent=13060470
8. Egeler RM, Webb D, Winiarski J, Janka G. HLH-
2004: Diagnostic and Therapeutic Guidelines for He-
mophagocytic Lymphohistiocytosis. 2006;(February).

246
Ndulos subcutneos y poliartritis destructiva
en paciente con artritis seronegativa grave
Hidalgo Jimnez, A; Jimnez-Ruz, FJ;
Mariscal Vzquez, G; Martnez Marcos, FJ.
Complejo Hospitalario Universitario. Huelva.

Caso clnico

Se trata de un varn de 53 aos. Entre sus antecedentes


principales destaca Poliartritis crnica seronegativa diag-
nosticada en 2010, para lo cual haba realizado varias
lneas de tratamiento (Metrotexate en dosis ascendente,
AINES y corticoides) con mala evolucin clnica. Desde
dicho diagnstico haba presentado varios episodios de
bursitis subacromial en el hombro derecho y artritis en el
tobillo izquierdo que precisaron infiltraciones como trata-
miento analgsico, desarrollando un trayecto fistuloso en Imagen 1.
ambas localizaciones. En 2011 requiri ingreso por artritis
sptica del hombro derecho, con aislamiento de Corynebac-
terium spp siendo tratado mediante artrocentesis y antibio-
terapia prolongada. Se aadi al tratamiento varios meses
despus Etanercept (Mantoux previamente negativo) que
finalmente se retir por ineficacia y respuesta inadecuada.
Cabe destacar la situacin previa al ingreso, ya que en los
ltimos meses haba presentado severos brotes articulares
de inflamacin marcada a nivel de ambas manos y miem-
bros inferiores (rodilla derecha y ambos tobillos) y la apari-
cin de ndulos subcutneos que progresaron en tamao.
El paciente ingresa en Febrero de 2013 tras los hallazgos
encontrados en una gammagrafa con galio realizada en el
mbito privado donde se describe un proceso patolgico
muy activo en el tobillo izquierdo y focos patolgicos en
el hombro y el pie derecho. Reinterrogndole de forma
dirigida sobre la clnica articular, afirma que, desde hace Imagen 2.
varios meses, dicha inflamacin es muy acusada a nivel
del tobillo izquierdo y de ambas manos, as como dolor y proximal, con orificio exudativo.
supuracin intermitente del trayecto fistuloso del hombro Llama la atencin la presencia de varios ndulos subcut-
derecho y del tobillo izquierdo. Destaca la aparicin de n- neos, el mayor de ellos situado en el antebrazo izquierdo
dulos subcutneos indoloros en el antebrazo que han ido (imagen 1), sin calor ni rubor, de consistencia semi-blanda,
aumentando de tamao. No refiere otra sintomatologa no adheridos a planos profundos. En el tobillo izquierdo
por aparatos, salvo astenia. se visualiza inflamacin y una tumoracin blanda a nivel
A la exploracin fsica inicial el paciente mantiene un buen de la articulacin metacarpo-falngica del 2 dedo. Por l-
estado general. La auscultacin cardiorrespiratoria as timo, en el tobillo izquierdo existe dolor e inflamacin y
como la exploracin abdominal es normal. En la explora- supuracin exudativa por un trayecto fistuloso.
cin articular se objetiva en el hombro derecho un orificio En las analticas iniciales tanto el hemograma como el
fistuloso limpio, sin exudacin en el momento de la explo- estudio de coagulacin no tenan alteraciones, con una
racin. Los codos y las muecas estn respetados, as como cifra de leucocitos dentro de los valores normales. En la
ambas articulaciones coxo-femorales. En la mano derecha bioqumica de ingreso los reactantes de fase aguda, PCR,
a nivel del 2 dedo existe inflamacin a nivel interfalngica Pro-calcitonina y VSG, se encuentran discretamente eleva-

247
CAPTULO 13
INFECCIONES POR MICOBACTERIAS

dos 5,83 mg/dL, 2,1 ng/dL y 85 mm/h respectivamente.


Se toman muestras superficiales de ambas fstulas, de los
ndulos subcutneos descritos, as como se realiza artro-
centesis del tobillo izquierdo..
La radiografa de trax realizada fue informada como nor-
mal. Las radiografas del pie derecho y de la mano derecha
mostraron artritis con destruccin sea de la articulacin
metatarso-falngica del 2 dedo y de la interfalange proxi- Imagen 3.
mal del 2 dedo respectivamente
Se solicitan pruebas de imagen complementarias para filiar de alteraciones en la piel es consistente con el diagnstico.
con mayor exactitud el nivel de afectacin osteo-articular En nuestro paciente se desestim posteriormente.
En primer lugar, obtenemos la RNM de tobillo izquierdo Otra posibilidad diagnostica es la artritis inducida por cris-
(imagen 2), describiendo osteomielitis y artritis sptica de tales (gota y pseudogota), aunque en este caso existe una
tobillo y retropi, con abscesos y trayectos fistulosos, teno- buena respuesta al tratamiento con antinflamatorios y el
sinovitis infecciosa del tibial posterior y abscesos de partes curso es ms agudo. Tambin, se puede tratar de una ar-
blandas. Adems, se realiza RNM del hombro derecho tritis reactiva, una exacerbacin de su enfermedad de base
(imagen 3), bursitis infecciosa subdeltoidea con componen- (con mayor afectacin de ciertas articulaciones), o metsta-
te muscular deltoideo y fistulizacin a piel. sis de un proceso neoplsico. Todas estas patologas fueron
La ecografa realizada de la tumoracin del antebrazo des- descartndose con el resultado de las pruebas complemen-
cribe una coleccin de 5 x 3,5 cm heterognea superficial a tarias realizadas.
los flexores y signos de tenosinovitis en la mueca. Lo especial del caso y a su vez distintivo radica en la pre-
Ante estos hallazgos consensuamos con el paciente la toma sencia de ndulos subcutneos concomitante a la clnica
de biopsias en quirfano del tobillo izquierdo y de la tumo- articular, adems de afectacin sinovial de mueca y tobi-
racin del antebrazo. llo. Inicialmente, la presencia de estos ndulos, se encua-
dra dentro de su enfermedad de base, pudiendo estar en
Diagnstico diferencial relacin con una exacerbacin de su patologa articular o
siendo la manifestacin clnica de un proceso infeccioso
Ante lo descrito previamente, el juicio sindrmico inicial subyacente, por lo que solicitamos biopsia programada en
del paciente, relaciona la presencia de ndulos subcut- quirfano. Los ndulos subcutneos en pacientes inmu-
neos, tenosinovitis y poliartritis destructiva con signos de nodeprimidos pueden ser una posible seal temprana de
infeccin, en un paciente inmunodeprimido por artropata una infeccin diseminada con un mal pronstico. Existen
seronegativa grave. diferentes entidades que podran ser las causantes de estos
La sospecha inicial obliga a descartar artritis sptica en una ndulos por lo que debemos establecer un diagnostico di-
o varias articulaciones requiriendo un abordaje amplio de ferencial. Entre las opciones deben estar aquellas enferme-
la etiologa de microorganismos infectivos a ese nivel. dades que causen linfagitis nodular, este sndrome es poco
Los microorganismos ms frecuentes en el diagnostico di- frecuente y puede estar causado por hongos, bacterias,
ferencial etiolgico deben incluir S.aureus, Streptococo spp, micobacterias, parsitos y virus. La forma clnica puede
bacterias gram negativas aerobias, Neisseria gonorroheae y cursar con la presencia de ndulos ascendentes con linfa-
otros microorganismos menos prevalentes como hongos y gitis o linfadenitis proximal o sin ellas. Para el diagnstico
Mycoplasma spp. Adems de los patgenos clsicos debemos definitivo se requiere la biopsia de la lesin.
pensar en bacterias de crecimiento lento o menos clsicas Especificamente la P. aeruginosa tambin se ha implicado
que puedan destruir la articulacin y afectar al hueso de como agente etiolgico de ndulos, aunque son ms carac-
forma insidiosa, como las micobacterias. tersticos en los miembros inferiores. Existen otros procesos
Otro diagnstico no infeccioso a tener en cuenta es la sarcoi- caracterizados por ndulos como la escabiosis nodular, la
dosis, que es la causa de lesiones seas en aproximadamen- sfilis secundaria o la angiomatosis bacilar.
te el 5% de los pacientes, estando la enfermedad articular Una entidad poco frecuente descrita en la literatura que
presente en un 25-50% de los casos de sarcoidosis, siendo provoca afectacin de la sinovial es la infeccin de la vaina
generalmente poliarticular. Las lesiones seas tpicas son tendinosa por micobacterias. La presentacin clnica suele
principalmente en manos y pies, inicialmente dolorosas, con ser la presencia de una masa indolente lo largo del tendn
gran destruccin del hueso en fases avanzadas y la presencia asociado con dolor y limitacin de los movimientos. Es

248
CAPTULO 13
INFECCIONES POR MICOBACTERIAS

una forma rara de afectacin extrapulmonar por M.tuber- La afectacin extrapulmonar se presenta en el 10-15% de
culosis. Esta infeccin de la vaina tendinosa con frecuen- los casos de tuberculosis, con mayor frecuencia en pacien-
cia se produce en la mano, el pie, el tobillo y la mueca, tes inmunodeprimidos. La afectacin muscular esqueltica
afectando especialmente a los compartimentos flexores. La de la tuberculosis es una forma de presentacin rara, que
infeccin de la vaina del tendn puede resultar de la dise- se sita en torno al 1-5% del total de casos, pese a lo cual
minacin hematgena del bacilo. ocupa el tercer lugar en frecuencia en cuanto a la afecta-
cin extrapulmonar (tras la afectacin linftica y genitou-
Evolucin rinaria).
La transmisin del M. tuberculosis suele producirse por va
Como parte del diagnostico diferencial en las muestras en- area hasta llegar as a la zona pulmonar. En cuanto a la
viadas a microbiologa (muestra de tobillo y tumoracin), tuberculosis musculoesqueltica, en la mayora de los ca-
se solicita estudio completo de micobacterias y otras bacte- sos, se produce una diseminacin por va hematgena a
rias de crecimiento defectivo. Adems, se solicita Mantoux partir de un foco primario, normalmente pulmonar, que
y estudio analtico ampliado. en circunstancias de inmunidad deprimida se reactiva;
Las pruebas solicitadas van llegando y nos informan desde aunque tambin hay otras vas de diseminacin, como la
el laboratorio de microbiologa que en las muestras recibi- linftica o por contigidad.
das del tobillo, la baciloscopia directa es positiva y en los La tuberculosis musculoesqueltica es ms frecuente en
cultivos extrados en quirfano de los ndulos y la puncin pacientes inmunodeprimidos. Las manifestaciones muscu-
aspiracin de dicha lesin se aisla M.tuberculosis. La prueba loesquelticas por M. tuberculosis se suelen presentar de for-
de la tuberculina realizada es positiva (20 mm x 15 mm), ma insidiosa, en ausencia de fiebre y sntomas generales.
apoyando el diagnstico. La bursitis tuberculosa puede ser un signo de reactivacin
La histologa de la biopsia obtenida en quirfano nos in- de la enfermedad. El diagnstico se suele realizar en fases
forma de ulceracin con inflamacin granulomatosa ne- avanzadas por la presencia de abscesos fros o fstulas.
crotizante, la presencia de macrfagos epitelioides, clulas Las artritis perifricas suelen presentarse en forma de mo-
gigantes tipo Langhans y necrosis caseosa, con linfocitos pe- noartritis (nicamente un 10% lo hace en forma poliarti-
rivasculares, caractersticas de los granulomas tuberculosos. cular), con clara predileccin por las grandes articulacio-
El paciente es dado de alta en planta con tratamiento tu- nes, entre ellas, las ms afectadas son la cadera y la rodilla,
berculosttico combinando Rifampicina + Isoniazida + y en menor frecuencia, el tobillo y la mueca, aunque
Pirazinamida + Etambutol a dosis estndar durante los puede afectarse cualquier articulacin, como en el caso de
dos meses iniciales. Se revisa en consultas externas men- nuestro paciente. Es una enfermedad grave, de larga du-
sualmente con evidente mejora de las lesiones cutneas, racin, que compromete al enfermo en su estado general.
desapareciendo casi por completo los ndulos. La poliar- Las articulaciones afectadas, generalmente destruidas por
tritis ha disminuido y los trayectos fistulosos del hombro y el proceso infeccioso crnico, curan con rigidez y anquilo-
la rodilla estn en fase de resolucin, con buen aspecto y sis de la articulacin.
sin componente exudativo. Aproximadamente el 50% de las osteoartritis tuberculosas
A los dos meses se suspende Etambutol y pirazinamida se acompaan de lesin tuberculosa pulmonar radiolgi-
completando un ao de tratamiento con rifampicina e iso- camente demostrable y el resto tienen una radiografa de
niazida con excelente tolerancia y resolucin casi completa trax normal.
de las lesiones. El mayor desafo en el diagnstico de la tuberculosis sea
es considerar el diagnstico, sobre todo porque no hay evi-
Diagnstico final dencia de enfermedad torcica activa en ms de la mitad
de los casos. Adems, los retrasos en el diagnstico son
Tuberculosis osteoarticular y de partes blandas en paciente comunes debido a la naturaleza indolora del hueso tu-
con artritis seronegativa grave en paciente inmunodepri- berculoso y enfermedad de las articulaciones. El diagns-
mido. tico se suele realizar en fases avanzadas por la presencia
de abscesos fros o fistulas. El diagnstico definitivo de la
Discusin tuberculosis musculoesqueltica se establece mediante la
visualizacin de los bacilos al microscopio y a travs del
La tuberculosis es una enfermedad infecciosa emergente cultivo del material infectado. La tincin de Zhiel Nielsen,
en nuestro medio, producida por el bacilo M. tuberculosis. por desgracia, es positiva en una minora de casos. El diag-

249
CAPTULO 13
INFECCIONES POR MICOBACTERIAS

nstico puede ser establecido por biopsia sinovial, aunque


el examen del lquido articular sinovial por lo general no
es til. Las imgenes de la TAC o la RMN ayudan a de-
limitar su extensin y la existencia de afectacin sea. La
prueba de la tuberculina suele ser positiva, aunque su valor
diagnstico es muy limitado. Un resultado positivo apoya
el diagnstico, aunque un resultado negativo no permite
exclurlo.

Bibliografa

1. Tuli S. General Principles of Osteoarticular Tubercu-


losis Clinical Orthop Relat Res 2002; 398: 11-19
2. Arias Miranda IM, Nu. Arias Miranda IM, Nuo Ma-
teo FJ, Babo Herriz J, Noval Menndez J, Galiana
Martn D. Artritis tuberculosa perifrica. An Med In-
terna 2004;21:412-3.
3. Caroti, M. Zanazzi, P. Rogasi, E. Fantoni, S. Farsetti,
G. Rosso, E. Bertoni, and M. Salvadori. Subcutaneous
nodules and infectious complications in renal allograft
recipients. Transplant Proc. 2010;42:1146-7
4. Cortez MV, Oliveira CM, Monte RL, Arajo JR,
Braga BB, Reis DZ, et al. HIV associated tuberculous
lymphadenitis: the importance of polymerase chain
reaction (PCR) as a complementary tool for the diag-
nosis of tuberculosis a study of 104 patients. An Bras
Dermatol 2011; 86:925-31.
5. Michel Nguessan ANOUMOU , Maurice KOUAME
, Thomas DAIX , Armand YEPIE. Tuberculosis te-
nosynovitis of the flexor tendons in the wrist: a case
report. Acta Orthop Traumatol Turc 2014;48:690-2.
6. Oshima M, Fukui A, Takakura Y. A case of tuber-
culous tenosynovitis in a patient with systemic lupus
erythmatosus. Hand Surg 2004;9:109-13
7. Barbagallo J1, Tager P, Ingleton R, Hirsch RJ, Wein-
berg JM.. Cutaneous tuberculosis: diagnosis and treat-
ment. Am J Clin Dermatol 2002; 3: 319-28
8. Pertuiset E. Tuberculosis of bone and joint members.
EMC-Rheumatologie Orthopdie 2004;463-86.

250
Varn joven con fiebre y derrame pericrdico
Chacn Mora, N; Roca Oporto, C;
Navarro Amuedo, MD; Luque Mrquez, R.
Hospital Universitario Virgen del Roco. Sevilla.

Caso clnico nesis por aparatos y sistemas.


A la exploracin fsica: febril, 39C, TA 130/70 mmHg,
Presentamos el caso de un varn de 37 aos, natural de FC 112 lpm. FR 12. SatO2 98% a FiO2 ambiente. No se
Rumana, residente en Espaa desde haca 6 aos. Fuma- palpaban adenopatas perifricas. La auscultacin cardia-
dor de unos 30 cigarrillos/da desde los 15 aos (33 pa- ca era normal, sin soplos ni roces. A la auscultacin pul-
quetes/ao). Trabaj en una fbrica de plsticos y estaba monar destacaba hipofonesis generalizada sin estertores.
diagnosticado de silicosis por exposicin laboral a chorros En la analtica de urgencias presentaba 9600 x10e9/L leu-
de arena. Present una tuberculosis pulmonar (TBP) 10 cocitos, 13.5% monocitos; resto normal. Funcin renal,
aos antes tratada en su pas durante seis meses con una ionograma y transaminasas dentro de la normalidad. PCR
pauta diaria que contena rifampicina. 112mg/l.
Acudi a urgencias por fiebre de 39C de una semana de El ECG no mostr alteraciones relevantes. En la radiogra-
evolucin, tos seca y odinofagia sin otra clnica a la anam- fa de trax (imagen 1) se objetiv aumento del ndice car-
diotorcico, imagen nodular en lbulo superior izquierdo
e infiltrado intersticial perihiliar bilateral. La TC de trax
(imagen 2) puso de manifiesto una fibrosis masiva progre-
siva secundaria a silicosis con tractos fibrticos residuales
tuberculosos superpuestos, afectacin infecciosa de la va
area distal y un derrame pericrdico severo confirmado
por ecocardiografa transtorcica que revel datos de ta-
ponamiento.

Diagnstico diferencial

A su ingreso se estableci el diagnstico diferencial entre


las causas de derrame pericrdico:

Imagen 1. Radiografa de trax PA al ingreso. Ndulo pulmonar silictico. Au- Causas infecciosas: cualquier microorganismo puede
mento del ndice cardiotorcico. Infiltrado intersticial bilateral en bases.
afectar al pericardio.
Virus, principalmente coxsackievirus. Tambin
hay que pensar en infeccin por VIH.
Bacterias: las ms frecuentemente aisladas en l-
quido pericrdico son Staphylococcus spp, S. pneumoniae y
Streptococcus spp. Menos frecuente, aunque a conside-
rar en nuestro paciente, es la pericarditis tuberculosa
(PTB).
Otros: Rickettsias, espiroquetas, hongos, parsitos,
Tropheryma whippelii o Chlamydia.
Causas neoplsicas: fundamentalmente cncer de
mama, de pulmn y el linfoma de Hodgkin.
Causas autoinmunes: las ms frecuentes el lupus erite-
matoso sistmico y la artritis reumatoide.
Causas metablicas: pericarditis urmica, hipotiroidis-
Imagen 2. TC de trax. Derrame pericrdico moderado-severo. mo.

251
CAPTULO 13
INFECCIONES POR MICOBACTERIAS

Otras: secundario a infarto agudo de miocardio, a ra- exudado fibrinoso, con presencia relativamente abundante
diacin, post-quirrgica, traumatolgica, farmacolgi- del bacilo, b) derrame sero-sanguinolento, c) reabsorcin
ca, quimioterapia. del exudado y formacin de granulomas caseificantes y d)
Idioptico: hasta en el 70% de los casos puede no es- adelgazamiento progresivo y fibrosis o calcificacin del pe-
tablecerse una etiologa del derrame. Se cree que la ricardio, pudiendo originar una pericarditis constrictiva (3).
mayora de ellos es de origen viral. Las manifestaciones clnicas iniciales suelen ser inespecfi-
cas y el comienzo de los sntomas insidioso con fiebre, su-
Evolucin doracin nocturna, prdida de peso y astenia, apareciendo
posteriormente tos, dolor torcico y disnea (3). Entre un
Se realiz una ventana pericrdica drenndose 150 ml de 18-46% de los casos desarrollarn pericarditis constrictiva
lquido sero-sanguinolento con 2380 leucocitos/ l (80% a pesar de instaurarse tratamiento adecuado (1).
mononucleares) con determinacin de ADA y glucosa El diagnstico de PTB es complejo y tardo debido a la na-
dentro de la normalidad. Ante la sospecha clnica de peri- turaleza paucibacilar de las muestras biolgicas y requiere
carditis tuberculosa se instaur tratamiento con 4 frma- el procesamiento de elevados volmenes de lquido peri-
cos antituberculosos de primera lnea (isoniazida, rifam- crdico (LP) (4). El diagnstico definitivo se establece por
picina, pirazinamida y etambutol) sin mejora del cuadro. cultivo de lquido o de tejido pericrdico. Dada la dificul-
El test de VIH fue negativo. Las baciloscopias y el cultivo tad para aislar el microorganismo, muchos autores definen
en medio de micobacterias de cuatro muestras de esputo PTB probable ante una enfermedad tuberculosa en otra
inducido fueron negativas. La baciloscopia del lquido pe- localizacin, en un paciente con pericarditis no explicada,
ricrdico fue negativa y en el cultivo del mismo, en medio exudado pericrdico linfocitario, biomarcadores de infec-
de micobacterias, creci M. tuberculosis resistente a rifampi- cin tuberculosa elevados y respuesta al tratamiento espe-
cina (R), isoniazida (I), pirazinamida (P) y etambutol (E) as cfico antituberculoso (3).
como a capreomicina y amikacina, siendo sensible al resto El lquido pericrdico es caractersticamente un exudado
de frmacos de segunda lnea. Se modific tratamiento a de predominio mononuclear con protenas elevadas. La
moxifloxacino, protionamida y linezolid y se aadi pred- sensibilidad de la baciloscopia del LP oscila entre 0-42%
nisona 50 mg/da. Se mantuvo el tratamiento antituber- y la del cultivo entre 56-67%, con un retraso de hasta tres
culoso durante 18 meses con buena evolucin clnica y se semanas en la obtencin de resultados (5). Aunque con di-
realiz seguimiento durante 24 meses sin recidiva. ferentes puntos de corte (30-72 U/L), la actividad de la
adenosindeaminasa (ADA) en el LP ha demostrado ser
Diagnstico final una herramienta til y econmica en el diagnstico de la
PTB con una sensibilidad entre el 84-100% (6,7). El es-
Pericarditis tuberculosa por M. tuberculosis multidrogorre- tudio anatomo-patolgico del tejido pericrdico muestra
sistente. granulomas caseificantes en el 50% (5). Con la aprobacin
de las tcnicas de deteccin de cidos nucleicos (PCR) en
Discusin muestras respiratorias y en lquido cefalorraqudeo, creca
la esperanza de mejorar el rendimiento diagnstico en la
La PTB es una es localizacin infrecuente de la enferme- PTB. Sin embargo, en dos estudios realizados en reas en-
dad tuberculosa (1-2%de pacientes con TBP). Constituye dmicas de tuberculosis, se compar la eficacia diagnstica
aproximadamente un 4% de todas las pericarditis en nues- de la PCR con el cultivo, el anlisis histolgico y la deter-
tro medio (1) y un 70% de las pericarditis constrictivas en minacin de ADA en el LP, no siendo la sensibilidad de la
pases en vas de desarrollo, donde se encuentra ligada a la PCR superior al 75% (6,8). En un estudio prospectivo en
infeccin por el VIH en la mitad de los casos (2). Sudfrica se ha comparado la precisin diagnstica de la
En la mayora de las ocasiones representa la reactivacin PCR (Xpert MTB/RIF) con la determinacin de ADA y
de la enfermedad tuberculosa, pasando desapercibido el de interfern gamma no estimulado (uIFN) en 151 pa-
foco primario de la infeccin. La afectacin pericrdica cientes con sospecha de PTB, 124 con diagnstico cierto.
se produce por diseminacin linftica de M. tuberculosis La sensibilidad del test Xpert MTB/RIF fue del 63,8% y
a travs de las cadenas ganglionares adyacentes al rbol la especificidad 100%; la determinacin de ADA (pun-
traqueo-bronquial o, menos frecuentemente, por contigi- to de corte 35 IU/ml) y de uIFN (punto de corte 44
dad desde una tuberculosis pulmonar o por diseminacin pg/ml) fueron del 95,7% con una especificidad del 84% y
hematgena. Se establecen cuatro estadios patolgicos: a) 96,3% respectivamente (9). Aunque la determinacin del

252
CAPTULO 13
INFECCIONES POR MICOBACTERIAS

uIFN es muy prometedora para establecer un diagnstico cia, reducir el taponamiento cardiaco ni el desarrollo de
rpido, su evaluacin se realiz en zonas con una preva- pericarditis constrictiva (10).
lencia de tuberculosis >30% y una alta incidencia de infec-
cin por VIH (74%).
Los frmacos activos frente a M. tuberculosis se dividen en
frmacos de primera lnea o grupo 1 (Isoniazida, Rifampi-
cina, Pirazinamida y Etambutol) y de segunda lnea, estos
a su vez en el grupo 2 (inyectables), 3 (fluoroquinolonas), 4
y 5; en orden decreciente de efectividad clnica y eviden-
cia cientfica (figura 1). Segn el patrn de resistencia a es-
tos frmacos antituberculosos se define como tuberculosis
multirresistente (MDR-TB) al aislamiento de M. tuberculosis
resistente al menos a Isoniazida (I) y Rifampicina (R); tu-
berculosis extremadamente resistente (XDR-TB) a aquel
aislamiento resistente al menos a I + R, a alguna fluoro-
quinolona y algn frmaco inyectable (aminoglucsidos o
capreomicina); y tuberculosis totalmente resistente (TDR-
TB) a aquel aislamiento resistente a todos los frmacos Figura 1. Frmacos antituberculosos de segunda lnea.
de primera y segunda lnea testados. En Espaa, en datos
del ao 2013, la incidencia de MDR-TB fue de 34 casos
anuales declarados (0,6% del total de casos - 5539 - de TB
declarados), en nmero decreciente a los aos previos (76
casos de MDR-TB en el ao 2008) y lejos an de cifras
estimadas para otros pases por la OMS de 5% de casos de
MDR-TB del total de casos de TB. La proporcin entre
nacidos en Espaa y fuera de Espaa para los casos de
MDR-TB fue similar (41,1% vs 56,8%) en el ao 2013. Las
recomendaciones de la OMS (guas del ao 2011) de trata-
miento para MDR-TB son: emplear al menos 4 frmacos
antituberculosos que sean efectivos; no emplear grupos de
frmacos con posibilidad de reactividad cruzada (slo un
tipo de fluoroquinolona o un tipo de inyectable, por ejem-
plo); eliminar frmacos no seguros e incluir siempre frma-
cos del grupo 1 al 5 en orden de efectividad. La duracin
del tratamiento de la fase intensiva se recomienda que sea
de 8 meses (o hasta la negativizacin del cultivo) con una
duracin total del tratamiento de 20 meses (o de 24 meses
si hay antecedentes de TB previa o tratamiento previo). En
la actualidad disponemos de nuevos frmacos antituber-
culosos en fase III, como la Bedaquilina (TMC-207), au-
torizada por la FDA en el ao 2012 para el tratamiento de
MDR-TB como parte de la terapia combinada cuando no
se disponen de otras opciones; Delamanid (OPC-67683) y
regmenes combinados incluyendo Moxifloxacino, Protio-
namida y Pirazinamida.
El papel de los corticoides en el tratamiento de la PTB no
est bien dilucidado. Numerosos estudios han sido publi-
cados pero en un reciente ensayo clnico en Sudfrica que
inclua 1400 adultos con PTB definitiva o probable, el uso
de corticoides no demostr mejorar la tasa de superviven-

253
CAPTULO 13
INFECCIONES POR MICOBACTERIAS

Bibliografa

1. Sagrista-Sauleda J, Permanyer-Miralda G, Soler-Soler


J. Tuberculous pericarditis: ten-year experience with a
prospective protocol for diagnosis and treatment. J Am
Coll Cardiol. 1988;11:724 8.
2. Reuter H, Burgess LJ, Doubell AF. Epidemiology of
pericardial effusions at a large academic hospital in
South Africa. Epidemiol Infect. 2005;133:393-9.
3. Mayosi BM, Burgess LJ, Doubell AF. Tuberculous pe-
ricarditis. Circulation. 2005;112:3608-16.
4. Mehta PK, Raj A, Singh N, Khuller GK. Diagnosis of
extrapulmonary tuberculosis by PCR. FEMS Immu-
nol Med Microbiol. 2012;66: 2036.
5. Reuter H, Burgess L, Van Vuuren W, Doubell A.
Diagnosing tuberculous pericarditis. Q J Med. 2006;
99:82739. DOI:10.1093/qjmed/hcl123
6. Lee JH, Lee CW, Lee SG, Yang HS, Hong MK, Kim
JJ et al. Comparison of polymerase chain reaction
with adenosine deaminase activity in pericardial fluid
for the diagnosis of tuberculous pericarditis. Am J
Med. 2002;113: 519-21.
7. Burgess LJ, Reuter H, Carstens ME, Taljaard JJ, Dou-
bell AF. The use of adenosine deaminase and interfe-
ron-gamma as diagnostic tools for tuberculous pericar-
ditis. Chest 2002;122:900-5.
8. Cegielski JP, Devlin BH, Morris AJ, Kitinya JN, Puli-
paka UP, Lema LE et al. Comparison of PCR, Cultu-
re, and Histopathology for Diagnosis of Tuberculous
Pericarditis. J Clin Microbiol. 1997;35: 3254-7.
9. Pandie S, Peter JG, Kerbelker ZS, Meldau R, Theron
G, Govender U et al. Diagnostic accuracy of quantita-
tive PCR(Xpert MTB/RIF) for tuberculous pericardi-
tis compared to adenosine deaminase and unstimula-
ted interferon- in a high burden setting: a prospective
study. BMC Medicine. 2014;12:101.
10. Mayosi BM, Ntsekhe M, Bosch J, Pandie S, Jung H,
Gumedze F et al. Prednisolone and Mycobacterium
indicus pranii in tuberculous pericarditis. Engl J Med.
2014;371: 1121.

254
Paciente con fiebre, ndulos pulmonares
y colestasis disociada
Garca Gmez, MJ; Lpez Snchez, MV;
Mercado Montoro, I; Duro Ruiz, G.
Complejo Hospitalario de Jan

Caso clnico normal. Ante la persistencia de la fiebre y empeoramiento


del estado general, se realiz interconsulta a la unidad de
Paciente de 63 aos, con antecedentes de fibrilacin au- enfermedades infecciosas.
ricular permanente no valvular, hipertrofia benigna de El paciente impresionaba de gravedad, con fiebre de 38.3
prstata y neoplasia vesical intraurotelial de alto grado. C, tensin arterial de 94/54 mmHg, frecuencia cardiaca
No presentaba alergias medicamentosas ni hbitos txi- de 102 latidos por minuto, auscultacin cardiorrespirato-
cos conocidos y segua tratamiento habitual con ateno- ria normal, abdomen no doloroso a la palpacin, sin evi-
lol 50mg/24h, digoxina 0,25mg/24h, AAS 300mg/24h, dencia de masas ni organomegalias, puopercusin renal
omeprazol 40mg/24h e instilaciones intravesicales peri- bilateral no dolorosa y miembros inferiores sin edemas.
dicas de Bacilo de Calmette-Gurin (BCG). Negaba clnica respiratoria. Se solicit analtica, en la que
Consulta en el servicio de urgencias por fiebre de 39 C de destacaba la presencia de gamma-glutamil-transpeptidasa
seis horas de evolucin sin foco evidente. Se diagnostica de (GGT) mayor de 400 U/l, fosfatasa alcalina de 200 U/l,
infeccin del tracto urinario y se indica tratamiento am- velocidad de sedimentacin globular (VSG) de 97 mm/h y
bulatorio con amoxicilina-clavulnico 2 gramos cada 12 PCR de 90 mg/L. Una nueva radiografa de trax puso de
horas. Tras dos semanas de tratamiento consulta de nuevo manifiesto la presencia de un patrn difuso micronodular.
por persistencia de la fiebre junto con polaquiuria y co-
luria, por lo que se decide ingreso hospitalario a cargo Diagnstico diferencial
del servicio de Urologa con el diagnstico de infeccin del
tracto urinario. Ante la presencia de fiebre, colestasis disociada y patrn
En las pruebas complementarias al ingreso destacaba au- pulmonar difuso micronodular el diagnstico diferencial
mento de PCR de 89 mg/L, fibringeno ligeramente ele- incluye procesos neoplsicos, autoinmunes e infecciosos.
vado (6,5 g/L) y hematuria, estando el resto de parmetros La afeccin pulmonar primaria por linfoma es rara y re-
del hemograma, funcin renal, electrolitos y coagulacin, presenta menos del 0.5% de todos los tumores pulmona-
dentro de la normalidad. Una radiografa de trax (Fig. res. Puede presentarse en forma de ndulos pulmonares
1) realizada 48 horas antes del ingreso se inform como mltiples y asociarse a fiebre prolongada y colestasis si
existe afectacin heptica concomitante. El carcinoma
bronquioloalveolar representa en torno al 5% de los carci-
nomas broncognicos. Se relaciona poco con el tabaquis-
mo y puede presentarse como masa pulmonar aislada o
como ndulos pulmonares mltiples. La granulomatosis
linfomatoide es una entidad infrecuente, consistente en
una vasculitis granulomatosa pulmonar manifestada en
forma de ndulos pulmonares mltiples y afectacin de
otros rganos (piel, rin, sistema nervioso, hgado). Se
trata de un proceso linfoproliferativo relacionado con in-
feccin por el virus de Epstein Barr y con linfoma B de
clulas grandes. Aunque todos estos procesos tumorales
no se pueden descartar en nuestro caso, la presencia de
neoplasias sincrnicas, aunque posible, es excepcional, por
lo que habra que pensar en una enfermedad pulmonar
metastsica. Sin embargo, se trata de un tumor urotelial no
Figura 1. Radiografa de trax: Patrn intersticial de predominio micronodulillar, infiltrante, en el que dicha afectacin metastsica, aunque
difuso y bilateral.
posible, es excepcional.

255
CAPTULO 13
INFECCIONES POR MICOBACTERIAS

En cuanto a las conectivopatas, tanto la enfermedad de


Wegener como la sarcoidosis podran explicar gran parte
de los hallazgos clnicos de nuestro caso. La enfermedad
de Wegener puede presentarse en forma de ndulos pul-
monares mltiples y sntomas sistmicos (fiebre, afectacin
del estado general); sin embargo, es frecuente la presencia
de compromiso de la va respiratoria superior y afectacin
renal con proteinuria y microhematuria. La sarcoidosis es
una enfermedad granulomatosa en la que la afectacin
pulmonar es muy frecuente, as como la fiebre y la coles-
tasis disociada por presencia de granulomas hepticos. Sin
embargo, en la radiografa de trax el hallazgo ms fre-
cuente es la presencia de adenopatas hiliares asociadas o
no a un patrn intersticial.
Las enfermedades por inhalacin, tipo silicosis u otras neu-
moconiosis, pueden descartarse en nuestro caso por la falta Figura 2. Tomografa axial computerizada. Patrn intersticial micronodular bilater-
al difuso compatible con TBC miliar.
de antecedentes epidemiolgicos y la normalidad en la ra-
diografa de trax previa.
De entre los procesos infecciosos, la histoplasmosis y coc- tro de la normalidad. Fibringeno 6.5 g/l, resto de la coa-
cidioidomicosis, aunque con cuadros clnicos compatibles, gulacin normal. Las enzimas hepticas variaron GOT
pueden descartarse por la ausencia del antecedente de via- 49-96 U/l, GPT45-72 U/l, GGT 401-640 U/l, FA 200-
je a zonas endmicas y/o datos de inmunosupresin se- 225 U/l, protenas 6.2-7.4 g/dl, resto normal. Los valores
vera. Del mismo modo, es altamente improbable que se de otros reactantes de fase aguda como PCR y VSG esta-
trate de una aspergilosis pulmonar invasiva, ya que afecta ban en torno a 89-65 mg/l y 97-54 mm/h respectivamen-
la mayora de las veces a individuos inmunodeprimidos o te. La serologa de hepatitis, VIH y autoinmunidad fueron
con enfermedad pulmonar obstructiva crnica (EPOC) en negativos. En la orina persista microhematuria, siendo el
fases avanzadas. La nocardiosis pulmonar es una entidad cultivo de orina negativo.
poco frecuente que tambin incide fundamentalmente en Debido a la presencia en la radiografa de trax de un pa-
pacientes inmunodeprimidos, con tratamientos prolon- trn intersticial de predominio micronodulillar, difuso y
gados con corticoides o EPOC. Otro proceso infeccioso bilateral, y ante el antecedente de instilacin endovesical
a considerar sera la presencia de mbolos pulmonares de BCG, se solicitaron basciloscopias seriadas en orina y
spticos originados en una endocarditis, aunque los n- cultivos de micobacterias cuyo resultado fue negativo. No
dulos suelen ser de mayor tamao. Sera conveniente la se pudieron realizar baciloscopias de esputo por falta de
realizacin de hemocultivos y un ecocardiograma. Habra expectoracin. Los hemocultivos fueron negativos. En el
que considerar la probabilidad de una neumona vrica, TAC de trax, abdomen y pelvis destacaba: adenopatas
sin embargo, la ms frecuente es la producida por el virus parahiliares bilaterales, patrn intersticial micronodular
varicela/zoster, siendo muy improbable sin la presencia de bilateral difuso de predominio en bases compatible con
las lesiones cutneas caractersticas. TBC miliar, pequea hernia hiatal, hgado homogneo de
Por ltimo, el diagnstico ms probable es la presencia de contornos lisos y hemangioma de L4 (Fig. 2).
una tuberculosis (TBC) miliar, tanto por el cuadro clnico Se inici tratamiento con isoniacida, rifampicina y etam-
como por la imagen radiolgica. butol as como con corticoides, presentando mejora cl-
nica con desaparicin de la fiebre y sndrome miccional
Evolucin por lo que fue dado de alta hospitalaria. En revisiones
posteriores, el paciente segua buena cumplimentacin del
Reinterrogado el paciente, conocimos que el inicio de la tratamiento, estaba asintomtico y analticamente se nor-
clnica haba coincidido con la administracin de la 6 do- malizaron las transaminasas. Complet tratamiento y se
sis de BCG intravesical. Se inici estudio, solicitando nue- realiz investigacin seriada de micobacterias post-trata-
vo control analtico, hemocultivos, cultivo de orina y TAC miento siendo estas negativas.
de trax, abdomen y pelvis.
Respecto al hemograma, los hallazgos se encontraban den-

256
CAPTULO 13
INFECCIONES POR MICOBACTERIAS

Diagnstico final Rabass Sol C. Tuberculosis miliar en paciente trata-


do con instilaciones intravesicales de bacilo de Calme-
Tuberculosis miliar secundaria a instilaciones intravesica- tte-Gurin. Med Intensiva 2006; 30: 116-9.
les de BCG (BCGitis) 3. Mignon F, Chevrire A, Mesurolle B, El Hajam
M,Morel H,Lacombe P. Miliary induced by intrave-
Discusin sical BCG immunotherapy for carcinoma of the blad-
der: CT Findings. J Radiol 2002; 83:368-71.
El bacilo de Calmette-Gurin (BCG) es una cepa viva ate- 4. Lamm DL. Efficacy and safety of bacille Calmet-
nuada de Mycobacterium bovis, una especie del grupo M. tu- te-Gurin immunotherapy in superficial bladder can-
berculosis complex. El tratamiento con instilaciones de BCG cer. Clin Infect Dis 2000; 31 Suppl 3: 86-90.
es ms efectivo que la mayora de agentes quimioterpicos 5. Durek C, Rsch-Gerdes S, Jocham D, Bhle A. Sensi-
intravesicales en la profilaxis y tratamiento del carcinoma tivity of BCG to modern antibiotics. Eur Urol 2000;
superficial de vejiga y el carcinoma in situ. Aunque suele 37 Suppl 1:21-5.
ser bien tolerada por la mayora de los pacientes, el trata- 6. Lamm DL, van der Meijden APM, Morales A, Bros-
miento con BCG puede provocar reacciones locales y sist- man SA, Catalona WJ, Herr HW, et al. Incidence and
micas, algunas de gravedad extrema. En un anlisis retros- treatment of complications of bacillus Calmette-Gu-
pectivo de 2602 pacientes, se evaluaron las complicaciones rin intravesical therapy in superficial bladder cancer. J
del tratamiento, siendo entre ellas la ms frecuente fiebre Urol 1992; 147:596-600.
> 39.5C en un 2.9% de los casos, seguido de hematuria
franca (1%), prostatitis granulomatosa (0.9%), neumonitis
o hepatitis (0.7%), artralgias (0.5%) epididimitis (0.4%),
sepsis (0.4%), rash (0.3%), obstruccin ureteral (0.3%), es-
pasmo vesical (0.2%), absceso renal (0.1%) y citopenias
(0.1%).
Este caso nos recuerda, que ante todo paciente con antece-
dentes de instilacin intravesical de BCG, que presente las
complicaciones descritas anteriormente, se debe sospechar
infeccin sistmica por M. bovis.
El diagnstico definitivo de TBC miliar requiere la detec-
cin microbiolgica de la micobacteria en cultivo de tejido
afectado, sin embargo en este caso no se realiz biopsia
pulmonar dado el cuadro clnico y hallazgos radiolgicos
compatibles, as como la evolucin favorable tras el inicio
del tratamiento antituberculoso.
Respecto al tratamiento, debe iniciarse lo ms pronto posi-
ble, mientras se espera el resultado de las pruebas de detec-
cin de la micobacteria, puesto que un retraso en el inicio
podra conducir a la muerte del paciente. Se debe suspen-
der la administracin de BCG y tratar con isoniazida ms
rifampicina ms etambutol durante 6 meses. M. bovis es
sensible a la mayora de antituberculosos con excepcin de
pirazinamida y cicloserina.

Bibliografa

1. Safdar N, Abad CL, Kaul DR, Jarrard D, and Saint


S. An Unintended Consequence. N Eng J Med 2008;
358: 1496-501
2. Del Castillo Durn Y, Bod Santos F, Castander Se-
rentill D, Jubert Montaperto P, Espinosa Valencia P,

257
Mujer de 74 aos con dolor lumbar no controlado
Soto Bentez, J; Lpez Alonso, B;
Garca Donaire, J; Brun Romero, F.
Hospital Universitario Puerta del Mar. Cdiz.

Caso clnico

Se presenta el caso de una mujer caucsica de 74 aos, sin


FRCV, hbitos txicos ni alergias, con nico antecedente
de neoplasia vesical in situ intervenida por RTU, y en estu-
dio desde 2012 por engrosamiento de urter derecho por
posible etiologa tumoral (citologas repetidamente negati-
vas). Acude a Urgencias por dolor lumbar de semanas de
evolucin de difcil control a pesar de opiceos, sin clnica
neurolgica. En la exploracin, la contraccin y el control
de esfnteres as como la sensibilidad rectal estn conser-
vados; a nivel motor global 5/5 sin prdida de fuerza en
ambas extremidades; la sensibilidad global se mantiene in-
tacta; y los reflejos cutneo-plantar flexor, y los osteotendi-
nosos rotuliano y aquleo tambin se encontraban intactos.
Inicialmente, en el hemograma se observa Hb 11,8 g/dl,
sin leucocitosis ni alteracin plaquetaria; y en bioqumica
destaca protena C reactiva (PCR) 60,9 mg/l y lactato des- Figura 1. Radiografa lateral de columna lumbosacra.
hidrogenasa (LDH) 523U/l. Se decide ingreso para estu-
dio y tratamiento del dolor.

Diagnstico diferencial

Dentro de los diagnsticos diferenciales cabra tener en


cuenta aquellas enfermedades que cursen con raquialgia
de caractersticas inflamatorias, tales como las espondi-
loartropatas y las neoplasias (principalmente metstasis
de primarios pulmonar, vesical y colorrectal). En cuanto al
posible origen tumoral, en las tomografas realizadas pre-
viamente no se encontr tumoracin a otro nivel que pu-
dieran sugerir un origen metastsico, as como la neoplasia
vesical in situ nunca se confirm. Asimismo, el hecho de
que en las metstasis sea habitual encontrar un aumento
de fosfatasa alcalina e hipercalcemia y que en las analticas
de nuestra paciente no aparecan estas alteraciones, nos Figura 2. Gammagrafa sea con captacin a nivel de L2-L3.
descartara el diagnstico tumoral.
Otro diagnstico diferencial seran las infecciones pige- tambin la etiologa fngica (Candida albicans), si bien sta
nas, cuya incidencia es superior a la tuberculosa, siendo suele darse en pacientes inmunodeprimidos sometidos a
el agente causal ms frecuente el Staphylococcus aureus, y en antibioterapia de amplio espectro y en usuarios de drogas
menor medida, los bacilos gramnegativos. por va parenteral, extremos que no se dan en nuestra pa-
Por otro lado, habra que descartar la espondilitis brucelar ciente.
ya que tras la sacroiletis es su localizacin osteoarticular Por ltimo, el mieloma mltiple cuya afectacin en esta
ms frecuente. Asimismo, a pesar de ser muy rara, valorar paciente se descarta por la ausencia de componente mo-

258
CAPTULO 13
INFECCIONES POR MICOBACTERIAS

noclonal en el proteinograma (<1% de los mielomas ml-


tiples no productores).

Evolucin

La paciente ingresa para estudio, encontrndose en el TC,


hallazgos compatibles con espondilodiscitis subaguda con
afectacin de cuerpos vertebrales L2-L3; menos probable-
mente afectacin metastsica; valorando segn la clnica
y la etiologa la posibilidad de tuberculosis. Asimismo, se
solicit una gammagrafa sea (Fig. 2) donde se aprecia-
ba hiperfijacin a dicho nivel; con realizacin posterior de
una resonancia magntica (RM) de columna vertebral en
la que se demuestran hallazgos compatibles con proceso
inflamatorio infeccioso discal L2-L3 con formacin de un
pequeo absceso prevertebral y otro dorsal (aproximada-
mente de 1cm) y datos de osteomielitis de los cuerpos L2-
L3 (Fig. 3). Figura 3. Resonancia Magntica de columna donde se observan abscesos
ventral y dorsal, as como osteomielitis vertebral.
A nivel analtico, destacar un hemograma con Hb 10.7g/
dl, VSG 89 mm/h, sin leucocitos ni alteraciones plaque-
tarias; en la coagulacin un fibringeno de 518 mg/dl; a
nivel bioqumico PCR 75,9 mg/l, con iones, funcin renal
normales; bilirrubina, GOT, GPT, gamma-GT y fosfatasa
alcalina normales; albmina 3,4; y marcadores tumorales
sin alteraciones. El uroanlisis demostraba intensa piuria
(1.219 leucocitos/mcl), con urocultivos negativos. La prue-
ba de Mantoux y la serologa de Brucella fueron negativas,
as como una radiografa de trax sin hallazgos a destacar.
Fue necesaria una biopsia guiada por TC, cuyos cultivos
bacterianos y las diferentes baciloscopias resultaron ne-
gativas, a expensas del resultado del cultivo en medio de
Lownstein. Figura 4. Atrofia de la pelvis renal secundarios a cambios inflamatorios asociados.
En la tomografa urolgica (Uro-TC), se encontraron ha-
llazgos de progresin de las lesiones en pelvis renal y urter lumbares con nueva toma de muestras para microbiologa.
derecho donde se apreciaban cambios inflamatorios aso- Tras la intervencin se inici tratamiento tuberculosttico
ciados y atrofia cortical renal derecha secundaria a lesin con Isoniazida, Rifampicina, Pirazinamida y Levofloxaci-
obstructiva a nivel del meato ureteral derecho (Fig. 4). no. La paciente recuper la funcionalidad, la deambula-
Adems de persistir la clnica, aumentaron los reactan- cin y la desaparicin del dolor neuroptico, por lo que fue
tes de fase aguda (RFA), y comenz con sintomatologa dada de alta con tratamiento emprico tuberculosttico y
relacionada con compromiso radicular (obliteracin de antiestafiloccico. Revisada en consulta se recibe resultado
formenes a nivel de L3) por lo que se decidi el inicio de Lowenstein donde haba crecido M. tuberculosis, confir-
de antibioterapia emprica con Ceftriaxona, Cloxacilina y mando el diagnstico de sospecha de Espondilodiscitis L2-
Rifampicina. La evolucin inicialmente es favorable, con L3 por M. tuberculosis.
disminucin de los RFA, pero tras la primera semana de
tratamiento, stos se estabilizan interrumpindose la me- Diagnstico final
jora. Se obtiene resultado de biopsia sea que se informa
como inflamatoria aprecindose algunas clulas de Lan- Espondilodiscitis L2-L3 por M. tuberculosis. Tuberculosis
ghans. Con la sospecha de origen tuberculoso, ante la falta urogenital.
de respuesta clnica completa, y la de compromiso radicu-
lar, precis descompresin y liberacin quirrgica de races

259
CAPTULO 13
INFECCIONES POR MICOBACTERIAS

Discusin El tratamiento habitual es la terapia antituberculosa du-


rante 6-9 meses. A pesar de la eficacia del tratamiento m-
La espondilodiscitis tuberculosa o Mal de Pott, es una de dico, el abordaje quirrgico es necesario en caso de com-
las formas de presentacin de las espondilodiscitis infeccio- presin medular o radicular, abscesos donde no es posible
sas que consisten en la colonizacin de cuerpo vertebral y el drenaje percutneo y en lesiones donde la destruccin
disco intervertebral con un microorganismo, en este caso, sea produce inestabilidad.
el M. tuberculosis. A pesar de su infrecuencia, es una de las El pronstico depender en gran medida de la precocidad
principales causas de tuberculosis extrapulmonar (1-5%), en el diagnstico y la rapidez en la instauracin de trata-
y representa el 50% de la infeccin tuberculosa osteoar- miento, lo que limitar la evolucin destructiva. A pesar
ticular. El principal factor predisponente es la inmunosu- de ello, difcilmente se conservar la estructura vertebral
presin, si bien en nuestro caso, se trata de un paciente indemne, lo que conllevar una raquialgia mecnica o in-
inmunocompentente. cluso afectacin grave de la estructura sea como secuelas,
La patognesis guarda relacin con reactivacin de focos que se dan hasta en un 40% de los casos.
hematgenos o diseminacin ganglionar prxima. En
nuestra paciente, coincidiendo con la clnica genitourina- Bibliografa
ria, as como que a nivel urolgico todas las pruebas com-
plementarias realizadas fueron negativas para malignidad, 1. Nolla Sol JM, Ariza Cardenal J. Infeccin osteoarti-
hay que plantear la posibilidad de esta patologa. A pesar cular. En: Medicina Interna. Farreras P, Rozman C (ed.).
de no haber sido confirmada la tuberculosis a dicho nivel, 15 Edicin. Barcelona. Elsevier. 2006;1030-35.
tras el tratamiento tuberculosttico, cur la espondilodisci- 2. Colmenero JD, Morata P. Subacute Osteomyelitis:
tis as como las alteraciones genitourinarias, observndose Tuberculous and Brucellar Vertebral Osteomyelitis.
la desaparicin de las alteraciones radiolgicas en el tracto En: Bone and Joint Infections. From Microbiology to Diagnos-
urinario en los controles posteriores, al igual que la piuria tics and Treatment. Zimmerli W. (ed.). Wiley Blackwell.
estril presente al ingreso. 2015; 241-252.
Su diagnstico depender de un alto grado de sospecha 3. Mateo L, Manzano J, Oliv A, Manterola JM, Perez
clnica. El comportamiento inespecfico y la ausencia de R, Tena X, et al. Tuberculosis osteoarticular: estudio
fiebre puede conllevar a un retraso diagnstico, debido a de 53 casos. Med Clin (Barc) 2007; 129:506-9.
que puede simular otras patologas. Dentro de la batera 4. Aguado Garca JM. Tuberculosis y otras infecciones
de pruebas a realizar, es fundamental la radiografa con- por micobacterias. En: Medicina Interna. Enfermedades
vencional donde se podr observar una disminucin de al- producidas por micobacterias. Barcelona. Masson, 2002:
tura del espacio intervertebral, erosiones y destruccin de 1789-97.
cuerpos vertebrales. Por otro lado, la resonancia magnti- 5. Lozano MC, Garca-Agudo L, Moreno R, Chozas N,
ca nos permitir, como beneficio respecto a la tomografa, Garca-Martos P. Espondilodiscitis tuberculosa en C-
una evaluacin del compromiso neurolgico y evidenciar diz (Espaa) durante 10 aos. Rev Med Chile. 2010;
cambios destructivos. Si bien estas pruebas de imagen son 138:1272-5.
necesarias, es imprescindible realizar un diagnstico etio-
lgico mediante cultivos y hemocultivos as como descartar
otras patologas tales como la brucelosis. Ante los resulta-
dos negativos de los cultivos y serologas, se debe obtener
una muestra de la vrtebra afecta para estudio microbiol-
gico y anatomopatolgico mediante puncin-biopsia guia-
da por TC que establece el diagnstico en un 70-80% de
los casos.
La clnica habitual suele ser de dolor e impotencia fun-
cional. En un 10-15% de los casos, hay afectacin a nivel
medular con clnica compresiva como, de hecho, se pre-
sent en nuestra paciente. Asimismo en el 70% pueden
aparecer abscesos locorregionales, como se aprecia en el
caso de nuestra paciente. La regin que se afecta con ms
frecuencia es la lumbar.

260
CAPTULO 14
INFECCIONES
POR PATGENOS ESPECIALES
El pasajero oculto
Badiola Gonzlez, J; Lpez Plana, MT;
Muoz Medina, L.
Hospital Clnico San Cecilio, Granada

Caso clnico 10e3/mcl, neutrfilos 36.1% (absolutos 1180 x 10e3/mcl),


linfocitos 55.5% (absolutos 1820 x 10e3/mcl), hemoglo-
Paciente de 43 aos de edad que acude a la urgencia por bina 11.6 g/dL con VCM 87 fL y 77000 plaquetas/mcl.
fiebre y malestar general de tres semanas de evolucin. El En la bioqumica haba una ligera elevacin de LDH (585
paciente no tena antecedentes mdico-quirrgicos fami- UI/L), enzimas hepticas (GOT 67 U/L, GPT 143 U/L)
liares ni personales de inters y nicamente refera la toma con colestasis disociada (GGT 319 U/L, FA 207 U/L y
ocasional de omeprazol y analgsicos habituales. Fumador bilirrubina total normal) y PCR de 114 mg/L. La funcin
activo de 1 paq/da desde los 12 aos y bebedor habitual renal y los iones estaban dentro de la normalidad. Se rea-
de cerveza y ocasional de ginebra. Viva con su madre en liz estudio de anemia y se encontr un hierro de 27 mcg/
zona rural de la provincia de Granada, siendo soltero y dl con una ferritina de 3581 ng/ml, transferrina 175 mg/
sin hijos. Se dedicaba a tareas agrcolas y ganaderas de dl, IST 12.4% reticulocitos 2.24% y un frotis de sangre
manera que tena contacto diario con animales (cabras, perifrica compatible con la normalidad. Por ltimo, en
perros y gallinas), no haba realizado viajes internacionales la coagulacin bsica la actividad de protrombina era del
recientes ni tenido contacto sexual de riesgo en los ltimos 71% y el INR 1.25.
12 meses. Se realiz ECG, radiografa de trax. en proyecciones pos-
La fiebre era diaria, de hasta 40 C, de unas 3 semanas de teroanterior y lateral, as como radiografa simple de ab-
duracin, de predominio vespertino y caractersticas bac- domen en decbito, siendo ambas normales. En la ecogra-
terimicas, acompandose de escalofros, sudoracin pro- fa abdominal destacaba hepatomegalia y esplenomegalia
fusa e incluso delirios nocturnos. Adems asociaba gran as- moderadas de ecogenicidad heterognea.
tenia, debilidad generalizada progresiva y prdida de peso Con los diagnsticos de fiebre de duracin intermedia,
no ponderada desde haca un ao, habindose acentuado pancitopenia y hepatoesplenomegalia se ingres en el Ser-
en los ltimos cuatro meses. En la anamnesis por rganos vicio de Enfermedades Infecciosas.
y aparatos, nicamente describa discreto dolor farngeo y
molestias abdominales inespecficas, con sensacin de dis- Diagnstico diferencial
tensin abdominal, as como tos seca sin expectoracin ni
disnea de larga evolucin que relacionaba con el hbito El diagnstico diferencial de un paciente con fiebre, panci-
tabquico. topenia y esplenomegalia no es infrecuente en un Servicio
En la exploracin fsica a su llegada, el paciente presen- de Enfermedades Infecciosas.
taba estabilidad hemodinmica con tendencia a la hipo- De una forma general nos podemos plantear tres orge-
tensin (Temperatura 36.4 C, TA 108/62 mmHg, FC 66 nes: causa infecciosa, causa tumoral y causa autoinmune,
lpm, FR 16 rpm y SpO2 98% basal) y buen estado general si bien no podremos olvidar otras miscelneas.
con discreta rubicundez facial. Normohidratado y normo- Dentro de las etiologas infecciosas nos encontramos
perfundido. Consciente y orientado, siendo la exploracin mltiples microorganismos que pueden cursar con dicho
neurolgica compatible con la normalidad. Orofaringe cuadro: a) virus: habra que descartar infeccin por VIH,
algo eritematosa sin exudado y no se palpaban adenopa- herpes virus (VHS, VEB, CMV, VH6) y Parvovirus B19
tas laterocervicales, preauriculares ni supraclaviculares, principalmente; b) pensando en una causa bacteriana es
axilares ni inguinales. La auscultacin cardiorrespiratoria necesario descartar la enfermedad por micobacterias (tu-
era compatible con la normalidad. A nivel de abdomen se berculosa y atpicas), en su forma diseminada, o la posibi-
palpaba hepato-esplenomegalia no dolorosa. En las extre- lidad de infeccin por bacterias intracelulares (Anaplasma,
midades no haba edemas, signos de TVP, lesiones en piel Ehrlichia...), adems, la sepsis por bacterias pigenas tam-
ni estigmas de picaduras. bin puede debutar con fiebre y citopenias; c) en cuanto
En las pruebas complementarias iniciales se detect en a parsitos ser necesario descartar infeccin por malaria,
el hemograma una pancitopenia con leucocitos 3280 x Toxoplasma o Leishmania.

262
CAPTULO 14
INFECCIONES
POR PATGENOS ESPECIALES

En el grupo de las enfermedades onco-hematolgicas es Dada la ausencia de diagnstico y que el paciente conti-
necesario tener en cuenta la posibilidad de un tumor de nuaba con fiebre diaria se decidi conjuntamente por los
rgano slido con infiltracin medular. Respecto a las en- Servicios de Infecciosas, Hematologa y Ciruga General,
fermedades hematolgicas, nos encontramos con un gran realizar esplenectoma ante la sospecha de sndrome lin-
abanico de posibilidades en un paciente con fiebre, cito- foproliferativo esplnico.
penias y esplenomegalia. Ser necesario descartar una en- Tras la intervencin, el paciente comenz a mejorar, con
fermedad linfoproliferativa (principalmente un linfoma es- disminucin de la fiebre, mejora del estado general y recu-
plnico y la tricoleucemia), la mielofibrosis o un sndrome peracin de las citopenias.
mielodisplsico. El informe de la anatoma patolgica del bazo, describa
Aunque menos probable, tambin debemos pensar en des- abundantes fenmenos de hemofagocitosis. Tras conocer
cartar las enfermedades autoinmunes o conectivopatas, este dato, se solicit la realizacin de PCR para Leishma-
principalmente el lupus eritematoso sistmico, la sarcoido- nia en bazo siendo sta positiva.
sis y con mucha menos frecuencia la artritis reumatoidea El paciente fue tratado con anfotericina B liposomal a las
como un sndrome de Felty. dosis habituales (3 ciclos de 5 mg/kg/da separados por 5
Dentro del grupo de las miscelneas, el cuadro clnico po- das) con muy buena evolucin clnica, recuperacin total
dra corresponder con un sndrome de linfohistiocitosis he- y sin recurrencia tras 2 aos de seguimiento.
mofagoctica. Aunque menos probable, tampoco se podra
descartar una histiocitosis o la hemoglobinuria paroxstica Diagnstico final
nocturna.
Con este amplio abanico de posibilidades debemos encajar Linfohistiocitosis hemofagoctica (LHH) secundaria a lei-
los datos que nos ofrece el paciente para llegar al diagns- shmaniasis.
tico definitivo. ste es un proceso paulatino en el que en
muchas ocasiones se necesitaran pruebas complementarias Discusin
especficas.
La LHH es un sndrome grave que se produce como con-
Evolucin secuencia de una excesiva activacin del sistema inmune,
existiendo una falta de regulacin de los macrfagos, los
Durante el ingreso se solicitaron serologas para virus he- cuales experimentan un aumento de actividad y una dis-
patotrpos, VIH, Parvovirus B19, VHS, VH6, les y Bruce- minucin de su control por parte de las clulas NK y los
lla que resultaron negativas, serologas para CMV y VEB linfocitos T citotxicos. Con mayor frecuencia afecta a
con IgG positiva e IgM negativa, y antgeno urinario para nios recin nacidos o menores de dos aos, debido a la
Leishmania spp que tambin fue negativo. Como parte del presencia de mutaciones genticas de transmisin recesi-
estudio se solicit adems un TAC toraco-abdominal, des- va, hablndose de LHH primaria o familiar. En cambio,
tacando nicamente hepato-esplenomegalia moderada sin la LHH secundaria o adquirida suele presentarse en nios
evidenciarse alteraciones pleuro-pulmonares, mediastni- mayores o adultos de cualquier edad, desencadenndose
cas, LOES hepticas ni adenopatas. por un proceso autoinmune, neoplsico o infeccioso fun-
Ante estos resultados, se realiz una puncin-aspiracin de damentalmente (tabla 1). No obstante, muchos de estos
mdula sea siendo inespecfica, donde no se observaron pacientes presentan un defecto gentico heterocigoto y no
fenmenos de hemofagocitosis, parsitos, displasia, datos desarrollan este sndrome hasta que aparezca dichos des-
de mielofibrosis ni infiltrado celular patolgico. encadenantes.
Durante su ingreso en la sala el paciente permaneci en La presentacin clnica se caracteriza por un sndrome
todo momento estable hemodinmicamente, pero con febril prolongado con hepatoesplenomegalia y citopenias,
fiebre alta diaria de hasta 40C de difcil control, siendo aunque progresivamente suele producir afectacin mul-
el estado general ligeramente afecto cuando permaneca tiorgnica.
apirtico. Se obtuvieron mltiples hemocultivos durante el La incidencia se estima en 1.2 casos/milln de individuos/
ingreso siendo todos negativos. ao, aunque probablemente sea una cifra subestimada. Su
Debido a la fiebre con elevacin de enzimas hepticas y baja incidencia y el cuadro de presentacin clnico y ana-
citopenias se decidi iniciar tratamiento emprico con do- ltico inespecfico hace que se retrase mucho el diagnstico
xiciclina que se retir a la semana por no evidenciarse me- de esta patologa.
jora ninguna. Para el diagnstico se han recomendado unos criterios

263
CAPTULO 14
INFECCIONES
POR PATGENOS ESPECIALES

INFECCIOSAS/SEPSIS:
Virus: VIH, VHB, CMV, VVZ, VHH-8, VHS, sarampin, gripe, parvovirus.
enfermedad y evitar as la administracin de un tratamien-
Bacterias: Brucella, Tuberculosis, Gram negativas (ms raro). to quimioterpico agresivo.
Parsitos: Leishmania, Malaria.
La leishmaniasis est descrita como un raro desencade-
Hongos.
NEOPLSICAS: nante del LHH. Hemos realizado una revisin bibliogrfi-
Linfomas: clulas T, anaplsico de clulas grandes. ca encontrando 56 casos descritos en la literatura mdica,
Leucemias: linfoblstica de clulas B, leucemias mieloides.
Slidas (ms raro). de ellos slo 8 en adultos (1). Las dificultades diagnsticas
SISTMICAS: que presentan el solapamiento clnico de ambas enferme-
Conectivopatas: Artritis Idioptica Juvenil, Enfermedad de Still del adul- dades y la ausencia de un protocolo teraputico, dificulta
to, LES, AR, Esclerosis sitmica, Polimiositis
Sarcoidosis. el manejo clnico.
Vasculitis: Panarteritis Nodosa.
A raz de varios casos, hemos realizado una revisin de los
INMUNODEFICIENCIAS:
Trasplantes: mdula sea, rin, hgado.
casos de LHH de adultos diagnosticados en nuestro centro
Esplenectoma. en los ltimos 5 aos, obteniendo 7 casos de los cuales en
Tabla 1. causas de linfohistiocitosis hemofagoctica secundaria. 3 (42.9%) se pudo de demostrar la Leishmania spp como
desencadenante. En los 3 casos el tratamiento se realiz
A. Diagnstico molecular mediante la deteccin de mutaciones genticas:
PRF1,UNC13D,Munc18-2,Rab27a,STX11,SH2D1A, oBIRC4
con anfotericina B liposomal +/- inmunosupresores, evi-

tando as tratamientos quimioterpicos y en todos ellos se
B. Cinco a ocho de los siguientes criterios:
consigui la curacin.
1. Fiebre 38.5C
En nuestra serie Leishmania spp aparece como la primera
2. Esplenomegalia
causa de LHH, pudiendo ser que en zonas endmicas di-
3. Citopenias (con afectacin de 2 3 series en sangre perifrica):
cha asociacin est infravalorada. Esto podra ser debido a
Hemoglobina <9 g/dL
que en areas de alta endemicidad de Leishmania spp a nivel
3 mundial, los recursos socio-sanitarios son escasos. Espaa
Plaquetas <100 10 /mL
3
continua siendo un pas endmico para Leishmania spp,
Neutrfilos <1 10 /mL
principalmente la cuenca mediterrnea, encontrndose
4. Hipertrigliceridemia >265 mg/dL y/o hipofibrinogenemia
<150mg/dL prevalencias de hasta 42.2% (2) en estudios de seropreva-
5. Hemofagocitos en mdula sea, bazo, ganglios linfticos o hgado lencia en zonas rurales de la provincia de Granada.
6. Disminucin o ausencia de actividad de clulas NK Creemos indicado realizar la investigacin exhaustiva de
7. Ferritina >500 ng/mL dicho parsito en las LHH diagnosticadas en zonas end-
8. Elevacin CD25 soluble micas, debido a que ello llevara un cambio en el trata-
miento y el pronstico. La realizacin del antgeno en orina
Tabla 2. Criterios diagnsticos empleados en el hlh-2004 trial. parece ser insuficiente, siendo necesario en caso negativo
la investigacin directa o por PCR de los reservorios.
clnicos/analticos (criterios HLH-2004 trial) y el estudio
gentico (tabla 2): fiebre mayor o igual de 38.5C, esple-
nomegalia, al menos dos citopenias en sangre perifrica,
hipertrigliceridemia y/o hipofibrinogenemia, presencia de
hemofagocitos en biopsia de mdula sea, esplnica, lin-
ftica o heptica, ferritina >500 ng/mL, y elevacin del
marcador CD25. Para realizar el diagnstico son necesa-
rios 5 de los 8 criterios, aunque dada la gravedad y elevada
mortalidad del cuadro, se ha propuesto iniciar tratamiento
en pacientes que cumplan 3 o 4 criterios clnicos y al me-
nos un marcador inmunolgico. Es conveniente recordar
que para realizar el diagnstico no es obligatoria la presen-
cia de fenmenos de hemofagocitosis.
La LHH es una afeccin progresiva y mortal sin tratamien-
to, por ello es fundamental establecer, en los casos secunda-
rios, la causa que lo ha producido para poder realizar un
tratamiento especfico, ya que se ha visto que disminuyen-
do o eliminando el desencadenante, se puede controlar la

264
CAPTULO 14
INFECCIONES
POR PATGENOS ESPECIALES

Bibliografa

1. Rajagopala S, Dutta U, Chandra KS, Bhatia P, Var-


ma N, Kochhar R. Visceral leishmaniasis associated
hemophagocytic lymphohistiocytosis-Case report and
systematic review. J Infect 2008; 56:381-8.
2. Acedo Sanchez C, Martin Sanchez J, Velez Bernal
ID, Sanchis Marin MC, Louassini M, Maldonado JA,
Morillas Marquez F. Leishmaniasis eco-epidemiology
in the Alpujarra region (Granada Province, southern
Spain) Int J Parasitol. 1996; 26:30310.
3. Hindupur S. Hemophagocytosis in hemophagocytic
lymphohistiocytosis. Am J Hematol 2005; 80:299-300.
4. Chandrakasan S, Filipovich AH: Hemophagocytic
lymphohistiocytosis: advances in pathophysiology,
diagnosis, and treatment. J Pediatr 2013; 163:1253-9.
5. Henter JI, Horne A, Arico M, Egeler RM, Filipovich
AH, Imashuku S, Ladisch S, McClain K, Webb D,
Winiarski J, Janka G: HLH-2004: diagnostic and the-
rapeutic guidelines for hemophagocytic lymphohistio-
cytosis. Pediatr Blood Cancer 2007, 48:124-31.

265
Varn de 38 aos con fiebre
y tumoracin axilar dolorosa
Lpez Crdenas, S; Rubio Marn, P;
Prez Corts, S.
Hospital de especialidades de Jerez de la Frontera
Bocanegra Muoz, C.
Clnica de la Salud. Cdiz.

Caso clnico

Varn de 39 aos, de origen boliviano, domiciliado en Je-


rez desde 5 aos antes, trabajador de la construccin. Sin
hbitos txicos, alergias ni viajes recientes al extranjero.
Antecedentes de dermatitis seborreica en cuero cabelludo.
No refera contacto con animales en los ltimos meses.
Acudi a su mdico de atencin primaria por presentar fie-
bre (38-39 C), escalofros, algn vmito y una tumoracin
dolorosa en la axila derecha. Se le prescribi tratamiento
con AINES.A pesar de ello, la fiebre continu, aumentan-
do el dolor y el tamao de la tumoracin, por lo que 4
das ms tarde acudi a urgencias, donde se le recomen-
d tratamiento ambulatorio con Amoxicilina/Clavulnico
2000/125 mg cada 12 horas. A pesar de ello, sigui con
fiebre y la tumoracin axilar creci y se hizo ms dolorosa,
por lo que acudi de nuevo a urgencias donde se decidi Figura 2. TAC torcico en el que podemos ver adenopata axilar derecha.
su ingreso.
El paciente presentaba regular estado general. Consciente, dea con unos 4 cm de dimetro mximo y signos flogticos
orientado y colaborador. Buena hidratacin de piel y mu- en la piel adyacente. Extremidades inferiores normales.
cosas. Normotenso. Bien perfundido. La auscultacin car- Los anlisis efectuados a su ingreso mostraron hemograma
diopulmonar era rigurosamente normal. Abdomen blan- y estudio de coagulacin normales. Las pruebas bioqumi-
do, depresible, no doloroso a la palpacin; sin crecimientos cas eran normales excepto por una PCR de 28,87 mg/dl
viscerales palpables. En axila derecha presentaba una tu- (normal < 0,5 mg/dl).
moracin dolorosa a la palpacin (figura 1), de forma ovoi- Las serologas para VHB, VHC, VHA, Coxiella burnetti, To-
xoplasma, Rickettsias, les, VIH, VEB y rubela resultaron
negativas, al igual que los hemocultivos en pico febril.
La ecografa de partes blandas de la regin axilar, mostr
una coleccin polilobulada de 4 cm en axila derecha, con
otras imgenes redondeadas muy hipoecognicas adyacen-
tes, algunas conservando hilio graso. Impresin: hidrosa-
denitis complicada.

Evolucin

El paciente con los hallazgos mostrados en la exploracin


y pruebas complementarias comenz tratamiento anti-
bitico con cloxacilina + ceftriaxona intravenosas siendo
sometido a una intervencin quirrgica para drenar la co-
leccin axilar visualizada en la ecografa. Se efectu una
incisin superficial que permiti apreciar una tumoracin
profunda que sugera tratarse de una adenopata axilar de
Figura 1. Conglomerado axilar adenoptico. gran tamao, por lo que no realiz ningn procedimiento

266
CAPTULO 14
INFECCIONES
POR PATGENOS ESPECIALES

en esos momentos, sugiriendo el cirujano la conveniencia nodular), por lo que en un paciente joven con fiebre
de completar el estudio del paciente. prolongada y este tipo de adenopata tambin debe de
Tras la intervencin el enfermo continu con fiebre elevada considerarse.
y dolor intenso en axila derecha, a pesar del tratamiento La enfermedad de Kikuchi-Fujimoto suele aparecer
antibitico. Ante esta evolucin se decidi la realizacin de en personas jvenes con fiebre sin foco aparente y es
una TAC toraco-abdominal (figura 2) que result normal, causa de adenopatas, que pueden ser supuradas.
salvo por la presencia de una lesin ocupante de espacio en
la axila derecha, de 5 x 3,5 cm, con bordes desdibujados Evolucin
en su aspecto externo. Presentaba zonas hipodensas en su
interior y tras la inyeccin de contraste mostr captacin Se inici tratamiento emprico con Claritromicina 500 mg,
perifrica del mismo. Diagnstico radiolgico: adenopata un comprimido cada 12 horas con adecuada tolerancia. En
abscesificada, sin descartar otra patologa subyacente. Se los das posteriores la evolucin fue adecuada fisulizando y
observaban adems otras mltiples y pequeas adenopa- drenando espontneamente. Se mantuvo tratamiento an-
tas adyacentes a la principal. tibitico hasta completar 14 das con desaparicin de toda
Ante estos datos, se decidi efectuar una puncin aspira- sintomatologa. En las revisiones posteriores el paciente
tiva de la lesin, obtenindose material purulento. Dos permanece asintomtico y sin nuevos signos o sntomas.
das despus la tumoracin se fustuliz y dren de forma
espontnea. Diagnstico final

Diagnstico diferencial Se recibieron los resultados de microbiologa y anatoma


patolgica del pus de la adenopata. La citologa era infla-
Con el diagnstico sindrmico de adenopata axilar su- matoria, mientras que los cultivos en medios de bacterias,
purada se tuvieron en cuenta las siguientes posibilidades hongos y micobacterias fueron negativos. Dada la sospecha
diagnsticas: clnica se efectu una PCR para Bartonella henselae que
result positiva. Con estos datos se efectu el diagnstico
Absceso axilar pigeno: generalmente secundario a final de enfermedad por araazo de gato. El paciente re-
hidrosadenitis, el germen que ms se asocia a este pro- cibi tratamiento oral con azitromicina 500 mg el primer
ceso es el S.aureus seguido de S.pyogenes. da y 250 mg los cuatro siguientes. A las 48 horas estaba
Infeccin por micobacterias. Como bien es sabido M. asintomtico y fue dado de alta.
tuberculosis es agente causal de adenopatas que hasta
en un 10% son supuradas. M. avium y M. scrofulaceum Discusin
tambin pueden causar linfadenitis, siendo frecuentes
en el segundo caso la abscesificacin y la supuracin Bartonella henselae es un bacilo Gram negativo, de crecimien-
espontnea. to lento y difcil de cultivar. Es el agente causal de la en-
Yersinia pestis debe ser considerada, pero no es endmi- fermedad por araazo de gato, que recibe su nombre por
ca en nuestro medio y aunque el paciente era bolivia- ser este animal el reservorio ms frecuente. La infeccin
no, no haba viajado fuera de Espaa. por este microbio es ms frecuente en los meses de prima-
Francisella tularensis, Bartonella henselae y Sporothrixs chenckii vera-verano y afecta habitualmente a inmunocompetentes
son microorganismos que que pueden producir cua- y menores de 20 aos. La afeccin en inmunocomprome-
dros de este tipo, asocindose las dos primeras al con- tidos suele producir una enfermedad grave y diseminada.
tacto con animales. En el interrogatorio dirigido el pa- El contagio suele ser por contacto directo con animales
ciente refiri que su hijo haba tenido un cachorro de colonizados por esta bacteria, sobre una zona de la piel
gato tres meses atrs, habiendo sufrido toda la familia daada. Las manifestaciones clnicas aparecen a los 3-10
algunos araazos del mismo. das tras el contacto en forma de lesin cutnea (lesin de
Chlamydia trachomatis es otra posible causa de adenopa- inoculacin), que puede ser muy diversa desde ppula a
tas supuradas, particularmente en la regin inguinal. ndulo y que en muchas ocasiones pasa desapercibida.
El paciente neg rotundamente haber tenido contac- Posteriormente aparece alguna adenopata en la zona de
tos sexuales de riesgo. drenaje del punto de inoculacin, las cuales suelen ser
El Linfoma de Hodgkin produce adenopatas fistuli- muy dolorosas y con signos inflamatorios en la piel supra-
zadas hasta en un 2 % de los casos (variedad esclerosis yacente. Hasta un 10% de estas adenopatas se fistulizan

267
CAPTULO 14
INFECCIONES
POR PATGENOS ESPECIALES

y supuran de forma espontnea. Otras menos frecuentes, Bibliografa


son hepatomegalia dolorosa (con granulomas necrotizan-
tes en la biopsia), alteraciones oculares (sndrome oculo- 1. Cruz Arns M, Vila lvarez J, Sa Requejo C, Muoz
glandular de Parinaud y neurorretinitis), manifestaciones Garca JC. Enfermedad por araazo de gato. Revista
neurolgicas (encefalopata, mielitis transversa, radiculitis Centro de Salud. 2001; 9:152-5
y ataxia cerebelosa), sntomas musculoesquelticos (mial- 2. Koehler JE, and Duncan LM. Case 30-2005: A
gias, artralgias) y otras consideradas como atpicas (endo- 56-Year-Old Man with Fever and Axillary Lymphade-
carditis, neumona, prpura trombtica trombocitopni- nopathy. N Engl J M. 2005 353;1605-13.
ca, hipercalcemia). 3. Lain P. Fraser, MD, DPhil. Suppurative Lymphadeni-
tis. Current Infectious Disease Reports. 2009, 11:383
El diagnstico se establece si se cumplen 3 de 4 criterios 8
establecidos que son: 4. Julie E. Reznicek, D.O., William J. Mason, M.D., Da-
niel R. Kaul, M.D., Saint S, and Bloch KC. Avoiding
1. Contacto con gatos (sin ser necesario la visualizacin a Rash Diagnosis. N Engl J Med. 2011; 364: 466-71.
de la lesin de inoculacin). 5. Wormser GP. Discovery of New Infectious Diseases
2. Serologa negativa para otros agentes, pus estril, PCR Bartonella Species. N Engl J Med. 2007 356;2346-7.
de B.henselae positiva o lesiones en bazo-hgado en 6. Davis BT, Thiim M, and Zukerberg LR. A 31-Year-
TAC. Old, HIV-PositiveMan with Rectal Pain. N Engl J
3. Serologa positiva para B.henselae >1:64. Med. 2006; 354: 284-9.
4. Biopsia con inflamacin granulomatosa o tincin War- 7. Camille N, Kattie MD. Zoonoses from cats. UpToDa-
thin-Starry. te 2010.
8. David H Spach, Sheldon Kaplan L. Microbiology, epi-
El tratamiento vara en funcin del cuadro clnico. En caso demiology, clinical manifestations, and diagnosis of cat
de linfadenitis el tratamiento de eleccin es un macrli- scratch disease. UpToDate 2010.
do (azitromicina o claritromicina), aunque muchas veces
resulta innecesario dada la tendencia que tiene la enfer-
medad a curar espontneamente. Si existe enfermedad
hepatoesplnica se aconseja tratamiento combinando (ri-
fampicina con azitromicina o gentamicina). En los casos de
afectacin neurolgica y neurorretinitis el tratamiento de
eleccin es la combinacin de doxiciclina con rifampicina.
Dada la baja incidencia de esta enfermedad en nuestro
medio y de que se trata de un proceso de curso en nume-
rosas ocasiones indolente, esta entidad es rara en la prc-
tica clnica diaria. Su forma de presentacin ms habitual
(adenopata dolorosa y supurada), es infrecuente en otros
procesos, por lo que ante este tipo de lesin y el anteceden-
te de contacto con gatos, debe de tenerse muy presente
esta posibilidad diagnstica. Es fundamental establecer un
diagnstico sindrmico, ya que como en este caso, permi-
te reducir considerablemente las opciones diagnsticas, al
ser escasas las causas de esta clase de adenopatas.
Como en tantas ocasiones hay que destacar la importancia
de una buena historia clnica, ya que nos puede aportar da-
tos que permitan el diagnstico. Numerosas ocasiones los
pacientes no aportan espontneamente informacin de uti-
lidad, por no considerarla relacionada con el cuadro clnico
(recordemos que el paciente no refiri a su ingreso contacto
con animales, un antecedente fundamental en el caso y que
no fue conocido hasta que se le pregunt de forma dirigida).

268
Fiebre, ictericia e insuficiencia renal
en un varn de 35 aos
Lpez Snchez, MV; Garca Gmez, MJ;
Navarro Marn, LJ; Herrero Rodrguez, C.
Complejo Hospitalario Ciudad de Jan

Caso Clnico biliar (colangitis y colecistitis), las hepatitis virales agudas,


abscesos hepticos y las infecciones hepticas por bacte-
Paciente de 35 aos, trabajador portuario, que consulta rias, hongos y parsitos. Otra posibilidad son las hepatitis
por fiebre de hasta 40C de seis das de evolucin junto a no infecciosas, como la hepatitis alcohlica y la hepatitis is-
debilidad generalizada, vmitos, dolor abdominal difuso, qumica (vasculitis, sndrome antifosfolpido catastrfico).
algn despeo diarreico, oliguria e ictericia. Como ante- La ecografa abdominal es anodina, muestra una va biliar
cedentes personales refiere consumir tabaco, cannabis y no dilatada, lo que excluye la colangitis, colecistitis y los
bebedor (76 gr alcohol/da). No ha realizado viajes al ex- abscesos hepticos. La elevacin de las transaminasas es
tranjero ni tiene contacto con animales. ligera (no supera en diez veces el valor normal) lo que va
En la exploracin fsica al ingreso: Consciente y orientado. en contra de hepatitis vrica aguda y hepatitis isqumica.
T 40C, TA: 140/70 mmHg. Destaca una marcada icte- Aunque el paciente consume alcohol a diario no hay datos
ricia subconjuntival junto con sangrado en mucosa oral. que sugieran un consumo excesivo (macrocitosis, araas
Auscultacin cardiorrespiratoria: normal. Abdomen: do- vasculares, hipertrofia parotdea, GOT/GPT<2, GGT
loroso a la palpacin sin signos de irritacin peritoneal. No con ligera elevacin). En este contexto de alcoholismo la
megalias. Resto de exploracin normal. fiebre puede ser de causa no infecciosa como en la hepatitis
alcohlica, pancreatitis aguda o el carcinoma hepatocelu-
Pruebas Complementarias: lar. La amilasa ligeramente elevada, la ecografa normal y
la elevacin leve de las transaminasas sin un claro predo-
Hemograma: hemoglobina 13.7 g/dl, leucocitos minio de GOT>GPT descartan estos procesos. Presenta
15000/mm3 (88% PMN) y plaquetas 35000/mm3. una hiperbilirrubinemia mixta sin un patrn definido: ni
Citomorfologa perifrica: neutrofilia con granulacin colestasis (Fosfatasa alcalina normal), ni lesin hepatoce-
txica, anisopoiquilocitosis discreta, sin esquistocito- lular difusa (las transaminasas no estn desproporcionada-
sis. mente elevadas con respecto a la fosfatasa alcalina).
Coagulacin: actividad de protrombina 70%, fibrin- La asociacin de un fracaso renal agudo oligrico con un
geno 7.5 g/l. sedimento urinario no nefrtico, con riones de tamao
Bioqumica: urea 151, mg/dl, creatinina 10 mg/dl, so- normal y parnquima ntegro segn la ecografa nos indi-
dio 132 mEq/l, potasio 3.2 mEq/l, creatinfosfocinasa ca que se trata de un cuadro prerrenal o parenquimatoso.
(CPK) 554 U/I, Bilirrubina total 17.04 mg/dl (bilirru- En este contexto, otra posibilidad diagnstica es el sndro-
bina directa 10,02 y bilirrubina indirecta 7,02 mg/dl), me hemoltico urmico, pues presenta insuficiencia renal
GOT 57 U/l, GPT 53 U/l, GGT 96 U/l, fosfatasa y trombopenia pero no anemia hemoltica ni esquistocitos
alcalina 80 U/I, protenas 6,1 g/dl, amilasa 110 U/l, en el frotis de sangre perifrica, ni explicara la afectacin
resto normal. PCR 79 mg/l. VSG 88 mm/h. heptica. Otra posibilidad diagnstica es el sndrome anti-
Sedimento de orina: microhematuria, nitritos positi- fosfolpido catastrfico por aparecer compromiso de varios
vos, leve leucocituria, bilirrubina positivo alto. rganos y desarrollo de las manifestaciones clnicas en una
Gasometra venosa: pH 7,38, pCO2 38mmHg, pO2 semana, pero es un cuadro que se presenta con una mayor
37 mmHg, BE -2,6. frecuencia en mujeres (72%) en el contexto de un sndrome
Radiografa de trax y ecografa abdominoplvica: sin antifosfolpido primario o secundario a un lupus eritema-
hallazgos patolgicos. toso sistmico.
Las infecciones hepticas por hongos son poco probables
Diagnstico Diferencial por no tratarse de un paciente inmunodeprimido ni neu-
tropnico. El no haber realizado viajes al extranjero hace
En el diagnstico diferencial inicial ante un paciente con poco probable la infeccin por trematodos hepticos como
fiebre e ictericia hay que incluir las causas infecciosas: va Clonorquiasis, Fasciola heptica y Opistorquiasis.

269
CAPTULO 14
INFECCIONES
POR PATGENOS ESPECIALES

Ante los hallazgos de fallo heptico y renal, trombopenia, Organizacin Mundial de la Salud ha estimado que ocu-
fiebre y hemorragia se sospecha un cuadro infeccioso con rren entre cuatro y cien casos por cada 100000 habitantes
repercusin sistmica. anualmente. Los reservorios ms importantes son mam-
El fracaso renal agudo con cifras de potasio normales, la feros pequeos que pueden transmitir la infeccin a los
elevacin de la isoenzima muscular de la creatinfosfoci- animales domsticos y a los humanos. Los seres humanos
nasa, (en ausencia de traumatismo, ejercicio fsico o inyec- se infectan de forma ocasional a travs del contacto directo
cin intramuscular reciente) y la presencia de hemorragias o indirecto con animales infectados, la epidemiologa de la
junto a un ambiente epidemiolgico concreto (trabajador infeccin humana est determinada por la naturaleza de
portuario) sugieren el diagnstico de leptospirosis. estos contactos, ya que la transmisin interhumana, aun-
El no haber realizado viajes al extranjero excluye otros que posible, es extremadamente rara. El contacto directo
cuadros infecciosos como Hantavirus, Dengue y Paludis- con animales o el contacto indirecto a travs del agua o
mo que comparten sntomas y patrones endmicos simila- el suelo contaminado pueden transmitir la infeccin al ser
res con la leptospirosis. Otra enfermedad a tener en cuenta humano, aunque la transmisin por contacto indirecto es
en el diagnstico diferencial es la rickettsiosis que puede lo ms comn. Son frecuentes las exposiciones ocupacio-
imitar la leptospirosis. nales (matarifes, granjeros, veterinarios) y las exposiciones
recreativas (nadadores, personas que acampan y pescado-
Evolucin res). Las leptospiras atraviesan la piel a travs de heridas
o abrasiones, pero tambin parecen capaces de atravesar
El paciente ingres en la Unidad de Cuidados Intensivos, la piel intacta, y ciertamente atraviesan las mucosas sanas.
durante las primeras 48 horas, evolucionando de forma La mayora de los pacientes tienen la forma leve y anic-
favorable con diurticos, antibioterapia emprica y trans- trica de la enfermedad (90%) y entre el 5 y 10% de los
fusin de plaquetas. que cursan con leptospirosis presentan la forma clsica,
Los hemocultivos, urocultivo y coprocultivo fueron negati- conocida fiebre icteroazomica o enfermedad de Weil,
vos. Se realizaron serologas frente a citomegalovirus, her- caracterizada por signos de afectacin heptica, renal y
pes, toxoplasma, varicela, virus de Epstein-Barr, rickettsias, vascular. En la enfermedad de Weil la afectacin hepti-
Borrelia burgdorferi, Coxiella burnetti, VIH, Yersinia pseudotuber- ca tiene algunas peculiaridades: aunque existe ictericia no
culosis y enterocolitica 0:3 y 0:9 que fueron negativas o in- hay evidencia de destruccin hepatocelular y la concen-
feccin pasada. Virus de la hepatitis: VHA IgM negativo, tracin de transaminasas rara vez supera las 200 U/l. La
HbsAg negativo, Anti Hbc negativo y Anti VHC negativo. bilirrubina habitualmente se mantiene cerca de 20 ng/dl
Evolucion de forma favorable en planta con antibiotera- y es de predominio directo. La leptospira causa una forma
pia de amplio espectro (meropenen, metronidazol y doxici- nica de insuficiencia renal aguda hipopotasmica, cuya
clina) con normalizacin de la funcin renal y de las cifras caracterstica principal es la alteracin en la reabsorcin
de plaquetas, con bilirrubina y transaminasas en descenso proximal de sodio. La aparicin de oliguria es un elemento
progresivo. A su alta el paciente se encontraba asintomti- de mal pronstico.
co y se mantuvo tratamiento domiciliario con doxiciclina, Las manifestaciones ms frecuentes de la forma ictrica
Se envi muestra al centro de referencia para leptospirosis icteroazomica son: insuficiencia renal (79%), ictericia
con resultado de IgM positivo. (70%), trombocitopenia (50-65%), elevacin de transami-
nasas (56%), meningitis (23%) y manifestaciones hemorr-
Diagnstico final gicas (11,6%). La sufusin hemorrgica subconjuntival de
estar presente es muy sugestiva de infeccin por lesptospira
Leptospirosis icteroazomica o enfermedad de Weil. pues raramente aparece en otros cuadros infecciosos. La
mayora de casos de leptospirosis son de leves a modera-
Discusin dos. Sin embargo, pueden aparecer complicaciones que
empeoran el pronstico (insuficiencia renal, uvetis, hemo-
La leptospira es una espiroqueta aerobia y dotada de movi- rragia, sndrome de dificultad respiratoria aguda con he-
miento, gram negativa. Se conocen dos especies Leptospira morragia pulmonar, miocarditis y rabdomilisis).
interrogans y Leptospira biflexa. L.interrogans es la nica especie Las anormalidades de laboratorio incluyen anemia, trom-
que causa leptospirosis, siendo una zoonosis de distribu- bocitopenia, leucocitosis con neutrofilia y un incremento
cin mundial, infectando a ms de 60 especies de mam- caracterstico en el nivel de creatinfosfocinasa, el cual pue-
feros. La incidencia no se conoce con precisin aunque la de aparecer precozmente.

270
CAPTULO 14
INFECCIONES
POR PATGENOS ESPECIALES

El diagnstico temprano resulta esencial puesto que el tra- les, silvestres o exticos. Siendo una patologa que tratada
tamiento antibitico logra mejores resultados si se admi- oportunamente favorece la evolucin del paciente, por lo
nistra al comienzo de la enfermedad. La falta de una prue- que debe contemplarse en el diagnstico diferencial de sn-
ba diagnstica adecuada constituye el principal obstculo dromes spticos con ictericia. Y que requiere un alto ndice
para el diagnstico precoz y la vigilancia epidemiolgica. de sospecha basado en la exposicin epidemiolgica y las
El diagnstico de certeza requiere el aislamiento de lep- manifestaciones clnicas, ya que los hallazgos clnicos y de
tospiras en sangre, orina o LCR. Las muestras de sangre y laboratorio son inespecficos.
LCR son en general positivas durante los primeros 10 das
de la enfermedad. El hemocultivo es poco sensible (aisla- Bibliografa
miento del microorganismo con xito entre un 5% y un
50% de los casos). Los cultivos de orina son positivos en la 1. Rojas A Gloria, Kong C Jorge, Donoso F Alejandro,
segunda semana de la enfermedad y siguen siendo positi- Prado D Priscilla. Una causa infrecuente de falla re-
vos para un mximo de 30 das despus de la resolucin de nal aguda e ictericia. Leptospirosis: caso clnico y re-
los sntomas. Sin embargo, el mtodo diagnstico ms co- visin de la literatura. Rev. chil. pediatr. [revista enla
mn es la serologa. La aglutinacin microscpica o prue- Internet]. 2001 Mayo; 72: 230-234. Disponible en:
ba de microaglutinacin (MAT) es el mtodo ms fiable http://www.scielo.cl/scielo.php?script=sci_arttex-
y muchos la consideran como una tcnica de referencia, t&pid=S0370-41062001000300008&lng=es.
con elevada sensibilidad y especificidad en humanos (92 y 2. Abdulkader RCRM, Seguro AC, Malheiro PS, Burd-
95%, respectivamente) pero presenta ciertas limitaciones, mann EA, Marcondes M. Peculiar electrolytic and
no siendo til para detectar la infeccin antes del sptimo hormonal abnormalities in acute renal failure due to
da de evolucin. Adems el rendimiento diagnstico de las leptospirosis. Am J Trop Med Hyg.1996;54:1-6.
pruebas serolgicas resulta todava ms limitado en regio- 3. Levett PN. Leptospirosis. Clin Microbiol Rev.
nes de alta endemicidad, puesto que los anticuerpos pro- 2001;14:296-326.
ducidos en respuesta a una infeccin pueden permanecer 4. Dupouey J, Faucher B, Edouard S, Richet H, Kodjo
detectables durante aos tras la exposicin. Las tcnicas A. Human leptospirosis: An emerging risk in Europe?.
moleculares tales como PCR en tiempo real y la amplifi- Comparative Inmunology, Microbiology and Infec-
cacin isotrmica mediada por LOOP (LAMP) se han de- tious Diseases 2014; 37: 77-83.
sarrollado para el diagnstico pero no estn ampliamente 5. Dupont H, Dupont-Perdrizet D, Perie JL, Zeh-
disponibles, aunque son prometedoras para el diagnstico ner-Hansen S. Leptospirosis: prognostic factors asso-
rpido y preciso. ciated with mortality. Clin Infect Dis. 1997;25:720-4.
En cuanto al tratamiento en los casos de leptospirosis grave 6. Schreier S, Doungchawee G, Chadsuthi S, et al. Lep-
en adultos hospitalizados la penicilina (1.5 millones de uni- tospirosis: current situation and trends of specific labo-
dades por va intravenosa cada seis horas), doxiciclina (100 ratory tests. Expert Rev Clin Immunol 2013; 9:263-80.
mg dos veces al da intravenosa) y cefalosporinas de tercera 7. Suputtamongkol Y, Niwattayakul K, Suttinont C, et
generacin son de eleccin (ceftriaxona 1-2 g una vez al al. An open, randomized, controlled trial of penicillin,
da o cefotaxima 1 g cada seis horas). La duracin del trata- doxycycline, and cefotaxime for patients with severe
miento de la enfermedad severa es generalmente siete das. leptospirosis. Clin Infect Dis 2004; 39:1417-24.
En los casos ms graves puede ser necesario tratamiento 8. Rossi A. Sndrome Antifosfolpido Catastrfico. He-
renal sustitutivo, soporte ventilatorio y transfusin de he- matologa. 2014;18:40- 7.
moderivados. No est claro que el tratamiento antibitico 9. Campistol J, Arias M, Ariceta G, Blasco M, Espinosa
reduzca la mortalidad pero s disminuye la probabilidad de M, Grinyo J, et al. Actualizacin en Sndrome Hemo-
progresin de la enfermedad a la forma severa ltico Urmico atpico: documento de consenso. Nefro-
La leptospirosis ha emergido como una enfermedad infec- loga 2013; 33: 27-45.
ciosa, entre tantas que hoy afectan al hombre. Tiene una
incidencia creciente y es sub-diagnosticada dentro de los
sndromes febriles, spticos e ictricos de otras etiologas.
En los pases industrializados constitua tradicionalmente
una enfermedad ocupacional pero en la actualidad afecta
tambin a adeptos del ecoturismo, viajeros y deportistas
que participan en actividades recreativas en medios rura-

271
Varn con fiebre, esplenomegalia y pancitopenia
Paniagua Garca, M; Palacios Baena, ZR;
Martnez Prez-Crespo, PM; Valiente Mndez, A.
Hospitales Universitarios Virgen Macarena y Virgen del Roco. Sevilla.

Caso Clnico

Varn de 41 aos, sin reacciones adversas a medicamentos


conocidas, fumador de 6 cigarros diarios, niega otros txi-
cos. Como antecedentes personales destacan una depre-
sin tras el fallecimiento de sus padres, y disfuncin erctil
sin causa orgnica. Ha sido intervenido de fstula perianal
y de osteosntesis de tobillo por accidente. No realiza trata-
miento de forma habitual. Procedente de una zona rural,
en contacto con ganado, perros y gatos. Niega consumo de
productos no pasteurizados, as como viajes recientes, pi-
caduras y contactos sexuales de riesgo. Antecedentes fami-
liares: padre fallecido a consecuencia de cncer de colon.
El paciente acude a Urgencias por fiebre de hasta 40C, TAC de abdomen.
cefalea e intensa astenia, de cinco das de evolucin, sin
otra sintomatologa acompaante. A la exploracin, he- B19, y tambin 2 serologa. Antgeno paldico y tincin
modinmicamente estable, taquicrdico y presenta hepa- Giemsa negativos. Se solicit serologa frente a Leishmania
tomegalia dolorosa. No afectacin cutnea ni articular. En spp, pero ante la falta de disponibilidad en el laboratorio
el control analtico se evidencia pancitopenia: Hemoglo- de Microbiologa sta no lleg a realizarse.
bina (Hb) 12.8 mg/dl, leucocitos 3.910/mm3, plaquetas Para el diagnstico del virus de la inmunodeficiencia hu-
43.000/mm3 (Normal: 150.000-450.000), aumento de mana (VIH) se realiz tanto serologa como determinacin
transaminasas: aspartato aminotransferasa -AST- 297 cuantitativa de la carga viral, y ambos resultaron negati-
U/L (N:0-37), alanina aminotransferasa -ALT- 354 U/L vos. Se solicitan Mantoux e interfern gamma (IFN-gam-
(N: 0-40), lactacto deshidrogenasa (LDH) 1536 U/L (N: < ma), que resultaron negativos. En el estudio microscpico
460) y protena C reactiva (PCR) 220 mg/L (N: 1-4). Fun- de muestras teidas con los colorantes de Ziehl-Neelsen no
cin renal e iones con valores dentro de la normalidad. Se se evidenciaron bacilos cido-alcohol resistentes (BAAR),
extraen hemocultivos seriados, urocultivos, y serologas. La y el cultivo de micobacterias obtenido en este momento
radiografa de trax sin imgenes de condensaciones. La fue tambin negativo tras el periodo de estudio de 46 das.
ecografa abdominal muestra el bazo en el lmite superior En el siguiente control analtico, se evidencia mejora tanto
de la normalidad (12.7 cm). Ante la sospecha de zoonosis de las transaminasas como de los parmetros inflamato-
habida cuenta de los antecedentes recogidos en la historia rios. Los reticulocitos fueron de 3.04% (N: 0.5-1.5), lo que
clnica, se inicia tratamiento con doxiciclina. Durante el orienta hacia un origen perifrico de la anemia. El test de
ingreso la fiebre fue bien tolerada, disminuyendo progre- Coombs directo fue positivo, lo que sugiere anemia hemo-
sivamente, hasta presentar nicamente un episodio diario, ltica. En el frotis de sangre perifrica, no se observan al-
sin sudoracin. No se produjeron episodios de sangrado teraciones morfolgicas que orientasen hacia el origen de
ni se evidenciaron hematomas. Los hemocultivos fueron la pancitopenia, por lo que se contacta con Hematologa.
repetidamente negativos, tambin result negativo el uro- Se realiza tanto aspirado como biopsia de mdula sea, se
cultivo. solicit estudio directo citohistoqumico, junto con tincio-
En las serologas, se constata una infeccin pasada por ci- nes y cultivo para descartar infeccin por micobacterias,
tomegalovirus (CMV) y por virus de Epstein-Barr (VEB), leishmaniasis y Brucella; no informndose la presencia de
virus hepatotropos negativos. Coxiella burnetii, Bartonella anormalidades en la tincin ni en la citometra.
quintana y B. henselae, Leptospira spp., Rickettsia spp., Bruce- Durante el tiempo de ingreso el paciente permanece asin-
lla sp., Toxoplasma gondii negativos y pendientes parvovirus tomtico, salvo por la presencia de episodios aislados bien

272
CAPTULO 14
INFECCIONES
POR PATGENOS ESPECIALES

tando empeoramiento de la funcin heptica: GOT y


GPT>1000 U/l, Bilirrubina total 8-16 mg/dl (N: 0.0-1.0)
a expensas de la fraccin directa, lactato deshidrogenasa
(LDH), fosfatasa alcalina (FA) y gamma glutamil transpep-
tidasa (GGT) elevadas.
Se realiza biopsia heptica transyugular, donde se eviden-
cia necrosis submasiva de origen probable txico, adems
de abundante pigmento frrico en las clulas de Kupffer.
En este contexto se solicita resonancia magntica heptica,
donde se aprecia moderada sobrecarga de hierro, sin im-
genes de masas. La muestra heptica se tie con tincin de
Rojo Congo, que fue negativa.

Diagnstico diferencial

El diagnstico diferencial que se plantea ante un varn


joven con fiebre elevada de origen comunitario, pancito-
penia y alteraciones hepticas, debe incluir fundamental-
mente patologa de causa infecciosa como la que se detalla,
y mxime en nuestro medio: bacterianas: zoonosis, infec-
ciones por micobacterias tuberculosas y no tuberculosas;
parsitos (malaria y leishmaniasis), virales (VIH, virus he-
patotropos, parvovirus B19). Por otro lado, enfermedades
hematolgicas con invasin maligna de MO (mielofibrosis,
leucemia, linfoma) o sndromes mielodisplsicos. Tambin
Amastigotes de Leishmania spp. entrara en el diagnstico diferencial el sndrome hemo-
fagoctico, adems de otras hepatopatas y enfermedades
tolerados de fiebre sin sudoracin que ceden con antitr- autoinmunes. Siempre se debe incluir en el diagnstico
micos, y se encuentra hemodinmicamente estable. Se diferencial posibles efectos txicos farmacolgicos o de
consensua con el paciente continuar el estudio de modo agentes qumicos (citotxicos, benzeno, antiepilpticos).
ambulatorio. Se plantean adems enfermedades infiltrativas, como la
Sin embargo, el paciente reingresa pasados 3 das por per- enfermedad de Gaucher, hemocromatosis o amiloidosis,
sistencia de la fiebre a pesar de tratamiento antipirtico, el como causas potenciales de esplenomegalia.
sndrome febril que empieza a ser mal tolerado. La explo-
racin fsica fue anodina, y en el control analtico persiste Evolucin
la pancitopenia. En el TAC de trax y abdomen realizado
se evidencian pequeas adenopatas mediastnicas de ca- Se repitieron las serologas, que resultaron positivas para
rcter inespecfico, esplenomegalia ligera (13.6 cm) y ade- IgG e IgM de parvovirus B19, resto negativas.
nopatas retrocrurales y periarticas. Se repite la biopsia de Dada la persistencia de la pancitopenia, se repite el estudio
mdula sea, en la que se aprecia una escasa representa- de mdula sea, donde se aprecia hemofagocitosis: proli-
cin de la serie mieloide y megacariocitos displsicos, con feracin de histiocitos de morfologa normal con intensa
fibrosis leve. La citologa descarta infiltracin linfomatosa. actividad fagoctica de clulas hemopoyticas. No se evi-
Se solicita estudio autoinmune, inicialmente los anticuer- dencian parsitos.
pos anti msculo liso y los anticuerpos antinucleares (ANA) Se confirma el diagnstico de sndrome hemofagoctico,
fueron positivos (patrn nucleolar 1/160), posteriormente atendiendo a los siguientes criterios: fiebre, citologa com-
se fueron normalizando. El proteinograma y los marca- patible, esplenomegalia y citopenias en las tres series. En el
dores tumorales fueron normales. Se constata elevacin estudio de mdula sea no se aprecia la presencia de par-
tanto de los triglicridos (TG) 211 mg/dl (N<150) como de sitos ni otros microorganismos. Se postula en este momen-
la ferritina, 4190 ng/ml (N 12-300 en varones). to como posible causante, la reactivacin de una infeccin
El paciente desarrolla un cuadro de ictericia, presen- por parvovirus B19 evidenciada en la serologa.

273
CAPTULO 14
INFECCIONES
POR PATGENOS ESPECIALES

Dada la mala evolucin clnica del paciente, con empeo- ganos (en nuestro paciente el hgado) y pancitopenia.
ramiento de la funcin heptica, se acepta por el Servicio Para su diagnstico, se utilizan los Criterios HLH-2004,
de Hematologa, inicindose tratamiento quimioterpico tienen que cumplirse cinco de los siguientes ocho: 1.Fie-
para el SHF con Etopsido y Dexametasona, segn esque- bre; 2.Esplenomegalia; 3.Citopenias de al menos dos lneas
ma de Protocolo HLH-94. Tras primer ciclo de quimiote- celulares; 4. Hipofibrinogenemia o hipertrigliceridemia;
rapia se produce una mejora de los parmetros hepticos, 5.Hiperferritinemia; 6.Aumento de los niveles de receptor
hasta llegar a una normalizacin completa, por lo que no soluble de interleukina 2 (IL-2); 7.Visualizacin de hemo-
fue necesario aadir ciclosporina. fagocitosis en mdula sea; 8. Ausencia o disminucin de
Dos meses ms tarde, el paciente recibe el alta de Hemato- la actividad de clulas NK.
loga, para seguimiento en consultas externas. Sin embar- El Gold Standard es el estudio de mdula sea: el aspira-
go, dada la persistencia de fiebre y pancitopenia, y a pesar do resulta positivo en el 84% de los casos, siendo la biopsia
de no estar incluido dentro del protocolo de actuacin ante menos efectiva (64%), pero permite descartar una neoplasia
un sndrome hemofagoctico, se repite estudio de mdula hematolgica de base. La hemofagocitosis puede no detec-
sea, y en la citologa se aprecian amastigotes de Leishmania tarse durante el curso de la enfermedad, ya que puede estar
spp., con ausencia de hemofagocitosis. Ante esta circuns- ausente en las fases iniciales o cuando existe diseritropoyesis.
tancia, y para datar la cronologa de la leishmaniasis, se Por tanto en casos de sospecha es necesario repetir el estu-
utiliza una muestra del paciente de la seroteca del labora- dio.
torio de Microbiologa, correspondiente al primer ingreso, Es obligatorio en todos los pacientes buscar el agente infec-
y dado que ya est disponible la tcnica, se solicita serolo- cioso desencadenante, siendo las ms frecuentes las infeccio-
ga frente a Leishmania spp. sta result ser positiva, lo que nes vricas, especialmente: VEB (50% de casos), virus de her-
nos indica que el paciente ya estaba infectado por Leish- pes simple -VHS- (2/3), CMV (8%). Se han descrito casos
mania spp. al comienzo del cuadro, a pesar de no haberse relacionados con virus de la hepatitis A, parvovirus B19 y los
documentado en los repetidos estudios de mdula sea. virus de la gripe. La mitad de las infecciones bacterianas se
producen por TBC, siendo menos frecuentes Rickettsia spp.,
Diagnstico final Staphylococcus spp. y E. coli. Por ltimo, se han descrito SHF
desencadenados por parsitos y hongos como en el caso de
El diagnstico final es de sndrome hemofagoctico secun- leishmaniasis, malaria, histoplasmosis y toxoplasmosis.
dario a leishmaniasis visceral (LV) en un paciente inmuno- La leishmaniasis visceral (LV) o kala-azar, constituye una
competente, que secundariamente desarrolla una hepati- causa curable de SHF. Se trata de una zoonosis provoca-
tis aguda con necrosis submasiva e insuficiencia heptica da por parsitos del gnero Leishmania spp., transmitida a
aguda. travs de la picadura del flebotomo, dptero de la familia
Psychodidae y subfamilia Phlebotominae. Leishmania spp.
Discusin sobrevive en el husped vertebrado como amastigotes in-
tracelulares. La especie principal responsable es Leishmania
El sndrome hemofagoctico (SHF) linfohistiocitosis hemo- infantum, endmica del Mediterrneo, cuyo principal reser-
fagoctica es una enfermedad autoinmune caracterizada vorio es el perro. El perodo de incubacin puede llegar
por una activacin incesante de linfocitos T (especialmen- a ser desde 2 semanas hasta 18 meses. La poblacin en
te CD8+) y macrfagos. Aunque es ms frecuente en pa- riesgo son fundamentalmente los lactantes y nios (edad
cientes peditricos, en los que se considera enfermedad media tres aos), y los sujetos inmunodeprimidos.
hereditaria, el 40% de los casos aparece en adultos, con La clnica de la LV es similar al SHF: fiebre, hepatomega-
una incidencia de 1/800.000 personas/ao, afectando es- lia y pancitopenia, por lo que a veces puede resultar difcil
pecialmente al sexo femenino (razn varn:mujer 1:7). La diferenciarlos. La hepatitis severa es una manifestacin ex-
edad media al diagnstico 50 aos. tremadamente rara.
De acuerdo a la etiologa, diferenciamos SHF primario El SHF asociado a Leishmaniasis visceral es una entidad
o gentico, y secundario o adquirido. En adultos la gran rara (se han descrito 30 casos en la literatura), indistingui-
mayora acontecen por confluencia de factores externos ble de otros SHF, pero siempre hay que tenerlo en conside-
desencadenantes y enfermedades predisponentes de base. racin para evitar tratamientos innecesarios, que pueden
La clnica de sospecha es inicialmente inespecfica, por lo resultar perjudiciales. ste es el caso de nuestro paciente,
que puede pasar desapercibida. Cursa con fiebre elevada, tratado con quimioterapia hasta el aislamiento de Leishma-
mantenida y no filiada, junto con afectacin de varios r- nia spp. en mdula sea.

274
CAPTULO 14
INFECCIONES
POR PATGENOS ESPECIALES

El diagnstico se basa en demostrar la presencia de parsi- la calcineurina, metotrexato o ciclofosfamida. Existe poca
tos en las clulas del sistema retculoendotelial, utilizando evidencia por el momento con respecto al uso de recambio
la tincin de Giemsa. Se aconseja la biopsia de mdula plasmtico y factores biolgicos.
sea, aunque no es infrecuente que los amastigotes de Lei- En cuanto a la quimioterapia, se sigue el Protocolo LHH-
shmania spp. no aparezcan hasta fases ms tardas de la en- 94 (dexametasona, etopsido y metotrexato si afectacin
fermedad: se han descrito entre un 46% y un 78% de falsos del sistema nervioso central). Si no hay remisin se podra
negativos en biopsias de mdula sea, ya que la sensibili- valorar transplante de progenitores hematopoyticos.
dad vara en funcin de la carga parasitaria. La biopsia A pesar de que nuestro paciente presentara infeccin coe-
esplnica tiene una sensibilidad de hasta el 98%, pero el xistente parvovirus B19, se lleg a la conclusin que la cau-
riesgo hemorrgico, dado el estado de hipocoagulacin y sa del SHF haba sido una infeccin por Leishmania spp., no
trombopenia, es elevado. La PCR de la muestra de mdula documentada en el estudio de mdula sea hasta inmuno-
sea presenta 100% de sensibilidad para Leishmania spp. La depresin por quimioterapia, pero ya presente en las fases
serologa tiene importante valor diagnstico en el pacien- iniciales del cuadro, atendiendo a los resultados de la sero-
te inmunocompetente, pero tiene menor sensibilidad en el loga. Dado que tanto IgG como IgM del parvovirus B19
inmunodeprimido. Los cultivos de sangre perifrica o de resultaron positivas, se piensa en una posible reactivacin
mdula sea tambin permiten llegar al diagnstico. de parvovirus, no responsable del SHF. No disponemos en
El tratamiento de la LV es complejo, ya que tanto el fr- nuestro laboratorio de tcnicas de PCR cuantitativas para
maco como la dosis y duracin varan en funcin de la parvovirus B19.
regin endmica. A pesar de concluir el tratamiento reco-
mendado, algunos pacientes sufren recadas (especialmen-
te los primeros seis meses), por lo que es recomendable un
seguimiento prolongado. En los pases mediterrneos, el
frmaco de eleccin es la anfotericina B (AmB) liposomal,
con rgimen teraputico de 3-5 mg/Kg/da intravenoso
(iv) los das 1 a 5, 14 y 21. La evolucin clsica con tra-
tamiento es la obtencin de apirexia dentro de las 24-48
horas siguientes, mejora rpida del estado general y nor-
malizacin de la trombopenia en la primera semana. Una
vez que la enfermedad aparece, es mortal sin tratamiento.
Con respecto al tratamiento del SHF secundario a LV,
se han propuesto diferentes esquemas teraputicos, pero
no existen protocolos especficos estandarizados. En au-
sencia de recada o de resistencia, los compuestos de anti-
monio pentavalente suponen el tratamiento de referencia.
La posologa recomendada por la Organizacin Mundial
de la Salud (OMS) es de 20 mg/kg/da durante 28 das,
prefiriendo la va intramuscular para evitar toxicidad. La
anfotericina B liposomal (3-5 mg/Kg/da iv) constituye el
tratamiento alternativo. Es muy importante garantizar las
medidas de apoyo vital (transfusin de hemoderivados, he-
parina profilctica) y la eliminacin de factores desen-
cadenantes. La inmunoglobulinas polivalentes han demos-
trados ser tiles en los SHF postinfecciosos.
El tratamiento precoz y agresivo con terapia inmunosupre-
sora es crucial, a pesar de los posibles efectos adversos. Los
glucocorticoides se suelen usar como tratamiento inicial,
indepedientemente de la causa. Se prefiere el uso de dexa-
metasona si existe afectacin del sistema nervioso central
(SNC). La ciclosporina se utiliza como tratamiento de in-
duccin, aunque tambin son eficaces los inhibidores de

275
CAPTULO 14
INFECCIONES
POR PATGENOS ESPECIALES

Bibliografa

1. Ramos-Casals M, Brito-Zern P, Lpez-Guillermo A,


Khamashta MA. Adult haemophagocytic syndrome.
Lancet. 2014; 383: 150316.
2. Sebastian F, Bode N, Bogdan C, Beutel K, Behnisch
W, Greiner J et al. Hemophagocytic Lymphohistio-
cytosis in Imported Pediatric Visceral Leishmaniasis in
a Nonendemic Area. The Journal of Pediatrics. 2014;
165:147-53.
3. Oudaina W, Assini K, El Ouardi M, Tligui H. Seve-
re macrophage activation syndrome following visceral
leishmaniasis in a child. Mdecine et Sant Tropicales.
2014; 24: 221-3.
4. Karrasch M, Felber J, Keller PM, Kletta C, Egerer R,
et al. Primary EpsteinBarr virus infection and proba-
ble parvovirus B19 reactivation resulting in fulminant
hepatitis and fulfilling five of eight criteria for hemo-
phagocytic lymphohistiocytosis. Int J Infect Dis. 2014;
28: 143-6.
5. Rosado F, Kim AS. Hemophagocytic Lymphohistio-
cytosis, an Update on Diagnosis and Pathogenesis. Am
J Clin Pathol. 2013; 139: 713-27.
6. Visentin S, Baudesson de Chanville A, Loosveld M,
Chambost H, Barlogis V. Infantile visceral leishmania-
sis, an etiology of easily curable hemophagocytic lym-
phohistiocytosis syndrome [Article in French]. Arch
Pediat. 2013; 20: 1225-9.
7. Bouguila J, Chabchoub I, Moncef Y, Mlika Y, Sa-
ghrouni F, Boughamoura L, Essoussi AS. Treatment
of severe hemophagocytic syndrome associated with
visceral leishmaniasis [Article in French]. Arch Pediat.
2010; 17:1566-70.
8. Mathur P, Samantaray JC, Samanta P. Fatal Haemo-
phagocytic syndrome and hepatitis associated with
visceral leishmaniasis. Indian J Med Microbiol. 2007;
25:416-8.

276
Fiebre y sndrome constitucional
en mujer joven inmunocompetente
Chacn Mora, N; Rodrguez Torres, P;
Caldern Cabrera, C; Aguilar Guisado, M.
Hospital Universitario Virgen del Roco. Sevilla.

Caso clnico

Presentamos el caso de una mujer de 36 aos que ingre-


s por fiebre y sndrome constitucional de tres meses de
evolucin. No presentaba antecedentes mdicos de inters,
salvo tabaquismo. En mayo de 2009 acudi a su mdico
de atencin primaria por fiebre vespertina de hasta 38.5C
acompaada de astenia, anorexia y prdida de peso. Se
realiz un hemograma y una bioqumica que mostraron
pancitopenia e hipertransaminasemia moderada, serolo-
gas para virus hepatotropos, CMV, VEB y Borrelia spp. que
fueron negativas, salvo inmunoglobulina G de VEB posi-
tiva, y ecografa abdominal donde se apreciaba hepato-es-
plenomegalia. Fue diagnosticada inicialmente de sndrome
mononuclesico y se prescribi tratamiento sintomtico.
Tras varias semanas no present mejora clnica y en suce-
sivos hemogramas se objetiv pancitopenia progresiva por
lo que se deriv a nuestro centro a principios de agosto de
2009.
A su ingreso presentaba aceptable estado general, con pa-
lidez cutnea. Se encontraba eupneica, afebril y normo-
tensa. La exploracin de la cavidad oral y la auscultacin
cardiopulmonar era normal. El abdomen era doloroso a
la palpacin en ambos hiponcondrios, palpndose hepa-
tomegalia de 6 cm por debajo del reborde costal y esple-
nomegalia de 9 cm. No tena edemas ni adenopatas peri-
fricas.
En el hemograma destacaba una pancitopenia con 2.27
x10e9/L leucocitos, 0.9 x10e9/L neutrfilos y 1.2 x10e9/L
linfocitos, hemoglobina de 100 g/L y 44 x10e9/L plaque-
tas. En el frotis de sangre perifrica se observaron algu-
nos dacriocitos y linfocitos activados (frmula leucocitaria:
30% de linfocitos, 14% de monocitos, 48% de neutrfilos, Imagen 1 y 2. TC abdominal. Hepato-espleomegalia gigantes.
7% bandas, 1% metamielocitos). En la bioqumica pre-
sentaba unas protenas totales de 9.3 g/dl, colinesterasa Varias tandas de hemocultivos fueron repetidamente ne-
de 3187 mU/ml y transaminasas normales. Haba una gativas.
elevacin de gammaglobulinas (3.7 g/dl, 48.5 %) de tipo La radiografa de trax fue normal. Se practic una TC
policlonal con inmunoglobulina G de 4160.00 mg/dl e in- con contraste intravenoso de cuello, trax y abdomen en
munoglobulina M de 351.00 mg/dl. La beta-2-microglo- el que se apreciaba hepatoesplenomegalia, con pequea
bulina fue de 3.82 mg/l, la PCR de 36.5 mg/l y la VSG de lesin hipodensa en hgado. No se observaron adenopatas
96.0 mm/h. Se solicitaron serologas para Leishmania ssp., ni otras alteraciones reseables.
VIH, virus hepatotropos, Brucella mellitensis, Coxiella burnetti,
virus herpes, les y Toxoplasma gondii.

277
CAPTULO 14
INFECCIONES
POR PATGENOS ESPECIALES

Diagnstico diferencial Sndrome hemofagoctico: se manifiesta con fiebre,


adenopatas, hepato-esplenomegalia, pancitopenia e
El abordaje diagnstico del paciente se plante con aquellas hipertransaminasemia.
entidades causantes de fiebre y sndrome constitucional con
hepatoesplenomegalia, pancitopenia, hipertransaminasemia Enfermedades por depsito:
e hipergammaglobulinemia policlonal. Entre las posibilidades
etiolgicas de este sndrome en nuestro medio destacamos: No suelen producir fiebre ni sndrome constitucional. La
enfermedad de Gaucher puede producir hepato-espleno-
Causas infecciosas: megalia, dolor y fracturas seas, anemia y trombocitope-
nia; y la amiloidosis, hepato-esplenomegalia, proteinuria e
Sndrome mononuclesico: por VEB, CMV, Toxoplas- hipertransaminasemia.
ma gondii, VIH. Fue el primer diagnstico que se con-
sider por su frecuencia y compatibilidad clnica aun- Evolucin
que se suele preceder de sntomas faringo-amigdalares.
Tuberculosis miliar: se presenta tpicamente con fie- El diagnstico de sospecha inicial fue de sndrome linfopro-
bre y sndrome constitucional de larga evolucin con liferativo. En primer lugar se realiz un aspirado-biopsia
esplenomegalia y leucopenia. Suele producirse en pa- de mdula sea: la citometra no detect clonalidad B ni
cientes inmunodeprimidos. otras alteraciones fenotpicas de linfocitos T y NK. La ana-
Leishmaniasis visceral: cursa con fiebre nocturna, he- toma patolgica objetiv una mdula sea hipocelular,
pato-esplenomegalia, adenopatas, infiltracin de la sin presencia de fibrosis. Ante la sospecha de linfoma es-
mdula sea con pancitopenia e hipergammaglobu- plnico se decidi realizar esplenectoma diagnstica y, a la
linemia policlonal. Es ms frecuente actualmente en espera del estudio anatomopatolgico, la paciente es dada
inmunodeprimidos en nuestro pas. de alta con tratamiento sintomtico para continuar el estu-
Brucelosis aguda: cursa con fiebre ondulante y sudora- dio de forma ambulatoria. Cinco das ms tarde reingresa
cin profusa, sndrome constitucional y artromialgias por presentar desde el alta, de forma progresiva, fiebre de
generalizadas, con adenopatas, hepatoesplenomega- hasta 39C con escalofros y dolor en hipocondrio dere-
lia, anemia y leucopenia. cho. Al tratarse de un sndrome febril postesplenectoma
Fiebre tifoidea: se presenta con fiebre alta prolongada, se trat de forma emprica con ceftriaxona y se practic
dolor abdominal, diarrea o estreimiento, hepato-es- TC abdominal que descart complicaciones secundarias
plenomegalia y lesiones cutneas. Su incidencia en a esplenectoma ni cambios respecto al previo. Se recibi
nuestro pas es casi anecdtica. el resultado del estudio anatomopatolgico del bazo que
En otros pases y en viajeros de zonas endmicas ha- mostr hematopoyesis extramedular y un importante in-
bra que considerar tambin la histoplasmosis y para- filtrado plasmocelular reactivo sin datos de clonalidad B.
coccidiomicosis diseminadas y la malaria, que pueden Ante la ausencia de diagnstico de hemopata maligna con
producir fiebre, organomegalias y pancitopenia entre las pruebas realizadas, que orientaban a un proceso reac-
otros sntomas; y la esquistosomiasis hepatoesplnica. tivo, se replante el diagnstico. Con la sospecha principal
de una posible leishmaniasis visceral (fiebre, hepato-esple-
Causas neoplsicas: nomegalia, pancitopenia e hipergammaglobulinemia poli-
clonal) se repiti el aspirado-biopsia de mdula sea en la
Sndromes linfoproliferativos: pueden debutar con fie- que, finalmente, se confirm la presencia de amastigotes de
bre, adenopatas, esplenomegalia e infiltracin de m- Leishmania ssp. Paralelamente se recibi el resultado de los
dula sea con pancitopenia. estudios serolgicos, siendo positivos para Leishmania ssp.
Sndromes mielodisplsicos: se caracterizan por sn- con un ttulo de Ig G por IFI de 1/256. Se comenz trata-
drome constitucional, pancitopenia y hemosiderosis miento con anfotericina B liposomal a dosis de 3 mg/Kg
aunque no suelen producir fiebre. con buena tolerancia. Tras las cinco primeras dosis que-
d afebril y progresivamente se fueron normalizando los
Causas inflamatorias: parmetros analticos. Complet dos dosis de tratamiento
ambulatorio consiguindose la curacin de la enfermedad.
Lupus eritematoso sistmico: puede producir fiebre sin fo-
calidad, adenopatas, afectacin cutnea y pancitopenia.

278
CAPTULO 14
INFECCIONES
POR PATGENOS ESPECIALES

Diagnstico final para el seguimiento al permanecer positiva hasta un ao


despus de la infeccin aguda al igual que ocurre con la se-
Leishmaniasis visceral hepato-esplnica. rologa [7]. La serologa puede ser til para el diagnstico.
Aunque la especificidad de las tcnicas de IFI y ELISA es
Discusin relativamente baja, cuando utilizan la protena recombi-
nante rk39, derivada de un antgeno especifico de L. dono-
La leishmaniasis visceral (LV) es una enfermedad produci- vani complex, en lugar de antgenos crudos de promastigotes
da por protozoos intracelulares estrictos del gnero Leish- de Leishmania ssp, la especificidad aumenta al 81% y 89%
mania que se transmiten a travs de la picadura de la mosca respectivamente [8]. En nuestro caso el diagnstico se ob-
hembra del gnero Phlebotomus. Hay dos tipos de LV que tuvo finalmente por visualizacin directa del parsito en
se diferencian en su mecanismo de transmisin: en la LV el aspirado y en la biopsia de la segunda mdula sea. La
antropontica el ser humano es el nico reservorio de la anatoma patolgica del bazo no fue diagnstica al no ser
infeccin y se observa principalmente en zonas de transmi- estudiada con la sospecha clnica de leishmaniasis.
sin de Leishmania donovani. En la LV zoontica la transmi- El tratamiento de eleccin en Europa es la anfotericina B
sin se produce desde un reservorio animal a un ser huma- liposomal debido a su eficacia y a su perfil de toxicidad, con
no y predomina en reas de transmisin de L. infantum [1]. una tasa de curacin del 95% a dosis de 3 mg/Kg durante
En Espaa se trata de una zoonosis causada por la especie los das del 1-5 y 10 en pacientes inmunocompetentes [9].
Leishmania infantum cuyo principal reservorio es el perro En nuestro caso se administr a 3 mg/kg los das 1 al 5, 14
domstico [2]. y 21 (dosis total de 21 mg/kg) segn recomendaciones de
En Europa la enfermedad se produce predominantemente la FDA [10] logrndose la curacin completa. La miltefo-
en nios menores de diez aos y en adultos inmunodepri- sina y la paramomicina son alternativas al tratamiento, son
midos [3], aunque se han descrito casos en sujetos inmu- activos frente al parsito con buen perfil de toxicidad. Los
nocompetentes en las zonas de mayor prevalencia [2, 4]. antimoniales pentavalentes son eficaces pero muy txicos,
Esta enfermedad se extiende por ms de 80 pases des- y slo deben utilizarse si no hay otra alternativa disponible.
tacando cinco focos epidemiolgicos: China. India, Asia Las estrategias de control de la enfermedad son diferen-
Central, Este de frica, cuenca Mediterrnea y Sudamri- tes en funcin del mecanismo principal de transmisin. En
ca. Alrededor del 90% de los casos ocurren en cinco pases: nuestro pas, al tratarse de una LVZ, la estrategia principal
India, Nepal, Bangladesh, Sudn y noreste de Brasil. Espa- se basa en el control del reservorio canino [3].
a es uno de los pases ms afectados de entre los pertene- El inters de nuestro caso radica en la baja prevalencia
cientes a la cuenca Mediterrnea [1]. de leishmaniasis visceral en pacientes inmunocompetentes
La infeccin es con frecuencia asintomtica, y los parsitos en nuestro medio, razn por la cual el ndice de sospecha
permanecen viables de por vida, pudiendo desarrollarse suele ser bajo. Por ese motivo se sospech como prime-
la enfermedad si se produce reactivacin debida a inmu- ra posibilidad el sndrome linfoproliferativo. Debido a la
nosupresin. El inicio de los sntomas suele ser insidioso, baja sensibilidad del aspirado de mdula en ausencia de
manifestndose con fiebre elevada, astenia, anorexia y pr- sospecha de la enfermedad, se retras el diagnstico al ser
dida de peso progresivas. La replicacin del parsito en el necesaria una segunda muestra.
sistema reticulo-endotelial produce esplenomegalia con o
sin hepatomegalia, adenopatas y pancitopenia, y la acti-
vacin policlonal de clulas B conduce a hipergammaglo-
bulinemia policlonal [5].
El diagnstico definitivo se basa en la visualizacin directa
de amastigotes en un aspirado o biopsia de mdula sea
mediante tincin de Giemsa o cultivo. La relativamente
baja (50-70%) sensibilidad del estudio de la mdula sea
hace aconsejable repetirlo si hay elevada sospecha clnica
y la primera muestra es negativa [6]. La sensibilidad es
mayor (98%) con una muestra de aspirado del bazo pero
el riesgo de sangrado hace que sea poco aconsejable. La
deteccin de antgenos de Leishmania ssp. en orina tiene una
sensibilidad en torno al 80-90%, sin embargo, no es til

279
CAPTULO 14
INFECCIONES
POR PATGENOS ESPECIALES

Bibliografa

1. Delacour H, Roche C, Roche B, Morand C, Koeck JL.


A Travel Misadventure Visceral Leishmaniasis in an
Immunocompetent Patient. JR Army Med Corps.
2010;156:169-71.
2. Pomara V, Muoz C, Domingo P. Increased liver en-
zymes and fever. Enferm Infecc Microbiol Clin. 2008;
26:667-8.
3. Palatnik-de-Sousa CB, Day MJ. One Health: The glo-
bal challenge of epidemic and endemic leishmaniasis.
Parasites & Vectors. 2011; 4:197.
4. Neghina R, Neghina AM, Merkler C, Marincu I, Mol-
dovan R, Iacobiciu I. Importation of visceral leishma-
niasis in returning Romanian workers from Spain.
Travel Med Infect Dis. 2009; 7:35-9.
5. Bauls AL, Bastien P, Pomares C, Arevalo J, Fisa R,
Hide M. Clinical pleiomorphism in human leishma-
niases, with special mention of asymptomatic infec-
tion. Clin Microbiol Infect. 2011; 17:145161.
6. Kouba M, Maloul I, Marrakchi C, Mdhaffar M,
Lahiani D, Hammami B et al. Hemophagocytic sy-
ndrome associated with visceral leishmaniasis in an
immunocompetent adult-case report and review of
the literature. Ann Hematol. 2012; 91:1143-5. DOI:
10.1007/s00277-011-1367-0.
7. Sundar S, Agrawal S, Pai K, Chance M, Hommel M.
Detection of leishmanial antigen in the urine of pa-
tients with visceral leishmaniasis by a latex agglutina-
tion test. Am J Trop Med Hyg. 2005; 73: 269-71.
8. Maia Z, Lirio M, Mistro S, Mendes CM, Mehta SR,
Badaro R. Comparative study of rK39 Leishmania
antigen for serodiagnosis of visceral leishmaniasis:
systematic review with meta-analysis. PloS Negl Trop
Dis. 2012;6(1):e1484.
9. Gradoni L, Soteriadou K, Louzir H et al. Drug re-
gimens for visceral leishmaniasis in Mediterranean
countries. Trop Med Int Health. 2008;13:1272-6.
10. Meyerhoff A. U.S. Food and Drug Administration
approval of AmBisome (liposomal amphotericin B)
for treatment of visceral leishmaniasis. Clin Infect Dis.
1999 Jan;28:42-8; discussion 49-51.

280
SAEI
SOCIEDAD ANDALUZA
DE ENFERMEDADES
INFECCIOSAS

You might also like